ΠΠΏΡΠ΅Π΄Π΅Π»Π΅Π½ΠΈΠ΅ ΠΊΠΎΠΎΡΠ΄ΠΈΠ½Π°Ρ Π²Π΅ΠΊΡΠΎΡΠ° Π·Π°Π΄Π°Π½Π½ΠΎΠ³ΠΎ ΠΊΠΎΠΎΡΠ΄ΠΈΠ½Π°ΡΠ°ΠΌΠΈ Π΅Π³ΠΎ Π½Π°ΡΠ°Π»ΡΠ½ΠΎΠΉ ΠΈ ΠΊΠΎΠ½Π΅ΡΠ½ΠΎΠΉ ΡΠΎΡΠΊΠΈ.
ΠΠΏΡΠ΅Π΄Π΅Π»Π΅Π½ΠΈΠ΅ ΠΊΠΎΠΎΡΠ΄ΠΈΠ½Π°Ρ Π²Π΅ΠΊΡΠΎΡΠ° Π·Π°Π΄Π°Π½Π½ΠΎΠ³ΠΎ ΠΊΠΎΠΎΡΠ΄ΠΈΠ½Π°ΡΠ°ΠΌΠΈ Π΅Π³ΠΎ Π½Π°ΡΠ°Π»ΡΠ½ΠΎΠΉ ΠΈ ΠΊΠΎΠ½Π΅ΡΠ½ΠΎΠΉ ΡΠΎΡΠΊΠΈ.ΠΠ°Π²ΠΈΠ³Π°ΡΠΈΡ ΠΏΠΎ ΡΡΡΠ°Π½ΠΈΡΠ΅:
- ΠΡΠ½ΠΎΠ²Π½ΠΎΠ΅ ΡΠΎΠΎΡΠ½ΠΎΡΠ΅Π½ΠΈΠ΅
- Π€ΠΎΡΠΌΡΠ»Ρ Π΄Π»Ρ ΠΎΠΏΡΠ΅Π΄Π΅Π»Π΅Π½ΠΈΠ΅ ΠΊΠΎΠΎΡΠ΄ΠΈΠ½Π°Ρ Π²Π΅ΠΊΡΠΎΡΠ° Π·Π°Π΄Π°Π½Π½ΠΎΠ³ΠΎ ΠΊΠΎΠΎΡΠ΄ΠΈΠ½Π°ΡΠ°ΠΌΠΈ Π΅Π³ΠΎ Π½Π°ΡΠ°Π»ΡΠ½ΠΎΠΉ ΠΈ ΠΊΠΎΠ½Π΅ΡΠ½ΠΎΠΉ ΡΠΎΡΠΊΠΈ
- Π΄Π»Ρ ΠΏΠ»ΠΎΡΠΊΠΈΡ Π·Π°Π΄Π°Ρ
- Π΄Π»Ρ ΠΏΡΠΎΡΡΡΠ°Π½ΡΡΠ²Π΅Π½Π½ΡΡ Π·Π°Π΄Π°Ρ
- Π΄Π»Ρ n -ΠΌΠ΅ΡΠ½ΠΎΠ³ΠΎ ΠΏΡΠΎΡΡΡΠ°Π½ΡΡΠ²Π°
- ΠΡΠΈΠΌΠ΅ΡΡ Π·Π°Π΄Π°Ρ
- ΠΏΠ»ΠΎΡΠΊΠ°Ρ Π·Π°Π΄Π°ΡΠ°
- ΠΏΡΠΎΡΡΡΠ°Π½ΡΡΠ²Π΅Π½Π½ΡΡ Π·Π°Π΄Π°ΡΠ°
- Π·Π°Π΄Π°ΡΠ° Π² n -ΠΌΠ΅ΡΠ½ΡΠΌ ΠΏΡΠΎΡΡΡΠ°Π½ΡΡΠ²Π΅
Π‘ΠΌΠΎΡΡΠΈΡΠ΅ ΡΠ°ΠΊΠΆΠ΅ ΠΎΠ½Π»Π°ΠΉΠ½ ΠΊΠ°Π»ΡΠΊΡΠ»ΡΡΠΎΡ Π΄Π»Ρ ΠΎΠΏΡΠ΅Π΄Π΅Π»Π΅Π½ΠΈΡ ΠΊΠΎΠΎΡΠ΄ΠΈΠ½Π°Ρ Π²Π΅ΠΊΡΠΎΡΠ° ΠΏΠΎ Π΄Π²ΡΠΌ ΡΠΎΡΠΊΠ°ΠΌ.
ΠΡΠ½ΠΎΠ²Π½ΠΎΠ΅ ΡΠΎΠΎΡΠ½ΠΎΡΠ΅Π½ΠΈΠ΅.Π§ΡΠΎΠ±Ρ Π½Π°ΠΉΡΠΈ ΠΊΠΎΠΎΡΠ΄ΠΈΠ½Π°ΡΡ Π²Π΅ΠΊΡΠΎΡΠ° AB, Π·Π½Π°Ρ ΠΊΠΎΠΎΡΠ΄ΠΈΠ½Π°ΡΡ Π΅Π³ΠΎ Π½Π°ΡΠ°Π»ΡΠ½ΠΎΠΉ ΡΠΎΡΠ΅ΠΊ Π ΠΈ ΠΊΠΎΠ½Π΅ΡΠ½ΠΎΠΉ ΡΠΎΡΠΊΠΈ Π, Π½Π΅ΠΎΠ±Ρ
ΠΎΠ΄ΠΈΠΌΠΎ ΠΈΠ· ΠΊΠΎΠΎΡΠ΄ΠΈΠ½Π°Ρ ΠΊΠΎΠ½Π΅ΡΠ½ΠΎΠΉ ΡΠΎΡΠΊΠΈ Π²ΡΡΠ΅ΡΡΡ ΡΠΎΠΎΡΠ²Π΅ΡΡΡΠ²ΡΡΡΠΈΠ΅ ΠΊΠΎΠΎΡΠ΄ΠΈΠ½Π°ΡΡ Π½Π°ΡΠ°Π»ΡΠ½ΠΎΠΉ ΡΠΎΡΠΊΠΈ.
Π€ΠΎΡΠΌΡΠ»Ρ ΠΎΠΏΡΠ΅Π΄Π΅Π»Π΅Π½ΠΈΡ ΠΊΠΎΠΎΡΠ΄ΠΈΠ½Π°Ρ Π²Π΅ΠΊΡΠΎΡΠ° Π·Π°Π΄Π°Π½Π½ΠΎΠ³ΠΎ ΠΊΠΎΠΎΡΠ΄ΠΈΠ½Π°ΡΠ°ΠΌΠΈ Π΅Π³ΠΎ Π½Π°ΡΠ°Π»ΡΠ½ΠΎΠΉ ΠΈ ΠΊΠΎΠ½Π΅ΡΠ½ΠΎΠΉ ΡΠΎΡΠΊΠΈ
Π€ΠΎΡΠΌΡΠ»Π° ΠΎΠΏΡΠ΅Π΄Π΅Π»Π΅Π½ΠΈΡ ΠΊΠΎΠΎΡΠ΄ΠΈΠ½Π°Ρ Π²Π΅ΠΊΡΠΎΡΠ° Π΄Π»Ρ ΠΏΠ»ΠΎΡΠΊΠΈΡ Π·Π°Π΄Π°Ρ
Π ΡΠ»ΡΡΠ°Π΅ ΠΏΠ»ΠΎΡΠΊΠΎΠΉ Π·Π°Π΄Π°ΡΠΈ Π²Π΅ΠΊΡΠΎΡ AB Π·Π°Π΄Π°Π½Π½ΡΠΉ ΠΊΠΎΠΎΡΠ΄ΠΈΠ½Π°ΡΠ°ΠΌΠΈ ΡΠΎΡΠ΅ΠΊ A(AxΒ ;Β Ay) ΠΈ B(BxΒ ;Β By) ΠΌΠΎΠΆΠ½ΠΎ Π½Π°ΠΉΡΠΈ Π²ΠΎΡΠΏΠΎΠ»ΡΠ·ΠΎΠ²Π°Π²ΡΠΈΡΡ ΡΠ»Π΅Π΄ΡΡΡΠ΅ΠΉ ΡΠΎΡΠΌΡΠ»ΠΎΠΉ
AB = {Bx — AxΒ ;Β By — Ay}
Π€ΠΎΡΠΌΡΠ»Π° ΠΎΠΏΡΠ΅Π΄Π΅Π»Π΅Π½ΠΈΡ ΠΊΠΎΠΎΡΠ΄ΠΈΠ½Π°Ρ Π²Π΅ΠΊΡΠΎΡΠ° Π΄Π»Ρ ΠΏΡΠΎΡΡΡΠ°Π½ΡΡΠ²Π΅Π½Π½ΡΡ Π·Π°Π΄Π°Ρ
Π ΡΠ»ΡΡΠ°Π΅ ΠΏΡΠΎΡΡΡΠ°Π½ΡΡΠ²Π΅Π½Π½ΠΎΠΉ Π·Π°Π΄Π°ΡΠΈ Π²Π΅ΠΊΡΠΎΡ AB Π·Π°Π΄Π°Π½Π½ΡΠΉ ΠΊΠΎΠΎΡΠ΄ΠΈΠ½Π°ΡΠ°ΠΌΠΈ ΡΠΎΡΠ΅ΠΊ A(AxΒ ;Β AyΒ ;Β Az) ΠΈ B(BxΒ ;Β ByΒ ;Β Bz) ΠΌΠΎΠΆΠ½ΠΎ Π½Π°ΠΉΡΠΈ Π²ΠΎΡΠΏΠΎΠ»ΡΠ·ΠΎΠ²Π°Π²ΡΠΈΡΡ ΡΠ»Π΅Π΄ΡΡΡΠ΅ΠΉ ΡΠΎΡΠΌΡΠ»ΠΎΠΉ
AB = {Bx — AxΒ ;Β By — AyΒ ;Β Bz — Az}
Π€ΠΎΡΠΌΡΠ»Π° ΠΎΠΏΡΠ΅Π΄Π΅Π»Π΅Π½ΠΈΡ ΠΊΠΎΠΎΡΠ΄ΠΈΠ½Π°Ρ Π²Π΅ΠΊΡΠΎΡΠ° Π΄Π»Ρ n -ΠΌΠ΅ΡΠ½ΠΎΠ³ΠΎ ΠΏΡΠΎΡΡΡΠ°Π½ΡΡΠ²Π°
Π ΡΠ»ΡΡΠ°Π΅ n-ΠΌΠ΅ΡΠ½ΠΎΠ³ΠΎ ΠΏΡΠΎΡΡΡΠ°Π½ΡΡΠ²Π° Π²Π΅ΠΊΡΠΎΡ AB Π·Π°Π΄Π°Π½Π½ΡΠΉ ΠΊΠΎΠΎΡΠ΄ΠΈΠ½Π°ΡΠ°ΠΌΠΈ ΡΠΎΡΠ΅ΠΊ A(A1Β ;Β A2Β ;Β .
..Β ;Β An) ΠΈ B(B1Β ;Β B2Β ;Β …Β ;Β Bn) ΠΌΠΎΠΆΠ½ΠΎ Π½Π°ΠΉΡΠΈ Π²ΠΎΡΠΏΠΎΠ»ΡΠ·ΠΎΠ²Π°Π²ΡΠΈΡΡ ΡΠ»Π΅Π΄ΡΡΡΠ΅ΠΉ ΡΠΎΡΠΌΡΠ»ΠΎΠΉ
AB = {B1 — A1Β ;Β B2 — A2Β ;Β …Β ;Β Bn — An}
ΠΡΠΈΠΌΠ΅ΡΡ Π·Π°Π΄Π°Ρ ΡΠ²ΡΠ·Π°Π½Π½ΡΡ Ρ ΠΎΠΏΡΠ΅Π΄Π΅Π»Π΅Π½ΠΈΠ΅ΠΌ ΠΊΠΎΠΎΡΠ΄ΠΈΠ½Π°Ρ Π²Π΅ΠΊΡΠΎΡΠ° ΠΏΠΎ Π΄Π²ΡΠΌ ΡΠΎΡΠΊΠ°ΠΌ
ΠΡΠΈΠΌΠ΅ΡΡ Π΄Π»Ρ ΠΏΠ»ΠΎΡΠΊΠΈΡ Π·Π°Π΄Π°Ρ
ΠΡΠΈΠΌΠ΅Ρ 1. ΠΠ°ΠΉΡΠΈ ΠΊΠΎΠΎΡΠ΄ΠΈΠ½Π°ΡΡ Π²Π΅ΠΊΡΠΎΡΠ° AB, Π΅ΡΠ»ΠΈ A(1; 4), B(3; 1).
Π Π΅ΡΠ΅Π½ΠΈΠ΅: AB = {3 — 1; 1 — 4} = {2; -3}.
ΠΡΠΈΠΌΠ΅Ρ 2. ΠΠ°ΠΉΡΠΈ ΠΊΠΎΠΎΡΠ΄ΠΈΠ½Π°ΡΡ ΡΠΎΡΠΊΠΈ B Π²Π΅ΠΊΡΠΎΡΠ° AB = {5; 1}, Π΅ΡΠ»ΠΈ ΠΊΠΎΠΎΡΠ΄ΠΈΠ½Π°ΡΡ ΡΠΎΡΠΊΠΈ A(3; -4).
Π Π΅ΡΠ΅Π½ΠΈΠ΅:
ABx = Bx — AxΒ Β Β =>Β Β Β Bx = ABx + AxΒ Β Β =>Β Β Β Bx = 5 + 3 = 8
ΠΡΠ²Π΅Ρ: B(8; -3).
ΠΡΠΈΠΌΠ΅Ρ 3. ΠΠ°ΠΉΡΠΈ ΠΊΠΎΠΎΡΠ΄ΠΈΠ½Π°ΡΡ ΡΠΎΡΠΊΠΈ A Π²Π΅ΠΊΡΠΎΡΠ° AB = {5; 1}, Π΅ΡΠ»ΠΈ ΠΊΠΎΠΎΡΠ΄ΠΈΠ½Π°ΡΡ ΡΠΎΡΠΊΠΈ B(3; -4).
Π Π΅ΡΠ΅Π½ΠΈΠ΅:
ABx = Bx — AxΒ Β Β =>Β Β Β Ax = Bx — ABxΒ Β Β =>Β Β Β Ax = 3 — 5 = -2
ABy = By — AyΒ Β Β =>Β Β Β Ay = By — AByΒ Β Β =>Β Β Β Ay = -4 — 1 = -5
ΠΡΠ²Π΅Ρ: A(-2; -5).
ΠΡΠΈΠΌΠ΅ΡΡ Π΄Π»Ρ ΠΏΡΠΎΡΡΡΠ°Π½ΡΡΠ²Π΅Π½Π½ΡΡ Π·Π°Π΄Π°Ρ
ΠΡΠΈΠΌΠ΅Ρ 4. ΠΠ°ΠΉΡΠΈ ΠΊΠΎΠΎΡΠ΄ΠΈΠ½Π°ΡΡ Π²Π΅ΠΊΡΠΎΡΠ° AB, Π΅ΡΠ»ΠΈ A(1; 4; 5), B(3; 1; 1).
Π Π΅ΡΠ΅Π½ΠΈΠ΅: AB = {3 — 1; 1 — 4; 1 — 5} = {2; -3; -4}.
Π Π΅ΡΠ΅Π½ΠΈΠ΅:
ABx = Bx — AxΒ Β Β =>Β Β Β Bx = ABx + AxΒ Β Β =>Β Β Β Bx = 5 + 3 = 8
ABy = By — AyΒ Β Β =>Β Β Β By = ABy + AyΒ Β Β =>Β Β Β By = 1 + (-4) = -3
ABz = Bz — AzΒ Β Β =>Β Β Β Bz = ABz + AzΒ Β Β =>Β Β Β Bz = 2 + 3 = 5
ΠΡΠ²Π΅Ρ: B(8; -3; 5).
ΠΡΠΈΠΌΠ΅Ρ 6. ΠΠ°ΠΉΡΠΈ ΠΊΠΎΠΎΡΠ΄ΠΈΠ½Π°ΡΡ ΡΠΎΡΠΊΠΈ A Π²Π΅ΠΊΡΠΎΡΠ° AB = {5; 1; 4}, Π΅ΡΠ»ΠΈ ΠΊΠΎΠΎΡΠ΄ΠΈΠ½Π°ΡΡ ΡΠΎΡΠΊΠΈ B(3; -4; 1).
Π Π΅ΡΠ΅Π½ΠΈΠ΅:
ABx = B x — AxΒ Β Β =>Β Β Β Ax = Bx — ABxΒ Β Β =>Β Β Β Ax = 3 — 5 = -2
ABy = By — AyΒ Β Β =>Β Β Β Ay = By — AByΒ Β Β =>Β Β Β Ay = -4 — 1 = -5
ABz = Bz — AzΒ Β Β =>Β Β Β Az = Bz — ABzΒ Β Β =>Β Β Β Az = 1 — 4 = -3
ΠΡΠ²Π΅Ρ: A(-2; -5; -3).
ΠΡΠΈΠΌΠ΅ΡΡ Π΄Π»Ρ n -ΠΌΠ΅ΡΠ½ΠΎΠ³ΠΎ ΠΏΡΠΎΡΡΡΠ°Π½ΡΡΠ²Π°
ΠΡΠΈΠΌΠ΅Ρ 7. ΠΠ°ΠΉΡΠΈ ΠΊΠΎΠΎΡΠ΄ΠΈΠ½Π°ΡΡ Π²Π΅ΠΊΡΠΎΡΠ° AB, Π΅ΡΠ»ΠΈ A(1; 4; 5; 5; -3), B(3; 0; 1; -2; 5).
Π Π΅ΡΠ΅Π½ΠΈΠ΅: AB = {3 — 1; 0 — 4; 1 — 5; -2 — 5; 5 — (-3)} = {2; -4; -4; -7; 8}.
ΠΡΠΈΠΌΠ΅Ρ 8. ΠΠ°ΠΉΡΠΈ ΠΊΠΎΠΎΡΠ΄ΠΈΠ½Π°ΡΡ ΡΠΎΡΠΊΠΈ B Π²Π΅ΠΊΡΠΎΡΠ° AB = {5; 1; 2; 1}, Π΅ΡΠ»ΠΈ ΠΊΠΎΠΎΡΠ΄ΠΈΠ½Π°ΡΡ ΡΠΎΡΠΊΠΈ A(3; -4; 3; 2).
Π Π΅ΡΠ΅Π½ΠΈΠ΅:
AB1 = B
AB2 = B2 — A2Β Β Β =>Β Β Β B2 = AB2 + A2Β Β Β =>Β Β Β B2 = 1 + (-4) = -3
AB3 = B3 — A3Β Β Β =>Β Β Β B3 = AB3 + A3Β Β Β =>Β Β Β B3 = 2 + 3 = 5
AB4 = B4 — A4Β Β Β =>Β Β Β B4 = AB4 + A4Β Β Β =>Β Β Β B4 = 1 + 2 = 3
ΠΡΠ²Π΅Ρ: B(8; -3; 5; 3).
ΠΡΠΈΠΌΠ΅Ρ 9. ΠΠ°ΠΉΡΠΈ ΠΊΠΎΠΎΡΠ΄ΠΈΠ½Π°ΡΡ ΡΠΎΡΠΊΠΈ A Π²Π΅ΠΊΡΠΎΡΠ° AB = {5; 1; 4; 5}, Π΅ΡΠ»ΠΈ ΠΊΠΎΠΎΡΠ΄ΠΈΠ½Π°ΡΡ ΡΠΎΡΠΊΠΈ B(3; -4; 1; 8).
Π Π΅ΡΠ΅Π½ΠΈΠ΅:
AB1 = B1 — A1Β Β Β =>Β Β Β A1 = B1 — AB1Β Β Β =>Β Β Β A1 = 3 — 5 = -2
AB2 = B2
AB3 = B3 — A3Β Β Β =>Β Β Β A3 = B3 — AB3Β Β Β =>Β Β Β A3 = 1 — 4 = -3
AB4 = B4 — A4Β Β Β =>Β Β Β A4 = B4 — AB4Β Β Β =>Β Β Β A4 = 8 — 5 = 3
ΠΡΠ²Π΅Ρ: A(-2; -5; -3; 3).
ΠΠ΅ΠΊΡΠΎΡΠ° ΠΠ΅ΠΊΡΠΎΡ: ΠΎΠΏΡΠ΅Π΄Π΅Π»Π΅Π½ΠΈΠ΅ ΠΈ ΠΎΡΠ½ΠΎΠ²Π½ΡΠ΅ ΠΏΠΎΠ½ΡΡΠΈΡ ΠΠΏΡΠ΅Π΄Π΅Π»Π΅Π½ΠΈΠ΅ ΠΊΠΎΠΎΡΠ΄ΠΈΠ½Π°Ρ Π²Π΅ΠΊΡΠΎΡΠ° Π·Π°Π΄Π°Π½Π½ΠΎΠ³ΠΎ ΠΊΠΎΠΎΡΠ΄ΠΈΠ½Π°ΡΠ°ΠΌΠΈ Π΅Π³ΠΎ Π½Π°ΡΠ°Π»ΡΠ½ΠΎΠΉ ΠΈ ΠΊΠΎΠ½Π΅ΡΠ½ΠΎΠΉ ΡΠΎΡΠΊΠΈ ΠΠΎΠ΄ΡΠ»Ρ Π²Π΅ΠΊΡΠΎΡΠ°. ΠΠ»ΠΈΠ½Π° Π²Π΅ΠΊΡΠΎΡΠ° ΠΠ°ΠΏΡΠ°Π²Π»ΡΡΡΠΈΠ΅ ΠΊΠΎΡΠΈΠ½ΡΡΡ Π²Π΅ΠΊΡΠΎΡΠ° Π Π°Π²Π΅Π½ΡΡΠ²ΠΎ Π²Π΅ΠΊΡΠΎΡΠΎΠ² ΠΡΡΠΎΠ³ΠΎΠ½Π°Π»ΡΠ½ΠΎΡΡΡ Π²Π΅ΠΊΡΠΎΡΠΎΠ² ΠΠΎΠ»Π»ΠΈΠ½Π΅Π°ΡΠ½ΠΎΡΡΡ Π²Π΅ΠΊΡΠΎΡΠΎΠ² ΠΠΎΠΌΠΏΠ»Π°Π½Π°ΡΠ½ΠΎΡΡΡ Π²Π΅ΠΊΡΠΎΡΠΎΠ² Π£Π³ΠΎΠ» ΠΌΠ΅ΠΆΠ΄Ρ Π²Π΅ΠΊΡΠΎΡΠ°ΠΌΠΈ ΠΡΠΎΠ΅ΠΊΡΠΈΡ Π²Π΅ΠΊΡΠΎΡΠ° Π‘Π»ΠΎΠΆΠ΅Π½ΠΈΠ΅ ΠΈ Π²ΡΡΠΈΡΠ°Π½ΠΈΠ΅ Π²Π΅ΠΊΡΠΎΡΠΎΠ² Π£ΠΌΠ½ΠΎΠΆΠ΅Π½ΠΈΠ΅ Π²Π΅ΠΊΡΠΎΡΠ° Π½Π° ΡΠΈΡΠ»ΠΎ Π‘ΠΊΠ°Π»ΡΡΠ½ΠΎΠ΅ ΠΏΡΠΎΠΈΠ·Π²Π΅Π΄Π΅Π½ΠΈΠ΅ Π²Π΅ΠΊΡΠΎΡΠΎΠ² ΠΠ΅ΠΊΡΠΎΡΠ½ΠΎΠ΅ ΠΏΡΠΎΠΈΠ·Π²Π΅Π΄Π΅Π½ΠΈΠ΅ Π²Π΅ΠΊΡΠΎΡΠΎΠ² Π‘ΠΌΠ΅ΡΠ°Π½Π½ΠΎΠ΅ ΠΏΡΠΎΠΈΠ·Π²Π΅Π΄Π΅Π½ΠΈΠ΅ Π²Π΅ΠΊΡΠΎΡΠΎΠ² ΠΠΈΠ½Π΅ΠΉΠ½ΠΎ Π·Π°Π²ΠΈΡΠΈΠΌΡΠ΅ ΠΈ Π»ΠΈΠ½Π΅ΠΉΠ½ΠΎ Π½Π΅Π·Π°Π²ΠΈΡΠΈΠΌΡΠ΅ Π²Π΅ΠΊΡΠΎΡΠ° Π Π°Π·Π»ΠΎΠΆΠ΅Π½ΠΈΠ΅ Π²Π΅ΠΊΡΠΎΡΠ° ΠΏΠΎ Π±Π°Π·ΠΈΡΡ
ΠΠ½Π»Π°ΠΉΠ½ ΠΊΠ°Π»ΡΠΊΡΠ»ΡΡΠΎΡΡ Ρ Π²Π΅ΠΊΡΠΎΡΠ°ΠΌΠΈ
ΠΠ½Π»Π°ΠΉΠ½ ΡΠΏΡΠ°ΠΆΠ½Π΅Π½ΠΈΡ Ρ Π²Π΅ΠΊΡΠΎΡΠ°ΠΌΠΈ Π½Π° ΠΏΠ»ΠΎΡΠΊΠΎΡΡΠΈ
ΠΠ½Π»Π°ΠΉΠ½ ΡΠΏΡΠ°ΠΆΠ½Π΅Π½ΠΈΡ Ρ Π²Π΅ΠΊΡΠΎΡΠ°ΠΌΠΈ Π² ΠΏΡΠΎΡΡΡΠ°Π½ΡΡΠ²Π΅
ΠΠ°ΠΊ Π½Π°ΠΉΡΠΈ ΠΊΠΎΠΎΡΠ΄ΠΈΠ½Π°ΡΡ Π²Π΅ΠΊΡΠΎΡΠ°
ΠΡΠ΅Π΄Π²Π°ΡΠΈΡΠ΅Π»ΡΠ½ΡΠ΅ ΡΠ²Π΅Π΄Π΅Π½ΠΈΡ
ΠΠ΄Π΅ΡΡ ΠΌΡ ΠΎΠ³ΡΠ°Π½ΠΈΡΠΈΠΌΡΡ Π΄Π²ΡΠΌΠ΅ΡΠ½ΡΠΌ ΡΠ»ΡΡΠ°Π΅ΠΌ. ΠΠ²Π΅Π΄Π΅Π½ΠΈΠ΅ ΠΏΠΎΠ½ΡΡΠΈΡ Π΄Π»Ρ ΡΡΠ΅Ρ
ΠΌΠ΅ΡΠ½ΠΎΠ³ΠΎ ΡΠ»ΡΡΠ°Ρ ΠΏΡΠΎΠ²ΠΎΠ΄ΠΈΡΡΡ Π°Π½Π°Π»ΠΎΠ³ΠΈΡΠ½ΠΎ.
ΠΠ»Ρ ΡΠΎΠ³ΠΎ, ΡΡΠΎΠ±Ρ Π²Π²Π΅ΡΡΠΈ ΠΏΠΎΠ½ΡΡΠΈΠ΅ ΠΊΠΎΠΎΡΠ΄ΠΈΠ½Π°Ρ Π²Π΅ΠΊΡΠΎΡΠ° ΡΠ½Π°ΡΠ°Π»Π° Π²Π²Π΅Π΄Π΅ΠΌ ΠΈ Π΄ΠΎΠΊΠ°ΠΆΠ΅ΠΌ ΡΠ»Π΅Π΄ΡΡΡΠΈΠ΅ Π»Π΅ΠΌΠΌΡ ΠΈ ΡΠ΅ΠΎΡΠ΅ΠΌΡ.
ΠΠ΅ΠΌΠΌΠ° 1: ΠΡΠ»ΠΈ Π²Π΅ΠΊΡΠΎΡΡ $\overrightarrow{a}$ ΠΈ $\overrightarrow{b}$ ΠΊΠΎΠ»Π»ΠΈΠ½Π΅Π°ΡΠ½Ρ, ΠΈ Π²Π΅ΠΊΡΠΎΡ $\overrightarrow{a}$ Π½Π΅ ΡΠ²Π»ΡΠ΅ΡΡΡ Π½ΡΠ»Π΅Π²ΡΠΌ, ΡΠΎ ΡΡΡΠ΅ΡΡΠ²ΡΠ΅Ρ Π΄Π΅ΠΉΡΡΠ²ΠΈΡΠ΅Π»ΡΠ½ΠΎΠ΅ ΡΠΈΡΠ»ΠΎ $k$, ΡΠ°ΠΊΠΎΠ΅ ΡΡΠΎ Π²ΡΠΏΠΎΠ»Π½ΡΠ΅ΡΡΡ ΡΠ°Π²Π΅Π½ΡΡΠ²ΠΎ$\overrightarrow{b}=k\overrightarrow{a}$
ΠΠΎΠΊΠ°Π·Π°ΡΠ΅Π»ΡΡΡΠ²ΠΎ.
ΠΠΎΠ·ΠΌΠΎΠΆΠ½Ρ Π΄Π²Π° ΡΠ»ΡΡΠ°Ρ:
$\overrightarrow{a}\uparrow \uparrow \overrightarrow{b}$
ΠΠ±ΠΎΠ·Π½Π°ΡΠΈΠΌ ΡΠΈΡΠ»ΠΎ $k$ ΡΠ»Π΅Π΄ΡΡΡΠΈΠΌ ΠΎΠ±ΡΠ°Π·ΠΎΠΌ: $k=\frac{|\overrightarrow{b}|}{|\overrightarrow{a}|}$. Π’Π°ΠΊ ΠΊΠ°ΠΊ Π²Π΅ΠΊΡΠΎΡΡ $\overrightarrow{a}$ ΠΈ $\overrightarrow{b}$ ΡΠΎΠ½Π°ΠΏΡΠ°Π²Π»Π΅Π½Ρ, Π° $k\ge 0$, ΡΠΎ Π²Π΅ΠΊΡΠΎΡΡ $k\overrightarrow{a}$ ΠΈ $\overrightarrow{b}$ ΡΠΎΠ½Π°ΠΏΡΠ°Π²Π»Π΅Π½Ρ. ΠΠ°Π»Π΅Π΅, ΠΈΠΌΠ΅Π΅ΠΌ, ΡΡΠΎ
\[\left|k\overrightarrow{a}\right|=\left|k\right|\left|\overrightarrow{a}\right|=\frac{|\overrightarrow{b}|}{|\overrightarrow{a}|}\left|\overrightarrow{a}\right|=|\overrightarrow{b}|\]ΠΠ· ΡΡΠΎΠ³ΠΎ Π²ΡΠ΅Π³ΠΎ ΡΠ»Π΅Π΄ΡΠ΅Ρ, ΡΡΠΎ $\overrightarrow{b}=k\overrightarrow{a}$.

$\overrightarrow{a}\uparrow \downarrow \overrightarrow{b}$
ΠΠ±ΠΎΠ·Π½Π°ΡΠΈΠΌ ΡΠΈΡΠ»ΠΎ $k$ ΡΠ»Π΅Π΄ΡΡΡΠΈΠΌ ΠΎΠ±ΡΠ°Π·ΠΎΠΌ: $k=-\frac{|\overrightarrow{b}|}{|\overrightarrow{a}|}$. Π’Π°ΠΊ ΠΊΠ°ΠΊ Π²Π΅ΠΊΡΠΎΡΡ $\overrightarrow{a}$ ΠΈ $\overrightarrow{b}$ ΠΏΡΠΎΡΠΈΠ²ΠΎΠΏΠΎΠ»ΠΎΠΆΠ½ΠΎ Π½Π°ΠΏΡΠ°Π²Π»Π΅Π½Π½ΡΠ΅, Π° $k \[\left|k\overrightarrow{a}\right|=\left|k\right|\left|\overrightarrow{a}\right|=\frac{|\overrightarrow{b}|}{|\overrightarrow{a}|}\left|\overrightarrow{a}\right|=|\overrightarrow{b}|\]
ΠΠ· ΡΡΠΎΠ³ΠΎ Π²ΡΠ΅Π³ΠΎ ΡΠ»Π΅Π΄ΡΠ΅Ρ, ΡΡΠΎ $\overrightarrow{b}=k\overrightarrow{a}$.
ΠΠ΅ΠΌΠΌΠ° Π΄ΠΎΠΊΠ°Π·Π°Π½Π°.
Π’Π΅ΠΎΡΠ΅ΠΌΠ° 1
ΠΡΠ±ΠΎΠΉ Π²Π΅ΠΊΡΠΎΡ ΠΌΠΎΠΆΠ½ΠΎ ΡΠ°Π·Π»ΠΎΠΆΠΈΡΡ ΠΏΠΎ Π΄Π²ΡΠΌ Π½Π΅ΠΊΠΎΠ»Π»ΠΈΠ½Π΅Π°ΡΠ½ΡΠΌ Π²Π΅ΠΊΡΠΎΡΠ°ΠΌ, ΠΏΡΠΈΡΠ΅ΠΌ ΠΊΠΎΡΡΡΠΈΡΠΈΠ΅Π½ΡΡ ΡΠ°Π·Π»ΠΎΠΆΠ΅Π½ΠΈΡ ΠΎΠΏΡΠ΅Π΄Π΅Π»ΡΡΡΡΡ Π΅Π΄ΠΈΠ½ΡΡΠ²Π΅Π½Π½ΡΠΌ ΠΎΠ±ΡΠ°Π·ΠΎΠΌ:
\[\overrightarrow{c}=m\overrightarrow{a}+n\overrightarrow{b}\]
ΠΠΎΠΊΠ°Π·Π°ΡΠ΅Π»ΡΡΡΠ²ΠΎ.
Π‘ΡΡΠ΅ΡΡΠ²ΠΎΠ²Π°Π½ΠΈΠ΅: ΠΠΎΠΊΠ°ΠΆΠ΅ΠΌ, ΡΡΠΎ ΡΠ°ΠΊΠΎΠ΅ ΡΠ°Π·Π»ΠΎΠΆΠ΅Π½ΠΈΠ΅ ΠΈΠΌΠ΅Π΅Ρ ΠΌΠ΅ΡΡΠΎ. ΠΠ΄Π΅ΡΡ Π²ΠΎΠ·ΠΌΠΎΠΆΠ½Ρ Π΄Π²Π° ΡΠ»ΡΡΠ°Ρ:
ΠΠ΅ΠΊΡΠΎΡ $\overrightarrow{c}$ ΠΊΠΎΠ»Π»ΠΈΠ½Π΅Π°ΡΠ΅Π½ (ΠΊ ΠΏΡΠΈΠΌΠ΅ΡΡ) Π²Π΅ΠΊΡΠΎΡΡ $\overrightarrow{b}$.

ΠΠΎ Π»Π΅ΠΌΠΌΠ΅ 1, Π±ΡΠ΄Π΅ΠΌ ΠΈΠΌΠ΅ΡΡ
\[\overrightarrow{c}=n\overrightarrow{b}\]ΠΠ½Π°ΡΠΈΡ, Π΅ΡΠ»ΠΈ ΡΠΈΡΠ»ΠΎ $m=0$, ΡΠΎ ΠΏΠΎΠ»ΡΡΠΈΠΌ
\[\overrightarrow{c}=m\overrightarrow{a}+n\overrightarrow{b}\]ΠΠ΅ΠΊΡΠΎΡ $\overrightarrow{c}$ Π½Π΅ ΠΊΠΎΠ»Π»ΠΈΠ½Π΅Π°ΡΠ΅Π½ Π²Π΅ΠΊΡΠΎΡΠ°ΠΌ $\overrightarrow{a}$ ΠΈ $\overrightarrow{b}$.
ΠΠΎΠ·ΡΠΌΠ΅ΠΌ ΠΏΡΠΎΠΈΠ·Π²ΠΎΠ»ΡΠ½ΡΡ ΡΠΎΡΠΊΡ $O$ ΠΈ ΠΎΡΠ»ΠΎΠΆΠΈΠΌ ΠΎΡ Π½Π΅Π΅ Π²Π΅ΠΊΡΠΎΡΡ $\overrightarrow{OB}=\overrightarrow{b},\ \overrightarrow{OA}=\overrightarrow{a}$ ΠΈ $\overrightarrow{OC}=\overrightarrow{c}$. ΠΡΡΡΡ ΠΡΠΎΠ²Π΅Π΄Π΅ΠΌ ΠΏΡΡΠΌΡΡ $CD||OB$ (ΡΠΈΡ. 1)
Π ΠΈΡΡΠ½ΠΎΠΊ 1. ΠΠ»Π»ΡΡΡΡΠ°ΡΠΈΡ ΡΠ΅ΠΎΡΠ΅ΠΌΡ 1
ΠΠΎ ΠΏΡΠ°Π²ΠΈΠ»Ρ ΡΡΠ΅ΡΠ³ΠΎΠ»ΡΠ½ΠΈΠΊΠ° Π΄Π»Ρ ΡΠ»ΠΎΠΆΠ΅Π½ΠΈΡ Π²Π΅ΠΊΡΠΎΡΠΎΠ², ΠΏΠΎΠ»ΡΡΠΈΠΌ
\[\overrightarrow{c}=\overrightarrow{OD}+\overrightarrow{DC}\]ΠΠΎ ΠΏΠΎΡΡΡΠΎΠ΅Π½ΠΈΡ, ΠΏΠΎΠ»ΡΡΠ°Π΅ΠΌ ΡΡΠΎ Π²Π΅ΠΊΡΠΎΡΡ $\overrightarrow{OD}||\overrightarrow{a}$ ΠΈ $\overrightarrow{DC}||\overrightarrow{b}$, ΡΠ»Π΅Π΄ΠΎΠ²Π°ΡΠ΅Π»ΡΠ½ΠΎ, ΠΏΠΎ Π»Π΅ΠΌΠΌΠ΅ 1, ΠΈΠΌΠ΅Π΅ΠΌ
\[\overrightarrow{OD}=m\overrightarrow{a},\ \overrightarrow{DC}=n\overrightarrow{b}\]ΠΠ½Π°ΡΠΈΡ
\[\overrightarrow{c}=m\overrightarrow{a}+n\overrightarrow{b}\]
ΠΠ΄ΠΈΠ½ΡΡΠ²Π΅Π½Π½ΠΎΡΡΡ: ΠΡΠ΅Π΄ΠΏΠΎΠ»ΠΎΠΆΠΈΠΌ ΠΏΡΠΎΡΠΈΠ²Π½ΠΎΠ΅, ΡΡΠΎ ΠΏΠΎΠΌΠΈΠΌΠΎ ΡΠ°Π·Π»ΠΎΠΆΠ΅Π½ΠΈΡ$\overrightarrow{c}=m\overrightarrow{a}+n\overrightarrow{b}$ ΡΡΡΠ΅ΡΡΠ²ΡΠ΅Ρ ΡΠ°Π·Π»ΠΎΠΆΠ΅Π½ΠΈΠ΅ $\overrightarrow{c}=m’\overrightarrow{a}+n’\overrightarrow{b}$.
ΠΡΡΡΠ΅ΠΌ ΡΡΠΈ Π΄Π²Π° ΡΠ°Π²Π΅Π½ΡΡΠ²Π° ΠΈΠ· Π΄ΡΡΠ³ Π΄ΡΡΠ³Π°:
ΠΠΎΠ»ΡΡΠ°Π΅ΠΌ ΡΠΈΡΡΠ΅ΠΌΡ:
Π ΠΈΡΡΠ½ΠΎΠΊ 2.
Π‘Π»Π΅Π΄ΠΎΠ²Π°ΡΠ΅Π»ΡΠ½ΠΎ, ΡΠ°Π·Π»ΠΎΠΆΠ΅Π½ΠΈΠ΅ Π΅Π΄ΠΈΠ½ΡΡΠ²Π΅Π½Π½ΠΎ.
Π’Π΅ΠΎΡΠ΅ΠΌΠ° Π΄ΠΎΠΊΠ°Π·Π°Π½Π°.
ΠΠΎΠΎΡΠ΄ΠΈΠ½Π°ΡΡ Π²Π΅ΠΊΡΠΎΡΠ°
Π Π°ΡΡΠΌΠΎΡΡΠΈΠΌ Π΄Π°Π»Π΅Π΅ ΡΠΈΡΡΠ΅ΠΌΡ ΠΊΠΎΠΎΡΠ΄ΠΈΠ½Π°Ρ. ΠΡ Π½Π°ΡΠ°Π»Π° ΠΊΠΎΠΎΡΠ΄ΠΈΠ½Π°Ρ $O$ Π² Π½Π°ΠΏΡΠ°Π²Π»Π΅Π½ΠΈΠΈ ΠΎΡΠΈ $Ox$ ΠΎΡΠ»ΠΎΠΆΠΈΠΌ Π²Π΅ΠΊΡΠΎΡ $\overrightarrow{i}$, Π° Π² Π½Π°ΠΏΡΠ°Π²Π»Π΅Π½ΠΈΠΈ ΠΎΡΠΈ $Oy$ ΠΎΡΠ»ΠΎΠΆΠΈΠΌ Π²Π΅ΠΊΡΠΎΡ $\overrightarrow{j}$, Π΄Π»ΠΈΠ½Ρ ΠΊΠΎΡΠΎΡΡΡ ΡΠ°Π²Π½Ρ Π΅Π΄ΠΈΠ½ΠΈΡΠ΅.
ΠΠΏΡΠ΅Π΄Π΅Π»Π΅Π½ΠΈΠ΅ 1
ΠΠ΅ΠΊΡΠΎΡΡ $\overrightarrow{i}$ ΠΈ $\overrightarrow{j}$ Π½Π°Π·ΡΠ²Π°ΡΡΡΡ ΠΊΠΎΠΎΡΠ΄ΠΈΠ½Π°ΡΠ½ΡΠΌΠΈ Π²Π΅ΠΊΡΠΎΡΠ°ΠΌΠΈ.
Π’Π°ΠΊ ΠΊΠ°ΠΊ Π²Π΅ΠΊΡΠΎΡΡ $\overrightarrow{i}$ ΠΈ $\overrightarrow{j}$ Π½Π΅ ΠΊΠΎΠ»Π»ΠΈΠ½Π΅Π°ΡΠ½Ρ ΡΠΎ, ΠΏΠΎ ΡΠ΅ΠΎΡΠ΅ΠΌΠ΅ 1, Π»ΡΠ±ΠΎΠΉ Π²Π΅ΠΊΡΠΎΡ ΠΌΠΎΠΆΠ½ΠΎ ΡΠ°Π·Π»ΠΎΠΆΠΈΡΡ Π² Π²ΠΈΠ΄Π΅ $\overrightarrow{c}=m\overrightarrow{i}+n\overrightarrow{j}$.
ΠΠΏΡΠ΅Π΄Π΅Π»Π΅Π½ΠΈΠ΅ 2
ΠΠΎΡΡΡΠΈΡΠΈΠ΅Π½ΡΡ ΡΠ°Π·Π»ΠΎΠΆΠ΅Π½ΠΈΡ Π²Π΅ΠΊΡΠΎΡΠ° $\overrightarrow{c}=m\overrightarrow{i}+n\overrightarrow{j}$ Π½Π°Π·ΡΠ²Π°ΡΡΡΡ ΠΊΠΎΠΎΡΠ΄ΠΈΠ½Π°ΡΠ°ΠΌΠΈ Π΄Π°Π½Π½ΠΎΠ³ΠΎ Π²Π΅ΠΊΡΠΎΡΠ° Π² Π΄Π°Π½Π½ΠΎΠΉ ΡΠΈΡΡΠ΅ΠΌΠ΅ ΠΊΠΎΠΎΡΠ΄ΠΈΠ½Π°Ρ, ΡΠΎ Π΅ΡΡΡ
\[\overrightarrow{c}=\{m,\ n\}\]
ΠΠΈΠ½Π΅ΠΉΠ½ΡΠ΅ ΠΎΠΏΠ΅ΡΠ°ΡΠΈΠΈ Π½Π°Π΄ Π²Π΅ΠΊΡΠΎΡΠ°ΠΌΠΈ
Π’Π΅ΠΎΡΠ΅ΠΌΠ° 2
Π’Π΅ΠΎΡΠ΅ΠΌΠ° ΠΎ ΡΡΠΌΠΌΠ΅ Π²Π΅ΠΊΡΠΎΡΠΎΠ²: ΠΠΎΠΎΡΠ΄ΠΈΠ½Π°ΡΡ ΡΡΠΌΠΌΡ Π²Π΅ΠΊΡΠΎΡΠΎΠ² ΡΠ°Π²Π½Ρ ΡΡΠΌΠΌΠ΅ ΡΠΎΠΎΡΠ²Π΅ΡΡΡΠ²ΡΡΡΠΈΡ
ΠΊΠΎΠΎΡΠ΄ΠΈΠ½Π°Ρ ΡΡΠΈΡ
Π²Π΅ΠΊΡΠΎΡΠΎΠ².
ΠΠΎΠΊΠ°Π·Π°ΡΠ΅Π»ΡΡΡΠ²ΠΎ.
ΠΠΎΠΊΠ°ΠΆΠ΅ΠΌ ΡΠ΅ΠΎΡΠ΅ΠΌΡ Π΄Π»Ρ Π΄Π²ΡΡ Π²Π΅ΠΊΡΠΎΡΠΎΠ². Π’Π΅ΠΎΡΠ΅ΠΌΠ° Π΄Π»Ρ Π±ΠΎΠ»ΡΡΠ΅Π³ΠΎ ΠΊΠΎΠ»ΠΈΡΠ΅ΡΡΠ²Π° Π²Π΅ΠΊΡΠΎΡΠΎΠ² Π΄ΠΎΠΊΠ°Π·ΡΠ²Π°Π΅ΡΡΡ Π°Π½Π°Π»ΠΎΠ³ΠΈΡΠ½ΠΎ. ΠΡΡΡΡ $\overrightarrow{a}=\left\{x_1,\ y_1\right\}$, $\overrightarrow{b}=\{x_2,\ y_2\}$, ΡΠΎΠ³Π΄Π°
Π‘Π»Π΅Π΄ΠΎΠ²Π°ΡΠ΅Π»ΡΠ½ΠΎ
Π’Π΅ΠΎΡΠ΅ΠΌΠ° Π΄ΠΎΠΊΠ°Π·Π°Π½Π°.
Π’Π΅ΠΎΡΠ΅ΠΌΠ° 3
Π’Π΅ΠΎΡΠ΅ΠΌΠ° ΠΎ ΡΠ°Π·Π½ΠΎΡΡΠΈ Π²Π΅ΠΊΡΠΎΡΠΎΠ²: ΠΠΎΠΎΡΠ΄ΠΈΠ½Π°ΡΡ ΡΠ°Π·Π½ΠΎΡΡΠΈ Π²Π΅ΠΊΡΠΎΡΠΎΠ² ΡΠ°Π²Π½Ρ ΡΠ°Π·Π½ΠΎΡΡΠΈ ΡΠΎΠΎΡΠ²Π΅ΡΡΡΠ²ΡΡΡΠΈΡ ΠΊΠΎΠΎΡΠ΄ΠΈΠ½Π°Ρ ΡΡΠΈΡ Π²Π΅ΠΊΡΠΎΡΠΎΠ².
ΠΠΎΠΊΠ°Π·Π°ΡΠ΅Π»ΡΡΡΠ²ΠΎ.
ΠΠΎΠΊΠ°ΠΆΠ΅ΠΌ ΡΠ΅ΠΎΡΠ΅ΠΌΡ Π΄Π»Ρ Π΄Π²ΡΡ Π²Π΅ΠΊΡΠΎΡΠΎΠ². Π’Π΅ΠΎΡΠ΅ΠΌΠ° Π΄Π»Ρ Π±ΠΎΠ»ΡΡΠ΅Π³ΠΎ ΠΊΠΎΠ»ΠΈΡΠ΅ΡΡΠ²Π° Π²Π΅ΠΊΡΠΎΡΠΎΠ² Π΄ΠΎΠΊΠ°Π·ΡΠ²Π°Π΅ΡΡΡ Π°Π½Π°Π»ΠΎΠ³ΠΈΡΠ½ΠΎ. ΠΡΡΡΡ $\overrightarrow{a}=\left\{x_1,\ y_1\right\}$, $\overrightarrow{b}=\{x_2,\ y_2\}$, ΡΠΎΠ³Π΄Π°
Π‘Π»Π΅Π΄ΠΎΠ²Π°ΡΠ΅Π»ΡΠ½ΠΎ
Π’Π΅ΠΎΡΠ΅ΠΌΠ° Π΄ΠΎΠΊΠ°Π·Π°Π½Π°.
Π’Π΅ΠΎΡΠ΅ΠΌΠ° 4
Π’Π΅ΠΎΡΠ΅ΠΌΠ° ΠΎ ΠΏΡΠΎΠΈΠ·Π²Π΅Π΄Π΅Π½ΠΈΠΈ Π²Π΅ΠΊΡΠΎΡΠ° Π½Π° ΡΠΈΡΠ»ΠΎ: ΠΠΎΠΎΡΠ΄ΠΈΠ½Π°ΡΡ ΠΏΡΠΎΠΈΠ·Π²Π΅Π΄Π΅Π½ΠΈΡ Π²Π΅ΠΊΡΠΎΡΠ° Π½Π° ΡΠΈΡΠ»ΠΎ ΡΠ°Π²Π½Ρ ΠΏΡΠΎΠΈΠ·Π²Π΅Π΄Π΅Π½ΠΈΡ ΡΠΎΠΎΡΠ²Π΅ΡΡΡΠ²ΡΡΡΠΈΡ ΠΊΠΎΠΎΡΠ΄ΠΈΠ½Π°Ρ ΡΡΠΎ ΡΠΈΡΠ»ΠΎ.
ΠΠΎΠΊΠ°Π·Π°ΡΠ΅Π»ΡΡΡΠ²ΠΎ.
ΠΡΡΡΡ $\overrightarrow{a}=\left\{x,\ y\right\}$, ΡΠΎΠ³Π΄Π° $\overrightarrow{a}=x\overrightarrow{i}+\ y\overrightarrow{j}.
$
Π‘Π»Π΅Π΄ΠΎΠ²Π°ΡΠ΅Π»ΡΠ½ΠΎ
Π’Π΅ΠΎΡΠ΅ΠΌΠ° Π΄ΠΎΠΊΠ°Π·Π°Π½Π°.
ΠΡΠΈΠΌΠ΅Ρ Π·Π°Π΄Π°ΡΠΈ Π½Π° Π½Π°Ρ ΠΎΠΆΠ΄Π΅Π½ΠΈΠ΅ ΠΊΠΎΠΎΡΠ΄ΠΈΠ½Π°Ρ Π²Π΅ΠΊΡΠΎΡΠ°
ΠΡΠΈΠΌΠ΅Ρ 1
ΠΡΡΡΡ $\overrightarrow{a}=\left\{3,\ 4\right\}$, $\overrightarrow{b}=\{2,\ -1\}$. ΠΠ°ΠΉΡΠΈ $\overrightarrow{a}+\overrightarrow{b}$, $\overrightarrow{a}-\overrightarrow{b}$ ΠΈ $3\overrightarrow{a}$.
Π Π΅ΡΠ΅Π½ΠΈΠ΅.
\[\overrightarrow{a}+\overrightarrow{b}=\left\{3+2,\ 4-1\right\}=\{5,\ 3\}\] \[\overrightarrow{a}-\overrightarrow{b}=\left\{3-2,\ 4+1\right\}=\{1,\ 5\}\] \[3\overrightarrow{a}=\left\{3\cdot 3,3\cdot 4\right\}=\{9,12\}\]
ΠΠ°ΠΊ ΠΎΠΏΡΠ΅Π΄Π΅Π»ΡΡΡΡΡ ΠΊΠΎΠΎΡΠ΄ΠΈΠ½Π°ΡΡ Π²Π΅ΠΊΡΠΎΡΠ° Π² ΠΏΡΠΎΡΡΡΠ°Π½ΡΡΠ²Π΅. ΠΠΎΠΎΡΠ΄ΠΈΠ½Π°ΡΡ Π²Π΅ΠΊΡΠΎΡΠ°. ΠΠ°ΠΏΡΠ°Π²Π»ΡΡΡΠΈΠ΅ ΠΊΠΎΡΠΈΠ½ΡΡΡ. Π‘Π²ΠΎΠΉΡΡΠ²Π° ΡΠΊΠ°Π»ΡΡΠ½ΠΎΠ³ΠΎ ΠΏΡΠΎΠΈΠ·Π²Π΅Π΄Π΅Π½ΠΈΡ Π²Π΅ΠΊΡΠΎΡΠΎΠ²
ΠΠ°Ρ
ΠΎΠΆΠ΄Π΅Π½ΠΈΠ΅ ΠΊΠΎΠΎΡΠ΄ΠΈΠ½Π°Ρ Π²Π΅ΠΊΡΠΎΡΠ° Π΄ΠΎΠ²ΠΎΠ»ΡΠ½ΠΎ ΡΠ°ΡΡΠΎ Π²ΡΡΡΠ΅ΡΠ°Π΅ΠΌΠΎΠ΅ ΡΡΠ»ΠΎΠ²ΠΈΠ΅ ΠΌΠ½ΠΎΠ³ΠΈΡ
Π·Π°Π΄Π°Ρ Π² ΠΌΠ°ΡΠ΅ΠΌΠ°ΡΠΈΠΊΠ΅. Π£ΠΌΠ΅Π½ΠΈΠ΅ Π½Π°Ρ
ΠΎΠ΄ΠΈΡΡ ΠΊΠΎΠΎΡΠ΄ΠΈΠ½Π°ΡΡ Π²Π΅ΠΊΡΠΎΡΠ° ΠΏΠΎΠΌΠΎΠΆΠ΅Ρ Π²Π°ΠΌ Π² Π΄ΡΡΠ³ΠΈΡ
, Π±ΠΎΠ»Π΅Π΅ ΡΠ»ΠΎΠΆΠ½ΡΡ
Π·Π°Π΄Π°ΡΠ°Ρ
ΡΠΎ ΡΡ
ΠΎΠΆΠ΅ΠΉ ΡΠ΅ΠΌΠ°ΡΠΈΠΊΠΎΠΉ. Π Π΄Π°Π½Π½ΠΎΠΉ ΡΡΠ°ΡΡΠ΅ ΠΌΡ ΡΠ°ΡΡΠΌΠΎΡΡΠΈΠΌ ΡΠΎΡΠΌΡΠ»Ρ Π½Π°Ρ
ΠΎΠΆΠ΄Π΅Π½ΠΈΡ ΠΊΠΎΠΎΡΠ΄ΠΈΠ½Π°Ρ Π²Π΅ΠΊΡΠΎΡΠ° ΠΈ Π½Π΅ΡΠΊΠΎΠ»ΡΠΊΠΎ Π·Π°Π΄Π°Ρ.
ΠΠ°Ρ ΠΎΠΆΠ΄Π΅Π½ΠΈΠ΅ ΠΊΠΎΠΎΡΠ΄ΠΈΠ½Π°Ρ Π²Π΅ΠΊΡΠΎΡΠ° Π² ΠΏΠ»ΠΎΡΠΊΠΎΡΡΠΈ
Π§ΡΠΎ ΡΠ°ΠΊΠΎΠ΅ ΠΏΠ»ΠΎΡΠΊΠΎΡΡΡ? ΠΠ»ΠΎΡΠΊΠΎΡΡΡΡ ΡΡΠΈΡΠ°Π΅ΡΡΡ Π΄Π²ΡΡ ΠΌΠ΅ΡΠ½ΠΎΠ΅ ΠΏΡΠΎΡΡΡΠ°Π½ΡΡΠ²ΠΎ, ΠΏΡΠΎΡΡΡΠ°Π½ΡΡΠ²ΠΎ Ρ Π΄Π²ΡΠΌΡ ΠΈΠ·ΠΌΠ΅ΡΠ΅Π½ΠΈΡΠΌΠΈ (ΠΈΠ·ΠΌΠ΅ΡΠ΅Π½ΠΈΠ΅ x ΠΈ ΠΈΠ·ΠΌΠ΅ΡΠ΅Π½ΠΈΠ΅ y). Π ΠΏΡΠΈΠΌΠ΅ΡΡ, Π±ΡΠΌΠ°Π³Π° β ΠΏΠ»ΠΎΡΠΊΠΎΡΡΡ. ΠΠΎΠ²Π΅ΡΡ Π½ΠΎΡΡΡ ΡΡΠΎΠ»Π° β ΠΏΠ»ΠΎΡΠΊΠΎΡΡΡ. ΠΠ°ΠΊΠ°Ρ-Π½ΠΈΠ±ΡΠ΄Ρ Π½Π΅ΠΎΠ±ΡΠ΅ΠΌΠ½Π°Ρ ΡΠΈΠ³ΡΡΠ° (ΠΊΠ²Π°Π΄ΡΠ°Ρ, ΡΡΠ΅ΡΠ³ΠΎΠ»ΡΠ½ΠΈΠΊ, ΡΡΠ°ΠΏΠ΅ΡΠΈΡ) ΡΠΎΠΆΠ΅ ΡΠ²Π»ΡΠ΅ΡΡΡ ΠΏΠ»ΠΎΡΠΊΠΎΡΡΡΡ. Π’Π°ΠΊΠΈΠΌ ΠΎΠ±ΡΠ°Π·ΠΎΠΌ, Π΅ΡΠ»ΠΈ Π² ΡΡΠ»ΠΎΠ²ΠΈΠΈ Π·Π°Π΄Π°ΡΠΈ Π½ΡΠΆΠ½ΠΎ Π½Π°ΠΉΡΠΈ ΠΊΠΎΠΎΡΠ΄ΠΈΠ½Π°ΡΡ Π²Π΅ΠΊΡΠΎΡΠ°, ΠΊΠΎΡΠΎΡΡΠΉ Π»Π΅ΠΆΠΈΡ Π½Π° ΠΏΠ»ΠΎΡΠΊΠΎΡΡΠΈ, ΡΡΠ°Π·Ρ Π²ΡΠΏΠΎΠΌΠΈΠ½Π°Π΅ΠΌ ΠΏΡΠΎ x ΠΈ y. ΠΠ°ΠΉΡΠΈ ΠΊΠΎΠΎΡΠ΄ΠΈΠ½Π°ΡΡ ΡΠ°ΠΊΠΎΠ³ΠΎ Π²Π΅ΠΊΡΠΎΡΠ° ΠΌΠΎΠΆΠ½ΠΎ ΡΠ»Π΅Π΄ΡΡΡΠΈΠΌ ΠΎΠ±ΡΠ°Π·ΠΎΠΌ: ΠΠΎΠΎΡΠ΄ΠΈΠ½Π°ΡΡ AB Π²Π΅ΠΊΡΠΎΡΠ° = (xB β xA; yB β xA). ΠΠ· ΡΠΎΡΠΌΡΠ»Ρ Π²ΠΈΠ΄Π½ΠΎ, ΡΡΠΎ ΠΎΡ ΠΊΠΎΠΎΡΠ΄ΠΈΠ½Π°Ρ ΠΊΠΎΠ½Π΅ΡΠ½ΠΎΠΉ ΡΠΎΡΠΊΠΈ Π½ΡΠΆΠ½ΠΎ ΠΎΡΠ½ΡΡΡ ΠΊΠΎΠΎΡΠ΄ΠΈΠ½Π°ΡΡ Π½Π°ΡΠ°Π»ΡΠ½ΠΎΠΉ ΡΠΎΡΠΊΠΈ.
ΠΡΠΈΠΌΠ΅Ρ:
- ΠΠ΅ΠΊΡΠΎΡ CD ΠΈΠΌΠ΅Π΅Ρ Π½Π°ΡΠ°Π»ΡΠ½ΡΠ΅ (5; 6) ΠΈ ΠΊΠΎΠ½Π΅ΡΠ½ΡΠ΅ (7; 8) ΠΊΠΎΠΎΡΠ΄ΠΈΠ½Π°ΡΡ.
- ΠΠ°ΠΉΡΠΈ ΠΊΠΎΠΎΡΠ΄ΠΈΠ½Π°ΡΡ ΡΠ°ΠΌΠΎΠ³ΠΎ Π²Π΅ΠΊΡΠΎΡΠ°.
- ΠΡΠΏΠΎΠ»ΡΠ·ΡΡ Π²ΡΡΠ΅ΡΠΏΠΎΠΌΡΠ½ΡΡΡΡ ΡΠΎΡΠΌΡΠ»Ρ, ΠΏΠΎΠ»ΡΡΠΈΠΌ ΡΠ»Π΅Π΄ΡΡΡΠ΅Π΅ Π²ΡΡΠ°ΠΆΠ΅Π½ΠΈΠ΅: CD = (7-5; 8-6) = (2; 2).
- Π’Π°ΠΊΠΈΠΌ ΠΎΠ±ΡΠ°Π·ΠΎΠΌ, ΠΊΠΎΠΎΡΠ΄ΠΈΠ½Π°ΡΡ CD Π²Π΅ΠΊΡΠΎΡΠ° = (2; 2).
- Π‘ΠΎΠΎΡΠ²Π΅ΡΡΡΠ²Π΅Π½Π½ΠΎ, x ΠΊΠΎΠΎΡΠ΄ΠΈΠ½Π°ΡΠ° ΡΠ°Π²Π½Π° Π΄Π²ΡΠΌ, y ΠΊΠΎΠΎΡΠ΄ΠΈΠ½Π°ΡΠ° β ΡΠΎΠΆΠ΅ Π΄Π²ΡΠΌ.

ΠΠ°Ρ ΠΎΠΆΠ΄Π΅Π½ΠΈΠ΅ ΠΊΠΎΠΎΡΠ΄ΠΈΠ½Π°Ρ Π²Π΅ΠΊΡΠΎΡΠ° Π² ΠΏΡΠΎΡΡΡΠ°Π½ΡΡΠ²Π΅
Π§ΡΠΎ ΡΠ°ΠΊΠΎΠ΅ ΠΏΡΠΎΡΡΡΠ°Π½ΡΡΠ²ΠΎ? ΠΡΠΎΡΡΡΠ°Π½ΡΡΠ²ΠΎ ΡΡΠΎ ΡΠΆΠ΅ ΡΡΠ΅Ρ ΠΌΠ΅ΡΠ½ΠΎΠ΅ ΠΈΠ·ΠΌΠ΅ΡΠ΅Π½ΠΈΠ΅, Π³Π΄Π΅ Π΄Π°Π½Ρ 3 ΠΊΠΎΠΎΡΠ΄ΠΈΠ½Π°ΡΡ: x, y, z. Π ΡΠ»ΡΡΠ°Π΅, Π΅ΡΠ»ΠΈ Π½ΡΠΆΠ½ΠΎ Π½Π°ΠΉΡΠΈ Π²Π΅ΠΊΡΠΎΡ, ΠΊΠΎΡΠΎΡΡΠΉ Π»Π΅ΠΆΠΈΡ Π² ΠΏΡΠΎΡΡΡΠ°Π½ΡΡΠ²Π΅, ΡΠΎΡΠΌΡΠ»Π° ΠΏΡΠ°ΠΊΡΠΈΡΠ΅ΡΠΊΠΈ Π½Π΅ ΠΌΠ΅Π½ΡΠ΅ΡΡΡ. ΠΠΎΠ±Π°Π²Π»ΡΠ΅ΡΡΡ ΡΠΎΠ»ΡΠΊΠΎ ΠΎΠ΄Π½Π° ΠΊΠΎΠΎΡΠ΄ΠΈΠ½Π°ΡΠ°. ΠΠ»Ρ Π½Π°Ρ ΠΎΠΆΠ΄Π΅Π½ΠΈΡ Π²Π΅ΠΊΡΠΎΡΠ° Π½ΡΠΆΠ½ΠΎ ΠΎΡ ΠΊΠΎΠΎΡΠ΄ΠΈΠ½Π°Ρ ΠΊΠΎΠ½ΡΠ° ΠΎΡΠ½ΡΡΡ ΠΊΠΎΠΎΡΠ΄ΠΈΠ½Π°ΡΡ Π½Π°ΡΠ°Π»Π°. AB = (xB β xA; yB β yA; zB β zA)
ΠΡΠΈΠΌΠ΅Ρ:
- ΠΠ΅ΠΊΡΠΎΡ DF ΠΈΠΌΠ΅Π΅Ρ Π½Π°ΡΠ°Π»ΡΠ½ΡΠ΅ (2; 3; 1) ΠΈ ΠΊΠΎΠ½Π΅ΡΠ½ΡΠ΅ (1; 5; 2).
- ΠΡΠΈΠΌΠ΅Π½ΡΡ Π²ΡΡΠ΅ΡΠΏΠΎΠΌΡΠ½ΡΡΡΡ ΡΠΎΡΠΌΡΠ»Ρ, ΠΏΠΎΠ»ΡΡΠΈΠΌ: ΠΠΎΠΎΡΠ΄ΠΈΠ½Π°ΡΡ Π²Π΅ΠΊΡΠΎΡΠ° DF = (1-2; 5-3; 2-1) = (-1; 2; 1).
- ΠΠΎΠΌΠ½ΠΈΡΠ΅, Π·Π½Π°ΡΠ΅Π½ΠΈΠ΅ ΠΊΠΎΠΎΡΠ΄ΠΈΠ½Π°Ρ ΠΌΠΎΠΆΠ΅Ρ Π±ΡΡΡ ΠΈ ΠΎΡΡΠΈΡΠ°ΡΠ΅Π»ΡΠ½ΡΠΌ, Π² ΡΡΠΎΠΌ Π½Π΅Ρ Π½ΠΈΠΊΠ°ΠΊΠΎΠΉ ΠΏΡΠΎΠ±Π»Π΅ΠΌΡ.
ΠΠ°ΠΊ Π½Π°ΠΉΡΠΈ ΠΊΠΎΠΎΡΠ΄ΠΈΠ½Π°ΡΡ Π²Π΅ΠΊΡΠΎΡΠ° ΠΎΠ½Π»Π°ΠΉΠ½?
ΠΡΠ»ΠΈ ΠΏΠΎ ΠΊΠ°ΠΊΠΈΠΌ-ΡΠΎ ΠΏΡΠΈΡΠΈΠ½Π°ΠΌ Π²Π°ΠΌ Π½Π΅ Ρ
ΠΎΡΠ΅ΡΡΡ Π½Π°Ρ
ΠΎΠ΄ΠΈΡΡ ΠΊΠΎΠΎΡΠ΄ΠΈΠ½Π°ΡΡ ΡΠ°ΠΌΠΎΡΡΠΎΡΡΠ΅Π»ΡΠ½ΠΎ, ΠΌΠΎΠΆΠ½ΠΎ Π²ΠΎΡΠΏΠΎΠ»ΡΠ·ΠΎΠ²Π°ΡΡΡΡ ΠΎΠ½Π»Π°ΠΉΠ½ ΠΊΠ°Π»ΡΠΊΡΠ»ΡΡΠΎΡΠΎΠΌ . ΠΠ»Ρ Π½Π°ΡΠ°Π»Π°, Π²ΡΠ±Π΅ΡΠΈΡΠ΅ ΡΠ°Π·ΠΌΠ΅ΡΠ½ΠΎΡΡΡ Π²Π΅ΠΊΡΠΎΡΠ°. Π Π°Π·ΠΌΠ΅ΡΠ½ΠΎΡΡΡ Π²Π΅ΠΊΡΠΎΡΠ° ΠΎΡΠ²Π΅ΡΠ°Π΅Ρ Π·Π° Π΅Π³ΠΎ ΠΈΠ·ΠΌΠ΅ΡΠ΅Π½ΠΈΡ. Π Π°Π·ΠΌΠ΅ΡΠ½ΠΎΡΡΡ 3 ΠΎΠ·Π½Π°ΡΠ°Π΅Ρ, ΡΡΠΎ Π²Π΅ΠΊΡΠΎΡ Π½Π°Ρ
ΠΎΠ΄ΠΈΡΡΡ Π² ΠΏΡΠΎΡΡΡΠ°Π½ΡΡΠ²Π΅, ΡΠ°Π·ΠΌΠ΅ΡΠ½ΠΎΡΡΡ 2 β ΡΡΠΎ Π½Π° ΠΏΠ»ΠΎΡΠΊΠΎΡΡΠΈ.
ΠΠ°Π»Π΅Π΅ Π²ΡΡΠ°Π²ΡΡΠ΅ ΠΊΠΎΠΎΡΠ΄ΠΈΠ½Π°ΡΡ ΡΠΎΡΠ΅ΠΊ Π² ΡΠΎΠΎΡΠ²Π΅ΡΡΡΠ²ΡΡΡΠΈΠ΅ ΠΏΠΎΠ»Ρ ΠΈ ΠΏΡΠΎΠ³ΡΠ°ΠΌΠΌΠ° ΠΎΠΏΡΠ΅Π΄Π΅Π»ΠΈΡ Π²Π°ΠΌ ΠΊΠΎΠΎΡΠ΄ΠΈΠ½Π°ΡΡ ΡΠ°ΠΌΠΎΠ³ΠΎ Π²Π΅ΠΊΡΠΎΡΠ°. ΠΡΠ΅ ΠΎΡΠ΅Π½Ρ ΠΏΡΠΎΡΡΠΎ.
ΠΠ°ΠΆΠ°Π² Π½Π° ΠΊΠ½ΠΎΠΏΠΊΡ, ΡΡΡΠ°Π½ΠΈΡΠ° Π°Π²ΡΠΎΠΌΠ°ΡΠΈΡΠ΅ΡΠΊΠΈ ΠΏΡΠΎΠΊΡΡΡΠΈΡΡΡ Π²Π½ΠΈΠ· ΠΈ Π²ΡΠ΄Π°ΡΡ Π²Π°ΠΌ ΠΏΡΠ°Π²ΠΈΠ»ΡΠ½ΡΠΉ ΠΎΡΠ²Π΅Ρ Π²ΠΌΠ΅ΡΡΠ΅ Ρ ΡΡΠ°ΠΏΠ°ΠΌΠΈ ΡΠ΅ΡΠ΅Π½ΠΈΡ.
Π Π΅ΠΊΠΎΠΌΠ΅Π½Π΄ΠΎΠ²Π°Π½ΠΎ Ρ ΠΎΡΠΎΡΠΎ ΠΈΠ·ΡΡΠΈΡΡ Π΄Π°Π½Π½ΡΡ ΡΠ΅ΠΌΡ, ΠΏΠΎΡΠΎΠΌΡ ΡΡΠΎ ΠΏΠΎΠ½ΡΡΠΈΠ΅ Π²Π΅ΠΊΡΠΎΡΠ° Π²ΡΡΡΠ΅ΡΠ°Π΅ΡΡΡ Π½Π΅ ΡΠΎΠ»ΡΠΊΠΎ Π² ΠΌΠ°ΡΠ΅ΠΌΠ°ΡΠΈΠΊΠ΅, Π½ΠΎ ΠΈ Π² ΡΠΈΠ·ΠΈΠΊΠ΅. Π‘ΡΡΠ΄Π΅Π½ΡΡ ΡΠ°ΠΊΡΠ»ΡΡΠ΅ΡΠ° ΠΠ½ΡΠΎΡΠΌΠ°ΡΠΈΠΎΠ½Π½ΡΡ Π’Π΅Ρ Π½ΠΎΠ»ΠΎΠ³ΠΈΠΉ ΡΠΎΠΆΠ΅ ΠΈΠ·ΡΡΠ°ΡΡ ΡΠ΅ΠΌΡ Π²Π΅ΠΊΡΠΎΡΠΎΠ², Π½ΠΎ Π½Π° Π±ΠΎΠ»Π΅Π΅ ΡΠ»ΠΎΠΆΠ½ΠΎΠΌ ΡΡΠΎΠ²Π½Π΅.
ΠΠ΅ΠΎΠ±Ρ ΠΎΠ΄ΠΈΠΌΠΎ ΠΎΠΏΡΠ΅Π΄Π΅Π»ΠΈΡΡ Π²Π΅ΠΊΡΠΎΡ Π½Π° ΠΏΠ»ΠΎΡΠΊΠΎΡΡΠΈ ΠΈΠ»ΠΈ Π² ΠΏΡΠΎΡΡΡΠ°Π½ΡΡΠ²Π΅, ΡΠΎ Π΅ΡΡΡ Π΄Π°ΡΡ ΠΈΠ½ΡΠΎΡΠΌΠ°ΡΠΈΡ ΠΎ Π΅Π³ΠΎ Π½Π°ΠΏΡΠ°Π²Π»Π΅Π½ΠΈΠΈ ΠΈ Π΄Π»ΠΈΠ½Π΅.
ΠΠΎΠΎΡΠ΄ΠΈΠ½Π°ΡΡ Π²Π΅ΠΊΡΠΎΡΠ°
ΠΡΡΡΡ Π·Π°Π΄Π°Π½Π° ΠΏΡΡΠΌΠΎΡΠ³ΠΎΠ»ΡΠ½Π°Ρ Π΄Π΅ΠΊΠ°ΡΡΠΎΠ²Π° ΡΠΈΡΡΠ΅ΠΌΠ° ΠΊΠΎΠΎΡΠ΄ΠΈΠ½Π°Ρ (ΠΠΠ‘Π) $x O y$ ΠΈ ΠΏΡΠΎΠΈΠ·Π²ΠΎΠ»ΡΠ½ΡΠΉ Π²Π΅ΠΊΡΠΎΡ $\overline{a}$, Π½Π°ΡΠ°Π»ΠΎ ΠΊΠΎΡΠΎΡΠΎΠ³ΠΎ ΡΠΎΠ²ΠΏΠ°Π΄Π°Π΅Ρ Ρ Π½Π°ΡΠ°Π»ΠΎΠΌ ΡΠΈΡΡΠ΅ΠΌΡ ΠΊΠΎΠΎΡΠ΄ΠΈΠ½Π°Ρ (ΡΠΈΡ. 1).
ΠΠΏΡΠ΅Π΄Π΅Π»Π΅Π½ΠΈΠ΅
ΠΠΎΠΎΡΠ΄ΠΈΠ½Π°ΡΠ°ΠΌΠΈ Π²Π΅ΠΊΡΠΎΡΠ° $\overline{a}$ Π½Π°Π·ΡΠ²Π°ΡΡΡΡ ΠΏΡΠΎΠ΅ΠΊΡΠΈΠΈ $a_{x}$ ΠΈ $a_{y}$ Π΄Π°Π½Π½ΠΎΠ³ΠΎ Π²Π΅ΠΊΡΠΎΡΠ° Π½Π° ΠΎΡΠΈ $O x$ ΠΈ $O y$ ΡΠΎΠΎΡΠ²Π΅ΡΡΡΠ²Π΅Π½Π½ΠΎ:
ΠΠ΅Π»ΠΈΡΠΈΠ½Π° $a_{x}$ Π½Π°Π·ΡΠ²Π°Π΅ΡΡΡ Π°Π±ΡΡΠΈΡΡΠΎΠΉ Π²Π΅ΠΊΡΠΎΡΠ° $\overline{a}$, Π° ΡΠΈΡΠ»ΠΎ $a_{y}$
— Π΅Π³ΠΎ ΠΎΡΠ΄ΠΈΠ½Π°ΡΠΎΠΉ .
Π’ΠΎ, ΡΡΠΎ Π²Π΅ΠΊΡΠΎΡ $\overline{a}$ ΠΈΠΌΠ΅Π΅Ρ ΠΊΠΎΠΎΡΠ΄ΠΈΠ½Π°ΡΡ
$a_{x}$ ΠΈ $a_{y}$,
Π·Π°ΠΏΠΈΡΡΠ²Π°Π΅ΡΡΡ ΡΠ»Π΅Π΄ΡΡΡΠΈΠΌ ΠΎΠ±ΡΠ°Π·ΠΎΠΌ: $\overline{a}=\left(a_{x} ; a_{y}\right)$.
ΠΡΠΈΠΌΠ΅Ρ
ΠΠ°ΠΏΠΈΡΡ $\overline{a}=(5 ;-2)$ ΠΎΠ·Π½Π°ΡΠ°Π΅Ρ, ΡΡΠΎ Π²Π΅ΠΊΡΠΎΡ $\overline{a}$ ΠΈΠΌΠ΅Π΅Ρ ΡΠ»Π΅Π΄ΡΡΡΠΈΠ΅ ΠΊΠΎΠΎΡΠ΄ΠΈΠ½Π°ΡΡ: Π°Π±ΡΡΠΈΡΡΠ° ΡΠ°Π²Π½Π° 5, ΠΎΡΠ΄ΠΈΠ½Π°ΡΠ° ΡΠ°Π²Π½Π° -2.
Π‘ΡΠΌΠΌΠ° Π΄Π²ΡΡ Π²Π΅ΠΊΡΠΎΡΠΎΠ², Π·Π°Π΄Π°Π½Π½ΡΡ ΠΊΠΎΠΎΡΠ΄ΠΈΠ½Π°ΡΠ°ΠΌΠΈ
ΠΡΡΡΡ Π·Π°Π΄Π°Π½Ρ $\overline{a}=\left(a_{x} ; a_{y}\right)$ ΠΈ $\overline{b}=\left(b_{x} ; b_{y}\right)$, ΡΠΎΠ³Π΄Π° Π²Π΅ΠΊΡΠΎΡ $\overline{c}=\overline{a}+\overline{b}$ ΠΈΠΌΠ΅Π΅Ρ ΠΊΠΎΠΎΡΠ΄ΠΈΠ½Π°ΡΡ $\left(a_{x}+b_{x} ; a_{y}+b_{y}\right)$ (ΡΠΈΡ. 2).
ΠΠΏΡΠ΅Π΄Π΅Π»Π΅Π½ΠΈΠ΅
Π§ΡΠΎΠ±Ρ Π½Π°ΠΉΡΠΈ ΡΡΠΌΠΌΡ Π΄Π²ΡΡ Π²Π΅ΠΊΡΠΎΡΠΎΠ² , Π·Π°Π΄Π°Π½Π½ΡΡ ΡΠ²ΠΎΠΈΠΌΠΈ ΠΊΠΎΠΎΡΠ΄ΠΈΠ½Π°ΡΠ°ΠΌΠΈ, Π½Π°Π΄ΠΎ ΡΠ»ΠΎΠΆΠΈΡΡ ΠΈΡ ΡΠΎΠΎΡΠ²Π΅ΡΡΡΠ²ΡΡΡΠΈΠ΅ ΠΊΠΎΠΎΡΠ΄ΠΈΠ½Π°ΡΡ.
ΠΡΠΈΠΌΠ΅Ρ
ΠΠ°Π΄Π°Π½ΠΈΠ΅. ΠΠ°Π΄Π°Π½Ρ $\overline{a}=(-3 ; 5)$ ΠΈ $\overline{b}=(0 ;-1)$. ΠΠ°ΠΉΡΠΈ ΠΊΠΎΠΎΡΠ΄ΠΈΠ½Π°ΡΡ Π²Π΅ΠΊΡΠΎΡΠ° $\overline{c}=\overline{a}+\overline{b}$
Π Π΅ΡΠ΅Π½ΠΈΠ΅. $\overline{c}=\overline{a}+\overline{b}=(-3 ; 5)+(0 ;-1)=(-3+0 ; 5+(-1))=(-3 ; 4)$
Π£ΠΌΠ½ΠΎΠΆΠ΅Π½ΠΈΠ΅ Π²Π΅ΠΊΡΠΎΡΠ° Π½Π° ΡΠΈΡΠ»ΠΎ
ΠΡΠ»ΠΈ Π·Π°Π΄Π°Π½ $\overline{a}=\left(a_{x} ; a_{y}\right)$, ΡΠΎ ΡΠΎΠ³Π΄Π° Π²Π΅ΠΊΡΠΎΡ
$m \overline{a}$ ΠΈΠΌΠ΅Π΅Ρ ΠΊΠΎΠΎΡΠ΄ΠΈΠ½Π°ΡΡ
$m \overline{a}=\left(m a_{x} ; m a_{y}\right)$, Π·Π΄Π΅ΡΡ
$m$ — Π½Π΅ΠΊΠΎΡΠΎΡΠΎΠ΅ ΡΠΈΡΠ»ΠΎ (ΡΠΈΡ.
3).
ΠΡΠΈΠΌΠ΅Ρ
ΠΠ°Π΄Π°Π½ΠΈΠ΅. ΠΠ΅ΠΊΡΠΎΡ $\overline{a}=(3 ;-2)$. ΠΠ°ΠΉΡΠΈ ΠΊΠΎΠΎΡΠ΄ΠΈΠ½Π°ΡΡ Π²Π΅ΠΊΡΠΎΡΠ° 2$\overline{a}$
Π Π΅ΡΠ΅Π½ΠΈΠ΅. $2 \overline{a}=2 \cdot(3 ;-2)=(2 \cdot 3 ; 2 \cdot(-2))=(6 ;-4)$
Π Π°ΡΡΠΌΠΎΡΡΠΈΠΌ Π΄Π°Π»Π΅Π΅ ΡΠ»ΡΡΠ°ΠΉ, ΠΊΠΎΠ³Π΄Π° Π½Π°ΡΠ°Π»ΠΎ Π²Π΅ΠΊΡΠΎΡΠ° Π½Π΅ ΡΠΎΠ²ΠΏΠ°Π΄Π°Π΅Ρ Ρ Π½Π°ΡΠ°Π»ΠΎΠΌ ΡΠΈΡΡΠ΅ΠΌΡ ΠΊΠΎΠΎΡΠ΄ΠΈΠ½Π°Ρ. ΠΡΠ΅Π΄ΠΏΠΎΠ»ΠΎΠΆΠΈΠΌ, ΡΡΠΎ Π² ΠΠΠ‘Π Π·Π°Π΄Π°Π½Ρ Π΄Π²Π΅ ΡΠΎΡΠΊΠΈ $A\left(a_{x} ; a_{y}\right)$ ΠΈ $B\left(b_{x} ; b_{y}\right)$. Π’ΠΎΠ³Π΄Π° ΠΊΠΎΠΎΡΠ΄ΠΈΠ½Π°ΡΡ Π²Π΅ΠΊΡΠΎΡΠ° $\overline{A B}=\left(x_{1} ; y_{1}\right)$ Π½Π°Ρ ΠΎΠ΄ΡΡΡΡ ΠΏΠΎ ΡΠΎΡΠΌΡΠ»Π°ΠΌ (ΡΠΈΡ. 4):
$x_{1}=b_{x}-a_{x}, y_{1}=b_{y}-a_{y}$
ΠΠΏΡΠ΅Π΄Π΅Π»Π΅Π½ΠΈΠ΅
Π§ΡΠΎΠ±Ρ Π½Π°ΠΉΡΠΈ ΠΊΠΎΠΎΡΠ΄ΠΈΠ½Π°ΡΡ Π²Π΅ΠΊΡΠΎΡΠ° , Π·Π°Π΄Π°Π½Π½ΠΎΠ³ΠΎ ΠΊΠΎΠΎΡΠ΄ΠΈΠ½Π°ΡΠ°ΠΌΠΈ Π½Π°ΡΠ°Π»Π° ΠΈ ΠΊΠΎΠ½ΡΠ°, Π½Π°Π΄ΠΎ ΠΎΡ ΠΊΠΎΠΎΡΠ΄ΠΈΠ½Π°Ρ ΠΊΠΎΠ½ΡΠ° ΠΎΡΠ½ΡΡΡ ΡΠΎΠΎΡΠ²Π΅ΡΡΡΠ²ΡΡΡΠΈΠ΅ ΠΊΠΎΠΎΡΠ΄ΠΈΠ½Π°ΡΡ Π½Π°ΡΠ°Π»Π°.
ΠΡΠΈΠΌΠ΅Ρ
ΠΠ°Π΄Π°Π½ΠΈΠ΅. ΠΠ°ΠΉΡΠΈ ΠΊΠΎΠΎΡΠ΄ΠΈΠ½Π°ΡΡ Π²Π΅ΠΊΡΠΎΡΠ° $\overline{A B}$, Π΅ΡΠ»ΠΈ $A(-4 ; 2), B(1 ;-3)$
Π Π΅ΡΠ΅Π½ΠΈΠ΅. $\overline{A B}=(1-(-4) ;-3-2)=(5 ;-5)$
ΠΠ°ΠΏΡΠ°Π²Π»ΡΡΡΠΈΠ΅ ΠΊΠΎΡΠΈΠ½ΡΡΡ
ΠΠΏΡΠ΅Π΄Π΅Π»Π΅Π½ΠΈΠ΅
ΠΠ°ΠΏΡΠ°Π²Π»ΡΡΡΠΈΠΌΠΈ ΠΊΠΎΡΠΈΠ½ΡΡΠ°ΠΌΠΈ Π²Π΅ΠΊΡΠΎΡΠ° Π½Π°Π·ΡΠ²Π°ΡΡΡΡ ΠΊΠΎΡΠΈΠ½ΡΡΡ ΡΠ³Π»ΠΎΠ², ΠΎΠ±ΡΠ°Π·ΠΎΠ²Π°Π½Π½ΡΡ
Π²Π΅ΠΊΡΠΎΡΠΎΠΌ Ρ
ΠΏΠΎΠ»ΠΎΠΆΠΈΡΠ΅Π»ΡΠ½ΡΠΌΠΈ Π½Π°ΠΏΡΠ°Π²Π»Π΅Π½ΠΈΡΠΌΠΈ ΠΎΡΠ΅ΠΉ ΠΊΠΎΠΎΡΠ΄ΠΈΠ½Π°Ρ.
{2}}}$
ΠΠ΄Π΅ΡΡ $\alpha$, $\beta$ ΠΈ $\gamma$ — ΡΠ³Π»Ρ, ΠΊΠΎΡΠΎΡΡΠ΅ ΡΠΎΡΡΠ°Π²Π»ΡΠ΅Ρ Π²Π΅ΠΊΡΠΎΡ Ρ ΠΏΠΎΠ»ΠΎΠΆΠΈΡΠ΅Π»ΡΠ½ΡΠΌΠΈ Π½Π°ΠΏΡΠ°Π²Π»Π΅Π½ΠΈΡΠΌΠΈ ΠΎΡΠ΅ΠΉ $O x$, $O y$ ΠΈ $O z$ ΡΠΎΠΎΡΠ²Π΅ΡΡΡΠ²Π΅Π½Π½ΠΎ.
ΠΠ° ΠΎΡΠΈ Π°Π±ΡΡΠΈΡΡ ΠΈ ΠΎΡΠ΄ΠΈΠ½Π°Ρ Π½Π°Π·ΡΠ²Π°ΡΡΡΡ ΠΊΠΎΠΎΡΠ΄ΠΈΠ½Π°ΡΠ°ΠΌΠΈ Π²Π΅ΠΊΡΠΎΡΠ° . ΠΠΎΠΎΡΠ΄ΠΈΠ½Π°ΡΡ Π²Π΅ΠΊΡΠΎΡΠ° ΠΎΠ±ΡΠ΅ΠΏΡΠΈΠ½ΡΡΠΎ ΡΠΊΠ°Π·ΡΠ²Π°ΡΡ Π² Π²ΠΈΠ΄Π΅ (Ρ , Ρ) , Π° ΡΠ°ΠΌ Π²Π΅ΠΊΡΠΎΡ ΠΊΠ°ΠΊ: =(Ρ , Ρ).
Π€ΠΎΡΠΌΡΠ»Π° ΠΎΠΏΡΠ΅Π΄Π΅Π»Π΅Π½ΠΈΡ ΠΊΠΎΠΎΡΠ΄ΠΈΠ½Π°Ρ Π²Π΅ΠΊΡΠΎΡΠ° Π΄Π»Ρ Π΄Π²ΡΡ ΠΌΠ΅ΡΠ½ΡΡ Π·Π°Π΄Π°Ρ.
Π ΡΠ»ΡΡΠ°Π΅ Π΄Π²ΡΡ ΠΌΠ΅ΡΠ½ΠΎΠΉ Π·Π°Π΄Π°ΡΠΈ Π²Π΅ΠΊΡΠΎΡ Ρ ΠΈΠ·Π²Π΅ΡΡΠ½ΡΠΌΠΈ ΠΊΠΎΠΎΡΠ΄ΠΈΠ½Π°ΡΠ°ΠΌΠΈ ΡΠΎΡΠ΅ΠΊ A(Ρ 1 ;Ρ 1) ΠΈ B(x 2 ; y 2 ) ΠΌΠΎΠΆΠ½ΠΎ Π²ΡΡΠΈΡΠ»ΠΈΡΡ:
= (x 2 — x 1 ; y 2 — y 1).
Π€ΠΎΡΠΌΡΠ»Π° ΠΎΠΏΡΠ΅Π΄Π΅Π»Π΅Π½ΠΈΡ ΠΊΠΎΠΎΡΠ΄ΠΈΠ½Π°Ρ Π²Π΅ΠΊΡΠΎΡΠ° Π΄Π»Ρ ΠΏΡΠΎΡΡΡΠ°Π½ΡΡΠ²Π΅Π½Π½ΡΡ Π·Π°Π΄Π°Ρ.
Π ΡΠ»ΡΡΠ°Π΅ ΠΏΡΠΎΡΡΡΠ°Π½ΡΡΠ²Π΅Π½Π½ΠΎΠΉ Π·Π°Π΄Π°ΡΠΈ Π²Π΅ΠΊΡΠΎΡ Ρ ΠΈΠ·Π²Π΅ΡΡΠ½ΡΠΌΠΈ ΠΊΠΎΠΎΡΠ΄ΠΈΠ½Π°ΡΠ°ΠΌΠΈ ΡΠΎΡΠ΅ΠΊ A(Ρ 1 ;Ρ 1 ; z 1 ) ΠΈ B(x 2 ; y 2 ; z 2 ) ΠΌΠΎΠΆΠ½ΠΎ Π²ΡΡΠΈΡΠ»ΠΈΡΡ ΠΏΡΠΈΠΌΠ΅Π½ΠΈΠ² ΡΠΎΡΠΌΡΠ»Ρ:
= (x 2 — x 1 ; y 2 — y 1 ; z 2 — z 1 ).
ΠΠΎΠΎΡΠ΄ΠΈΠ½Π°ΡΡ Π΄Π°ΡΡ Π²ΡΠ΅ΠΎΠ±ΡΠ΅ΠΌΠ»ΡΡΡΡ Ρ Π°ΡΠ°ΠΊΡΠ΅ΡΠΈΡΡΠΈΠΊΡ Π²Π΅ΠΊΡΠΎΡΠ°, ΠΏΠΎΡΠΊΠΎΠ»ΡΠΊΡ ΠΏΠΎ ΠΊΠΎΠΎΡΠ΄ΠΈΠ½Π°ΡΠ°ΠΌ Π΅ΡΡΡ Π²ΠΎΠ·ΠΌΠΎΠΆΠ½ΠΎΡΡΡ ΠΏΠΎΡΡΡΠΎΠΈΡΡ ΠΈ ΡΠ°ΠΌ Π²Π΅ΠΊΡΠΎΡ. ΠΠ½Π°Ρ ΠΊΠΎΠΎΡΠ΄ΠΈΠ½Π°ΡΡ, Π»Π΅Π³ΠΊΠΎ Π²ΡΡΠΈΡΠ»ΠΈΡΡ ΠΈ Π΄Π»ΠΈΠ½Ρ Π²Π΅ΠΊΡΠΎΡΠ° . (Π‘Π²ΠΎΠΉΡΡΠ²ΠΎ 3, ΠΏΡΠΈΠ²Π΅Π΄Π΅Π½Π½ΠΎΠ΅ Π½ΠΈΠΆΠ΅).
Π‘Π²ΠΎΠΉΡΡΠ²Π° ΠΊΠΎΠΎΡΠ΄ΠΈΠ½Π°Ρ Π²Π΅ΠΊΡΠΎΡΠ°.
1. ΠΡΠ±ΡΠ΅ ΡΠ°Π²Π½ΡΠ΅ Π²Π΅ΠΊΡΠΎΡΡ Π² Π΅Π΄ΠΈΠ½ΠΎΠΉ ΡΠΈΡΡΠ΅ΠΌΠ΅ ΠΊΠΎΠΎΡΠ΄ΠΈΠ½Π°Ρ ΠΈΠΌΠ΅ΡΡ ΡΠ°Π²Π½ΡΠ΅ ΠΊΠΎΠΎΡΠ΄ΠΈΠ½Π°ΡΡ .
2. ΠΠΎΠΎΡΠ΄ΠΈΠ½Π°ΡΡ ΠΊΠΎΠ»Π»ΠΈΠ½Π΅Π°ΡΠ½ΡΡ Π²Π΅ΠΊΡΠΎΡΠΎΠ² ΠΏΡΠΎΠΏΠΎΡΡΠΈΠΎΠ½Π°Π»ΡΠ½Ρ. ΠΡΠΈ ΡΡΠ»ΠΎΠ²ΠΈΠΈ, ΡΡΠΎ Π½ΠΈ ΠΎΠ΄ΠΈΠ½ ΠΈΠ· Π²Π΅ΠΊΡΠΎΡΠΎΠ² Π½Π΅ ΡΠ°Π²Π΅Π½ Π½ΡΠ»Ρ.
3. ΠΠ²Π°Π΄ΡΠ°Ρ Π΄Π»ΠΈΠ½Ρ Π»ΡΠ±ΠΎΠ³ΠΎ Π²Π΅ΠΊΡΠΎΡΠ° ΡΠ°Π²Π΅Π½ ΡΡΠΌΠΌΠ΅ ΠΊΠ²Π°Π΄ΡΠ°ΡΠΎΠ² Π΅Π³ΠΎ ΠΊΠΎΠΎΡΠ΄ΠΈΠ½Π°Ρ .
4.ΠΡΠΈ ΠΎΠΏΠ΅ΡΠ°ΡΠΈΠΈ ΡΠΌΠ½ΠΎΠΆΠ΅Π½ΠΈΡ Π²Π΅ΠΊΡΠΎΡΠ° Π½Π° Π΄Π΅ΠΉΡΡΠ²ΠΈΡΠ΅Π»ΡΠ½ΠΎΠ΅ ΡΠΈΡΠ»ΠΎ ΠΊΠ°ΠΆΠ΄Π°Ρ Π΅Π³ΠΎ ΠΊΠΎΠΎΡΠ΄ΠΈΠ½Π°ΡΠ° ΡΠΌΠ½ΠΎΠΆΠ°Π΅ΡΡΡ Π½Π° ΡΡΠΎ ΡΠΈΡΠ»ΠΎ.
5. ΠΡΠΈ ΠΎΠΏΠ΅ΡΠ°ΡΠΈΠΈ ΡΠ»ΠΎΠΆΠ΅Π½ΠΈΡ Π²Π΅ΠΊΡΠΎΡΠΎΠ² Π²ΡΡΠΈΡΠ»ΡΠ΅ΠΌ ΡΡΠΌΠΌΡ ΡΠΎΠΎΡΠ²Π΅ΡΡΡΠ²ΡΡΡΠΈΠ΅ ΠΊΠΎΠΎΡΠ΄ΠΈΠ½Π°ΡΡ Π²Π΅ΠΊΡΠΎΡΠΎΠ² .
6. Π‘ΠΊΠ°Π»ΡΡΠ½ΠΎΠ΅ ΠΏΡΠΎΠΈΠ·Π²Π΅Π΄Π΅Π½ΠΈΠ΅ Π΄Π²ΡΡ Π²Π΅ΠΊΡΠΎΡΠΎΠ² ΡΠ°Π²Π½ΡΠ΅ΡΡΡ ΡΡΠΌΠΌΠ΅ ΠΏΡΠΎΠΈΠ·Π²Π΅Π΄Π΅Π½ΠΈΠΉ ΠΈΡ ΡΠΎΠΎΡΠ²Π΅ΡΡΡΠ²ΡΡΡΠΈΡ ΠΊΠΎΠΎΡΠ΄ΠΈΠ½Π°Ρ.
ΠΠ»Ρ Π½Π°ΡΠ°Π»Π° Π΄Π°Π΄ΠΈΠΌ ΠΎΠΏΡΠ΅Π΄Π΅Π»Π΅Π½ΠΈΠ΅ ΠΊΠΎΠΎΡΠ΄ΠΈΠ½Π°Ρ Π²Π΅ΠΊΡΠΎΡΠ° Π² Π·Π°Π΄Π°Π½Π½ΠΎΠΉ ΡΠΈΡΡΠ΅ΠΌΠ΅ ΠΊΠΎΠΎΡΠ΄ΠΈΠ½Π°Ρ. Π§ΡΠΎΠ±Ρ Π²Π²Π΅ΡΡΠΈ Π΄Π°Π½Π½ΠΎΠ΅ ΠΏΠΎΠ½ΡΡΠΈΠ΅, ΠΎΠΏΡΠ΅Π΄Π΅Π»ΠΈΠΌ ΡΡΠΎ ΠΌΡ Π½Π°Π·ΡΠ²Π°Π΅ΠΌ ΠΏΡΡΠΌΠΎΡΠ³ΠΎΠ»ΡΠ½ΠΎΠΉ ΠΈΠ»ΠΈ Π΄Π΅ΠΊΠ°ΡΡΠΎΠ²ΠΎΠΉ ΡΠΈΡΡΠ΅ΠΌΠΎΠΉ ΠΊΠΎΠΎΡΠ΄ΠΈΠ½Π°Ρ.
ΠΠΏΡΠ΅Π΄Π΅Π»Π΅Π½ΠΈΠ΅ 1
ΠΡΡΠΌΠΎΡΠ³ΠΎΠ»ΡΠ½Π°Ρ ΡΠΈΡΡΠ΅ΠΌΠ° ΠΊΠΎΠΎΡΠ΄ΠΈΠ½Π°Ρ ΠΏΡΠ΅Π΄ΡΡΠ°Π²Π»ΡΠ΅Ρ ΠΈΠ· ΡΠ΅Π±Ρ ΠΏΡΡΠΌΠΎΠ»ΠΈΠ½Π΅ΠΉΠ½ΡΡ ΡΠΈΡΡΠ΅ΠΌΡ ΠΊΠΎΠΎΡΠ΄ΠΈΠ½Π°Ρ Ρ Π²Π·Π°ΠΈΠΌΠ½ΠΎ ΠΏΠ΅ΡΠΏΠ΅Π½Π΄ΠΈΠΊΡΠ»ΡΡΠ½ΡΠΌΠΈ ΠΎΡΡΠΌΠΈ Π½Π° ΠΏΠ»ΠΎΡΠΊΠΎΡΡΠΈ ΠΈΠ»ΠΈ Π² ΠΏΡΠΎΡΡΡΠ°Π½ΡΡΠ²Π΅.
Π‘ ΠΏΠΎΠΌΠΎΡΡΡ Π²Π²Π΅Π΄Π΅Π½ΠΈΡ ΠΏΡΡΠΌΠΎΡΠ³ΠΎΠ»ΡΠ½ΠΎΠΉ ΡΠΈΡΡΠ΅ΠΌΡ ΠΊΠΎΠΎΡΠ΄ΠΈΠ½Π°Ρ Π½Π° ΠΏΠ»ΠΎΡΠΊΠΎΡΡΠΈ ΠΈΠ»ΠΈ Π² ΡΡΠ΅Ρ ΠΌΠ΅ΡΠ½ΠΎΠΌ ΠΏΡΠΎΡΡΡΠ°Π½ΡΡΠ²Π΅ ΡΡΠ°Π½ΠΎΠ²ΠΈΡΡΡ Π²ΠΎΠ·ΠΌΠΎΠΆΠ½ΡΠΌ ΠΎΠΏΠΈΡΡΠ²Π°Π½ΠΈΠ΅ Π³Π΅ΠΎΠΌΠ΅ΡΡΠΈΡΠ΅ΡΠΊΠΈΡ ΡΠΈΠ³ΡΡ Π²ΠΌΠ΅ΡΡΠ΅ Ρ ΠΈΡ ΡΠ²ΠΎΠΉΡΡΠ²Π°ΠΌΠΈ ΠΏΡΠΈ ΠΏΠΎΠΌΠΎΡΠΈ ΡΡΠ°Π²Π½Π΅Π½ΠΈΠΉ ΠΈ Π½Π΅ΡΠ°Π²Π΅Π½ΡΡΠ², ΡΠΎ Π΅ΡΡΡ ΠΈΡΠΏΠΎΠ»ΡΠ·ΠΎΠ²Π°ΡΡ Π°Π»Π³Π΅Π±ΡΠ°ΠΈΡΠ΅ΡΠΊΠΈΠ΅ ΠΌΠ΅ΡΠΎΠ΄Ρ ΠΏΡΠΈ ΡΠ΅ΡΠ΅Π½ΠΈΠΈ Π³Π΅ΠΎΠΌΠ΅ΡΡΠΈΡΠ΅ΡΠΊΠΈΡ Π·Π°Π΄Π°Ρ.
Π’Π΅ΠΌ ΡΠ°ΠΌΡΠΌ, ΠΌΡ ΠΌΠΎΠΆΠ΅ΠΌ ΠΏΡΠΈΠ²ΡΠ·Π°ΡΡ ΠΊ Π·Π°Π΄Π°Π½Π½ΠΎΠΉ ΡΠΈΡΡΠ΅ΠΌΠ΅ ΠΊΠΎΠΎΡΠ΄ΠΈΠ½Π°Ρ Π²Π΅ΠΊΡΠΎΡΡ. ΠΡΠΎ Π·Π½Π°ΡΠΈΡΠ΅Π»ΡΠ½ΠΎ ΡΠ°ΡΡΠΈΡΠΈΡ Π½Π°ΡΠΈ Π²ΠΎΠ·ΠΌΠΎΠΆΠ½ΠΎΡΡΠΈ ΠΏΡΠΈ ΡΠ΅ΡΠ΅Π½ΠΈΠΈ ΠΎΠΏΡΠ΅Π΄Π΅Π»Π΅Π½Π½ΡΡ Π·Π°Π΄Π°Ρ
ΠΡΡΠΌΠΎΡΠ³ΠΎΠ»ΡΠ½Π°Ρ ΡΠΈΡΡΠ΅ΠΌΠ° ΠΊΠΎΠΎΡΠ΄ΠΈΠ½Π°Ρ Π½Π° ΠΏΠ»ΠΎΡΠΊΠΎΡΡΠΈ ΠΎΠ±ΡΡΠ½ΠΎ ΠΎΠ±ΠΎΠ·Π½Π°ΡΠ°Π΅ΡΡΡ O x y , Π³Π΄Π΅ O x ΠΈ O y β ΠΎΡΠΈ ΠΊΠΎΠΎΡΠ΄Π½Π°Ρ. ΠΡΡ O x Π½Π°Π·ΡΠ²Π°ΡΡ ΠΎΡΡΡ Π°Π±ΡΡΠΈΡΡ, Π° ΠΎΡΡ O y β ΠΎΡΡΡ ΠΎΡΠ΄ΠΈΠ½Π°Ρ (Π² ΠΏΡΠΎΡΡΡΠ°Π½ΡΡΠ²Π΅ ΠΏΠΎΡΠ²Π»ΡΠ΅ΡΡΡ Π΅ΡΡ ΠΎΠ΄Π½Π° ΠΎΡΡ O z , ΠΊΠΎΡΠΎΡΠ°Ρ ΠΏΠ΅ΡΠΏΠ΅Π½Π΄ΠΈΠΊΡΠ»ΡΡΠ½Π° ΠΈ O x ΠΈ O y).
ΠΡΠΈΠΌΠ΅Ρ 1
ΠΡΠ°ΠΊ, Π½Π°ΠΌ Π΄Π°Π½Π° ΠΏΡΡΠΌΠΎΡΠ³ΠΎΠ»ΡΠ½Π°Ρ Π΄Π΅ΠΊΠ°ΡΡΠΎΠ²Π° ΡΠΈΡΡΠ΅ΠΌΠ° ΠΊΠΎΠΎΡΠ΄ΠΈΠ½Π°Ρ O x y Π½Π° ΠΏΠ»ΠΎΡΠΊΠΎΡΡΠΈ Π΅ΡΠ»ΠΈ ΠΌΡ ΠΎΡΠ»ΠΎΠΆΠΈΠΌ ΠΎΡ Π½Π°ΡΠ°Π»Π° ΠΊΠΎΠΎΡΠ΄ΠΈΠ½Π°Ρ Π²Π΅ΠΊΡΠΎΡΡ i β ΠΈ j β , Π½Π°ΠΏΡΠ°Π²Π»Π΅Π½ΠΈΠ΅ ΠΊΠΎΡΠΎΡΡΡ
ΡΠΎΠΎΡΠ²Π΅ΡΡΡΠ²Π΅Π½Π½ΠΎ ΡΠΎΠ²ΠΏΠ°Π΄Π΅Ρ Ρ ΠΏΠΎΠ»ΠΎΠΆΠΈΡΠ΅Π»ΡΠ½ΡΠΌΠΈ Π½Π°ΠΏΡΠ°Π²Π»Π΅Π½ΠΈΡΠΌΠΈ ΠΎΡΠ΅ΠΉ O x ΠΈ O y , ΠΈ ΠΈΡ
Π΄Π»ΠΈΠ½Π° Π±ΡΠ΄Π΅Ρ ΡΠ°Π²Π½Π° ΡΡΠ»ΠΎΠ²Π½ΠΎΠΉ Π΅Π΄ΠΈΠ½ΠΈΡΠ΅, ΠΌΡ ΠΏΠΎΠ»ΡΡΠΈΠΌ ΠΊΠΎΠΎΡΠ΄ΠΈΠ½Π°ΡΠ½ΡΠ΅ Π²Π΅ΠΊΡΠΎΡΡ.
Π’ΠΎ Π΅ΡΡΡ Π² Π΄Π°Π½Π½ΠΎΠΌ ΡΠ»ΡΡΠ°Π΅ i β ΠΈ j β ΡΠ²Π»ΡΡΡΡΡ ΠΊΠΎΠΎΡΠ΄ΠΈΠ½Π°ΡΠ½ΡΠΌΠΈ Π²Π΅ΠΊΡΠΎΡΠ°ΠΌΠΈ.
ΠΠΎΠΎΡΠ΄ΠΈΠ½Π°ΡΠ½ΡΠ΅ Π²Π΅ΠΊΡΠΎΡΡ
ΠΠΏΡΠ΅Π΄Π΅Π»Π΅Π½ΠΈΠ΅ 2ΠΠ΅ΠΊΡΠΎΡΡ i β ΠΈ j β Π½Π°Π·ΡΠ²Π°ΡΡΡΡ ΠΊΠΎΠΎΡΠ΄ΠΈΠ½Π°ΡΠ½ΡΠΌΠΈ Π²Π΅ΠΊΡΠΎΡΠ°ΠΌΠΈ Π΄Π»Ρ Π·Π°Π΄Π°Π½Π½ΠΎΠΉ ΡΠΈΡΡΠ΅ΠΌΡ ΠΊΠΎΠΎΡΠ΄ΠΈΠ½Π°Ρ.
ΠΡΠΈΠΌΠ΅Ρ 2
ΠΡΠΊΠ»Π°Π΄ΡΠ²Π°Π΅ΠΌ ΠΎΡ Π½Π°ΡΠ°Π»Π° ΠΊΠΎΠΎΡΠ΄ΠΈΠ½Π°Ρ ΠΏΡΠΎΠΈΠ·Π²ΠΎΠ»ΡΠ½ΡΠΉ Π²Π΅ΠΊΡΠΎΡ a β . ΠΠΏΠΈΡΠ°ΡΡΡ Π½Π° Π³Π΅ΠΎΠΌΠ΅ΡΡΠΈΡΠ΅ΡΠΊΠΎΠ΅ ΠΎΠΏΡΠ΅Π΄Π΅Π»Π΅Π½ΠΈΠ΅ ΠΎΠΏΠ΅ΡΠ°ΡΠΈΠΉ Π½Π°Π΄ Π²Π΅ΠΊΡΠΎΡΠ°ΠΌΠΈ, Π²Π΅ΠΊΡΠΎΡ a β ΠΌΠΎΠΆΠ΅Ρ Π±ΡΡΡ ΠΏΡΠ΅Π΄ΡΡΠ°Π²Π»Π΅Π½ Π² Π²ΠΈΠ΄Π΅ a β = a x Β· i β + a y Β· j β , Π³Π΄Π΅ ΠΊΠΎΡΡΡΠΈΡΠΈΠ΅Π½ΡΡ a x ΠΈ a y — Π΅Π΄ΠΈΠ½ΡΡΠ²Π΅Π½Π½ΡΠ΅ Π² ΡΠ²ΠΎΠ΅ΠΌ ΡΠΎΠ΄Π΅, ΠΈΡ Π΅Π΄ΠΈΠ½ΡΡΠ²Π΅Π½Π½ΠΎΡΡΡ Π΄ΠΎΡΡΠ°ΡΠΎΡΠ½ΠΎ ΠΏΡΠΎΡΡΠΎ Π΄ΠΎΠΊΠ°Π·Π°ΡΡ ΠΌΠ΅ΡΠΎΠ΄ΠΎΠΌ ΠΎΡ ΠΏΡΠΎΡΠΈΠ²Π½ΠΎΠ³ΠΎ.
Π Π°Π·Π»ΠΎΠΆΠ΅Π½ΠΈΠ΅ Π²Π΅ΠΊΡΠΎΡΠ°
ΠΠΏΡΠ΅Π΄Π΅Π»Π΅Π½ΠΈΠ΅ 3Π Π°Π·Π»ΠΎΠΆΠ΅Π½ΠΈΠ΅ΠΌ Π²Π΅ΠΊΡΠΎΡΠ° a β ΠΏΠΎ ΠΊΠΎΠΎΡΠ΄ΠΈΠ½Π°ΡΠ½ΡΠΌ Π²Π΅ΠΊΡΠΎΡΠ°ΠΌ i β ΠΈ j β Π½Π° ΠΏΠ»ΠΎΡΠΊΠΎΡΡΠΈ Π½Π°Π·ΡΠ²Π°Π΅ΡΡΡ ΠΏΡΠ΅Π΄ΡΡΠ°Π²Π»Π΅Π½ΠΈΠ΅ Π²ΠΈΠ΄Π° a β = a x Β· i β + a y Β· j β .
ΠΠΏΡΠ΅Π΄Π΅Π»Π΅Π½ΠΈΠ΅ 4
ΠΠΎΡΡΡΠΈΡΠΈΠ΅Π½ΡΡ a x ΠΈ a y Π½Π°Π·ΡΠ²Π°ΡΡΡΡ ΠΊΠΎΠΎΡΠ΄ΠΈΠ½Π°ΡΠ°ΠΌΠΈ Π²Π΅ΠΊΡΠΎΡΠ° Π² Π΄Π°Π½Π½ΠΎΠΉ ΡΠΈΡΡΠ΅ΠΌΠ΅ ΠΊΠΎΠΎΡΠ΄ΠΈΠ½Π°Ρ Π½Π° ΠΏΠ»ΠΎΡΠΊΠΎΡΡΠΈ.
ΠΠΎΠΎΡΠ΄ΠΈΠ½Π°ΡΡ Π²Π΅ΠΊΡΠΎΡΠ° Π² Π΄Π°Π½Π½ΠΎΠΉ ΡΠΈΡΡΠ΅ΠΌΠ΅ ΠΊΠΎΠΎΡΠ΄ΠΈΠ½Π°Ρ ΠΏΡΠΈΠ½ΡΡΠΎ Π·Π°ΠΏΠΈΡΡΠ²Π°ΡΡ Π² ΠΊΡΡΠ³Π»ΡΡ ΡΠΊΠΎΠ±ΠΊΠ°Ρ , ΡΠ΅ΡΠ΅Π· Π·Π°ΠΏΡΡΡΡ, ΠΏΡΠΈ ΡΡΠΎΠΌ Π·Π°Π΄Π°Π½Π½ΡΠ΅ ΠΊΠΎΠΎΡΠ΄ΠΈΠ½Π°ΡΡ ΡΠ»Π΅Π΄ΡΠ΅Ρ ΠΎΡΠ΄Π΅Π»ΡΡΡ ΠΎΡ ΠΎΠ±ΠΎΠ·Π½Π°ΡΠ΅Π½ΠΈΡ Π²Π΅ΠΊΡΠΎΡΠ° Π·Π½Π°ΠΊΠΎΠΌ ΡΠ°Π²Π΅Π½ΡΡΠ²Π°. Π ΠΏΡΠΈΠΌΠ΅ΡΡ, Π·Π°ΠΏΠΈΡΡ a β = (2 ; — 3) ΠΎΠ·Π½Π°ΡΠ°Π΅Ρ, ΡΡΠΎ Π²Π΅ΠΊΡΠΎΡ a β ΠΈΠΌΠ΅Π΅Ρ ΠΊΠΎΠΎΡΠ΄ΠΈΠ½Π°ΡΡ (2 ; — 3) Π² Π΄Π°Π½Π½ΠΎΠΉ ΡΠΈΡΡΠ΅ΠΌΠ΅ ΠΊΠΎΠΎΡΠ΄ΠΈΠ½Π°Ρ ΠΈ ΠΌΠΎΠΆΠ΅Ρ Π±ΡΡΡ ΠΏΡΠ΅Π΄ΡΡΠ°Π²Π»Π΅Π½ Π² Π²ΠΈΠ΄Π΅ ΡΠ°Π·Π»ΠΎΠΆΠ΅Π½ΠΈΡ ΠΏΠΎ ΠΊΠΎΠΎΡΠ΄ΠΈΠ½Π°ΡΠ½ΡΠΌ Π²Π΅ΠΊΡΠΎΡΠ°ΠΌ i β ΠΈ j β ΠΊΠ°ΠΊ a β = 2 Β· i β — 3 Β· j β .
ΠΠ°ΠΌΠ΅ΡΠ°Π½ΠΈΠ΅
Π‘Π»Π΅Π΄ΡΠ΅Ρ ΠΎΠ±ΡΠ°ΡΠΈΡΡ Π²Π½ΠΈΠΌΠ°Π½ΠΈΠ΅, ΡΡΠΎ ΠΏΠΎΡΡΠ΄ΠΎΠΊ Π·Π°ΠΏΠΈΡΠΈ ΠΊΠΎΠΎΡΠ΄ΠΈΠ½Π°Ρ, ΠΈΠΌΠ΅Π΅Ρ Π²Π°ΠΆΠ½ΠΎΠ΅ Π·Π½Π°ΡΠ΅Π½ΠΈΠ΅, Π΅ΡΠ»ΠΈ Π²Ρ Π·Π°ΠΏΠΈΡΠΈΡΠ΅ ΠΊΠΎΠΎΡΠ΄ΠΈΠ½Π°ΡΡ Π²Π΅ΠΊΡΠΎΡΠ° Π² Π΄ΡΡΠ³ΠΎΠΌ ΠΏΠΎΡΡΠ΄ΠΊΠ΅, Π²Ρ ΠΏΠΎΠ»ΡΡΠΈΡΠ΅ ΡΠΎΠ²Π΅ΡΡΠ΅Π½Π½ΠΎ Π΄ΡΡΠ³ΠΎΠΉ Π²Π΅ΠΊΡΠΎΡ.
ΠΠΏΠΈΡΠ°ΡΡΡ Π½Π° ΠΎΠΏΡΠ΅Π΄Π΅Π»Π΅Π½ΠΈΡ ΠΊΠΎΠΎΡΠ΄ΠΈΠ½Π°Ρ Π²Π΅ΠΊΡΠΎΡΠ° ΠΈ ΠΈΡ ΡΠ°Π·Π»ΠΎΠΆΠ΅Π½ΠΈΡ ΡΡΠ°Π½ΠΎΠ²ΠΈΡΡΡ ΠΎΡΠ΅Π²ΠΈΠ΄Π½ΡΠΌ, ΡΡΠΎ Π΅Π΄ΠΈΠ½ΠΈΡΠ½ΡΠ΅ Π²Π΅ΠΊΡΠΎΡΡ i β ΠΈ j β ΠΈΠΌΠ΅ΡΡ ΠΊΠΎΠΎΡΠ΄ΠΈΠ½Π°ΡΡ (1 ; 0) ΠΈ (0 ; 1) ΡΠΎΠΎΡΠ²Π΅ΡΡΡΠ²Π΅Π½Π½ΠΎ, ΠΈ ΠΎΠ½ΠΈ ΠΌΠΎΠ³ΡΡ Π±ΡΡΡ ΠΏΡΠ΅Π΄ΡΡΠ°Π²Π»Π΅Π½Ρ Π² Π²ΠΈΠ΄Π΅ ΡΠ»Π΅Π΄ΡΡΡΠΈΡ ΡΠ°Π·Π»ΠΎΠΆΠ΅Π½ΠΈΠΉ i β = 1 Β· i β + 0 Β· j β ; j β = 0 Β· i β + 1 Β· j β .
Π’Π°ΠΊΠΆΠ΅ ΠΈΠΌΠ΅Π΅Ρ ΠΌΠ΅ΡΡΠΎ Π±ΡΡΡ Π½ΡΠ»Π΅Π²ΠΎΠΉ Π²Π΅ΠΊΡΠΎΡ 0 β Ρ ΠΊΠΎΠΎΡΠ΄ΠΈΠ½Π°ΡΠ°ΠΌΠΈ (0 ; 0) ΠΈ ΡΠ°Π·Π»ΠΎΠΆΠ΅Π½ΠΈΠ΅ΠΌ 0 β = 0 Β· i β + 0 Β· j β .
Π Π°Π²Π½ΡΠ΅ ΠΈ ΠΏΡΠΎΡΠΈΠ²ΠΎΠΏΠΎΠ»ΠΎΠΆΠ½ΡΠ΅ Π²Π΅ΠΊΡΠΎΡΡ
ΠΠΏΡΠ΅Π΄Π΅Π»Π΅Π½ΠΈΠ΅ 5ΠΠ΅ΠΊΡΠΎΡΡ a β ΠΈ b β ΡΠ°Π²Π½Ρ ΡΠΎΠ³Π΄Π°, ΠΊΠΎΠ³Π΄Π° ΠΈΡ ΡΠΎΠΎΡΠ²Π΅ΡΡΡΠ²ΡΡΡΠΈΠ΅ ΠΊΠΎΠΎΡΠ΄ΠΈΠ½Π°ΡΡ ΡΠ°Π²Π½Ρ.
ΠΠΏΡΠ΅Π΄Π΅Π»Π΅Π½ΠΈΠ΅ 6
ΠΡΠΎΡΠΈΠ²ΠΎΠΏΠΎΠ»ΠΎΠΆΠ½ΡΠΌ Π²Π΅ΠΊΡΠΎΡΠΎΠΌ Π½Π°Π·ΡΠ²Π°Π΅ΡΡΡ Π²Π΅ΠΊΡΠΎΡ ΠΏΡΠΎΡΠΈΠ²ΠΎΠΏΠΎΠ»ΠΎΠΆΠ½ΡΠΉ Π΄Π°Π½Π½ΠΎΠΌΡ.
ΠΡΡΡΠ΄Π° ΡΠ»Π΅Π΄ΡΠ΅Ρ, ΡΡΠΎ ΠΊΠΎΠΎΡΠ΄ΠΈΠ½Π°ΡΡ ΡΠ°ΠΊΠΎΠ³ΠΎ Π²Π΅ΠΊΡΠΎΡΠ° Π±ΡΠ΄ΡΡ ΠΏΡΠΎΡΠΈΠ²ΠΎΠΏΠΎΠ»ΠΎΠΆΠ½Ρ ΠΊΠΎΠΎΡΠ΄ΠΈΠ½Π°ΡΠ°ΠΌ Π΄Π°Π½Π½ΠΎΠ³ΠΎ Π²Π΅ΠΊΡΠΎΡΠ°, ΡΠΎ Π΅ΡΡΡ, — a β = (- a x ; — a y) .
ΠΡΠ΅ Π²ΡΡΠ΅ΠΈΠ·Π»ΠΎΠΆΠ΅Π½Π½ΠΎΠ΅ ΠΌΠΎΠΆΠ½ΠΎ Π°Π½Π°Π»ΠΎΠ³ΠΈΡΠ½ΠΎ ΠΎΠΏΡΠ΅Π΄Π΅Π»ΠΈΡΡ ΠΈ Π΄Π»Ρ ΠΏΡΡΠΌΠΎΡΠ³ΠΎΠ»ΡΠ½ΠΎΠΉ ΡΠΈΡΡΠ΅ΠΌΡ ΠΊΠΎΠΎΡΠ΄ΠΈΠ½Π°Ρ, Π·Π°Π΄Π°Π½Π½ΠΎΠΉ Π² ΡΡΠ΅Ρ ΠΌΠ΅ΡΠ½ΠΎΠΌ ΠΏΡΠΎΡΡΡΠ°Π½ΡΡΠ²Π΅. Π ΡΠ°ΠΊΠΎΠΉ ΡΠΈΡΡΠ΅ΠΌΠ΅ ΠΊΠΎΠΎΡΠ΄ΠΈΠ½Π°Ρ ΠΈΠΌΠ΅Π΅Ρ ΠΌΠ΅ΡΡΠΎ Π±ΡΡΡ ΡΡΠΎΠΉΠΊΠ° ΠΊΠΎΠΎΡΠ΄ΠΈΠ½Π°ΡΠ½ΡΡ Π²Π΅ΠΊΡΠΎΡΠΎΠ² i β , j β , k β , Π° ΠΏΡΠΎΠΈΠ·Π²ΠΎΠ»ΡΠ½ΡΠΉ Π²Π΅ΠΊΡΠΎΡ a β ΡΠ°ΡΠΊΠ»Π°Π΄ΡΠ²Π°Π΅ΡΡΡ Π½Π΅ ΠΏΠΎ Π΄Π²ΡΠΌ, Π° ΡΠΆΠ΅ ΠΏΠΎ ΡΡΠ΅ΠΌ ΠΊΠΎΠΎΡΠ΄ΠΈΠ½Π°ΡΠ°ΠΌ, ΠΏΡΠΈΡΠ΅ΠΌ Π΅Π΄ΠΈΠ½ΡΡΠ²Π΅Π½Π½ΡΠΌ ΠΎΠ±ΡΠ°Π·ΠΎΠΌ ΠΈ ΠΈΠΌΠ΅Π΅Ρ Π²ΠΈΠ΄ a β = a x Β· i β + a y Β· j β + a z Β· k β , Π° ΠΊΠΎΡΡΡΠΈΡΠΈΠ΅Π½ΡΡ ΡΡΠΎΠ³ΠΎ ΡΠ°Π·Π»ΠΎΠΆΠ΅Π½ΠΈΡ (a x ; a y ; a z) Π½Π°Π·ΡΠ²Π°ΡΡΡΡ ΠΊΠΎΠΎΡΠ΄ΠΈΠ½Π°ΡΠ°ΠΌΠΈ Π²Π΅ΠΊΡΠΎΡΠ° Π² Π΄Π°Π½Π½ΠΎΠΉ (ΡΡΠ΅Ρ ΠΌΠ΅ΡΠ½ΠΎΠΉ) ΡΠΈΡΡΠ΅ΠΌΠ΅ ΠΊΠΎΠΎΡΠ΄ΠΈΠ½Π°Ρ.
Π‘Π»Π΅Π΄ΠΎΠ²Π°ΡΠ΅Π»ΡΠ½ΠΎ, ΠΊΠΎΠΎΡΠ΄ΠΈΠ½Π°ΡΠ½ΡΠ΅ Π²Π΅ΠΊΡΠΎΡΡ Π² ΡΡΠ΅Ρ
ΠΌΠ΅ΡΠ½ΠΎΠΌ ΠΏΡΠΎΡΡΡΠ°Π½ΡΡΠ²Π΅ ΠΏΡΠΈΠ½ΠΈΠΌΠ°ΡΡ ΡΠ°ΠΊΠΆΠ΅ Π·Π½Π°ΡΠ΅Π½ΠΈΠ΅ 1 ΠΈ ΠΈΠΌΠ΅ΡΡ ΠΊΠΎΠΎΡΠ΄ΠΈΠ½Π°ΡΡ i β = (1 ; 0 ; 0) , j β = (0 ; 1 ; 0) , k β = (0 ; 0 ; 1) , ΠΊΠΎΠΎΡΠ΄ΠΈΠ½Π°ΡΡ Π½ΡΠ»Π΅Π²ΠΎΠ³ΠΎ Π²Π΅ΠΊΡΠΎΡΠ° ΡΠ°ΠΊΠΆΠ΅ ΡΠ°Π²Π½Ρ Π½ΡΠ»Ρ 0 β = (0 ; 0 ; 0) , ΠΈ Π² ΡΠ°ΠΊΠΎΠΌ ΡΠ»ΡΡΠ°Π΅ Π΄Π²Π° Π²Π΅ΠΊΡΠΎΡΠ° Π±ΡΠ΄ΡΡ ΡΡΠΈΡΠ°ΡΡΡΡ ΡΠ°Π²Π½ΡΠΌΠΈ, Π΅ΡΠ»ΠΈ Π²ΡΠ΅ ΡΡΠΈ ΡΠΎΠΎΡΠ²Π΅ΡΡΡΠ²ΡΡΡΠΈΠ΅ ΠΊΠΎΠΎΡΠ΄ΠΈΠ½Π°ΡΡ Π²Π΅ΠΊΡΠΎΡΠΎΠ² ΠΌΠ΅ΠΆΠ΄Ρ ΡΠΎΠ±ΠΎΠΉ ΡΠ°Π²Π½Ρ a β = b β β a x = b x , a y = b y , a z = b z , ΠΈ ΠΊΠΎΠΎΡΠ΄ΠΈΠ½Π°ΡΡ ΠΏΡΠΎΡΠΈΠ²ΠΎΠΏΠΎΠ»ΠΎΠΆΠ½ΠΎΠ³ΠΎ Π²Π΅ΠΊΡΠΎΡΠ° a β ΠΏΡΠΎΡΠΈΠ²ΠΎΠΏΠΎΠ»ΠΎΠΆΠ½Ρ ΡΠΎΠΎΡΠ²Π΅ΡΡΡΠ²ΡΡΡΠΈΠΌ ΠΊΠΎΠΎΡΠ΄ΠΈΠ½Π°ΡΠ°ΠΌ Π²Π΅ΠΊΡΠΎΡΠ° a β , ΡΠΎ Π΅ΡΡΡ, — a β = (- a x ; — a y ; — a z) .
Π§ΡΠΎΠ±Ρ Π²Π²Π΅ΡΡΠΈ Π΄Π°Π½Π½ΠΎΠ΅ ΠΎΠΏΡΠ΅Π΄Π΅Π»Π΅Π½ΠΈΠ΅, ΡΡΠ΅Π±ΡΠ΅ΡΡΡ ΠΏΠΎΠΊΠ°Π·Π°ΡΡ Π² Π΄Π°Π½Π½ΠΎΠΉ ΡΠΈΡΡΠ΅ΠΌΠ΅ ΠΊΠΎΠΎΡΠ΄ΠΈΠ½Π°Ρ ΡΠ²ΡΠ·Ρ ΠΊΠΎΠΎΡΠ΄ΠΈΠ½Π°Ρ ΡΠΎΡΠΊΠΈ ΠΈ ΠΊΠΎΠΎΡΠ΄ΠΈΠ½Π°Ρ Π²Π΅ΠΊΡΠΎΡΠ°.
ΠΡΡΡΡ Π½Π°ΠΌ Π΄Π°Π½Π° Π½Π΅ΠΊΠΎΡΠΎΡΠ°Ρ ΠΏΡΡΠΌΠΎΡΠ³ΠΎΠ»ΡΠ½Π°Ρ Π΄Π΅ΠΊΠ°ΡΡΠΎΠ²Π° ΡΠΈΡΡΠ΅ΠΌΠ° ΠΊΠΎΠΎΡΠ΄ΠΈΠ½Π°Ρ O x y ΠΈ Π½Π° Π½Π΅ΠΉ Π·Π°Π΄Π°Π½Π° ΠΏΡΠΎΠΈΠ·Π²ΠΎΠ»ΡΠ½Π°Ρ ΡΠΎΡΠΊΠ° M Ρ ΠΊΠΎΠΎΡΠ΄ΠΈΠ½Π°ΡΠ°ΠΌΠΈ M (x M ; y M) .
ΠΠΏΡΠ΅Π΄Π΅Π»Π΅Π½ΠΈΠ΅ 7
ΠΠ΅ΠΊΡΠΎΡ O M β Π½Π°Π·ΡΠ²Π°Π΅ΡΡΡ ΡΠ°Π΄ΠΈΡΡ-Π²Π΅ΠΊΡΠΎΡΠΎΠΌ ΡΠΎΡΠΊΠΈ M .
ΠΠΏΡΠ΅Π΄Π΅Π»ΠΈΠΌ, ΠΊΠ°ΠΊΠΈΠ΅ ΠΊΠΎΠΎΡΠ΄ΠΈΠ½Π°ΡΡ Π² Π΄Π°Π½Π½ΠΎΠΉ ΡΠΈΡΡΠ΅ΠΌΠ΅ ΠΊΠΎΠΎΡΠ΄ΠΈΠ½Π°Ρ ΠΈΠΌΠ΅Π΅Ρ ΡΠ°Π΄ΠΈΡΡ-Π²Π΅ΠΊΡΠΎΡ ΡΠΎΡΠΊΠΈ
ΠΠ΅ΠΊΡΠΎΡ O M β ΠΈΠΌΠ΅Π΅Ρ Π²ΠΈΠ΄ ΡΡΠΌΠΌΡ O M β = O M x β + O M y β = x M Β· i β + y M Β· j β , Π³Π΄Π΅ ΡΠΎΡΠΊΠΈ M x ΠΈ M y ΡΡΠΎ ΠΏΡΠΎΠ΅ΠΊΡΠΈΠΈ ΡΠΎΡΠΊΠΈ Π Π½Π° ΠΊΠΎΠΎΡΠ΄ΠΈΠ½Π°ΡΠ½ΡΠ΅ ΠΏΡΡΠΌΡΠ΅ Ox ΠΈ Oy ΡΠΎΠΎΡΠ²Π΅ΡΡΡΠ²Π΅Π½Π½ΠΎ (Π΄Π°Π½Π½ΡΠ΅ ΡΠ°ΡΡΡΠΆΠ΄Π΅Π½ΠΈΡ ΡΠ»Π΅Π΄ΡΡΡ ΠΈΠ· ΠΎΠΏΡΠ΅Π΄Π΅Π»Π΅Π½ΠΈΡ ΠΏΡΠΎΠ΅ΠΊΡΠΈΡ ΡΠΎΡΠΊΠΈ Π½Π° ΠΏΡΡΠΌΡΡ), Π° i β ΠΈ j β — ΠΊΠΎΠΎΡΠ΄ΠΈΠ½Π°ΡΠ½ΡΠ΅ Π²Π΅ΠΊΡΠΎΡΡ, ΡΠ»Π΅Π΄ΠΎΠ²Π°ΡΠ΅Π»ΡΠ½ΠΎ, Π²Π΅ΠΊΡΠΎΡ O M β ΠΈΠΌΠ΅Π΅Ρ ΠΊΠΎΠΎΡΠ΄ΠΈΠ½Π°ΡΡ (x M ; y M) Π² Π΄Π°Π½Π½ΠΎΠΉ ΡΠΈΡΡΠ΅ΠΌΠ΅ ΠΊΠΎΠΎΡΠ΄ΠΈΠ½Π°Ρ.
ΠΠ½Π°ΡΠ΅ Π³ΠΎΠ²ΠΎΡΡ, ΠΊΠΎΠΎΡΠ΄ΠΈΠ½Π°ΡΡ ΡΠ°Π΄ΠΈΡΡ-Π²Π΅ΠΊΡΠΎΡΠ° ΡΠΎΡΠΊΠΈ Π ΡΠ°Π²Π½Ρ ΡΠΎΠΎΡΠ²Π΅ΡΡΡΠ²ΡΡΡΠΈΠΌ ΠΊΠΎΠΎΡΠ΄ΠΈΠ½Π°ΡΠ°ΠΌ ΡΠΎΡΠΊΠΈ Π Π² ΠΏΡΡΠΌΠΎΡΠ³ΠΎΠ»ΡΠ½ΠΎΠΉ Π΄Π΅ΠΊΠ°ΡΡΠΎΠ²ΠΎΠΉ ΡΠΈΡΡΠ΅ΠΌΠ΅ ΠΊΠΎΠΎΡΠ΄ΠΈΠ½Π°Ρ.
ΠΠ½Π°Π»ΠΎΠ³ΠΈΡΠ½ΠΎ Π² ΡΡΠ΅Ρ ΠΌΠ΅ΡΠ½ΠΎΠΌ ΠΏΡΠΎΡΡΡΠ°Π½ΡΡΠ²Π΅ ΡΠ°Π΄ΠΈΡΡ-Π²Π΅ΠΊΡΠΎΡ ΡΠΎΡΠΊΠΈ M (x M ; y M ; z M) ΡΠ°Π·Π»Π°Π³Π°Π΅ΡΡΡ ΠΏΠΎ ΠΊΠΎΠΎΡΠ΄ΠΈΠ½Π°ΡΠ½ΡΠΌ Π²Π΅ΠΊΡΠΎΡΠ°ΠΌ ΠΊΠ°ΠΊ O M β = O M x β + O M y β + O M z β = x M Β· i β + y M Β· j β + z M Β· k β , ΡΠ»Π΅Π΄ΠΎΠ²Π°ΡΠ΅Π»ΡΠ½ΠΎ, O M β = (x M ; y M ; z M) .
ΠΡΠ»ΠΈ Π²Ρ Π·Π°ΠΌΠ΅ΡΠΈΠ»ΠΈ ΠΎΡΠΈΠ±ΠΊΡ Π² ΡΠ΅ΠΊΡΡΠ΅, ΠΏΠΎΠΆΠ°Π»ΡΠΉΡΡΠ°, Π²ΡΠ΄Π΅Π»ΠΈΡΠ΅ Π΅Ρ ΠΈ Π½Π°ΠΆΠΌΠΈΡΠ΅ Ctrl+Enter
ΠΠΎ ΡΠΈΡ
ΠΏΠΎΡ ΡΡΠΈΡΠ°Π»ΠΎΡΡ, ΡΡΠΎ Π²Π΅ΠΊΡΠΎΡΡ
ΡΠ°ΡΡΠΌΠ°ΡΡΠΈΠ²Π°ΡΡΡΡ Π² ΠΏΡΠΎΡΡΡΠ°Π½ΡΡΠ²Π΅. ΠΠ°ΡΠΈΠ½Π°Ρ
Ρ ΡΡΠΎΠ³ΠΎ ΠΌΠΎΠΌΠ΅Π½ΡΠ° Π±ΡΠ΄ΠΈΠΌ ΡΡΠΈΡΠ°ΡΡ, ΡΡΠΎ Π²ΡΠ΅
Π²Π΅ΠΊΡΠΎΡΡ ΡΠ°ΡΡΠΌΠ°ΡΡΠΈΠ²Π°ΡΡΡΡ Π½Π° ΠΏΠ»ΠΎΡΠΊΠΎΡΡΠΈ.
ΠΡΠ΄Π΅ΠΌ ΡΠ°ΠΊΠΆΠ΅ ΠΏΠΎΠ»Π°Π³Π°ΡΡ, ΡΡΠΎ Π½Π° ΠΏΠ»ΠΎΡΠΊΠΎΡΡΠΈ
Π·Π°Π΄Π°Π½Π° ΠΠ΅ΠΊΠ°ΡΡΠΎΠ²Π° ΡΠΈΡΡΠ΅ΠΌΠ° ΠΊΠΎΠΎΡΠ΄ΠΈΠ½Π°Ρ
(Π΄Π°ΠΆΠ΅ Π΅ΡΠ»ΠΈ ΠΎΠ± ΡΡΠΎΠΌ Π½Π΅ Π³ΠΎΠ²ΠΎΡΠΈΡΡΡ),
ΠΏΡΠ΅Π΄ΡΡΠ°Π²Π»ΡΡΡΠ°Ρ Π΄Π²Π΅ Π²Π·Π°ΠΈΠΌΠ½ΠΎ ΠΏΠ΅ΡΠΏΠ΅Π½Π΄ΠΈΠΊΡΠ»ΡΡΠ½ΡΠ΅
ΡΠΈΡΠ»ΠΎΠ²ΡΠ΅ ΠΎΡΠΈ β Π³ΠΎΡΠΈΠ·ΠΎΠ½ΡΠ°Π»ΡΠ½Π°Ρ ΠΎΡΡ
ΠΈ
Π²Π΅ΡΡΠΈΠΊΠ°Π»ΡΠ½Π°Ρ ΠΎΡΡ.
Π’ΠΎΠ³Π΄Π° ΠΊΠ°ΠΆΠ΄ΠΎΠΉ ΡΠΎΡΠΊΠ΅
Π½Π°
ΠΏΠ»ΠΎΡΠΊΠΎΡΡΠΈ ΡΡΠ°Π²ΠΈΡΡΡ Π² ΡΠΎΠΎΡΠ²Π΅ΡΡΡΠ²ΠΈΠ΅ ΠΏΠ°ΡΠ°
ΡΠΈΡΠ΅Π»
,
ΠΊΠΎΡΠΎΡΡΠ΅ ΡΠ²Π»ΡΡΡΡΡ Π΅Π΅ ΠΊΠΎΠΎΡΠ΄ΠΈΠ½Π°ΡΠ°ΠΌΠΈ.
ΠΠ±ΡΠ°ΡΠ½ΠΎ, ΠΊΠ°ΠΆΠ΄ΠΎΠΉ ΠΏΠ°ΡΠ΅ ΡΠΈΡΠ΅Π»
ΡΠΎΠΎΡΠ²Π΅ΡΡΡΠ²ΡΠ΅Ρ ΡΠΎΡΠΊΠ° ΠΏΠ»ΠΎΡΠΊΠΎΡΡΠΈ ΡΠ°ΠΊΠ°Ρ,
ΡΡΠΎ ΠΏΠ°ΡΠ° ΡΠΈΡΠ΅Π»
ΡΠ²Π»ΡΡΡΡΡ Π΅Π΅ ΠΊΠΎΠΎΡΠ΄ΠΈΠ½Π°ΡΠ°ΠΌΠΈ.
ΠΠ· ΡΠ»Π΅ΠΌΠ΅Π½ΡΠ°ΡΠ½ΠΎΠΉ Π³Π΅ΠΎΠΌΠ΅ΡΡΠΈΠΈ ΠΈΠ·Π²Π΅ΡΡΠ½ΠΎ,
ΡΡΠΎ Π΅ΡΠ»ΠΈ Π½Π° ΠΏΠ»ΠΎΡΠΊΠΎΡΡΠΈ ΠΈΠΌΠ΅ΡΡΡΡ Π΄Π²Π΅ ΡΠΎΡΠΊΠΈ
ΠΈ
,
ΡΠΎ ΡΠ°ΡΡΡΠΎΡΠ½ΠΈΠ΅
ΠΌΠ΅ΠΆΠ΄Ρ
ΡΡΠΈΠΌΠΈ ΡΠΎΡΠΊΠ°ΠΌΠΈ Π²ΡΡΠ°ΠΆΠ°Π΅ΡΡΡ ΡΠ΅ΡΠ΅Π· ΠΈΡ
ΠΊΠΎΠΎΡΠ΄ΠΈΠ½Π°ΡΡ ΠΏΠΎ ΡΠΎΡΠΌΡΠ»Π΅
ΠΡΡΡΡ Π½Π° ΠΏΠ»ΠΎΡΠΊΠΎΡΡΠΈ Π·Π°Π΄Π°Π½Π° ΠΠ΅ΠΊΠ°ΡΡΠΎΠ²Π°
ΡΠΈΡΡΠ΅ΠΌΠ° ΠΊΠΎΠΎΡΠ΄ΠΈΠ½Π°Ρ.
ΠΡΡ ΠΎΡΠΈ
Π±ΡΠ΄Π΅ΠΌ ΠΎΠ±ΠΎΠ·Π½Π°ΡΠ°ΡΡ ΡΠΈΠΌΠ²ΠΎΠ»ΠΎΠΌ,
Π° ΠΎΡΡ ΠΎΡΠΈΡΠΈΠΌΠ²ΠΎΠ»ΠΎΠΌ.
ΠΡΠΎΠ΅ΠΊΡΠΈΡ ΠΏΡΠΎΠΈΠ·Π²ΠΎΠ»ΡΠ½ΠΎΠ³ΠΎΠ²Π΅ΠΊΡΠΎΡΠ°Π½Π° ΠΎΡΡΠ±ΡΠ΄Π΅ΠΌ ΠΎΠ±ΠΎΠ·Π½Π°ΡΠ°ΡΡ ΡΠΈΠΌΠ²ΠΎΠ»ΠΎΠΌ
,
Π° ΠΏΡΠΎΠ΅ΠΊΡΠΈΡ Π½Π° ΠΎΡΡΡΠΈΠΌΠ²ΠΎΠ»ΠΎΠΌ
.
ΠΡΡΡΡ — ΠΏΡΠΎΠΈΠ·Π²ΠΎΠ»ΡΠ½ΡΠΉ Π²Π΅ΠΊΡΠΎΡ Π½Π° ΠΏΠ»ΠΎΡΠΊΠΎΡΡΠΈ. ΠΠΌΠ΅Π΅Ρ ΠΌΠ΅ΡΡΠΎ ΡΠ»Π΅Π΄ΡΡΡΠ°Ρ ΡΠ΅ΠΎΡΠ΅ΠΌΠ°.
Π’Π΅ΠΎΡΠ΅ΠΌΠ° 22.
ΠΠ»Ρ Π»ΡΠ±ΠΎΠ³ΠΎ Π²Π΅ΠΊΡΠΎΡΠ°
Π½Π° ΠΏΠ»ΠΎΡΠΊΠΎΡΡΠΈ ΡΡΡΠ΅ΡΡΠ²ΡΠ΅Ρ ΠΏΠ°ΡΠ° ΡΠΈΡΠ΅Π»
.
ΠΡΠΈ ΡΡΠΎΠΌ
,
.
ΠΠΎΠΊΠ°Π·Π°ΡΠ΅Π»ΡΡΡΠ²ΠΎ.
ΠΡΡΡΡ Π΄Π°Π½ Π²Π΅ΠΊΡΠΎΡ. ΠΡΠ»ΠΎΠΆΠΈΠΌ Π²Π΅ΠΊΡΠΎΡΠΎΡ Π½Π°ΡΠ°Π»Π° ΠΊΠΎΠΎΡΠ΄ΠΈΠ½Π°Ρ. ΠΠ±ΠΎΠ·Π½Π°ΡΠΈΠΌ ΡΠ΅ΡΠ΅Π·Π²Π΅ΠΊΡΠΎΡ-ΠΏΡΠΎΠ΅ΠΊΡΠΈΡ Π²Π΅ΠΊΡΠΎΡΠ°Π½Π° ΠΎΡΡ, Π° ΡΠ΅ΡΠ΅Π·Π²Π΅ΠΊΡΠΎΡ-ΠΏΡΠΎΠ΅ΠΊΡΠΈΡ Π²Π΅ΠΊΡΠΎΡΠ°Π½Π° ΠΎΡΡ. Π’ΠΎΠ³Π΄Π°, ΠΊΠ°ΠΊ Π²ΠΈΠ΄Π½ΠΎ ΠΈΠ· ΡΠΈΡΡΠ½ΠΊΠ° 21, ΠΈΠΌΠ΅Π΅Ρ ΠΌΠ΅ΡΡΠΎ ΡΠ°Π²Π΅Π½ΡΡΠ²ΠΎ
.
Π‘ΠΎΠ³Π»Π°ΡΠ½ΠΎ ΡΠ΅ΠΎΡΠ΅ΠΌΠ΅ 9,
,
.
ΠΠ±ΠΎΠ·Π½Π°ΡΠΈΠΌ
,
.
Π’ΠΎΠ³Π΄Π° ΠΏΠΎΠ»ΡΡΠ°Π΅ΠΌ
.
ΠΡΠ°ΠΊ, Π΄ΠΎΠΊΠ°Π·Π°Π½ΠΎ, ΡΡΠΎ Π΄Π»Ρ Π»ΡΠ±ΠΎΠ³ΠΎ Π²Π΅ΠΊΡΠΎΡΠ°
ΡΡΡΠ΅ΡΡΠ²ΡΠ΅Ρ ΠΏΠ°ΡΠ° ΡΠΈΡΠ΅Π»
ΡΠ°ΠΊΠΈΡ
, ΡΡΠΎ ΡΠΏΡΠ°Π²Π΅Π΄Π»ΠΈΠ²ΠΎ ΡΠ°Π²Π΅Π½ΡΡΠ²ΠΎ
,
,
.
ΠΡΠΈ Π΄ΡΡΠ³ΠΎΠΌ ΡΠ°ΡΠΏΠΎΠ»ΠΎΠΆΠ΅Π½ΠΈΠΈ Π²Π΅ΠΊΡΠΎΡΠ° ΠΎΡΠ½ΠΎΡΠΈΡΠ΅Π»ΡΠ½ΠΎ ΠΎΡΠ΅ΠΉ Π΄ΠΎΠΊΠ°Π·Π°ΡΠ΅Π»ΡΡΡΠ²ΠΎ Π°Π½Π°Π»ΠΎΠ³ΠΈΡΠ½ΠΎ.
ΠΠΏΡΠ΅Π΄Π΅Π»Π΅Π½ΠΈΠ΅.
ΠΠ°ΡΠ° ΡΠΈΡΠ΅Π»
ΠΈΡΠ°ΠΊΠΈΡ
, ΡΡΠΎ
,
Π½Π°Π·ΡΠ²Π°ΡΡΡΡ ΠΊΠΎΠΎΡΠ΄ΠΈΠ½Π°ΡΠ°ΠΌΠΈ Π²Π΅ΠΊΡΠΎΡΠ°.
Π§ΠΈΡΠ»ΠΎΠ½Π°Π·ΡΠ²Π°Π΅ΡΡΡ ΠΈΠΊΡΠΎΠ²ΠΎΠΉ ΠΊΠΎΠΎΡΠ΄ΠΈΠ½Π°ΡΠΎΠΉ, Π° ΡΠΈΡΠ»ΠΎΠΈΠ³ΡΠ΅ΠΊΠΎΠ²ΠΎΠΉ ΠΊΠΎΠΎΡΠ΄ΠΈΠ½Π°ΡΠΎΠΉ.
ΠΠΏΡΠ΅Π΄Π΅Π»Π΅Π½ΠΈΠ΅.
ΠΠ°ΡΠ° ΠΎΡΡΠΎΠ² ΠΎΡΠ΅ΠΉ ΠΊΠΎΠΎΡΠ΄ΠΈΠ½Π°Ρ
Π½Π°Π·ΡΠ²Π°Π΅ΡΡΡ ΠΎΡΡΠΎΠ½ΠΎΡΠΌΠΈΡΠΎΠ²Π°Π½Π½ΡΠΌ Π±Π°Π·ΠΈΡΠΎΠΌ
Π½Π° ΠΏΠ»ΠΎΡΠΊΠΎΡΡΠΈ. ΠΡΠ΅Π΄ΡΡΠ°Π²Π»Π΅Π½ΠΈΠ΅ Π»ΡΠ±ΠΎΠ³ΠΎ
Π²Π΅ΠΊΡΠΎΡΠ°Π² Π²ΠΈΠ΄Π΅
Π½Π°Π·ΡΠ²Π°Π΅ΡΡΡ ΡΠ°Π·Π»ΠΎΠΆΠ΅Π½ΠΈΠ΅ΠΌ Π²Π΅ΠΊΡΠΎΡΠ°ΠΏΠΎ Π±Π°Π·ΠΈΡΡ
.
ΠΠ΅ΠΏΠΎΡΡΠ΅Π΄ΡΡΠ²Π΅Π½Π½ΠΎ ΠΈΠ· ΠΎΠΏΡΠ΅Π΄Π΅Π»Π΅Π½ΠΈΡ ΠΊΠΎΠΎΡΠ΄ΠΈΠ½Π°Ρ Π²Π΅ΠΊΡΠΎΡΠ° ΡΠ»Π΅Π΄ΡΠ΅Ρ, ΡΡΠΎ Π΅ΡΠ»ΠΈ ΠΊΠΎΠΎΡΠ΄ΠΈΠ½Π°ΡΡ Π²Π΅ΠΊΡΠΎΡΠΎΠ² ΡΠ°Π²Π½Ρ, ΡΠΎ ΡΠ°Π²Π½Ρ ΠΈ ΡΠ°ΠΌΠΈ Π²Π΅ΠΊΡΠΎΡΡ. Π‘ΠΏΡΠ°Π²Π΅Π΄Π»ΠΈΠ²ΠΎ ΡΠ°ΠΊΠΆΠ΅ ΠΈ ΠΎΠ±ΡΠ°ΡΠ½ΠΎΠ΅ ΡΡΠ²Π΅ΡΠΆΠ΄Π΅Π½ΠΈΠ΅.
Π’Π΅ΠΎΡΠ΅ΠΌΠ°.
Π Π°Π²Π½ΡΠ΅ Π²Π΅ΠΊΡΠΎΡΡ ΠΈΠΌΠ΅ΡΡ ΡΠ°Π²Π½ΡΠ΅ ΠΊΠΎΠΎΡΠ΄ΠΈΠ½Π°ΡΡ.
ΠΠΎΠΊΠ°Π·Π°ΡΠ΅Π»ΡΡΡΠ²ΠΎ.
,
ΠΈ
.
ΠΠΎΠΊΠ°ΠΆΠ΅ΠΌ, ΡΡΠΎ
,
.
ΠΠ· ΡΠ°Π²Π΅Π½ΡΡΠ²Π° Π²Π΅ΠΊΡΠΎΡΠΎΠ² ΡΠ»Π΅Π΄ΡΠ΅Ρ, ΡΡΠΎ
.
ΠΠΎΠΏΡΡΡΠΈΠΌ, ΡΡΠΎ
,
Π°
.
Π’ΠΎΠ³Π΄Π°
ΠΈ Π·Π½Π°ΡΠΈΡ
,
ΡΡΠΎ Π½Π΅ Π²Π΅ΡΠ½ΠΎ. ΠΠ½Π°Π»ΠΎΠ³ΠΈΡΠ½ΠΎ, Π΅ΡΠ»ΠΈ
,
Π½ΠΎ
,
ΡΠΎ
.
ΠΡΡΡΠ΄Π°
,
ΡΡΠΎ Π½Π΅ Π²Π΅ΡΠ½ΠΎ. ΠΠ°ΠΊΠΎΠ½Π΅Ρ, Π΅ΡΠ»ΠΈ Π΄ΠΎΠΏΡΡΡΠΈΡΡ,
ΡΡΠΎ
ΠΈ
,
ΡΠΎ ΠΏΠΎΠ»ΡΡΠ°Π΅ΠΌ, ΡΡΠΎ
.
ΠΡΠΎ ΠΎΠ·Π½Π°ΡΠ°Π΅Ρ, ΡΡΠΎ Π²Π΅ΠΊΡΠΎΡΡ
ΠΈΠΊΠΎΠ»Π»ΠΈΠ½Π΅Π°ΡΠ΅Ρ. ΠΠΎ ΡΡΠΎ Π½Π΅ Π²Π΅ΡΠ½ΠΎ, ΡΠ°ΠΊ ΠΊΠ°ΠΊ
ΠΎΠ½ΠΈ ΠΏΠ΅ΡΠΏΠ΅Π½Π΄ΠΈΠΊΡΠ»ΡΡΠ½Ρ. Π‘Π»Π΅Π΄ΠΎΠ²Π°ΡΠ΅Π»ΡΠ½ΠΎ,
ΠΎΡΡΠ°Π΅ΡΡΡ, ΡΡΠΎ
,
,
ΡΡΠΎ ΠΈ ΡΡΠ΅Π±ΠΎΠ²Π°Π»ΠΎΡΡ Π΄ΠΎΠΊΠ°Π·Π°ΡΡ.
Π’Π°ΠΊΠΈΠΌ ΠΎΠ±ΡΠ°Π·ΠΎΠΌ, ΠΊΠΎΠΎΡΠ΄ΠΈΠ½Π°ΡΡ Π²Π΅ΠΊΡΠΎΡΠ°
ΠΏΠΎΠ»Π½ΠΎΡΡΡΡ ΠΎΠΏΡΠ΅Π΄Π΅Π»ΡΡΡ ΡΠ°ΠΌ Π²Π΅ΠΊΡΠΎΡ. ΠΠ½Π°Ρ
ΠΊΠΎΠΎΡΠ΄ΠΈΠ½Π°ΡΡ
ΠΈΠ²Π΅ΠΊΡΠΎΡΠ°ΠΌΠΎΠΆΠ½ΠΎ ΠΏΠΎΡΡΡΠΎΠΈΡΡ ΡΠ°ΠΌ Π²Π΅ΠΊΡΠΎΡ, ΠΏΠΎΡΡΡΠΎΠΈΠ² Π²Π΅ΠΊΡΠΎΡΡ
ΠΈ
ΠΈ ΡΠ»ΠΎΠΆΠΈΠ² ΠΈΡ
. ΠΠΎΡΡΠΎΠΌΡ ΡΠ°ΡΡΠΎ ΡΠ°ΠΌ Π²Π΅ΠΊΡΠΎΡΠΎΠ±ΠΎΠ·Π½Π°ΡΠ°ΡΡ Π² Π²ΠΈΠ΄Π΅ ΠΏΠ°ΡΡ Π΅Π³ΠΎ ΠΊΠΎΠΎΡΠ΄ΠΈΠ½Π°Ρ ΠΈ
ΠΏΠΈΡΡΡ
.
Π’Π°ΠΊΠ°Ρ Π·Π°ΠΏΠΈΡΡ ΠΎΠ·Π½Π°ΡΠ°Π΅Ρ, ΡΡΠΎ
.
ΠΠ΅ΠΏΠΎΡΡΠ΅Π΄ΡΡΠ²Π΅Π½Π½ΠΎ ΠΈΠ· ΠΎΠΏΡΠ΅Π΄Π΅Π»Π΅Π½ΠΈΡ ΠΊΠΎΠΎΡΠ΄ΠΈΠ½Π°Ρ Π²Π΅ΠΊΡΠΎΡΠ° ΡΠ»Π΅Π΄ΡΠ΅Ρ ΡΠ»Π΅Π΄ΡΡΡΠ°Ρ ΡΠ΅ΠΎΡΠ΅ΠΌΠ°.
Π’Π΅ΠΎΡΠ΅ΠΌΠ°.
ΠΡΠΈ ΡΠ»ΠΎΠΆΠ΅Π½ΠΈΠΈ Π²Π΅ΠΊΡΠΎΡΠΎΠ² ΠΈΡ ΠΊΠΎΠΎΡΠ΄ΠΈΠ½Π°ΡΡ ΡΠΊΠ»Π°Π΄ΡΠ²Π°ΡΡΡΡ Π° ΠΏΡΠΈ ΡΠΌΠ½ΠΎΠΆΠ΅Π½ΠΈΠΈ Π²Π΅ΠΊΡΠΎΡΠ° Π½Π° ΡΠΈΡΠ»ΠΎ Π΅Π³ΠΎ ΠΊΠΎΠΎΡΠ΄ΠΈΠ½Π°ΡΡ ΡΠΌΠ½ΠΎΠΆΠ°ΡΡΡΡ Π½Π° ΡΡΠΎ ΡΠΈΡΠ»ΠΎ. ΠΠ°ΠΏΠΈΡΡΠ²Π°ΡΡΡΡ ΡΡΠΈ ΡΡΠ²Π΅ΡΠΆΠ΄Π΅Π½ΠΈΡ Π² Π²ΠΈΠ΄Π΅
.
ΠΠΎΠΊΠ°Π·Π°ΡΠ΅Π»ΡΡΡΠ²ΠΎ.
,
Π’Π΅ΠΎΡΠ΅ΠΌΠ°.
ΠΡΡΡΡ
,
ΠΏΡΠΈΡΠ΅ΠΌ Π½Π°ΡΠ°Π»ΠΎ Π²Π΅ΠΊΡΠΎΡΠ° ΡΠΎΡΠΊΠ°ΠΈΠΌΠ΅Π΅Ρ ΠΊΠΎΠΎΡΠ΄ΠΈΠ½Π°ΡΡ
,
Π° ΠΊΠΎΠ½Π΅Ρ Π²Π΅ΠΊΡΠΎΡΠ° Π΅ΡΡΡ ΡΠΎΡΠΊΠ°
.
Π’ΠΎΠ³Π΄Π° ΠΊΠΎΠΎΡΠ΄ΠΈΠ½Π°ΡΡ Π²Π΅ΠΊΡΠΎΡΠ° ΡΠ²ΡΠ·Π°Π½Ρ Ρ
ΠΊΠΎΠΎΡΠ΄ΠΈΠ½Π°ΡΠ°ΠΌΠΈ Π΅Π³ΠΎ ΠΊΠΎΠ½ΡΠΎΠ² ΡΠ»Π΅Π΄ΡΡΡΠΈΠΌΠΈ
ΡΠΎΠΎΡΠ½ΠΎΡΠ΅Π½ΠΈΡΠΌΠΈ
,
.
ΠΠΎΠΊΠ°Π·Π°ΡΠ΅Π»ΡΡΡΠ²ΠΎ.
ΠΡΡΡΡ
ΠΈ ΠΏΡΡΡΡ Π²Π΅ΠΊΡΠΎΡ-ΠΏΡΠΎΠ΅ΠΊΡΠΈΡ Π²Π΅ΠΊΡΠΎΡΠ°Π½Π° ΠΎΡΡΡΠΎΠ½Π°ΠΏΡΠ°Π²Π»Π΅Π½ Ρ ΠΎΡΡΡ(ΡΠΌ. ΡΠΈΡ. 22). Π’ΠΎΠ³Π΄Π°
ΡΠ°ΠΊ
ΠΊΠ°ΠΊ Π΄Π»ΠΈΠ½Π° ΠΎΡΡΠ΅Π·ΠΊΠ° Π½Π° ΡΠΈΡΠ»ΠΎΠ²ΠΎΠΉ ΠΎΡΠΈΡΠ°Π²Π½Π° ΠΊΠΎΠΎΡΠ΄ΠΈΠ½Π°ΡΠ΅ ΠΏΡΠ°Π²ΠΎΠ³ΠΎ ΠΊΠΎΠ½ΡΠ° ΠΌΠΈΠ½ΡΡ
ΠΊΠΎΠΎΡΠ΄ΠΈΠ½Π°ΡΠ° Π»Π΅Π²ΠΎΠ³ΠΎ ΠΊΠΎΠ½ΡΠ°.
ΠΡΠ»ΠΈ Π²Π΅ΠΊΡΠΎΡ
ΠΏΡΠΎΡΠΈΠ²ΠΎΠ½Π°ΠΏΡΠ°Π²Π»Π΅Π½ ΠΎΡΠΈ(ΠΊΠ°ΠΊ Π½Π° Π ΠΈΡ. 23), ΡΠΎ
Π ΠΈΡ. 23.
ΠΡΠ»ΠΈ
,
ΡΠΎ Π² ΡΡΠΎΠΌ ΡΠ»ΡΡΠ°Π΅
ΠΈ ΡΠΎΠ³Π΄Π° ΠΏΠΎΠ»ΡΡΠ°Π΅ΠΌ
.
Π’Π°ΠΊΠΈΠΌ ΠΎΠ±ΡΠ°Π·ΠΎΠΌ, ΠΏΡΠΈ Π»ΡΠ±ΠΎΠΌ ΡΠ°ΡΠΏΠΎΠ»ΠΎΠΆΠ΅Π½ΠΈΠΈ
Π²Π΅ΠΊΡΠΎΡΠ°
ΠΎΡΠ½ΠΎΡΠΈΡΠ΅Π»ΡΠ½ΠΎ
ΠΎΡΠ΅ΠΉ ΠΊΠΎΠΎΡΠ΄ΠΈΠ½Π°Ρ Π΅Π³ΠΎ ΠΊΠΎΠΎΡΠ΄ΠΈΠ½Π°ΡΠ°ΡΠ°Π²Π½Π°
.
ΠΠ½Π°Π»ΠΎΠ³ΠΈΡΠ½ΠΎ Π΄ΠΎΠΊΠ°Π·ΡΠ²Π°Π΅ΡΡΡ, ΡΡΠΎ
.
ΠΡΠΈΠΌΠ΅Ρ.
ΠΠ°Π½Ρ ΠΊΠΎΠΎΡΠ΄ΠΈΠ½Π°ΡΡ ΠΊΠΎΠ½ΡΠΎΠ² Π²Π΅ΠΊΡΠΎΡΠ°
:
.
ΠΠ°ΠΉΡΠΈ ΠΊΠΎΠΎΡΠ΄ΠΈΠ½Π°ΡΡ Π²Π΅ΠΊΡΠΎΡΠ°
.
Π Π΅ΡΠ΅Π½ΠΈΠ΅.
Π ΡΠ»Π΅Π΄ΡΡΡΠ΅ΠΉ ΡΠ΅ΠΎΡΠ΅ΠΌΠ΅ ΠΏΡΠΈΠ²ΠΎΠ΄ΠΈΡΡΡ Π²ΡΡΠ°ΠΆΠ΅Π½ΠΈΠ΅ Π΄Π»ΠΈΠ½Ρ Π²Π΅ΠΊΡΠΎΡΠ° ΡΠ΅ΡΠ΅Π· Π΅Π³ΠΎ ΠΊΠΎΠΎΡΠ΄ΠΈΠ½Π°ΡΡ.
Π’Π΅ΠΎΡΠ΅ΠΌΠ° 15.
ΠΡΡΡΡ
.Π’ΠΎΠ³Π΄Π°
.
ΠΠΎΠΊΠ°Π·Π°ΡΠ΅Π»ΡΡΡΠ²ΠΎ.
ΠΡΡΡΡ ΠΈ- Π²Π΅ΠΊΡΠΎΡ-ΠΏΡΠΎΠ΅ΠΊΡΠΈΠΈ Π²Π΅ΠΊΡΠΎΡΠ°Π½Π° ΠΎΡΠΈΠΈ, ΡΠΎΠΎΡΠ²Π΅ΡΡΡΠ²Π΅Π½Π½ΠΎ. Π’ΠΎΠ³Π΄Π°, ΠΊΠ°ΠΊ ΠΏΠΎΠΊΠ°Π·Π°Π½ΠΎ ΠΏΡΠΈ Π΄ΠΎΠΊΠ°Π·Π°ΡΠ΅Π»ΡΡΡΠ²Π΅ ΡΠ΅ΠΎΡΠ΅ΠΌΡ 9, ΠΈΠΌΠ΅Π΅Ρ ΠΌΠ΅ΡΡΠΎ ΡΠ°Π²Π΅Π½ΡΡΠ²ΠΎ
.
ΠΡΠΈ ΡΡΠΎΠΌ, Π²Π΅ΠΊΡΠΎΡΡ ΠΈΠ²Π·Π°ΠΈΠΌΠ½ΠΎ ΠΏΠ΅ΡΠΏΠ΅Π½Π΄ΠΈΠΊΡΠ»ΡΡΠ½Ρ. ΠΡΠΈ ΡΠ»ΠΎΠΆΠ΅Π½ΠΈΠΈ ΡΡΠΈΡ Π²Π΅ΠΊΡΠΎΡΠΎΠ² ΠΏΠΎ ΠΏΡΠ°Π²ΠΈΠ»Ρ ΡΡΠ΅ΡΠ³ΠΎΠ»ΡΠ½ΠΈΠΊΠ° ΠΏΠΎΠ»ΡΡΠ°Π΅ΠΌ ΠΏΡΡΠΌΠΎΡΠ³ΠΎΠ»ΡΠ½ΡΠΉ ΡΡΠ΅ΡΠ³ΠΎΠ»ΡΠ½ΠΈΠΊ (ΡΠΌ. Π ΠΈΡ. 24).
ΠΠΎ ΡΠ΅ΠΎΡΠ΅ΠΌΠ΅ ΠΠΈΡΠ°Π³ΠΎΡΠ° ΠΈΠΌΠ΅Π΅ΠΌ
.
,
.
Π‘Π»Π΅Π΄ΠΎΠ²Π°ΡΠ΅Π»ΡΠ½ΠΎ
,
.
.
.
ΠΡΠΈΠΌΠ΅Ρ.
.ΠΠ°ΠΉΡΠΈ.
ΠΠ²Π΅Π΄Π΅ΠΌ ΠΏΠΎΠ½ΡΡΠΈΠ΅ Π½Π°ΠΏΡΠ°Π²Π»ΡΡΡΠΈΡ ΠΊΠΎΡΠΈΠ½ΡΡΠΎΠ² Π²Π΅ΠΊΡΠΎΡΠ°.
ΠΠΏΡΠ΅Π΄Π΅Π»Π΅Π½ΠΈΠ΅.
ΠΡΡΡΡ Π²Π΅ΠΊΡΠΎΡ
ΡΠΎΡΡΠ°Π²Π»ΡΠ΅Ρ Ρ ΠΎΡΡΡΡΠ³ΠΎΠ»,
Π° Ρ ΠΎΡΡΡΡΠ³ΠΎΠ»(ΡΠΌ. Π ΠΈΡ. 25).
,
.
Π‘Π»Π΅Π΄ΠΎΠ²Π°ΡΠ΅Π»ΡΠ½ΠΎ,
Π’Π°ΠΊ ΠΊΠ°ΠΊ Π΄Π»Ρ Π»ΡΠ±ΠΎΠ³ΠΎ Π²Π΅ΠΊΡΠΎΡΠ° ΠΈΠΌΠ΅Π΅Ρ ΠΌΠ΅ΡΡΠΎ ΡΠ°Π²Π΅Π½ΡΡΠ²ΠΎ
,
ΠΠ΄Π΅ — ΠΎΡΡ Π²Π΅ΠΊΡΠΎΡΠ°, ΡΠΎ Π΅ΡΡΡ Π²Π΅ΠΊΡΠΎΡ Π΅Π΄ΠΈΠ½ΠΈΡΠ½ΠΎΠΉ Π΄Π»ΠΈΠ½Ρ, ΡΠΎΠ½Π°ΠΏΡΠ°Π²Π»Π΅Π½Π½ΡΠΉ Ρ Π²Π΅ΠΊΡΠΎΡΠΎΠΌ, ΡΠΎ
ΠΠ΅ΠΊΡΠΎΡ
ΠΎΠΏΡΠ΅Π΄Π΅Π»ΡΠ΅Ρ Π½Π°ΠΏΡΠ°Π²Π»Π΅Π½ΠΈΠ΅ Π²Π΅ΠΊΡΠΎΡΠ°.
ΠΠ³ΠΎ ΠΊΠΎΠΎΡΠ΄ΠΈΠ½Π°ΡΡ
ΠΈ
Π½Π°Π·ΡΠ²Π°ΡΡΡΡ Π½Π°ΠΏΡΠ°Π²Π»ΡΡΡΠΈΠΌΠΈ ΠΊΠΎΡΠΈΠ½ΡΡΠ°ΠΌΠΈ
Π²Π΅ΠΊΡΠΎΡΠ°.
ΠΠ°ΠΏΡΠ°Π²Π»ΡΡΡΠΈΠ΅ ΠΊΠΎΡΠΈΠ½ΡΡΡ Π²Π΅ΠΊΡΠΎΡΠ° ΠΌΠΎΠΆΠ½ΠΎ
Π²ΡΡΠ°Π·ΠΈΡΡ ΡΠ΅ΡΠ΅Π· Π΅Π³ΠΎ ΠΊΠΎΠΎΡΠ΄ΠΈΠ½Π°ΡΡ ΠΏΠΎ ΡΠΎΡΠΌΡΠ»Π°ΠΌ
,
.
ΠΠΌΠ΅Π΅Ρ ΠΌΠ΅ΡΡΠΎ ΡΠΎΠΎΡΠ½ΠΎΡΠ΅Π½ΠΈΠ΅
.
ΠΠΎ Π½Π°ΡΡΠΎΡΡΠ΅Π³ΠΎ ΠΌΠΎΠΌΠ΅Π½ΡΠ° Π² ΡΡΠΎΠΌ ΠΏΠ°ΡΠ°Π³ΡΠ°ΡΠ΅ ΡΡΠΈΡΠ°Π»ΠΎΡΡ, ΡΡΠΎ Π²ΡΠ΅ Π²Π΅ΠΊΡΠΎΡΡ ΡΠ°ΡΠΏΠΎΠ»Π°Π³Π°ΡΡΡΡ Π² ΠΎΠ΄Π½ΠΎΠΉ ΠΈ ΡΠΎΠΉ ΠΆΠ΅ ΠΏΠ»ΠΎΡΠΊΠΎΡΡΠΈ. Π’Π΅ΠΏΠ΅ΡΡ ΡΠ΄Π΅Π»Π°Π΅ΠΌ ΠΎΠ±ΠΎΠ±ΡΠ΅Π½ΠΈΠ΅ Π΄Π»Ρ Π²Π΅ΠΊΡΠΎΡΠΎΠ² Π² ΠΏΡΠΎΡΡΡΠ°Π½ΡΡΠ²Π΅.
ΠΡΠ΄Π΅ΠΌ ΡΡΠΈΡΠ°ΡΡ, ΡΡΠΎ Π² ΠΏΡΠΎΡΡΡΠ°Π½ΡΡΠ²Π΅ Π·Π°Π΄Π°Π½Π° ΠΠ΅ΠΊΠ°ΡΡΠΎΠ²Π° ΡΠΈΡΡΠ΅ΠΌΠ° ΠΊΠΎΠΎΡΠ΄ΠΈΠ½Π°Ρ Ρ ΠΎΡΡΠΌΠΈ ,ΠΈ.
ΠΡΡΡ ΠΎΡΠ΅ΠΉ ,ΠΈΠ±ΡΠ΄Π΅ΠΌ ΠΎΠ±ΠΎΠ·Π½Π°ΡΠ°ΡΡ ΡΠΈΠΌΠ²ΠΎΠ»Π°ΠΌΠΈ,ΠΈ, ΡΠΎΠΎΡΠ²Π΅ΡΡΡΠ²Π΅Π½Π½ΠΎ (Π ΠΈΡ. 26).
ΠΠΎΠΆΠ½ΠΎ ΠΏΠΎΠΊΠ°Π·Π°ΡΡ, ΡΡΠΎ Π²ΡΠ΅ ΠΏΠΎΠ½ΡΡΠΈΡ ΠΈ ΡΠΎΡΠΌΡΠ»Ρ, ΠΊΠΎΡΠΎΡΡΠ΅ Π±ΡΠ»ΠΈ ΠΏΠΎΠ»ΡΡΠ΅Π½Ρ Π΄Π»Ρ Π²Π΅ΠΊΡΠΎΡΠΎΠ² Π½Π° ΠΏΠ»ΠΎΡΠΊΠΎΡΡΠΈ, ΠΎΠ±ΠΎΠ±ΡΠ°ΡΡΡΡ Π΄Π»Ρ
Π ΠΈΡ.
26.
Π²Π΅ΠΊΡΠΎΡΠΎΠ² Π² ΠΏΡΠΎΡΡΡΠ°Π½ΡΡΠ²Π΅. Π’ΡΠΎΠΉΠΊΠ° Π²Π΅ΠΊΡΠΎΡΠΎΠ²
Π½Π°Π·ΡΠ²Π°Π΅ΡΡΡ ΠΎΡΡΠΎΠ½ΠΎΡΠΌΠΈΡΠΎΠ²Π°Π½Π½ΡΠΌ Π±Π°Π·ΠΈΡΠΎΠΌ
Π² ΠΏΡΠΎΡΡΡΠ°Π½ΡΡΠ²Π΅.
ΠΡΡΡΡ ,ΠΈ- Π²Π΅ΠΊΡΠΎΡ-ΠΏΡΠΎΠ΅ΠΊΡΠΈΠΈ Π²Π΅ΠΊΡΠΎΡΠ°Π½Π° ΠΎΡΠΈ,ΠΈ, ΡΠΎΠΎΡΠ²Π΅ΡΡΡΠ²Π΅Π½Π½ΠΎ. Π’ΠΎΠ³Π΄Π°
.
Π ΡΠ²ΠΎΡ ΠΎΡΠ΅ΡΠ΅Π΄Ρ
,
,
.
ΠΡΠ»ΠΈ ΠΎΠ±ΠΎΠ·Π½Π°ΡΠΈΡΡ
,
,
,
Π’ΠΎ ΠΏΠΎΠ»ΡΡΠ°Π΅ΠΌ ΡΠ°Π²Π΅Π½ΡΡΠ²ΠΎ
.
ΠΠΎΡΡΡΠΈΡΠΈΠ΅Π½ΡΡ ΠΏΠ΅ΡΠ΅Π΄ Π±Π°Π·ΠΈΡΠ½ΡΠΌΠΈ Π²Π΅ΠΊΡΠΎΡΠ°ΠΌΠΈ ,ΠΈΠ½Π°Π·ΡΠ²Π°ΡΡΡΡ ΠΊΠΎΠΎΡΠ΄ΠΈΠ½Π°ΡΠ°ΠΌΠΈ Π²Π΅ΠΊΡΠΎΡΠ°. Π’Π°ΠΊΠΈΠΌ ΠΎΠ±ΡΠ°Π·ΠΎΠΌ, Π΄Π»Ρ Π»ΡΠ±ΠΎΠ³ΠΎ Π²Π΅ΠΊΡΠΎΡΠ°Π² ΠΏΡΠΎΡΡΡΠ°Π½ΡΡΠ²Π΅ ΡΡΡΠ΅ΡΡΠ²ΡΠ΅Ρ ΡΡΠΎΠΉΠΊΠ° ΡΠΈΡΠ΅Π»,,, Π½Π°Π·ΡΠ²Π°Π΅ΠΌΡΡ ΠΊΠΎΠΎΡΠ΄ΠΈΠ½Π°ΡΠ°ΠΌΠΈ Π²Π΅ΠΊΡΠΎΡΠ°ΡΠ°ΠΊΠΈΡ , ΡΡΠΎ Π΄Π»Ρ ΡΡΠΎΠ³ΠΎ Π²Π΅ΠΊΡΠΎΡΠ° ΡΠΏΡΠ°Π²Π΅Π΄Π»ΠΈΠ²ΠΎ ΠΏΡΠ΅Π΄ΡΡΠ°Π²Π»Π΅Π½ΠΈΠ΅
.
ΠΠ΅ΠΊΡΠΎΡ
Π² ΡΡΠΎΠΌ ΡΠ»ΡΡΠ°Π΅ ΡΠ°ΠΊΠΆΠ΅ ΠΎΠ±ΠΎΠ·Π½Π°ΡΠ°ΡΡ Π² Π²ΠΈΠ΄Π΅
.
ΠΡΠΈ ΡΡΠΎΠΌ, ΠΊΠΎΠΎΡΠ΄ΠΈΠ½Π°ΡΡ Π²Π΅ΠΊΡΠΎΡΠ° ΡΠ°Π²Π½Ρ
ΠΏΡΠΎΠ΅ΠΊΡΠΈΡΠΌ ΡΡΠΎΠ³ΠΎ Π²Π΅ΠΊΡΠΎΡΠ° Π½Π° ΠΊΠΎΠΎΡΠ΄ΠΈΠ½Π°ΡΠ½ΡΠ΅
ΠΎΡΠΈ
,
,
,
Π³Π΄Π΅ — ΡΠ³ΠΎΠ» ΠΌΠ΅ΠΆΠ΄Ρ Π²Π΅ΠΊΡΠΎΡΠΎΠΌΠΈ ΠΎΡΡΡ,- ΡΠ³ΠΎΠ» ΠΌΠ΅ΠΆΠ΄Ρ Π²Π΅ΠΊΡΠΎΡΠΎΠΌΠΈ ΠΎΡΡΡ,- ΡΠ³ΠΎΠ» ΠΌΠ΅ΠΆΠ΄Ρ Π²Π΅ΠΊΡΠΎΡΠΎΠΌΠΈ ΠΎΡΡΡ.
ΠΠ»ΠΈΠ½Π° Π²Π΅ΠΊΡΠΎΡΠ° Π²ΡΡΠ°ΠΆΠ°Π΅ΡΡΡ ΡΠ΅ΡΠ΅Π· Π΅Π³ΠΎ ΠΊΠΎΠΎΡΠ΄ΠΈΠ½Π°ΡΡ ΠΏΠΎ ΡΠΎΡΠΌΡΠ»Π΅
.
Π‘ΠΏΡΠ°Π²Π΅Π΄Π»ΠΈΠ²Ρ ΡΡΠ²Π΅ΡΠΆΠ΄Π΅Π½ΠΈΡ ΠΎ ΡΠΎΠΌ, ΡΡΠΎ
ΡΠ°Π²Π½ΡΠ΅ Π²Π΅ΠΊΡΠΎΡΡ ΠΈΠΌΠ΅ΡΡ ΡΠ°Π²Π½ΡΠ΅ ΠΊΠΎΠΎΡΠ΄ΠΈΠ½Π°ΡΡ,
ΠΏΡΠΈ ΡΠ»ΠΎΠΆΠ΅Π½ΠΈΠΈ Π²Π΅ΠΊΡΠΎΡΠΎΠ² ΠΈΡ
ΠΊΠΎΠΎΡΠ΄ΠΈΠ½Π°ΡΡ
ΡΠΊΠ»Π°Π΄ΡΠ²Π°ΡΡΡΡ, Π° ΠΏΡΠΈ ΡΠΌΠ½ΠΎΠΆΠ΅Π½ΠΈΠΈ Π²Π΅ΠΊΡΠΎΡΠ°
Π½Π° ΡΠΈΡΠ»ΠΎ Π΅Π³ΠΎ ΠΊΠΎΠΎΡΠ΄ΠΈΠ½Π°ΡΡ ΡΠΌΠ½ΠΎΠΆΠ°ΡΡΡΡ Π½Π°
ΡΡΠΎ ΡΠΈΡΠ»ΠΎ.
2\), ΠΎΡΠΊΡΠ΄Π° ΠΏΠΎΠ»ΡΡΠ°Π΅ΠΌ ΡΡΠ΅Π±ΡΠ΅ΠΌΠΎΠ΅ ΡΠ°Π²Π΅Π½ΡΡΠ²ΠΎ.
Β
Π£ΡΠ²Π΅ΡΠΆΠ΄Π΅Π½ΠΈΠ΅
ΠΡΠ»ΠΈ Π² ΠΏΡΡΠΌΠΎΡΠ³ΠΎΠ»ΡΠ½ΠΎΠΉ ΡΠΈΡΡΠ΅ΠΌΠ΅ ΠΊΠΎΠΎΡΠ΄ΠΈΠ½Π°Ρ ΡΠΎΡΠΊΠ° \(M\) β ΡΠ΅ΡΠ΅Π΄ΠΈΠ½Π° ΠΎΡΡΠ΅Π·ΠΊΠ° \(PQ\), Π³Π΄Π΅ \(P(x_1;y_1), \ Q(x_2;y_2)\), ΡΠΎ
\[M\left(\dfrac{x_1 + x_2}{2}; \dfrac{y_1 + y_2}{2}\right)\]
ΠΠΎΠΊΠ°Π·Π°ΡΠ΅Π»ΡΡΡΠ²ΠΎ
ΠΡΡΡΡ \(M(a;b)\).
Β
1) ΠΡΡΡΡ \(PQ\parallel Oy \Rightarrow x_1=x_2=a\). ΠΠ½Π°ΡΠΈΡ, \(a=\dfrac{x_1+x_2}2=\dfrac{a+a}2\) β Π²Π΅ΡΠ½ΠΎ.
Β
Π’.ΠΊ. \(PM=MQ\), ΡΠ»Π΅Π΄ΠΎΠ²Π°ΡΠ΅Π»ΡΠ½ΠΎ, \(|y_2-b|=|y_1-b| \Rightarrow y_2-b=y_1-b\) ΠΈΠ»ΠΈ \(y_2-b=b-y_1\), ΡΡΠΎ ΡΠ°Π²Π½ΠΎΡΠΈΠ»ΡΠ½ΠΎ \(y_2=y_1\) ΠΈΠ»ΠΈ \(b=\dfrac{y_1+y_2}2\). ΠΠ΅ΡΠ²ΠΎΠ΅ ΡΠ°Π²Π΅Π½ΡΡΠ²ΠΎ Π½Π΅Π²ΠΎΠ·ΠΌΠΎΠΆΠ½ΠΎ (Ρ.ΠΊ. ΡΠΎΠ³Π΄Π° ΡΠΎΡΠΊΠΈ \(P\) ΠΈ \(Q\) ΡΠΎΠ²ΠΏΠ°Π΄Π°ΡΡ).
Β
2) Π‘Π»ΡΡΠ°ΠΉ \(PQ\parallel Ox \Rightarrow y_1=y_2=b\) Π΄ΠΎΠΊΠ°Π·ΡΠ²Π°Π΅ΡΡΡ Π°Π½Π°Π»ΠΎΠ³ΠΈΡΠ½ΠΎ.
Β
3) \(x_1\ne x_2, y_1\ne y_2\).
Β
Π’ΠΎΠ³Π΄Π° \(Ma=b\) β ΡΡΠ΅Π΄Π½ΡΡ Π»ΠΈΠ½ΠΈΡ ΡΡΠ°ΠΏΠ΅ΡΠΈΠΈ \(x_1PQx_2\), ΡΠ»Π΅Π΄ΠΎΠ²Π°ΡΠ΅Π»ΡΠ½ΠΎ, ΡΠ°Π²Π½Π° ΠΏΠΎΠ»ΡΡΡΠΌΠΌΠ΅ ΠΎΡΠ½ΠΎΠ²Π°Π½ΠΈΠΉ, ΡΠΎ Π΅ΡΡΡ \(b=\dfrac{y_1+y_2}2\).
Β
ΠΠ½Π°Π»ΠΎΠ³ΠΈΡΠ½ΠΎ \(a=\dfrac{x_1+x_2}2\).
Β
\[{\Large{\text{ΠΠ΅ΠΊΡΠΎΡΡ Π½Π° ΠΊΠΎΠΎΡΠ΄ΠΈΠ½Π°ΡΠ½ΠΎΠΉ ΠΏΠ»ΠΎΡΠΊΠΎΡΡΠΈ}}}\]
ΠΠ΅ΠΌΠΌΠ°
ΠΡΠ»ΠΈ Π²Π΅ΠΊΡΠΎΡΡ \(\overrightarrow a\) ΠΈ \(\overrightarrow b\) ΠΊΠΎΠ»Π»ΠΈΠ½Π΅Π°ΡΠ½Ρ, ΡΠΎ ΡΡΡΠ΅ΡΡΠ²ΡΠ΅Ρ ΡΠ°ΠΊΠΎΠ΅ ΡΠΈΡΠ»ΠΎ \(\lambda\ne 0\), ΡΡΠΎ \(\overrightarrow a=\lambda\overrightarrow b\).
Β
ΠΠΎΠΊΠ°Π·Π°ΡΠ΅Π»ΡΡΡΠ²ΠΎ
1) ΠΡΠ»ΠΈ \(\overrightarrow a\uparrow \uparrow \overrightarrow b\).
Β
Π Π°ΡΡΠΌΠΎΡΡΠΈΠΌ Π²Π΅ΠΊΡΠΎΡ \(\dfrac1{|\overrightarrow a|}\overrightarrow a\). ΠΠ°Π½Π½ΡΠΉ Π²Π΅ΠΊΡΠΎΡ ΡΠΎΠ½Π°Π²ΠΏΡΠ°Π²Π»Π΅Π½ Ρ \(\overrightarrow a\), Π° Π΅Π³ΠΎ Π΄Π»ΠΈΠ½Π° ΡΠ°Π²Π½Π° \(1\). Π’ΠΎΠ³Π΄Π° Π²Π΅ΠΊΡΠΎΡ \(\dfrac{|\overrightarrow b|}{|\overrightarrow a|}\overrightarrow a\) ΡΠ°ΠΊΠΆΠ΅ ΡΠΎΠ½Π°ΠΏΡΠ°Π²Π»Π΅Π½ Ρ \(\overrightarrow a\), Π½ΠΎ Π΅Π³ΠΎ Π΄Π»ΠΈΠ½Π° ΡΠ°Π²Π½Π° \(|\overrightarrow b|\). Π’ΠΎ Π΅ΡΡΡ ΡΠ°Π²Π΅Π½ Π²Π΅ΠΊΡΠΎΡΡ \(\overrightarrow b\).
Β
2) ΠΡΠ»ΠΈ \(\overrightarrow a\uparrow \downarrow \overrightarrow b\).
Β
ΠΠ½Π°Π»ΠΎΠ³ΠΈΡΠ½ΠΎ Π΄ΠΎΠΊΠ°Π·ΡΠ²Π°Π΅ΡΡΡ, ΡΡΠΎ \(\overrightarrow b=-\dfrac{|\overrightarrow b|}{|\overrightarrow a|}\overrightarrow a\).
Β
ΠΠΏΡΠ΅Π΄Π΅Π»Π΅Π½ΠΈΠ΅
ΠΡΠ»ΠΈ Π²Π΅ΠΊΡΠΎΡ \(\overrightarrow p\) ΠΏΡΠ΅Π΄ΡΡΠ°Π²Π»Π΅Π½ ΠΊΠ°ΠΊ Π»ΠΈΠ½Π΅ΠΉΠ½Π°Ρ ΠΊΠΎΠΌΠ±ΠΈΠ½Π°ΡΠΈΡ Π΄Π²ΡΡ
Π²Π΅ΠΊΡΠΎΡΠΎΠ²: \(\overrightarrow p=\alpha\overrightarrow a+\beta
\overrightarrow b\), ΡΠΎ Π³ΠΎΠ²ΠΎΡΡΡ, ΡΡΠΎ Π²Π΅ΠΊΡΠΎΡ \(\overrightarrow p\) ΡΠ°Π·Π»ΠΎΠΆΠ΅Π½ ΠΏΠΎ Π²Π΅ΠΊΡΠΎΡΠ°ΠΌ \(\overrightarrow a\) ΠΈ \(\overrightarrow b\).
Β
\(\alpha, \beta\) β ΠΊΠΎΡΡΡΠΈΡΠΈΠ΅Π½ΡΡ ΡΠ°Π·Π»ΠΎΠΆΠ΅Π½ΠΈΡ. Β
ΠΡΡΡΡ Π²Π΅ΠΊΡΠΎΡΡ \(\overrightarrow i\), \(\overrightarrow j\) β Π²Π΅ΠΊΡΠΎΡΡ, Π΄Π»ΠΈΠ½Ρ ΠΊΠΎΡΠΎΡΡΡ ΡΠ°Π²Π½Ρ \(1\), Π° Π½Π°ΠΏΡΠ°Π²Π»Π΅Π½ΠΈΠ΅ ΡΠΎΠ²ΠΏΠ°Π΄Π°Π΅Ρ Ρ Π½Π°ΠΏΡΠ°Π²Π»Π΅Π½ΠΈΠ΅ΠΌ ΠΎΡΠ΅ΠΉ \(Ox\) ΠΈ \(Oy\) ΡΠΎΠΎΡΠ²Π΅ΡΡΡΠ²Π΅Π½Π½ΠΎ. Π’Π°ΠΊΠΈΠ΅ Π²Π΅ΠΊΡΠΎΡΡ Π½Π°Π·ΡΠ²Π°ΡΡΡΡ Π΅Π΄ΠΈΠ½ΠΈΡΠ½ΡΠΌΠΈ Π²Π΅ΠΊΡΠΎΡΠ°ΠΌΠΈ.
Β
Π’ΠΎΠ³Π΄Π° Π΅ΡΠ»ΠΈ \(\overrightarrow p=a\overrightarrow i+b\overrightarrow j\), ΡΠΎ \(\{a;b\}\) β ΠΊΠΎΠΎΡΠ΄ΠΈΠ½Π°ΡΡ Π²Π΅ΠΊΡΠΎΡΠ° \(\overrightarrow p\).
Β
Π‘Π²ΠΎΠΉΡΡΠ²Π° ΠΊΠΎΠΎΡΠ΄ΠΈΠ½Π°Ρ Π²Π΅ΠΊΡΠΎΡΠ°
1. Π Π°Π²Π½ΡΠ΅ Π²Π΅ΠΊΡΠΎΡΡ ΠΈΠΌΠ΅ΡΡ ΡΠ°Π²Π½ΡΠ΅ ΠΊΠΎΠΎΡΠ΄ΠΈΠ½Π°ΡΡ.
Β
2. ΠΠΎΠΎΡΠ΄ΠΈΠ½Π°ΡΡ ΡΡΠΌΠΌΡ Π²Π΅ΠΊΡΠΎΡΠΎΠ² ΡΠ°Π²Π½Ρ ΡΡΠΌΠΌΠ΅ ΠΊΠΎΠΎΡΠ΄ΠΈΠ½Π°Ρ ΠΊΠ°ΠΆΠ΄ΠΎΠ³ΠΎ Π²Π΅ΠΊΡΠΎΡΠ°: Π΅ΡΠ»ΠΈ \(\overrightarrow a\{x_1;y_1\}, \ \overrightarrow b\{x_2;y_2\}\), ΡΠΎ \(\overrightarrow a+\overrightarrow b=\{x_1+x_2;y_1+y_2\}\).
Β
3. ΠΠ°ΠΆΠ΄Π°Ρ ΠΊΠΎΠΎΡΠ΄ΠΈΠ½Π°ΡΠ° ΠΏΡΠΎΠΈΠ·Π²Π΅Π΄Π΅Π½ΠΈΡ Π²Π΅ΠΊΡΠΎΡΠ° Π½Π° ΡΠΈΡΠ»ΠΎ ΡΠ°Π²Π½Π° ΠΏΡΠΎΠΈΠ·Π²Π΅Π΄Π΅Π½ΠΈΡ ΡΠΎΠΎΡΠ²Π΅ΡΡΡΠ²ΡΡΡΠ΅ΠΉ ΠΊΠΎΠΎΡΠ΄ΠΈΠ½Π°ΡΡ Π΄Π°Π½Π½ΠΎΠ³ΠΎ Π²Π΅ΠΊΡΠΎΡΠ° Π½Π° ΡΡΠΎ ΡΠΈΡΠ»ΠΎ: \(\overrightarrow a\{x;y\}, \ \lambda \) β ΡΠΈΡΠ»ΠΎ, ΡΠΎ \(\lambda\overrightarrow a\{\lambda x;\lambda y\}\).
2}\).
Β
\[{\Large{\text{Π‘ΠΊΠ°Π»ΡΡΠ½ΠΎΠ΅ ΠΏΡΠΎΠΈΠ·Π²Π΅Π΄Π΅Π½ΠΈΠ΅ Π²Π΅ΠΊΡΠΎΡΠΎΠ²}}}\]
ΠΠΏΡΠ΅Π΄Π΅Π»Π΅Π½ΠΈΠ΅
ΠΡΡΡΡ ΠΎΡ ΠΎΠ΄Π½ΠΎΠΉ ΡΠΎΡΠΊΠΈ ΠΎΡΠ»ΠΎΠΆΠ΅Π½Ρ Π΄Π²Π° Π²Π΅ΠΊΡΠΎΡΠ° \(\overrightarrow {AB}\) ΠΈ \(\overrightarrow {AC}\). Π’ΠΎΠ³Π΄Π° ΡΠ³ΠΎΠ» ΠΌΠ΅ΠΆΠ΄Ρ ΡΡΠΈΠΌΠΈ Π²Π΅ΠΊΡΠΎΡΠ°ΠΌΠΈ β ΡΡΠΎ ΡΠ³ΠΎΠ» \(\angle BAC\), Π½Π΅ ΠΏΡΠ΅Π²ΡΡΠ°ΡΡΠΈΠΉ ΡΠ°Π·Π²Π΅ΡΠ½ΡΡΠΎΠ³ΠΎ ΡΠ³Π»Π°.
Β
Π‘ΠΊΠ°Π»ΡΡΠ½ΠΎΠ΅ ΠΏΡΠΎΠΈΠ·Π²Π΅Π΄Π΅Π½ΠΈΠ΅ Π²Π΅ΠΊΡΠΎΡΠΎΠ² \(\overrightarrow a\) ΠΈ \(\overrightarrow b\) β ΡΡΠΎ ΡΠΈΡΠ»ΠΎ, ΡΠ°Π²Π½ΠΎΠ΅ ΠΏΡΠΎΠΈΠ·Π²Π΅Π΄Π΅Π½ΠΈΡ Π΄Π»ΠΈΠ½ ΡΡΠΈΡ
Π²Π΅ΠΊΡΠΎΡΠΎΠ² Π½Π° ΠΊΠΎΡΠΈΠ½ΡΡ ΡΠ³Π»Π° ΠΌΠ΅ΠΆΠ΄Ρ Π½ΠΈΠΌΠΈ.
ΠΠ±ΠΎΠ·Π½Π°ΡΠ΅Π½ΠΈΠ΅: \(\overrightarrow a\cdot \overrightarrow b\) ΠΈΠ»ΠΈ \((\overrightarrow a, \overrightarrow b)\). \[(\overrightarrow a, \overrightarrow b)=|\overrightarrow a|\cdot
|\overrightarrow b|\cdot \cos\widehat{(\overrightarrow a,
\overrightarrow b)}\]
Π‘Π»Π΅Π΄ΡΡΠ²ΠΈΡ
1. ΠΡΠ»ΠΈ Π½Π΅Π½ΡΠ»Π΅Π²ΡΠ΅ Π²Π΅ΠΊΡΠΎΡΡ Π²Π·Π°ΠΈΠΌΠ½ΠΎ ΠΏΠ΅ΡΠΏΠ΅Π½Π΄ΠΈΠΊΡΠ»ΡΡΠ½Ρ, ΡΠΎ ΠΊΠΎΡΠΈΠ½ΡΡ ΡΠ³Π»Π° ΠΌΠ΅ΠΆΠ΄Ρ Π½ΠΈΠΌΠΈ ΡΠ°Π²Π΅Π½ Π½ΡΠ»Ρ, ΡΠ»Π΅Π΄ΠΎΠ²Π°ΡΠ΅Π»ΡΠ½ΠΎ, ΠΈ ΠΈΡ ΡΠΊΠ°Π»ΡΡΠ½ΠΎΠ΅ ΠΏΡΠΎΠΈΠ·Π²Π΅Π΄Π΅Π½ΠΈΠ΅ ΡΠ°Π²Π½ΠΎ Π½ΡΠ»Ρ.
Β
2. ΠΡΠ»ΠΈ ΡΠ³ΠΎΠ» ΠΌΠ΅ΠΆΠ΄Ρ Π½Π΅Π½ΡΠ»Π΅Π²ΡΠΌΠΈ Π²Π΅ΠΊΡΠΎΡΠ°ΠΌΠΈ ΠΎΡΡΡΡΠΉ, ΡΠΎ ΡΠΊΠ°Π»ΡΡΠ½ΠΎΠ΅ ΠΏΡΠΎΠΈΠ·Π²Π΅Π΄Π΅Π½ΠΈΠ΅ ΠΏΠΎΠ»ΠΎΠΆΠΈΡΠ΅Π»ΡΠ½ΠΎ.
2=0
\Leftrightarrow |\overrightarrow a|=0\).
Β
2. ΠΠ΅ΡΠ΅ΠΌΠ΅ΡΡΠΈΡΠ΅Π»ΡΠ½ΡΠΉ Π·Π°ΠΊΠΎΠ½: \(\overrightarrow a\cdot \overrightarrow b=\overrightarrow b\cdot \overrightarrow a\).
Β
3. Π Π°ΡΠΏΡΠ΅Π΄Π΅Π»ΠΈΡΠ΅Π»ΡΠ½ΡΠΉ Π·Π°ΠΊΠΎΠ½: \(\overrightarrow a \cdot (\overrightarrow b+\overrightarrow c)=\overrightarrow a\cdot \overrightarrow b+\overrightarrow a\cdot \overrightarrow c\).
Β
4. Π‘ΠΎΡΠ΅ΡΠ°ΡΠ΅Π»ΡΠ½ΡΠΉ Π·Π°ΠΊΠΎΠ½: \((\lambda\overrightarrow a)\cdot \overrightarrow b=\lambda (\overrightarrow a\cdot \overrightarrow b)\).
ΠΠ°ΠΊ Π½Π°ΠΉΡΠΈ ΠΊΠΎΠΎΡΠ΄ΠΈΠ½Π°ΡΡ Π²Π΅ΠΊΡΠΎΡΠ° Π² ΡΡΠ΅Ρ ΠΌΠ΅ΡΠ½ΠΎΠΌ ΠΏΡΠΎΡΡΡΠ°Π½ΡΡΠ²Π΅
ΠΡΠ΅ ΠΈΠ· ΡΠΊΠΎΠ»ΡΠ½ΠΎΠ³ΠΎ ΠΊΡΡΡΠ° Π°Π»Π³Π΅Π±ΡΡ ΠΈ Π³Π΅ΠΎΠΌΠ΅ΡΡΠΈΠΈ ΠΌΡ Π·Π½Π°Π΅ΠΌ ΠΎ ΠΏΠΎΠ½ΡΡΠΈΠΈ ΡΡΠ΅Ρ ΠΌΠ΅ΡΠ½ΠΎΠ³ΠΎ ΠΏΡΠΎΡΡΡΠ°Π½ΡΡΠ²Π°. ΠΡΠ»ΠΈ ΡΠ°Π·ΠΎΠ±ΡΠ°ΡΡΡΡ, ΡΠ°ΠΌ ΡΠ΅ΡΠΌΠΈΠ½ Β«ΡΡΠ΅Ρ ΠΌΠ΅ΡΠ½ΠΎΠ΅ ΠΏΡΠΎΡΡΡΠ°Π½ΡΡΠ²ΠΎΒ» ΠΎΠΏΡΠ΅Π΄Π΅Π»ΡΠ΅ΡΡΡ ΠΊΠ°ΠΊ ΡΠΈΡΡΠ΅ΠΌΠ° ΠΊΠΎΠΎΡΠ΄ΠΈΠ½Π°Ρ Ρ ΡΡΠ΅ΠΌΡ ΠΈΠ·ΠΌΠ΅ΡΠ΅Π½ΠΈΡΠΌΠΈ (ΡΡΠΎ Π·Π½Π°ΡΡ Π²ΡΠ΅). ΠΠΎ ΡΡΡΠΈ, ΠΎΠΏΠΈΡΠ°ΡΡ Π»ΡΠ±ΠΎΠΉ ΠΎΠ±ΡΠ΅ΠΌΠ½ΡΠΉ ΠΎΠ±ΡΠ΅ΠΊΡ ΠΌΠΎΠΆΠ½ΠΎ ΠΏΡΠΈ ΠΏΠΎΠΌΠΎΡΠΈ Π΄Π»ΠΈΠ½Ρ, ΡΠΈΡΠΈΠ½Ρ ΠΈ Π²ΡΡΠΎΡΡ Π² ΠΊΠ»Π°ΡΡΠΈΡΠ΅ΡΠΊΠΎΠΌ ΠΏΠΎΠ½ΠΈΠΌΠ°Π½ΠΈΠΈ. ΠΠ΄Π½Π°ΠΊΠΎ Π΄Π°Π²Π°ΠΉΡΠ΅, ΠΊΠ°ΠΊ Π³ΠΎΠ²ΠΎΡΠΈΡΡΡ, ΠΊΠΎΠΏΠ½Π΅ΠΌ Π½Π΅ΡΠΊΠΎΠ»ΡΠΊΠΎ Π³Π»ΡΠ±ΠΆΠ΅.
Π§ΡΠΎ ΡΠ°ΠΊΠΎΠ΅ ΡΡΠ΅Ρ ΠΌΠ΅ΡΠ½ΠΎΠ΅ ΠΏΡΠΎΡΡΡΠ°Π½ΡΡΠ²ΠΎ
ΠΠ°ΠΊ ΡΠΆΠ΅ ΡΡΠ°Π»ΠΎ ΡΡΠ½ΠΎ, ΠΏΠΎΠ½ΠΈΠΌΠ°Π½ΠΈΠ΅ ΡΡΠ΅Ρ
ΠΌΠ΅ΡΠ½ΠΎΠ³ΠΎ ΠΏΡΠΎΡΡΡΠ°Π½ΡΡΠ²Π° ΠΈ ΠΎΠ±ΡΠ΅ΠΊΡΠΎΠ², ΡΠΏΠΎΡΠΎΠ±Π½ΡΡ
ΡΡΡΠ΅ΡΡΠ²ΠΎΠ²Π°ΡΡ Π²Π½ΡΡΡΠΈ Π½Π΅Π³ΠΎ, ΠΎΠΏΡΠ΅Π΄Π΅Π»ΡΠ΅ΡΡΡ ΡΡΠ΅ΠΌΡ ΠΎΡΠ½ΠΎΠ²Π½ΡΠΌΠΈ ΠΏΠΎΠ½ΡΡΠΈΡΠΌΠΈ.
ΠΡΠ°Π²Π΄Π°, Π² ΡΠ»ΡΡΠ°Π΅ Ρ ΡΠΎΡΠΊΠΎΠΉ ΡΡΠΎ ΠΈΠΌΠ΅Π½Π½ΠΎ ΡΡΠΈ Π·Π½Π°ΡΠ΅Π½ΠΈΡ, Π° Π² ΡΠ»ΡΡΠ°Π΅ Ρ ΠΏΡΡΠΌΡΠΌΠΈ, ΠΊΡΠΈΠ²ΡΠΌΠΈ, Π»ΠΎΠΌΠ°Π½ΡΠΌΠΈ Π»ΠΈΠ½ΠΈΡΠΌΠΈ ΠΈΠ»ΠΈ ΠΎΠ±ΡΠ΅ΠΌΠ½ΡΠΌΠΈ ΠΎΠ±ΡΠ΅ΠΊΡΠ°ΠΌΠΈ ΡΠΎΠΎΡΠ²Π΅ΡΡΡΠ²ΡΡΡΠΈΡ
ΠΊΠΎΠΎΡΠ΄ΠΈΠ½Π°Ρ ΠΌΠΎΠΆΠ΅Ρ Π±ΡΡΡ Π±ΠΎΠ»ΡΡΠ΅.
Π Π΄Π°Π½Π½ΠΎΠΌ ΡΠ»ΡΡΠ°Π΅ Π²ΡΠ΅ Π·Π°Π²ΠΈΡΠΈΡ ΠΈΠΌΠ΅Π½Π½ΠΎ ΠΎΡ ΡΠΈΠΏΠ° ΠΎΠ±ΡΠ΅ΠΊΡΠ° ΠΈ ΠΏΡΠΈΠΌΠ΅Π½ΡΠ΅ΠΌΠΎΠΉ ΡΠΈΡΡΠ΅ΠΌΡ ΠΊΠΎΠΎΡΠ΄ΠΈΠ½Π°Ρ. Π‘Π΅Π³ΠΎΠ΄Π½Ρ Π½Π°ΠΈΠ±ΠΎΠ»Π΅Π΅ ΡΠ°ΡΠΏΡΠΎΡΡΡΠ°Π½Π΅Π½Π½ΠΎΠΉ (ΠΊΠ»Π°ΡΡΠΈΡΠ΅ΡΠΊΠΎΠΉ) ΡΡΠΈΡΠ°Π΅ΡΡΡ ΠΠ΅ΠΊΠ°ΡΡΠΎΠ²Π° ΡΠΈΡΡΠ΅ΠΌΠ°, ΠΊΠΎΡΠΎΡΡΡ ΠΈΠ½ΠΎΠ³Π΄Π° Π΅ΡΠ΅ Π½Π°Π·ΡΠ²Π°ΡΡ ΠΏΡΡΠΌΠΎΡΠ³ΠΎΠ»ΡΠ½ΠΎΠΉ. ΠΠ½Π° ΠΈ Π½Π΅ΠΊΠΎΡΠΎΡΡΠ΅ Π΄ΡΡΠ³ΠΈΠ΅ ΡΠ°Π·Π½ΠΎΠ²ΠΈΠ΄Π½ΠΎΡΡΠΈ Π±ΡΠ΄ΡΡ ΡΠ°ΡΡΠΌΠΎΡΡΠ΅Π½Ρ Π½Π΅ΡΠΊΠΎΠ»ΡΠΊΠΎ ΠΏΠΎΠ·ΠΆΠ΅.
ΠΡΠΎΠΌΠ΅ Π²ΡΠ΅Π³ΠΎ ΠΏΡΠΎΡΠ΅Π³ΠΎ, Π·Π΄Π΅ΡΡ Π½ΡΠΆΠ½ΠΎ ΡΠ°Π·Π³ΡΠ°Π½ΠΈΡΠΈΠ²Π°ΡΡ Π°Π±ΡΡΡΠ°ΠΊΡΠ½ΡΠ΅ ΠΏΠΎΠ½ΡΡΠΈΡ (Π΅ΡΠ»ΠΈ ΠΌΠΎΠΆΠ½ΠΎ ΡΠ°ΠΊ ΡΠΊΠ°Π·Π°ΡΡ, Π±Π΅ΡΡΠΎΡΠΌΠ΅Π½Π½ΡΠ΅) Π²ΡΠΎΠ΄Π΅ ΡΠΎΡΠ΅ΠΊ, ΠΏΡΡΠΌΡΡ ΠΈΠ»ΠΈ ΠΏΠ»ΠΎΡΠΊΠΎΡΡΠ΅ΠΉ ΠΈ ΡΠΈΠ³ΡΡΡ, ΠΎΠ±Π»Π°Π΄Π°ΡΡΠΈΠ΅ ΠΊΠΎΠ½Π΅ΡΠ½ΡΠΌΠΈ ΡΠ°Π·ΠΌΠ΅ΡΠ°ΠΌΠΈ ΠΈΠ»ΠΈ Π΄Π°ΠΆΠ΅ ΠΎΠ±ΡΠ΅ΠΌΠΎΠΌ. ΠΠ»Ρ ΠΊΠ°ΠΆΠ΄ΠΎΠ³ΠΎ ΠΈΠ· ΡΠ°ΠΊΠΈΡ ΠΎΠΏΡΠ΅Π΄Π΅Π»Π΅Π½ΠΈΠΉ ΡΡΡΠ΅ΡΡΠ²ΡΡΡ ΠΈ ΡΠ²ΠΎΠΈ ΡΡΠ°Π²Π½Π΅Π½ΠΈΡ, ΠΎΠΏΠΈΡΡΠ²Π°ΡΡΠΈΠ΅ ΠΈΡ Π²ΠΎΠ·ΠΌΠΎΠΆΠ½ΠΎΠ΅ ΠΏΠΎΠ»ΠΎΠΆΠ΅Π½ΠΈΠ΅ Π² ΡΡΠ΅Ρ ΠΌΠ΅ΡΠ½ΠΎΠΌ ΠΏΡΠΎΡΡΡΠ°Π½ΡΡΠ²Π΅. ΠΠΎ ΡΠ΅ΠΉΡΠ°Ρ Π½Π΅ ΠΎΠ± ΡΡΠΎΠΌ.
ΠΠΎΠ½ΡΡΠΈΠ΅ ΡΠΎΡΠΊΠΈ Π² ΡΡΠ΅Ρ ΠΌΠ΅ΡΠ½ΠΎΠΌ ΠΏΡΠΎΡΡΡΠ°Π½ΡΡΠ²Π΅
ΠΠ»Ρ Π½Π°ΡΠ°Π»Π° ΠΎΠΏΡΠ΅Π΄Π΅Π»ΠΈΠΌΡΡ, ΡΡΠΎ ΠΏΡΠ΅Π΄ΡΡΠ°Π²Π»ΡΠ΅Ρ ΡΠΎΠ±ΠΎΠΉ ΡΠΎΡΠΊΠ° Π² ΡΡΠ΅Ρ
ΠΌΠ΅ΡΠ½ΠΎΠΌ ΠΏΡΠΎΡΡΡΠ°Π½ΡΡΠ²Π΅. Π ΠΎΠ±ΡΠ΅ΠΌ-ΡΠΎ, Π΅Π΅ ΠΌΠΎΠΆΠ½ΠΎ Π½Π°Π·Π²Π°ΡΡ Π½Π΅ΠΊΠΎΠΉ ΠΎΡΠ½ΠΎΠ²Π½ΠΎΠΉ Π΅Π΄ΠΈΠ½ΠΈΡΠ΅ΠΉ, ΠΎΠΏΡΠ΅Π΄Π΅Π»ΡΡΡΠ΅ΠΉ Π»ΡΠ±ΡΡ ΠΏΠ»ΠΎΡΠΊΡΡ ΠΈΠ»ΠΈ ΠΎΠ±ΡΠ΅ΠΌΠ½ΡΡ ΡΠΈΠ³ΡΡΡ, ΠΏΡΡΠΌΡΡ, ΠΎΡΡΠ΅Π·ΠΎΠΊ, Π²Π΅ΠΊΡΠΎΡ, ΠΏΠ»ΠΎΡΠΊΠΎΡΡΡ ΠΈ Ρ.
Π΄.
Π‘Π°ΠΌΠ° ΠΆΠ΅ ΡΠΎΡΠΊΠ° Ρ Π°ΡΠ°ΠΊΡΠ΅ΡΠΈΠ·ΡΠ΅ΡΡΡ ΡΡΠ΅ΠΌΡ ΠΎΡΠ½ΠΎΠ²Π½ΡΠΌΠΈ ΠΊΠΎΠΎΡΠ΄ΠΈΠ½Π°ΡΠ°ΠΌΠΈ. ΠΠ»Ρ Π½ΠΈΡ Π² ΠΏΡΡΠΌΠΎΡΠ³ΠΎΠ»ΡΠ½ΠΎΠΉ ΡΠΈΡΡΠ΅ΠΌΠ΅ ΠΏΡΠΈΠΌΠ΅Π½ΡΡΡΡΡ ΡΠΏΠ΅ΡΠΈΠ°Π»ΡΠ½ΡΠ΅ Π½Π°ΠΏΡΠ°Π²Π»ΡΡΡΠΈΠ΅, Π½Π°Π·ΡΠ²Π°Π΅ΠΌΡΠ΅ ΠΎΡΡΠΌΠΈ X, Y ΠΈ Z, ΠΏΡΠΈΡΠ΅ΠΌ ΠΏΠ΅ΡΠ²ΡΠ΅ Π΄Π²Π΅ ΠΎΡΠΈ ΡΠ»ΡΠΆΠ°Ρ Π΄Π»Ρ Π²ΡΡΠ°ΠΆΠ΅Π½ΠΈΡ Π³ΠΎΡΠΈΠ·ΠΎΠ½ΡΠ°Π»ΡΠ½ΠΎΠ³ΠΎ ΠΏΠΎΠ»ΠΎΠΆΠ΅Π½ΠΈΡ ΠΎΠ±ΡΠ΅ΠΊΡΠ°, Π° ΡΡΠ΅ΡΡΡ ΠΎΡΠ½ΠΎΡΠΈΡΡΡ ΠΊ Π²Π΅ΡΡΠΈΠΊΠ°Π»ΡΠ½ΠΎΠΌΡ Π·Π°Π΄Π°Π½ΠΈΡ ΠΊΠΎΠΎΡΠ΄ΠΈΠ½Π°Ρ. ΠΡΡΠ΅ΡΡΠ²Π΅Π½Π½ΠΎ, Π΄Π»Ρ ΡΠ΄ΠΎΠ±ΡΡΠ²Π° Π²ΡΡΠ°ΠΆΠ΅Π½ΠΈΡ ΠΏΠΎΠ»ΠΎΠΆΠ΅Π½ΠΈΡ ΠΎΠ±ΡΠ΅ΠΊΡΠ° ΠΎΡΠ½ΠΎΡΠΈΡΠ΅Π»ΡΠ½ΠΎ Π½ΡΠ»Π΅Π²ΡΡ ΠΊΠΎΠΎΡΠ΄ΠΈΠ½Π°Ρ Π² ΡΠΈΡΡΠ΅ΠΌΠ΅ ΠΏΡΠΈΠ½ΡΡΡ ΠΏΠΎΠ»ΠΎΠΆΠΈΡΠ΅Π»ΡΠ½ΡΠ΅ ΠΈ ΠΎΡΡΠΈΡΠ°ΡΠ΅Π»ΡΠ½ΡΠ΅ Π·Π½Π°ΡΠ΅Π½ΠΈΡ. ΠΠ΄Π½Π°ΠΊΠΎ ΠΆΠ΅ ΡΠ΅Π³ΠΎΠ΄Π½Ρ ΠΌΠΎΠΆΠ½ΠΎ Π½Π°ΠΉΡΠΈ ΠΈ Π΄ΡΡΠ³ΠΈΠ΅ ΡΠΈΡΡΠ΅ΠΌΡ.
Π Π°Π·Π½ΠΎΠ²ΠΈΠ΄Π½ΠΎΡΡΠΈ ΡΠΈΡΡΠ΅ΠΌ ΠΊΠΎΠΎΡΠ΄ΠΈΠ½Π°Ρ
ΠΠ°ΠΊ ΡΠΆΠ΅ Π³ΠΎΠ²ΠΎΡΠΈΠ»ΠΎΡΡ, ΠΏΡΡΠΌΠΎΡΠ³ΠΎΠ»ΡΠ½Π°Ρ ΡΠΈΡΡΠ΅ΠΌΠ° ΠΊΠΎΠΎΡΠ΄ΠΈΠ½Π°Ρ, ΡΠΎΠ·Π΄Π°Π½Π½Π°Ρ ΠΠ΅ΠΊΠ°ΡΡΠΎΠΌ, ΡΠ΅Π³ΠΎΠ΄Π½Ρ ΡΠ²Π»ΡΠ΅ΡΡΡ ΠΎΡΠ½ΠΎΠ²Π½ΠΎΠΉ. Π’Π΅ΠΌ Π½Π΅ ΠΌΠ΅Π½Π΅Π΅ Π² Π½Π΅ΠΊΠΎΡΠΎΡΡΡ ΠΌΠ΅ΡΠΎΠ΄ΠΈΠΊΠ°Ρ Π·Π°Π΄Π°Π½ΠΈΡ ΠΌΠ΅ΡΡΠΎΠΏΠΎΠ»ΠΎΠΆΠ΅Π½ΠΈΡ ΠΎΠ±ΡΠ΅ΠΊΡΠ° Π² ΡΡΠ΅Ρ ΠΌΠ΅ΡΠ½ΠΎΠΌ ΠΏΡΠΎΡΡΡΠ°Π½ΡΡΠ²Π΅ ΠΏΡΠΈΠΌΠ΅Π½ΡΡΡΡΡ ΠΈ Π½Π΅ΠΊΠΎΡΠΎΡΡΠ΅ Π΄ΡΡΠ³ΠΈΠ΅ ΡΠ°Π·Π½ΠΎΠ²ΠΈΠ΄Π½ΠΎΡΡΠΈ.
ΠΠ°ΠΈΠ±ΠΎΠ»Π΅Π΅ ΠΈΠ·Π²Π΅ΡΡΠ½ΡΠΌΠΈ ΡΡΠΈΡΠ°ΡΡΡΡ ΡΠΈΠ»ΠΈΠ½Π΄ΡΠΈΡΠ΅ΡΠΊΠ°Ρ ΠΈ ΡΡΠ΅ΡΠΈΡΠ΅ΡΠΊΠ°Ρ ΡΠΈΡΡΠ΅ΠΌΡ. ΠΡΠ»ΠΈΡΠΈΠ΅ ΠΎΡ ΠΊΠ»Π°ΡΡΠΈΡΠ΅ΡΠΊΠΎΠΉ ΡΠΎΡΡΠΎΠΈΡ Π² ΡΠΎΠΌ, ΡΡΠΎ ΠΏΡΠΈ Π·Π°Π΄Π°Π½ΠΈΠΈ ΡΠ΅Ρ
ΠΆΠ΅ ΡΡΠ΅Ρ
Π²Π΅Π»ΠΈΡΠΈΠ½, ΠΎΠΏΡΠ΅Π΄Π΅Π»ΡΡΡΠΈΡ
ΠΌΠ΅ΡΡΠΎΠΏΠΎΠ»ΠΎΠΆΠ΅Π½ΠΈΠ΅ ΡΠΎΡΠΊΠΈ Π² ΡΡΠ΅Ρ
ΠΌΠ΅ΡΠ½ΠΎΠΌ ΠΏΡΠΎΡΡΡΠ°Π½ΡΡΠ²Π΅, ΠΎΠ΄Π½ΠΎ ΠΈΠ· Π·Π½Π°ΡΠ΅Π½ΠΈΠΉ ΡΠ²Π»ΡΠ΅ΡΡΡ ΡΠ³Π»ΠΎΠ²ΡΠΌ.
ΠΠ½ΡΠΌΠΈ ΡΠ»ΠΎΠ²Π°ΠΌΠΈ, Π² ΡΠ°ΠΊΠΈΡ
ΡΠΈΡΡΠ΅ΠΌΠ°Ρ
ΠΈΡΠΏΠΎΠ»ΡΠ·ΡΠ΅ΡΡΡ ΠΎΠΊΡΡΠΆΠ½ΠΎΡΡΡ, ΡΠΎΠΎΡΠ²Π΅ΡΡΡΠ²ΡΡΡΠ°Ρ ΡΠ³Π»Ρ Π² 360 Π³ΡΠ°Π΄ΡΡΠΎΠ². ΠΡΡΡΠ΄Π° ΠΈ ΡΠΏΠ΅ΡΠΈΡΠΈΡΠ½ΠΎΠ΅ Π·Π°Π΄Π°Π½ΠΈΠ΅ ΠΊΠΎΠΎΡΠ΄ΠΈΠ½Π°Ρ, Π²ΠΊΠ»ΡΡΠ°ΡΡΠ΅Π΅ ΡΠ°ΠΊΠΈΠ΅ ΡΠ»Π΅ΠΌΠ΅Π½ΡΡ, ΠΊΠ°ΠΊ ΡΠ°Π΄ΠΈΡΡ, ΡΠ³ΠΎΠ» ΠΈ ΠΎΠ±ΡΠ°Π·ΡΡΡΠ°Ρ. ΠΠΎΠΎΡΠ΄ΠΈΠ½Π°ΡΡ Π² ΡΡΠ΅Ρ
ΠΌΠ΅ΡΠ½ΠΎΠΌ ΠΏΡΠΎΡΡΡΠ°Π½ΡΡΠ²Π΅ (ΡΠΈΡΡΠ΅ΠΌΠ΅) ΡΠ°ΠΊΠΎΠ³ΠΎ ΡΠΈΠΏΠ° ΠΏΠΎΠ΄ΡΠΈΠ½ΡΡΡΡΡ Π½Π΅ΡΠΊΠΎΠ»ΡΠΊΠΎ Π΄ΡΡΠ³ΠΈΠΌ Π·Π°ΠΊΠΎΠ½ΠΎΠΌΠ΅ΡΠ½ΠΎΡΡΡΠΌ. ΠΡ
Π·Π°Π΄Π°Π½ΠΈΠ΅ Π² Π΄Π°Π½Π½ΠΎΠΌ ΡΠ»ΡΡΠ°Π΅ ΠΊΠΎΠ½ΡΡΠΎΠ»ΠΈΡΡΠ΅ΡΡΡ ΠΏΡΠ°Π²ΠΈΠ»ΠΎΠΌ ΠΏΡΠ°Π²ΠΎΠΉ ΡΡΠΊΠΈ: Π΅ΡΠ»ΠΈ ΡΠΎΠ²ΠΌΠ΅ΡΡΠΈΡΡ Π±ΠΎΠ»ΡΡΠΎΠΉ ΠΈ ΡΠΊΠ°Π·Π°ΡΠ΅Π»ΡΠ½ΡΠΉ ΠΏΠ°Π»Π΅Ρ Ρ ΠΎΡΡΠΌΠΈ X ΠΈ Y, ΡΠΎΠΎΡΠ²Π΅ΡΡΡΠ²Π΅Π½Π½ΠΎ, ΠΎΡΡΠ°Π»ΡΠ½ΡΠ΅ ΠΏΠ°Π»ΡΡΡ Π² ΠΈΠ·ΠΎΠ³Π½ΡΡΠΎΠΌ ΠΏΠΎΠ»ΠΎΠΆΠ΅Π½ΠΈΠΈ ΡΠΊΠ°ΠΆΡΡ Π½Π° Π½Π°ΠΏΡΠ°Π²Π»Π΅Π½ΠΈΠ΅ ΠΎΡΠΈ Z.
ΠΠΎΠ½ΡΡΠΈΠ΅ ΠΏΡΡΠΌΠΎΠΉ Π² ΡΡΠ΅Ρ ΠΌΠ΅ΡΠ½ΠΎΠΌ ΠΏΡΠΎΡΡΡΠ°Π½ΡΡΠ²Π΅
Π’Π΅ΠΏΠ΅ΡΡ Π½Π΅ΡΠΊΠΎΠ»ΡΠΊΠΎ ΡΠ»ΠΎΠ² ΠΎ ΡΠΎΠΌ, ΡΡΠΎ ΠΏΡΠ΅Π΄ΡΡΠ°Π²Π»ΡΠ΅Ρ ΡΠΎΠ±ΠΎΠΉ ΠΏΡΡΠΌΠ°Ρ Π² ΡΡΠ΅Ρ ΠΌΠ΅ΡΠ½ΠΎΠΌ ΠΏΡΠΎΡΡΡΠ°Π½ΡΡΠ²Π΅. ΠΡΡ ΠΎΠ΄Ρ ΠΈΠ· ΠΎΡΠ½ΠΎΠ²Π½ΠΎΠ³ΠΎ ΠΏΠΎΠ½ΡΡΠΈΡ ΠΏΡΡΠΌΠΎΠΉ, ΡΡΠΎ Π½Π΅ΠΊΠ°Ρ Π±Π΅ΡΠΊΠΎΠ½Π΅ΡΠ½Π°Ρ Π»ΠΈΠ½ΠΈΡ, ΠΏΡΠΎΠ²Π΅Π΄Π΅Π½Π½Π°Ρ ΡΠ΅ΡΠ΅Π· ΡΠΎΡΠΊΡ ΠΈΠ»ΠΈ Π΄Π²Π΅, Π½Π΅ ΡΡΠΈΡΠ°Ρ ΠΌΠ½ΠΎΠΆΠ΅ΡΡΠ²Π° ΡΠΎΡΠ΅ΠΊ, ΡΠ°ΡΠΏΠΎΠ»ΠΎΠΆΠ΅Π½Π½ΡΡ Π² ΠΏΠΎΡΠ»Π΅Π΄ΠΎΠ²Π°ΡΠ΅Π»ΡΠ½ΠΎΡΡΠΈ, Π½Π΅ ΠΈΠ·ΠΌΠ΅Π½ΡΡΡΠ΅ΠΉ ΠΏΡΡΠΌΠΎΠ΅ ΠΏΡΠΎΡ ΠΎΠΆΠ΄Π΅Π½ΠΈΠ΅ Π»ΠΈΠ½ΠΈΠΈ ΡΠ΅ΡΠ΅Π· Π½ΠΈΡ .
ΠΡΠ»ΠΈ ΠΏΠΎΡΠΌΠΎΡΡΠ΅ΡΡ Π½Π° ΠΏΡΡΠΌΡΡ, ΠΏΡΠΎΠ²Π΅Π΄Π΅Π½Π½ΡΡ ΡΠ΅ΡΠ΅Π· Π΄Π²Π΅ ΡΠΎΡΠΊΠΈ Π² ΡΡΠ΅Ρ
ΠΌΠ΅ΡΠ½ΠΎΠΌ ΠΏΡΠΎΡΡΡΠ°Π½ΡΡΠ²Π΅, ΠΏΡΠΈΠ΄Π΅ΡΡΡ ΡΡΠΈΡΡΠ²Π°ΡΡ ΠΏΠΎ ΡΡΠΈ ΠΊΠΎΠΎΡΠ΄ΠΈΠ½Π°ΡΡ ΠΎΠ±Π΅ΠΈΡ
ΡΠΎΡΠ΅ΠΊ.
Π’ΠΎ ΠΆΠ΅ ΡΠ°ΠΌΠΎΠ΅ ΠΎΡΠ½ΠΎΡΠΈΡΡΡ ΠΊ ΠΎΡΡΠ΅Π·ΠΊΠ°ΠΌ ΠΈ Π²Π΅ΠΊΡΠΎΡΠ°ΠΌ. ΠΠΎΡΠ»Π΅Π΄Π½ΠΈΠ΅ ΠΎΠΏΡΠ΅Π΄Π΅Π»ΡΡΡ Π±Π°Π·ΠΈΡ ΡΡΠ΅Ρ
ΠΌΠ΅ΡΠ½ΠΎΠ³ΠΎ ΠΏΡΠΎΡΡΡΠ°Π½ΡΡΠ²Π° ΠΈ Π΅Π³ΠΎ ΡΠ°Π·ΠΌΠ΅ΡΠ½ΠΎΡΡΡ.
ΠΠΏΡΠ΅Π΄Π΅Π»Π΅Π½ΠΈΠ΅ Π²Π΅ΠΊΡΠΎΡΠΎΠ² ΠΈ Π±Π°Π·ΠΈΡΠ° ΡΡΠ΅Ρ ΠΌΠ΅ΡΠ½ΠΎΠ³ΠΎ ΠΏΡΠΎΡΡΡΠ°Π½ΡΡΠ²Π°
ΠΠ°ΠΊ ΠΏΡΠΈΠ½ΡΡΠΎ ΡΡΠΈΡΠ°ΡΡ, Π² ΡΡΠ΅Ρ ΠΌΠ΅ΡΠ½ΠΎΠΉ ΡΠΈΡΡΠ΅ΠΌΠ΅ ΠΊΠΎΠΎΡΠ΄ΠΈΠ½Π°Ρ ΠΌΠΎΠΆΠ΅Ρ ΡΡΡΠ΅ΡΡΠ²ΠΎΠ²Π°ΡΡ ΡΡΠΈ ΠΎΡΠ½ΠΎΠ²Π½ΡΡ Π²Π΅ΠΊΡΠΎΡΠ°, ΠΊΠΎΡΠΎΡΡΠ΅ ΠΎΠΏΡΠ΅Π΄Π΅Π»ΡΡΡ Π±Π°Π·ΠΈΡ. ΠΡΠΈ ΡΡΠΎΠΌ Π±Π°Π·ΠΈΡΠΎΠ² Ρ ΡΠΎΠΎΡΠ²Π΅ΡΡΡΠ²ΡΡΡΠΈΠΌΠΈ Π½Π΅Π·Π°Π²ΠΈΡΠΈΠΌΡΠΌΠΈ ΡΡΠ΅ΠΌΡ Π²Π΅ΠΊΡΠΎΡΠ°ΠΌΠΈ ΠΌΠΎΠΆΠ΅Ρ Π±ΡΡΡ Π±Π΅ΡΡΠΈΡΠ»Π΅Π½Π½ΠΎΠ΅ ΠΌΠ½ΠΎΠΆΠ΅ΡΡΠ²ΠΎ.
ΠΠ°ΠΌΠ΅ΡΡΡΠ΅, ΡΡΠΎ ΠΌΠΎΠ³ΡΡ Π±ΡΡΡ ΡΠΎΠ»ΡΠΊΠΎ ΡΡΠΈ Π²Π΅ΠΊΡΠΎΡΠ°, Π½ΠΎ Π²ΠΎΡ ΡΡΠΎΠ΅ΠΊ Π²Π΅ΠΊΡΠΎΡΠΎΠ² ΠΌΠΎΠΆΠ½ΠΎ ΠΎΠΏΡΠ΅Π΄Π΅Π»ΠΈΡΡ ΡΠΊΠΎΠ»ΡΠΊΠΎ ΡΠ³ΠΎΠ΄Π½ΠΎ. Π Π°Π·ΠΌΠ΅ΡΠ½ΠΎΡΡΡ ΠΏΡΠΎΡΡΡΠ°Π½ΡΡΠ²Π° ΠΎΠΏΡΠ΅Π΄Π΅Π»ΡΠ΅ΡΡΡ ΠΊΠΎΠ»ΠΈΡΠ΅ΡΡΠ²ΠΎΠΌ Π»ΠΈΠ½Π΅ΠΉΠ½ΠΎ-Π½Π΅Π·Π°Π²ΠΈΡΠΈΠΌΡΡ Π²Π΅ΠΊΡΠΎΡΠΎΠ² (Π² Π½Π°ΡΠ΅ΠΌ ΡΠ»ΡΡΠ°Π΅ β ΡΡΠΈ). Π ΠΏΡΠΎΡΡΡΠ°Π½ΡΡΠ²ΠΎ, Π² ΠΊΠΎΡΠΎΡΠΎΠΌ ΠΈΠΌΠ΅Π΅ΡΡΡ ΠΊΠΎΠ½Π΅ΡΠ½ΠΎΠ΅ ΡΠΈΡΠ»ΠΎ ΡΠ°ΠΊΠΈΡ Π²Π΅ΠΊΡΠΎΡΠΎΠ², Π½Π°Π·ΡΠ²Π°Π΅ΡΡΡ ΠΊΠΎΠ½Π΅ΡΠ½ΠΎΠΌΠ΅ΡΠ½ΡΠΌ.
ΠΠ°Π²ΠΈΡΠΈΠΌΡΠ΅ ΠΈ Π½Π΅Π·Π°Π²ΠΈΡΠΈΠΌΡΠ΅ Π²Π΅ΠΊΡΠΎΡΡ
Π§ΡΠΎ ΠΊΠ°ΡΠ°Π΅ΡΡΡ ΠΎΠΏΡΠ΅Π΄Π΅Π»Π΅Π½ΠΈΡ Π·Π°Π²ΠΈΡΠΈΠΌΡΡ ΠΈ Π½Π΅Π·Π°Π²ΠΈΡΠΈΠΌΡΡ Π²Π΅ΠΊΡΠΎΡΠΎΠ², Π»ΠΈΠ½Π΅ΠΉΠ½ΠΎ-Π½Π΅Π·Π°Π²ΠΈΡΠΈΠΌΡΠΌΠΈ ΠΏΡΠΈΠ½ΡΡΠΎ ΡΡΠΈΡΠ°ΡΡ Π²Π΅ΠΊΡΠΎΡΡ, ΡΠ²Π»ΡΡΡΠΈΠ΅ΡΡ ΠΏΡΠΎΠ΅ΠΊΡΠΈΡΠΌΠΈ (Π½Π°ΠΏΡΠΈΠΌΠ΅Ρ, Π²Π΅ΠΊΡΠΎΡΡ ΠΎΡΠΈ X, ΡΠΏΡΠΎΠ΅ΡΠΈΡΠΎΠ²Π°Π½Π½ΡΠ΅ Π½Π° ΠΎΡΡ Y).
ΠΠ°ΠΊ ΡΠΆΠ΅ ΠΏΠΎΠ½ΡΡΠ½ΠΎ, Π»ΡΠ±ΠΎΠΉ ΡΠ΅ΡΠ²Π΅ΡΡΡΠΉ Π²Π΅ΠΊΡΠΎΡ ΡΠ²Π»ΡΠ΅ΡΡΡ Π·Π°Π²ΠΈΡΠΈΠΌΡΠΌ (ΡΠ΅ΠΎΡΠΈΡ Π»ΠΈΠ½Π΅ΠΉΠ½ΡΡ
ΠΏΡΠΎΡΡΡΠ°Π½ΡΡΠ²).
Π Π²ΠΎΡ ΡΡΠΈ Π½Π΅Π·Π°Π²ΠΈΡΠΈΠΌΡΡ
Π²Π΅ΠΊΡΠΎΡΠ° Π² ΡΡΠ΅Ρ
ΠΌΠ΅ΡΠ½ΠΎΠΌ ΠΏΡΠΎΡΡΡΠ°Π½ΡΡΠ²Π΅ Π² ΠΎΠ±ΡΠ·Π°ΡΠ΅Π»ΡΠ½ΠΎΠΌ ΠΏΠΎΡΡΠ΄ΠΊΠ΅ Π½Π΅ Π΄ΠΎΠ»ΠΆΠ½Ρ Π»Π΅ΠΆΠ°ΡΡ Π² ΠΎΠ΄Π½ΠΎΠΉ ΠΏΠ»ΠΎΡΠΊΠΎΡΡΠΈ. ΠΡΠΎΠΌΠ΅ ΡΠΎΠ³ΠΎ, Π΅ΡΠ»ΠΈ ΠΎΠΏΡΠ΅Π΄Π΅Π»ΡΡΡ Π½Π΅Π·Π°Π²ΠΈΡΠΈΠΌΡΠ΅ Π²Π΅ΠΊΡΠΎΡΡ Π² ΡΡΠ΅Ρ
ΠΌΠ΅ΡΠ½ΠΎΠΌ ΠΏΡΠΎΡΡΡΠ°Π½ΡΡΠ²Π΅, ΠΎΠ½ΠΈ Π½Π΅ ΠΌΠΎΠ³ΡΡ ΡΠ²Π»ΡΡΡΡΡ, ΡΠ°ΠΊ ΡΠΊΠ°Π·Π°ΡΡ, ΠΎΠ΄ΠΈΠ½ ΠΏΡΠΎΠ΄ΠΎΠ»ΠΆΠ΅Π½ΠΈΠ΅ΠΌ Π΄ΡΡΠ³ΠΎΠ³ΠΎ. ΠΠ°ΠΊ ΡΠΆΠ΅ ΠΏΠΎΠ½ΡΡΠ½ΠΎ, Π² ΡΠ°ΡΡΠΌΠ°ΡΡΠΈΠ²Π°Π΅ΠΌΠΎΠΌ Π½Π°ΠΌΠΈ ΡΠ»ΡΡΠ°Π΅ Ρ ΡΡΠ΅ΠΌΡ ΠΈΠ·ΠΌΠ΅ΡΠ΅Π½ΠΈΡΠΌΠΈ, ΡΠΎΠ³Π»Π°ΡΠ½ΠΎ ΠΎΠ±ΡΠ΅ΠΉ ΡΠ΅ΠΎΡΠΈΠΈ, ΠΌΠΎΠΆΠ½ΠΎ ΠΏΠΎΡΡΡΠΎΠΈΡΡ ΠΈΡΠΊΠ»ΡΡΠΈΡΠ΅Π»ΡΠ½ΠΎ ΡΠΎΠ»ΡΠΊΠΎ ΡΡΠΎΠΉΠΊΠΈ Π»ΠΈΠ½Π΅ΠΉΠ½ΠΎ-Π½Π΅Π·Π°Π²ΠΈΡΠΈΠΌΡΡ
Π²Π΅ΠΊΡΠΎΡΠΎΠ² Π² ΠΎΠΏΡΠ΅Π΄Π΅Π»Π΅Π½Π½ΠΎΠΉ ΡΠΈΡΡΠ΅ΠΌΠ΅ ΠΊΠΎΠΎΡΠ΄ΠΈΠ½Π°Ρ (Π±Π΅Π· ΡΠ°Π·Π½ΠΈΡΡ, ΠΊΠ°ΠΊΠΎΠ³ΠΎ ΡΠΈΠΏΠ°).
ΠΠ»ΠΎΡΠΊΠΎΡΡΡ Π² ΡΡΠ΅Ρ ΠΌΠ΅ΡΠ½ΠΎΠΌ ΠΏΡΠΎΡΡΡΠ°Π½ΡΡΠ²Π΅
ΠΡΠ»ΠΈ ΡΠ°ΡΡΠΌΠ°ΡΡΠΈΠ²Π°ΡΡ ΠΏΠΎΠ½ΡΡΠΈΠ΅ ΠΏΠ»ΠΎΡΠΊΠΎΡΡΠΈ, Π½Π΅ Π²Π΄Π°Π²Π°ΡΡΡ Π² ΠΌΠ°ΡΠ΅ΠΌΠ°ΡΠΈΡΠ΅ΡΠΊΠΈΠ΅ ΠΎΠΏΡΠ΅Π΄Π΅Π»Π΅Π½ΠΈΡ, Π΄Π»Ρ Π±ΠΎΠ»Π΅Π΅ ΠΏΡΠΎΡΡΠΎΠ³ΠΎ ΠΏΠΎΠ½ΠΈΠΌΠ°Π½ΠΈΡ ΡΡΠΎΠ³ΠΎ ΡΠ΅ΡΠΌΠΈΠ½Π°, ΡΠ°ΠΊΠΎΠΉ ΠΎΠ±ΡΠ΅ΠΊΡ ΠΌΠΎΠΆΠ½ΠΎ ΡΠ°ΡΡΠΌΠ°ΡΡΠΈΠ²Π°ΡΡ ΠΈΡΠΊΠ»ΡΡΠΈΡΠ΅Π»ΡΠ½ΠΎ ΠΊΠ°ΠΊ Π΄Π²ΡΠΌΠ΅ΡΠ½ΡΠΉ. ΠΠ½ΡΠΌΠΈ ΡΠ»ΠΎΠ²Π°ΠΌΠΈ, ΡΡΠΎ Π±Π΅ΡΠΊΠΎΠ½Π΅ΡΠ½Π°Ρ ΡΠΎΠ²ΠΎΠΊΡΠΏΠ½ΠΎΡΡΡ ΡΠΎΡΠ΅ΠΊ, Ρ ΠΊΠΎΡΠΎΡΡΡ ΠΎΠ΄Π½Π° ΠΈΠ· ΠΊΠΎΠΎΡΠ΄ΠΈΠ½Π°Ρ ΡΠ²Π»ΡΠ΅ΡΡΡ ΠΏΠΎΡΡΠΎΡΠ½Π½ΠΎΠΉ (ΠΊΠΎΠ½ΡΡΠ°Π½ΡΠΎΠΉ).
Π ΠΏΡΠΈΠΌΠ΅ΡΡ, ΠΏΠ»ΠΎΡΠΊΠΎΡΡΡΡ ΠΌΠΎΠΆΠ½ΠΎ Π½Π°Π·Π²Π°ΡΡ Π»ΡΠ±ΠΎΠ΅ ΠΊΠΎΠ»ΠΈΡΠ΅ΡΡΠ²ΠΎ ΡΠΎΡΠ΅ΠΊ Ρ ΡΠ°Π·Π½ΡΠΌΠΈ ΠΊΠΎΠΎΡΠ΄ΠΈΠ½Π°ΡΠ°ΠΌΠΈ ΠΏΠΎ ΠΎΡΡΠΌ X ΠΈ Y, Π½ΠΎ ΠΎΠ΄ΠΈΠ½Π°ΠΊΠΎΠ²ΡΠΌΠΈ ΠΊΠΎΠΎΡΠ΄ΠΈΠ½Π°ΡΠ°ΠΌΠΈ ΠΏΠΎ ΠΎΡΠΈ Z. Π Π»ΡΠ±ΠΎΠΌ ΡΠ»ΡΡΠ°Π΅ ΠΎΠ΄Π½Π° ΠΈΠ· ΡΡΠ΅Ρ
ΠΌΠ΅ΡΠ½ΡΡ
ΠΊΠΎΠΎΡΠ΄ΠΈΠ½Π°Ρ ΠΎΡΡΠ°Π΅ΡΡΡ Π½Π΅ΠΈΠ·ΠΌΠ΅Π½Π½ΠΎΠΉ.
ΠΠ΄Π½Π°ΠΊΠΎ ΡΡΠΎ, ΡΠ°ΠΊ ΡΠΊΠ°Π·Π°ΡΡ, ΠΎΠ±ΡΠΈΠΉ ΡΠ»ΡΡΠ°ΠΉ. Π Π½Π΅ΠΊΠΎΡΠΎΡΡΡ
ΡΠΈΡΡΠ°ΡΠΈΡΡ
ΡΡΠ΅Ρ
ΠΌΠ΅ΡΠ½ΠΎΠ΅ ΠΏΡΠΎΡΡΡΠ°Π½ΡΡΠ²ΠΎ ΠΌΠΎΠΆΠ΅Ρ ΠΏΠ΅ΡΠ΅ΡΠ΅ΠΊΠ°ΡΡΡΡ ΠΏΠ»ΠΎΡΠΊΠΎΡΡΡΡ ΠΏΠΎ Π²ΡΠ΅ΠΌ ΠΎΡΡΠΌ.
Π‘ΡΡΠ΅ΡΡΠ²ΡΠ΅Ρ Π»ΠΈ Π±ΠΎΠ»Π΅Π΅ ΡΡΠ΅Ρ ΠΈΠ·ΠΌΠ΅ΡΠ΅Π½ΠΈΠΉ
ΠΠΎΠΏΡΠΎΡ ΠΎ ΡΠΎΠΌ, ΡΠΊΠΎΠ»ΡΠΊΠΎ ΠΌΠΎΠΆΠ΅Ρ ΡΡΡΠ΅ΡΡΠ²ΠΎΠ²Π°ΡΡ ΠΈΠ·ΠΌΠ΅ΡΠ΅Π½ΠΈΠΉ, Π΄ΠΎΡΡΠ°ΡΠΎΡΠ½ΠΎ ΠΈΠ½ΡΠ΅ΡΠ΅ΡΠ΅Π½. ΠΠ°ΠΊ ΡΡΠΈΡΠ°Π΅ΡΡΡ, ΠΌΡ ΠΆΠΈΠ²Π΅ΠΌ Π½Π΅ Π² ΡΡΠ΅Ρ ΠΌΠ΅ΡΠ½ΠΎΠΌ Ρ ΠΊΠ»Π°ΡΡΠΈΡΠ΅ΡΠΊΠΎΠΉ ΡΠΎΡΠΊΠΈ Π·ΡΠ΅Π½ΠΈΡ ΠΏΡΠΎΡΡΡΠ°Π½ΡΡΠ²Π΅, Π° Π² ΡΠ΅ΡΡΡΠ΅Ρ ΠΌΠ΅ΡΠ½ΠΎΠΌ. ΠΡΠΎΠΌΠ΅ ΠΈΠ·Π²Π΅ΡΡΠ½ΡΡ Π²ΡΠ΅ΠΌ Π΄Π»ΠΈΠ½Ρ, ΡΠΈΡΠΈΠ½Ρ ΠΈ Π²ΡΡΠΎΡΡ, ΡΠ°ΠΊΠΎΠ΅ ΠΏΡΠΎΡΡΡΠ°Π½ΡΡΠ²ΠΎ Π²ΠΊΠ»ΡΡΠ°Π΅Ρ Π² ΡΠ΅Π±Ρ Π΅ΡΠ΅ ΠΈ Π²ΡΠ΅ΠΌΡ ΡΡΡΠ΅ΡΡΠ²ΠΎΠ²Π°Π½ΠΈΡ ΠΎΠ±ΡΠ΅ΠΊΡΠ°, ΠΏΡΠΈΡΠ΅ΠΌ Π²ΡΠ΅ΠΌΡ ΠΈ ΠΏΡΠΎΡΡΡΠ°Π½ΡΡΠ²ΠΎ ΠΌΠ΅ΠΆΠ΄Ρ ΡΠΎΠ±ΠΎΠΉ Π²Π·Π°ΠΈΠΌΠΎΡΠ²ΡΠ·Π°Π½Ρ Π΄ΠΎΡΡΠ°ΡΠΎΡΠ½ΠΎ ΡΠΈΠ»ΡΠ½ΠΎ. ΠΡΠΎ Π΄ΠΎΠΊΠ°Π·Π°Π» Π΅ΡΠ΅ ΠΠΉΠ½ΡΡΠ΅ΠΉΠ½ Π² ΡΠ²ΠΎΠ΅ΠΉ ΡΠ΅ΠΎΡΠΈΠΈ ΠΎΡΠ½ΠΎΡΠΈΡΠ΅Π»ΡΠ½ΠΎΡΡΠΈ, Ρ ΠΎΡΡ ΡΡΠΎ Π±ΠΎΠ»ΡΡΠ΅ ΠΎΡΠ½ΠΎΡΠΈΡΡΡ ΠΊ ΡΠΈΠ·ΠΈΠΊΠ΅, Π½Π΅ΠΆΠ΅Π»ΠΈ ΠΊ Π°Π»Π³Π΅Π±ΡΠ΅ ΠΈ Π³Π΅ΠΎΠΌΠ΅ΡΡΠΈΠΈ.
ΠΠ½ΡΠ΅ΡΠ΅ΡΠ΅Π½ ΠΈ ΡΠΎΡ ΡΠ°ΠΊΡ, ΡΡΠΎ ΡΠ΅Π³ΠΎΠ΄Π½Ρ ΡΡΠ΅Π½ΡΠ΅ ΡΠΆΠ΅ Π΄ΠΎΠΊΠ°Π·Π°Π»ΠΈ ΡΡΡΠ΅ΡΡΠ²ΠΎΠ²Π°Π½ΠΈΠ΅ ΠΊΠ°ΠΊ ΠΌΠΈΠ½ΠΈΠΌΡΠΌ Π΄Π²Π΅Π½Π°Π΄ΡΠ°ΡΠΈ ΠΈΠ·ΠΌΠ΅ΡΠ΅Π½ΠΈΠΉ. ΠΠΎΠ½Π΅ΡΠ½ΠΎ, ΠΏΠΎΠ½ΡΡΡ, ΡΡΠΎ ΠΎΠ½ΠΈ ΡΠΎΠ±ΠΎΠΉ ΠΏΡΠ΅Π΄ΡΡΠ°Π²Π»ΡΡΡ, ΡΠΌΠΎΠΆΠ΅Ρ Π΄Π°Π»Π΅ΠΊΠΎ Π½Π΅ ΠΊΠ°ΠΆΠ΄ΡΠΉ, ΠΏΠΎΡΠΊΠΎΠ»ΡΠΊΡ ΡΡΠΎ ΠΎΡΠ½ΠΎΡΠΈΡΡΡ ΡΠΊΠΎΡΠ΅Π΅ ΠΊ Π½Π΅ΠΊΠΎΠΉ Π°Π±ΡΡΡΠ°ΠΊΡΠ½ΠΎΠΉ ΠΎΠ±Π»Π°ΡΡΠΈ, ΠΊΠΎΡΠΎΡΠ°Ρ Π½Π°Ρ
ΠΎΠ΄ΠΈΡΡΡ Π²Π½Π΅ ΡΠ΅Π»ΠΎΠ²Π΅ΡΠ΅ΡΠΊΠΎΠ³ΠΎ Π²ΠΎΡΠΏΡΠΈΡΡΠΈΡ ΠΌΠΈΡΠ°. Π’Π΅ΠΌ Π½Π΅ ΠΌΠ΅Π½Π΅Π΅ ΡΠ°ΠΊΡ ΠΎΡΡΠ°Π΅ΡΡΡ ΡΠ°ΠΊΡΠΎΠΌ. Π Π½Π΅ Π·ΡΡ ΠΆΠ΅ ΠΌΠ½ΠΎΠ³ΠΈΠ΅ Π°Π½ΡΡΠΎΠΏΠΎΠ»ΠΎΠ³ΠΈ ΠΈ ΠΈΡΡΠΎΡΠΈΠΊΠΈ ΡΡΠ²Π΅ΡΠΆΠ΄Π°ΡΡ, ΡΡΠΎ Π½Π°ΡΠΈ ΠΏΡΠ°ΡΡΡΡ ΠΌΠΎΠ³Π»ΠΈ ΠΈΠΌΠ΅ΡΡ Π½Π΅ΠΊΠΈΠ΅ ΡΠΏΠ΅ΡΠΈΡΠΈΡΠ½ΡΠ΅ ΡΠ°Π·Π²ΠΈΡΡΠ΅ ΠΎΡΠ³Π°Π½Ρ ΡΡΠ²ΡΡΠ² Π²ΡΠΎΠ΄Π΅ ΡΡΠ΅ΡΡΠ΅Π³ΠΎ Π³Π»Π°Π·Π°, ΠΊΠΎΡΠΎΡΡΠ΅ ΠΏΠΎΠΌΠΎΠ³Π°Π»ΠΈ Π²ΠΎΡΠΏΡΠΈΠ½ΠΈΠΌΠ°ΡΡ ΠΌΠ½ΠΎΠ³ΠΎΠΌΠ΅ΡΠ½ΡΡ Π΄Π΅ΠΉΡΡΠ²ΠΈΡΠ΅Π»ΡΠ½ΠΎΡΡΡ, Π° Π½Π΅ ΠΈΡΠΊΠ»ΡΡΠΈΡΠ΅Π»ΡΠ½ΠΎ ΡΡΠ΅Ρ
ΠΌΠ΅ΡΠ½ΠΎΠ΅ ΠΏΡΠΎΡΡΡΠ°Π½ΡΡΠ²ΠΎ.
ΠΡΡΠ°ΡΠΈ ΡΠΊΠ°Π·Π°ΡΡ, ΡΠ΅Π³ΠΎΠ΄Π½Ρ ΡΡΡΠ΅ΡΡΠ²ΡΠ΅Ρ Π΄ΠΎΡΡΠ°ΡΠΎΡΠ½ΠΎ ΠΌΠ½ΠΎΠ³ΠΎ ΠΌΠ½Π΅Π½ΠΈΠΉ ΠΏΠΎ ΠΏΠΎΠ²ΠΎΠ΄Ρ ΡΠΎΠ³ΠΎ, ΡΡΠΎ ΡΠΊΡΡΡΠ°ΡΠ΅Π½ΡΠΎΡΠΈΠΊΠ° ΡΠΎΠΆΠ΅ ΡΠ²Π»ΡΠ΅ΡΡΡ ΠΎΠ΄Π½ΠΈΠΌ ΠΈΠ· ΠΏΡΠΎΡΠ²Π»Π΅Π½ΠΈΠΉ Π²ΠΎΡΠΏΡΠΈΡΡΠΈΡ ΠΌΠ½ΠΎΠ³ΠΎΠΌΠ΅ΡΠ½ΠΎΠ³ΠΎ ΠΌΠΈΡΠ°, ΠΈ ΡΠΎΠΌΡ ΠΌΠΎΠΆΠ½ΠΎ Π½Π°ΠΉΡΠΈ Π΄ΠΎΡΡΠ°ΡΠΎΡΠ½ΠΎ ΠΌΠ½ΠΎΠ³ΠΎ ΠΏΠΎΠ΄ΡΠ²Π΅ΡΠΆΠ΄Π΅Π½ΠΈΠΉ.
ΠΠ°ΠΌΠ΅ΡΡΡΠ΅, ΡΡΠΎ ΡΠΎΠ²ΡΠ΅ΠΌΠ΅Π½Π½ΡΠΌΠΈ Π±Π°Π·ΠΎΠ²ΡΠΌΠΈ ΡΡΠ°Π²Π½Π΅Π½ΠΈΡΠΌΠΈ ΠΈ ΡΠ΅ΠΎΡΠ΅ΠΌΠ°ΠΌΠΈ ΠΎΠΏΠΈΡΠ°ΡΡ ΠΌΠ½ΠΎΠ³ΠΎΠΌΠ΅ΡΠ½ΡΠ΅ ΠΏΡΠΎΡΡΡΠ°Π½ΡΡΠ²Π°, ΠΎΡΠ»ΠΈΡΠ°ΡΡΠΈΠ΅ΡΡ ΠΎΡ Π½Π°ΡΠ΅Π³ΠΎ ΡΠ΅ΡΡΡΠ΅Ρ ΠΌΠ΅ΡΠ½ΠΎΠ³ΠΎ ΠΌΠΈΡΠ°, ΡΠΎΠΆΠ΅ Π½Π΅ Π²ΡΠ΅Π³Π΄Π° ΠΏΡΠ΅Π΄ΡΡΠ°Π²Π»ΡΠ΅ΡΡΡ Π²ΠΎΠ·ΠΌΠΎΠΆΠ½ΡΠΌ. ΠΠ° ΠΈ Π½Π°ΡΠΊΠ° Π² ΡΡΠΎΠΉ ΠΎΠ±Π»Π°ΡΡΠΈ ΠΎΡΠ½ΠΎΡΠΈΡΡΡ ΡΠΊΠΎΡΠ΅Π΅ ΠΊ ΠΎΠ±Π»Π°ΡΡΠΈ ΡΠ΅ΠΎΡΠΈΠΉ ΠΈ ΠΏΡΠ΅Π΄ΠΏΠΎΠ»ΠΎΠΆΠ΅Π½ΠΈΠΉ, Π½Π΅ΠΆΠ΅Π»ΠΈ ΠΊ ΡΠΎΠΌΡ, ΡΡΠΎ ΠΌΠΎΠΆΠ½ΠΎ ΡΠ²Π½ΠΎ ΠΎΡΡΡΠΈΡΡ ΠΈΠ»ΠΈ, ΡΠ°ΠΊ ΡΠΊΠ°Π·Π°ΡΡ, ΠΏΠΎΡΡΠΎΠ³Π°ΡΡ ΠΈΠ»ΠΈ ΡΠ²ΠΈΠ΄Π΅ΡΡ Π²ΠΎΠΎΡΠΈΡ. Π’Π΅ΠΌ Π½Π΅ ΠΌΠ΅Π½Π΅Π΅ ΠΊΠΎΡΠ²Π΅Π½Π½ΡΠ΅ Π΄ΠΎΠΊΠ°Π·Π°ΡΠ΅Π»ΡΡΡΠ²Π° ΡΡΡΠ΅ΡΡΠ²ΠΎΠ²Π°Π½ΠΈΡ ΠΌΠ½ΠΎΠ³ΠΎΠΌΠ΅ΡΠ½ΡΡ ΠΌΠΈΡΠΎΠ², Π² ΠΊΠΎΡΠΎΡΡΡ ΠΌΠΎΠΆΠ΅Ρ ΡΡΡΠ΅ΡΡΠ²ΠΎΠ²Π°ΡΡ ΡΠ΅ΡΡΡΠ΅ ΠΈ Π±ΠΎΠ»Π΅Π΅ ΠΈΠ·ΠΌΠ΅ΡΠ΅Π½ΠΈΠΉ, ΡΠ΅Π³ΠΎΠ΄Π½Ρ Π½ΠΈ Ρ ΠΊΠΎΠ³ΠΎ Π½Π΅ Π²ΡΠ·ΡΠ²Π°ΡΡ ΡΠΎΠΌΠ½Π΅Π½ΠΈΠΉ.
ΠΠ°ΠΊΠ»ΡΡΠ΅Π½ΠΈΠ΅
Π ΡΠ΅Π»ΠΎΠΌ ΠΆΠ΅, ΠΌΡ ΠΎΡΠ΅Π½Ρ ΠΊΡΠ°ΡΠΊΠΎ ΡΠ°ΡΡΠΌΠΎΡΡΠ΅Π»ΠΈ ΠΎΡΠ½ΠΎΠ²Π½ΡΠ΅ ΠΏΠΎΠ½ΡΡΠΈΡ, ΠΎΡΠ½ΠΎΡΡΡΠΈΠ΅ΡΡ ΠΊ ΡΡΠ΅Ρ
ΠΌΠ΅ΡΠ½ΠΎΠΌΡ ΠΏΡΠΎΡΡΡΠ°Π½ΡΡΠ²Ρ ΠΈ Π±Π°Π·ΠΎΠ²ΡΠΌ ΠΎΠΏΡΠ΅Π΄Π΅Π»Π΅Π½ΠΈΡΠΌ. ΠΡΡΠ΅ΡΡΠ²Π΅Π½Π½ΠΎ, ΡΡΡΠ΅ΡΡΠ²ΡΠ΅Ρ ΠΌΠ½ΠΎΠΆΠ΅ΡΡΠ²ΠΎ ΡΠ°ΡΡΠ½ΡΡ
ΡΠ»ΡΡΠ°Π΅Π², ΡΠ²ΡΠ·Π°Π½Π½ΡΡ
Ρ ΡΠ°Π·Π½ΡΠΌΠΈ ΡΠΈΡΡΠ΅ΠΌΠ°ΠΌΠΈ ΠΊΠΎΠΎΡΠ΄ΠΈΠ½Π°Ρ. Π ΡΠΎΠΌΡ ΠΆΠ΅ ΠΌΡ ΠΏΠΎΡΡΠ°ΡΠ°Π»ΠΈΡΡ ΠΎΡΠΎΠ±ΠΎ Π½Π΅ Π»Π΅Π·ΡΡ Π² ΠΌΠ°ΡΠ΅ΠΌΠ°ΡΠΈΡΠ΅ΡΠΊΠΈΠ΅ Π΄Π΅Π±ΡΠΈ Π΄Π»Ρ ΠΎΠ±ΡΡΡΠ½Π΅Π½ΠΈΡ ΠΎΡΠ½ΠΎΠ²Π½ΡΡ
ΡΠ΅ΡΠΌΠΈΠ½ΠΎΠ² ΡΠΎΠ»ΡΠΊΠΎ Π΄Π»Ρ ΡΠΎΠ³ΠΎ, ΡΡΠΎΠ±Ρ Π²ΠΎΠΏΡΠΎΡ, ΡΠ²ΡΠ·Π°Π½Π½ΡΠΉ Ρ Π½ΠΈΠΌΠΈ, Π±ΡΠ» ΠΏΠΎΠ½ΡΡΠ΅Π½ Π»ΡΠ±ΠΎΠΌΡ ΡΠΊΠΎΠ»ΡΠ½ΠΈΠΊΡ (ΡΠ°ΠΊ ΡΠΊΠ°Π·Π°ΡΡ, ΠΎΠ±ΡΡΡΠ½Π΅Π½ΠΈΠ΅ Β«Π½Π° ΠΏΠ°Π»ΡΡΠ°Ρ
Β»).
Π’Π΅ΠΌ Π½Π΅ ΠΌΠ΅Π½Π΅Π΅, Π΄ΡΠΌΠ°Π΅ΡΡΡ, Π΄Π°ΠΆΠ΅ ΠΈΠ· ΡΠ°ΠΊΠΈΡ ΠΏΡΠΎΡΡΡΡ ΡΡΠ°ΠΊΡΠΎΠ²ΠΎΠΊ ΠΌΠΎΠΆΠ½ΠΎ ΡΠ΄Π΅Π»Π°ΡΡ Π²ΡΠ²ΠΎΠ΄ ΠΎ ΠΌΠ°ΡΠ΅ΠΌΠ°ΡΠΈΡΠ΅ΡΠΊΠΎΠΌ Π°ΡΠΏΠ΅ΠΊΡΠ΅ Π²ΡΠ΅Ρ ΡΠΎΡΡΠ°Π²Π»ΡΡΡΠΈΡ , Π²Ρ ΠΎΠ΄ΡΡΠΈΡ Π² Π±Π°Π·ΠΎΠ²ΡΠΉ ΡΠΊΠΎΠ»ΡΠ½ΡΠΉ ΠΊΡΡΡ Π°Π»Π³Π΅Π±ΡΡ ΠΈ Π³Π΅ΠΎΠΌΠ΅ΡΡΠΈΠΈ.
Π‘ΡΡΠ΅ΡΡΠ²ΡΠ΅Ρ Π΄Π²Π° ΡΠΏΠΎΡΠΎΠ±Π° ΡΠ΅ΡΠ΅Π½ΠΈΡ Π·Π°Π΄Π°Ρ ΠΏΠΎ ΡΡΠ΅ΡΠ΅ΠΎΠΌΠ΅ΡΡΠΈΠΈ
ΠΠ΅ΡΠ²ΡΠΉ β ΠΊΠ»Π°ΡΡΠΈΡΠ΅ΡΠΊΠΈΠΉ β ΡΡΠ΅Π±ΡΠ΅Ρ ΠΎΡΠ»ΠΈΡΠ½ΠΎΠ³ΠΎ Π·Π½Π°Π½ΠΈΡ Π°ΠΊΡΠΈΠΎΠΌ ΠΈ ΡΠ΅ΠΎΡΠ΅ΠΌ ΡΡΠ΅ΡΠ΅ΠΎΠΌΠ΅ΡΡΠΈΠΈ, Π»ΠΎΠ³ΠΈΠΊΠΈ, ΡΠΌΠ΅Π½ΠΈΡ ΠΏΠΎΡΡΡΠΎΠΈΡΡ ΡΠ΅ΡΡΠ΅ΠΆ ΠΈ ΡΠ²Π΅ΡΡΠΈ ΠΎΠ±ΡΠ΅ΠΌΠ½ΡΡ Π·Π°Π΄Π°ΡΡ ΠΊ ΠΏΠ»Π°Π½ΠΈΠΌΠ΅ΡΡΠΈΡΠ΅ΡΠΊΠΎΠΉ. Π‘ΠΏΠΎΡΠΎΠ± Ρ ΠΎΡΠΎΡ ΡΠ΅ΠΌ, ΡΡΠΎ ΡΠ°Π·Π²ΠΈΠ²Π°Π΅Ρ ΠΌΠΎΠ·Π³ΠΈ ΠΈ ΠΏΡΠΎΡΡΡΠ°Π½ΡΡΠ²Π΅Π½Π½ΠΎΠ΅ Π²ΠΎΠΎΠ±ΡΠ°ΠΆΠ΅Π½ΠΈΠ΅.
ΠΡΡΠ³ΠΎΠΉ ΠΌΠ΅ΡΠΎΠ΄ β ΠΏΡΠΈΠΌΠ΅Π½Π΅Π½ΠΈΠ΅ Π²Π΅ΠΊΡΠΎΡΠΎΠ² ΠΈ ΠΊΠΎΠΎΡΠ΄ΠΈΠ½Π°Ρ. ΠΡΠΎ ΠΏΡΠΎΡΡΡΠ΅ ΡΠΎΡΠΌΡΠ»Ρ, Π°Π»Π³ΠΎΡΠΈΡΠΌΡ ΠΈ ΠΏΡΠ°Π²ΠΈΠ»Π°. ΠΠ½ ΠΎΡΠ΅Π½Ρ ΡΠ΄ΠΎΠ±Π΅Π½, ΠΎΡΠΎΠ±Π΅Π½Π½ΠΎ ΠΊΠΎΠ³Π΄Π° Π²ΡΠ΅ΠΌΠ΅Π½ΠΈ Π΄ΠΎ ΡΠΊΠ·Π°ΠΌΠ΅Π½Π° ΠΌΠ°Π»ΠΎ, Π° ΡΠ΅ΡΠΈΡΡ Π·Π°Π΄Π°ΡΡ Ρ ΠΎΡΠ΅ΡΡΡ.
ΠΡΠ»ΠΈ Π²Ρ ΠΎΡΠ²ΠΎΠΈΠ»ΠΈ Π²Π΅ΠΊΡΠΎΡΡ Π½Π° ΠΏΠ»ΠΎΡΠΊΠΎΡΡΠΈ ΠΈ Π΄Π΅ΠΉΡΡΠ²ΠΈΡ Ρ Π½ΠΈΠΌΠΈ β ΡΠΎ ΠΈ Ρ Π²Π΅ΠΊΡΠΎΡΠ°ΠΌΠΈ Π² ΠΏΡΠΎΡΡΡΠ°Π½ΡΡΠ²Π΅ ΡΠ°Π·Π±Π΅ΡΠ΅ΡΠ΅ΡΡ. ΠΠ½ΠΎΠ³ΠΈΠ΅ ΠΏΠΎΠ½ΡΡΠΈΡ ΠΎΠΊΠ°ΠΆΡΡΡΡ Π·Π½Π°ΠΊΠΎΠΌΡΠΌΠΈ.
Π‘ΠΈΡΡΠ΅ΠΌΠ° ΠΊΠΎΠΎΡΠ΄ΠΈΠ½Π°Ρ Π² ΠΏΡΠΎΡΡΡΠ°Π½ΡΡΠ²Π΅ΠΡΠ±Π΅ΡΠ΅ΠΌ Π½Π°ΡΠ°Π»ΠΎ ΠΊΠΎΠΎΡΠ΄ΠΈΠ½Π°Ρ. ΠΡΠΎΠ²Π΅Π΄Π΅ΠΌ ΡΡΠΈ Π²Π·Π°ΠΈΠΌΠ½ΠΎ ΠΏΠ΅ΡΠΏΠ΅Π½Π΄ΠΈΠΊΡΠ»ΡΡΠ½ΡΠ΅ ΠΎΡΠΈ X, Y ΠΈ Z. ΠΠ°Π΄Π°Π΄ΠΈΠΌ ΡΠ΄ΠΎΠ±Π½ΡΠΉ ΠΌΠ°ΡΡΡΠ°Π±.
ΠΠΎΠ»ΡΡΠΈΠ»Π°ΡΡ ΡΠΈΡΡΠ΅ΠΌΠ° ΠΊΠΎΠΎΡΠ΄ΠΈΠ½Π°Ρ Π² ΡΡΠ΅Ρ
ΠΌΠ΅ΡΠ½ΠΎΠΌ ΠΏΡΠΎΡΡΡΠ°Π½ΡΡΠ²Π΅.
Π’Π΅ΠΏΠ΅ΡΡ ΠΊΠ°ΠΆΠ΄Π°Ρ Π΅Π³ΠΎ ΡΠΎΡΠΊΠ° Ρ
Π°ΡΠ°ΠΊΡΠ΅ΡΠΈΠ·ΡΠ΅ΡΡΡ ΡΡΠ΅ΠΌΡ ΡΠΈΡΠ»Π°ΠΌΠΈ β ΠΊΠΎΠΎΡΠ΄ΠΈΠ½Π°ΡΠ°ΠΌΠΈ ΠΏΠΎ X, Y ΠΈ Z. ΠΠ°ΠΏΡΠΈΠΌΠ΅Ρ, Π·Π°ΠΏΠΈΡΡ M(β1; 3; 2) ΠΎΠ·Π½Π°ΡΠ°Π΅Ρ, ΡΡΠΎ ΠΊΠΎΠΎΡΠ΄ΠΈΠ½Π°ΡΠ° ΡΠΎΡΠΊΠΈ M ΠΏΠΎ X (Π°Π±ΡΡΠΈΡΡΠ°) ΡΠ°Π²Π½Π° β1, ΠΊΠΎΠΎΡΠ΄ΠΈΠ½Π°ΡΠ° ΠΏΠΎ Y (ΠΎΡΠ΄ΠΈΠ½Π°ΡΠ°) ΡΠ°Π²Π½Π° 3, Π° ΠΊΠΎΠΎΡΠ΄ΠΈΠ½Π°ΡΠ° ΠΏΠΎ Z (Π°ΠΏΠΏΠ»ΠΈΠΊΠ°ΡΠ°) ΡΠ°Π²Π½Π° 2.
ΠΠ΅ΠΊΡΠΎΡΡ Π² ΠΏΡΠΎΡΡΡΠ°Π½ΡΡΠ²Π΅ ΠΎΠΏΡΠ΅Π΄Π΅Π»ΡΡΡΡΡ ΡΠ°ΠΊ ΠΆΠ΅, ΠΊΠ°ΠΊ ΠΈ Π½Π° ΠΏΠ»ΠΎΡΠΊΠΎΡΡΠΈ. ΠΡΠΎ Π½Π°ΠΏΡΠ°Π²Π»Π΅Π½Π½ΡΠ΅ ΠΎΡΡΠ΅Π·ΠΊΠΈ, ΠΈΠΌΠ΅ΡΡΠΈΠ΅ Π½Π°ΡΠ°Π»ΠΎ ΠΈ ΠΊΠΎΠ½Π΅Ρ. Π’ΠΎΠ»ΡΠΊΠΎ Π² ΠΏΡΠΎΡΡΡΠ°Π½ΡΡΠ²Π΅ Π²Π΅ΠΊΡΠΎΡ Π·Π°Π΄Π°Π΅ΡΡΡ ΡΡΠ΅ΠΌΡ ΠΊΠΎΠΎΡΠ΄ΠΈΠ½Π°ΡΠ°ΠΌΠΈ x, y ΠΈ z:
ΠΠ°ΠΊ Π½Π°ΠΉΡΠΈ ΠΊΠΎΠΎΡΠ΄ΠΈΠ½Π°ΡΡ Π²Π΅ΠΊΡΠΎΡΠ°? ΠΠ°ΠΊ ΠΈ Π½Π° ΠΏΠ»ΠΎΡΠΊΠΎΡΡΠΈ β ΠΈΠ· ΠΊΠΎΠΎΡΠ΄ΠΈΠ½Π°ΡΡ ΠΊΠΎΠ½ΡΠ° Π²ΡΡΠΈΡΠ°Π΅ΠΌ ΠΊΠΎΠΎΡΠ΄ΠΈΠ½Π°ΡΡ Π½Π°ΡΠ°Π»Π°.
ΠΠ»ΠΈΠ½Π° Π²Π΅ΠΊΡΠΎΡΠ° Π² ΠΏΡΠΎΡΡΡΠ°Π½ΡΡΠ²Π΅ β ΡΡΠΎ ΡΠ°ΡΡΡΠΎΡΠ½ΠΈΠ΅ ΠΌΠ΅ΠΆΠ΄Ρ ΡΠΎΡΠΊΠ°ΠΌΠΈ A ΠΈ B. ΠΠ°Ρ ΠΎΠ΄ΠΈΡΡΡ ΠΊΠ°ΠΊ ΠΊΠΎΡΠ΅Π½Ρ ΠΊΠ²Π°Π΄ΡΠ°ΡΠ½ΡΠΉ ΠΈΠ· ΡΡΠΌΠΌΡ ΠΊΠ²Π°Π΄ΡΠ°ΡΠΎΠ² ΠΊΠΎΠΎΡΠ΄ΠΈΠ½Π°Ρ Π²Π΅ΠΊΡΠΎΡΠ°.
ΠΡΡΡΡ ΡΠΎΡΠΊΠ° M β ΡΠ΅ΡΠ΅Π΄ΠΈΠ½Π° ΠΎΡΡΠ΅Π·ΠΊΠ° AB. ΠΠ΅ ΠΊΠΎΠΎΡΠ΄ΠΈΠ½Π°ΡΡ Π½Π°Ρ ΠΎΠ΄ΡΡΡΡ ΠΏΠΎ ΡΠΎΡΠΌΡΠ»Π΅:
ΠΠ»Ρ ΡΠ»ΠΎΠΆΠ΅Π½ΠΈΡ Π²Π΅ΠΊΡΠΎΡΠΎΠ² ΠΏΡΠΈΠΌΠ΅Π½ΡΠ΅ΠΌ ΡΠΆΠ΅ Π·Π½Π°ΠΊΠΎΠΌΡΠ΅ ΠΏΡΠ°Π²ΠΈΠ»ΠΎ ΡΡΠ΅ΡΠ³ΠΎΠ»ΡΠ½ΠΈΠΊΠ° ΠΈ ΠΏΡΠ°Π²ΠΈΠ»ΠΎ ΠΏΠ°ΡΠ°Π»Π»Π΅Π»ΠΎΠ³ΡΠ°ΠΌΠΌΠ°
Π‘ΡΠΌΠΌΠ° Π²Π΅ΠΊΡΠΎΡΠΎΠ², ΠΈΡ
ΡΠ°Π·Π½ΠΎΡΡΡ, ΠΏΡΠΎΠΈΠ·Π²Π΅Π΄Π΅Π½ΠΈΠ΅ Π²Π΅ΠΊΡΠΎΡΠ° Π½Π° ΡΠΈΡΠ»ΠΎ ΠΈ ΡΠΊΠ°Π»ΡΡΠ½ΠΎΠ΅ ΠΏΡΠΎΠΈΠ·Π²Π΅Π΄Π΅Π½ΠΈΠ΅ Π²Π΅ΠΊΡΠΎΡΠΎΠ² ΠΎΠΏΡΠ΅Π΄Π΅Π»ΡΡΡΡΡ ΡΠ°ΠΊ ΠΆΠ΅, ΠΊΠ°ΠΊ ΠΈ Π½Π° ΠΏΠ»ΠΎΡΠΊΠΎΡΡΠΈ.
Π’ΠΎΠ»ΡΠΊΠΎ ΠΊΠΎΠΎΡΠ΄ΠΈΠ½Π°Ρ Π½Π΅ Π΄Π²Π΅, Π° ΡΡΠΈ. ΠΠΎΠ·ΡΠΌΠ΅ΠΌ Π²Π΅ΠΊΡΠΎΡΡ ΠΈ .
ΠΡΠΎΠΈΠ·Π²Π΅Π΄Π΅Π½ΠΈΠ΅ Π²Π΅ΠΊΡΠΎΡΠ° Π½Π° ΡΠΈΡΠ»ΠΎ:
Π‘ΠΊΠ°Π»ΡΡΠ½ΠΎΠ΅ ΠΏΡΠΎΠΈΠ·Π²Π΅Π΄Π΅Π½ΠΈΠ΅ Π²Π΅ΠΊΡΠΎΡΠΎΠ²:
ΠΠΎΡΠΈΠ½ΡΡ ΡΠ³Π»Π° ΠΌΠ΅ΠΆΠ΄Ρ Π²Π΅ΠΊΡΠΎΡΠ°ΠΌΠΈ:
ΠΠΎΡΠ»Π΅Π΄Π½ΡΡ ΡΠΎΡΠΌΡΠ»Π° ΡΠ΄ΠΎΠ±Π½Π° Π΄Π»Ρ Π½Π°Ρ ΠΎΠΆΠ΄Π΅Π½ΠΈΡ ΡΠ³Π»Π° ΠΌΠ΅ΠΆΠ΄Ρ ΠΏΡΡΠΌΡΠΌΠΈ Π² ΠΏΡΠΎΡΡΡΠ°Π½ΡΡΠ²Π΅. ΠΡΠΎΠ±Π΅Π½Π½ΠΎ Π΅ΡΠ»ΠΈ ΡΡΠΈ ΠΏΡΡΠΌΡΠ΅ β ΡΠΊΡΠ΅ΡΠΈΠ²Π°ΡΡΡΡ. ΠΠ°ΠΏΠΎΠΌΠ½ΠΈΠΌ, ΡΡΠΎ ΡΠ°ΠΊ Π½Π°Π·ΡΠ²Π°ΡΡΡΡ ΠΏΡΡΠΌΡΠ΅, ΠΊΠΎΡΠΎΡΡΠ΅ Π½Π΅ ΠΏΠ°ΡΠ°Π»Π»Π΅Π»ΡΠ½Ρ ΠΈ Π½Π΅ ΠΏΠ΅ΡΠ΅ΡΠ΅ΠΊΠ°ΡΡΡΡ. ΠΠ½ΠΈ Π»Π΅ΠΆΠ°Ρ Π² ΠΏΠ°ΡΠ°Π»Π»Π΅Π»ΡΠ½ΡΡ ΠΏΠ»ΠΎΡΠΊΠΎΡΡΡΡ .
1. Π ΠΊΡΠ±Π΅ ABCDA1B1C1D1 ΡΠΎΡΠΊΠΈ E ΠΈ K β ΡΠ΅ΡΠ΅Π΄ΠΈΠ½Ρ ΡΠ΅Π±Π΅Ρ ΡΠΎΠΎΡΠ²Π΅ΡΡΡΠ²Π΅Π½Π½ΠΎ A1B1 ΠΈ B1C1. ΠΠ°ΠΉΠ΄ΠΈΡΠ΅ ΠΊΠΎΡΠΈΠ½ΡΡ ΡΠ³Π»Π° ΠΌΠ΅ΠΆΠ΄Ρ ΠΏΡΡΠΌΡΠΌΠΈ AE ΠΈ BK.
ΠΡΠ»ΠΈ Π²Π°ΠΌ Π΄ΠΎΡΡΠ°Π»ΡΡ ΠΊΡΠ± β Π·Π½Π°ΡΠΈΡ, ΠΏΠΎΠ²Π΅Π·Π»ΠΎ. ΠΠ½ ΠΎΡΠ»ΠΈΡΠ½ΠΎ Π²ΠΏΠΈΡΡΠ²Π°Π΅ΡΡΡ Π² ΠΏΡΡΠΌΠΎΡΠ³ΠΎΠ»ΡΠ½ΡΡ ΡΠΈΡΡΠ΅ΠΌΡ ΠΊΠΎΠΎΡΠ΄ΠΈΠ½Π°Ρ. Π‘ΡΡΠΎΠΈΠΌ ΡΠ΅ΡΡΠ΅ΠΆ:
ΠΠ»ΠΈΠ½Π° ΡΠ΅Π±ΡΠ° ΠΊΡΠ±Π° Π½Π΅ Π΄Π°Π½Π°. ΠΠ°ΠΊΠΎΠΉ Π±Ρ ΠΎΠ½Π° Π½ΠΈ Π±ΡΠ»Π°, ΡΠ³ΠΎΠ» ΠΌΠ΅ΠΆΠ΄Ρ AE ΠΈ BK ΠΎΡ Π½Π΅Π΅ Π½Π΅ Π·Π°Π²ΠΈΡΠΈΡ. ΠΠΎΡΡΠΎΠΌΡ Π²ΠΎΠ·ΡΠΌΠ΅ΠΌ Π΅Π΄ΠΈΠ½ΠΈΡΠ½ΡΠΉ ΠΊΡΠ±, Π²ΡΠ΅ ΡΠ΅Π±ΡΠ° ΠΊΠΎΡΠΎΡΠΎΠ³ΠΎ ΡΠ°Π²Π½Ρ 1.
ΠΡΡΠΌΡΠ΅ AE ΠΈ BK β ΡΠΊΡΠ΅ΡΠΈΠ²Π°ΡΡΡΡ. ΠΠ°ΠΉΠ΄Π΅ΠΌ ΡΠ³ΠΎΠ» ΠΌΠ΅ΠΆΠ΄Ρ Π²Π΅ΠΊΡΠΎΡΠ°ΠΌΠΈ ΠΈ .
ΠΠ»Ρ ΡΡΠΎΠ³ΠΎ Π½ΡΠΆΠ½Ρ ΠΈΡ
ΠΊΠΎΠΎΡΠ΄ΠΈΠ½Π°ΡΡ.
ΠΠ°ΠΏΠΈΡΠ΅ΠΌ ΠΊΠΎΠΎΡΠ΄ΠΈΠ½Π°ΡΡ Π²Π΅ΠΊΡΠΎΡΠΎΠ²:
ΠΈ Π½Π°ΠΉΠ΄Π΅ΠΌ ΠΊΠΎΡΠΈΠ½ΡΡ ΡΠ³Π»Π° ΠΌΠ΅ΠΆΠ΄Ρ Π²Π΅ΠΊΡΠΎΡΠ°ΠΌΠΈ ΠΈ :
2. Π ΠΏΡΠ°Π²ΠΈΠ»ΡΠ½ΠΎΠΉ ΡΠ΅ΡΡΡΠ΅Ρ ΡΠ³ΠΎΠ»ΡΠ½ΠΎΠΉ ΠΏΠΈΡΠ°ΠΌΠΈΠ΄Π΅ SABCD, Π²ΡΠ΅ ΡΠ΅Π±ΡΠ° ΠΊΠΎΡΠΎΡΠΎΠΉ ΡΠ°Π²Π½Ρ 1, ΡΠΎΡΠΊΠΈ E, K β ΡΠ΅ΡΠ΅Π΄ΠΈΠ½Ρ ΡΠ΅Π±Π΅Ρ SB ΠΈ SC ΡΠΎΠΎΡΠ²Π΅ΡΡΡΠ²Π΅Π½Π½ΠΎ. ΠΠ°ΠΉΠ΄ΠΈΡΠ΅ ΠΊΠΎΡΠΈΠ½ΡΡ ΡΠ³Π»Π° ΠΌΠ΅ΠΆΠ΄Ρ ΠΏΡΡΠΌΡΠΌΠΈ AE ΠΈ BK.
ΠΡΡΡΠ΅ Π²ΡΠ΅Π³ΠΎ Π²ΡΠ±ΡΠ°ΡΡ Π½Π°ΡΠ°Π»ΠΎ ΠΊΠΎΠΎΡΠ΄ΠΈΠ½Π°Ρ Π² ΡΠ΅Π½ΡΡΠ΅ ΠΎΡΠ½ΠΎΠ²Π°Π½ΠΈΡ ΠΏΠΈΡΠ°ΠΌΠΈΠ΄Ρ, Π° ΠΎΡΠΈ X ΠΈ Y ΡΠ΄Π΅Π»Π°ΡΡ ΠΏΠ°ΡΠ°Π»Π»Π΅Π»ΡΠ½ΡΠΌΠΈ ΡΡΠΎΡΠΎΠ½Π°ΠΌ ΠΎΡΠ½ΠΎΠ²Π°Π½ΠΈΡ.
ΠΠΎΠΎΡΠ΄ΠΈΠ½Π°ΡΡ ΡΠΎΡΠ΅ΠΊ A, B ΠΈ C Π½Π°ΠΉΡΠΈ Π»Π΅Π³ΠΊΠΎ:
ΠΠ· ΠΏΡΡΠΌΠΎΡΠ³ΠΎΠ»ΡΠ½ΠΎΠ³ΠΎ ΡΡΠ΅ΡΠ³ΠΎΠ»ΡΠ½ΠΈΠΊΠ° AOS Π½Π°ΠΉΠ΄Π΅ΠΌ
ΠΠΎΠΎΡΠ΄ΠΈΠ½Π°ΡΡ Π²Π΅ΡΡΠΈΠ½Ρ ΠΏΠΈΡΠ°ΠΌΠΈΠ΄Ρ:
Π’ΠΎΡΠΊΠ° E β ΡΠ΅ΡΠ΅Π΄ΠΈΠ½Π° SB, Π° K β ΡΠ΅ΡΠ΅Π΄ΠΈΠ½Π° SC. ΠΠΎΡΠΏΠΎΠ»ΡΠ·ΡΠ΅ΠΌΡΡ ΡΠΎΡΠΌΡΠ»ΠΎΠΉ Π΄Π»Ρ ΠΊΠΎΠΎΡΠ΄ΠΈΠ½Π°Ρ ΡΠ΅ΡΠ΅Π΄ΠΈΠ½Ρ ΠΎΡΡΠ΅Π·ΠΊΠ° ΠΈ Π½Π°ΠΉΠ΄Π΅ΠΌ ΠΊΠΎΠΎΡΠ΄ΠΈΠ½Π°ΡΡ ΡΠΎΡΠ΅ΠΊ E ΠΈ K.
ΠΠ°ΠΉΠ΄Π΅ΠΌ ΠΊΠΎΠΎΡΠ΄ΠΈΠ½Π°ΡΡ Π²Π΅ΠΊΡΠΎΡΠΎΠ² ΠΈ
ΠΈ ΡΠ³ΠΎΠ» ΠΌΠ΅ΠΆΠ΄Ρ Π½ΠΈΠΌΠΈ:
ΠΠΎΠΊΠ°ΠΆΠ΅ΠΌ ΡΠ΅ΠΏΠ΅ΡΡ, ΠΊΠ°ΠΊ Π²ΠΏΠΈΡΠ°ΡΡ ΡΠΈΡΡΠ΅ΠΌΡ ΠΊΠΎΠΎΡΠ΄ΠΈΠ½Π°Ρ Π² ΡΡΠ΅ΡΠ³ΠΎΠ»ΡΠ½ΡΡ ΠΏΡΠΈΠ·ΠΌΡ:
3. Π ΠΏΡΠ°Π²ΠΈΠ»ΡΠ½ΠΎΠΉ ΡΡΠ΅ΡΠ³ΠΎΠ»ΡΠ½ΠΎΠΉ ΠΏΡΠΈΠ·ΠΌΠ΅ ABCA1B1C1, Π²ΡΠ΅ ΡΠ΅Π±ΡΠ° ΠΊΠΎΡΠΎΡΠΎΠΉ ΡΠ°Π²Π½Ρ 1, ΡΠΎΡΠΊΠ° D β ΡΠ΅ΡΠ΅Π΄ΠΈΠ½Π° ΡΠ΅Π±ΡΠ° A1B1.
ΠΠ°ΠΉΠ΄ΠΈΡΠ΅ ΠΊΠΎΡΠΈΠ½ΡΡ ΡΠ³Π»Π° ΠΌΠ΅ΠΆΠ΄Ρ ΠΏΡΡΠΌΡΠΌΠΈ AD ΠΈ BC1
ΠΡΡΡΡ ΡΠΎΡΠΊΠ° A β Π½Π°ΡΠ°Π»ΠΎ ΠΊΠΎΠΎΡΠ΄ΠΈΠ½Π°Ρ. ΠΠΎΠ·ΡΠΌΠ΅ΠΌ ΠΎΡΡ X ΠΏΠ°ΡΠ°Π»Π»Π΅Π»ΡΠ½ΠΎ ΡΡΠΎΡΠΎΠ½Π΅ BC, Π° ΠΎΡΡ Y ΠΏΠ΅ΡΠΏΠ΅Π½Π΄ΠΈΠΊΡΠ»ΡΡΠ½ΠΎ Π΅ΠΉ. ΠΡΡΠ³ΠΈΠΌΠΈ ΡΠ»ΠΎΠ²Π°ΠΌΠΈ, Π½Π° ΠΎΡΠΈ Y Π±ΡΠ΄Π΅Ρ Π»Π΅ΠΆΠ°ΡΡ ΠΎΡΡΠ΅Π·ΠΎΠΊ AH, ΡΠ²Π»ΡΡΡΠΈΠΉΡΡ Π²ΡΡΠΎΡΠΎΠΉ ΡΡΠ΅ΡΠ³ΠΎΠ»ΡΠ½ΠΈΠΊΠ° ABC. ΠΠ°ΡΠΈΡΡΠ΅ΠΌ ΠΎΡΠ΄Π΅Π»ΡΠ½ΠΎ Π½ΠΈΠΆΠ½Π΅Π΅ ΠΎΡΠ½ΠΎΠ²Π°Π½ΠΈΠ΅ ΠΏΡΠΈΠ·ΠΌΡ.
ΠΠ°ΠΏΠΈΡΠ΅ΠΌ ΠΊΠΎΠΎΡΠ΄ΠΈΠ½Π°ΡΡ ΡΠΎΡΠ΅ΠΊ:
Π’ΠΎΡΠΊΠ° D β ΡΠ΅ΡΠ΅Π΄ΠΈΠ½Π° A1B1. ΠΠ½Π°ΡΠΈΡ, ΠΏΠΎΠ»ΡΠ·ΡΠ΅ΠΌΡΡ ΡΠΎΡΠΌΡΠ»Π°ΠΌΠΈ Π΄Π»Ρ ΠΊΠΎΠΎΡΠ΄ΠΈΠ½Π°Ρ ΡΠ΅ΡΠ΅Π΄ΠΈΠ½Ρ
ΠΎΡΡΠ΅Π·ΠΊΠ°.
ΠΠ°ΠΉΠ΄Π΅ΠΌ ΠΊΠΎΠΎΡΠ΄ΠΈΠ½Π°ΡΡ Π²Π΅ΠΊΡΠΎΡΠΎΠ² ΠΈ , Π° Π·Π°ΡΠ΅ΠΌ ΡΠ³ΠΎΠ» ΠΌΠ΅ΠΆΠ΄Ρ Π½ΠΈΠΌΠΈ:
Π‘ΠΌΠΎΡΡΠΈΡΠ΅, ΠΊΠ°ΠΊ Π»Π΅Π³ΠΊΠΎ Ρ ΠΏΠΎΠΌΠΎΡΡΡ Π²Π΅ΠΊΡΠΎΡΠΎΠ² ΠΈ ΠΊΠΎΠΎΡΠ΄ΠΈΠ½Π°Ρ Π½Π°ΠΉΡΠΈ ΡΠ³ΠΎΠ» ΠΌΠ΅ΠΆΠ΄Ρ ΠΏΡΡΠΌΡΠΌΠΈ. Π Π΅ΡΠ»ΠΈ ΡΡΠ΅Π±ΡΠ΅ΡΡΡ Π½Π°ΠΉΡΠΈ ΡΠ³ΠΎΠ» ΠΌΠ΅ΠΆΠ΄Ρ ΠΏΠ»ΠΎΡΠΊΠΎΡΡΡΠΌΠΈ ΠΈΠ»ΠΈ ΠΌΠ΅ΠΆΠ΄Ρ ΠΏΡΡΠΌΠΎΠΉ ΠΈ ΠΏΠ»ΠΎΡΠΊΠΎΡΡΡΡ? ΠΠ»Ρ ΡΠ΅ΡΠ΅Π½ΠΈΡ ΠΏΠΎΠ΄ΠΎΠ±Π½ΡΡ Π·Π°Π΄Π°Ρ Π½Π°ΠΌ ΠΏΠΎΠ½Π°Π΄ΠΎΠ±ΠΈΡΡΡ ΡΡΠ°Π²Π½Π΅Π½ΠΈΠ΅ ΠΏΠ»ΠΎΡΠΊΠΎΡΡΠΈ Π² ΠΏΡΠΎΡΡΡΠ°Π½ΡΡΠ²Π΅.
ΠΠ»ΠΎΡΠΊΠΎΡΡΡ Π² ΠΏΡΠΎΡΡΡΠ°Π½ΡΡΠ²Π΅ Π·Π°Π΄Π°Π΅ΡΡΡ ΡΡΠ°Π²Π½Π΅Π½ΠΈΠ΅ΠΌ:
ΠΠ΄Π΅ΡΡ ΡΠΈΡΠ»Π° A, B ΠΈ C β ΠΊΠΎΠΎΡΠ΄ΠΈΠ½Π°ΡΡ Π²Π΅ΠΊΡΠΎΡΠ°, ΠΏΠ΅ΡΠΏΠ΅Π½Π΄ΠΈΠΊΡΠ»ΡΡΠ½ΠΎΠ³ΠΎ ΡΡΠΎΠΉ ΠΏΠ»ΠΎΡΠΊΠΎΡΡΠΈ. ΠΠ³ΠΎ Π½Π°Π·ΡΠ²Π°ΡΡ Π½ΠΎΡΠΌΠ°Π»ΡΡ ΠΊ ΠΏΠ»ΠΎΡΠΊΠΎΡΡΠΈ.
ΠΠΌΠ΅ΡΡΠΎ x, y ΠΈ z ΠΌΠΎΠΆΠ½ΠΎ ΠΏΠΎΠ΄ΡΡΠ°Π²ΠΈΡΡ Π² ΡΡΠ°Π²Π½Π΅Π½ΠΈΠ΅ ΠΊΠΎΠΎΡΠ΄ΠΈΠ½Π°ΡΡ Π»ΡΠ±ΠΎΠΉ ΡΠΎΡΠΊΠΈ, ΠΏΡΠΈΠ½Π°Π΄Π»Π΅ΠΆΠ°ΡΠ΅ΠΉ Π΄Π°Π½Π½ΠΎΠΉ ΠΏΠ»ΠΎΡΠΊΠΎΡΡΠΈ.
ΠΠΎΠ»ΡΡΠΈΡΡΡ Π²Π΅ΡΠ½ΠΎΠ΅ ΡΠ°Π²Π΅Π½ΡΡΠ²ΠΎ.
ΠΠ»ΠΎΡΠΊΠΎΡΡΡ Π² ΠΏΡΠΎΡΡΡΠ°Π½ΡΡΠ²Π΅ ΠΌΠΎΠΆΠ½ΠΎ ΠΏΡΠΎΠ²Π΅ΡΡΠΈ ΡΠ΅ΡΠ΅Π· Π»ΡΠ±ΡΠ΅ ΡΡΠΈ ΡΠΎΡΠΊΠΈ, Π½Π΅ Π»Π΅ΠΆΠ°ΡΠΈΠ΅ Π½Π° ΠΎΠ΄Π½ΠΎΠΉ ΠΏΡΡΠΌΠΎΠΉ. ΠΠΎΡΡΠΎΠΌΡ Π΄Π»Ρ ΡΠΎΠ³ΠΎ, ΡΡΠΎΠ±Ρ Π½Π°ΠΏΠΈΡΠ°ΡΡ ΡΡΠ°Π²Π½Π΅Π½ΠΈΠ΅ ΠΏΠ»ΠΎΡΠΊΠΎΡΡΠΈ, Π±Π΅ΡΠ΅ΠΌ ΠΊΠΎΠΎΡΠ΄ΠΈΠ½Π°ΡΡ ΡΡΠ΅Ρ ΠΏΡΠΈΠ½Π°Π΄Π»Π΅ΠΆΠ°ΡΠΈΡ Π΅ΠΉ ΡΠΎΡΠ΅ΠΊ. ΠΠΎΠ΄ΡΡΠ°Π²Π»ΡΠ΅ΠΌ ΠΈΡ ΠΏΠΎ ΠΎΡΠ΅ΡΠ΅Π΄ΠΈ Π² ΡΡΠ°Π²Π½Π΅Π½ΠΈΠ΅ ΠΏΠ»ΠΎΡΠΊΠΎΡΡΠΈ. Π Π΅ΡΠ°Π΅ΠΌ ΠΏΠΎΠ»ΡΡΠ΅Π½Π½ΡΡ ΡΠΈΡΡΠ΅ΠΌΡ.
ΠΠΎΠΊΠ°ΠΆΠ΅ΠΌ, ΠΊΠ°ΠΊ ΡΡΠΎ Π΄Π΅Π»Π°Π΅ΡΡΡ.
ΠΠ°ΠΏΠΈΡΠ΅ΠΌ ΡΡΠ°Π²Π½Π΅Π½ΠΈΠ΅ ΠΏΠ»ΠΎΡΠΊΠΎΡΡΠΈ, ΠΏΡΠΎΡ ΠΎΠ΄ΡΡΠ΅ΠΉ ΡΠ΅ΡΠ΅Π· ΡΠΎΡΠΊΠΈ M (1; 0; 1), N (2; β2; 0) ΠΈ K (4; 1; 2).
Π£ΡΠ°Π²Π½Π΅Π½ΠΈΠ΅ ΠΏΠ»ΠΎΡΠΊΠΎΡΡΠΈ Π²ΡΠ³Π»ΡΠ΄ΠΈΡ ΡΠ°ΠΊ:
ΠΠΎΠ΄ΡΡΠ°Π²ΠΈΠΌ Π² Π½Π΅Π³ΠΎ ΠΏΠΎ ΠΎΡΠ΅ΡΠ΅Π΄ΠΈ ΠΊΠΎΠΎΡΠ΄ΠΈΠ½Π°ΡΡ ΡΠΎΡΠ΅ΠΊ M, N ΠΈ K.
Π’ΠΎ Π΅ΡΡΡ A + C + D = 0.
ΠΠ½Π°Π»ΠΎΠ³ΠΈΡΠ½ΠΎ Π΄Π»Ρ ΡΠΎΡΠΊΠΈ K:
ΠΠΎΠ»ΡΡΠΈΠ»ΠΈ ΡΠΈΡΡΠ΅ΠΌΡ ΠΈΠ· ΡΡΠ΅Ρ ΡΡΠ°Π²Π½Π΅Π½ΠΈΠΉ:
Π Π½Π΅ΠΉ ΡΠ΅ΡΡΡΠ΅ Π½Π΅ΠΈΠ·Π²Π΅ΡΡΠ½ΡΡ : A, B, C ΠΈ D. ΠΠΎΡΡΠΎΠΌΡ ΠΎΠ΄Π½Ρ ΠΈΠ· Π½ΠΈΡ ΠΌΡ Π²ΡΠ±Π΅ΡΠ΅ΠΌ ΡΠ°ΠΌΠΈ, Π° Π΄ΡΡΠ³ΠΈΠ΅ Π²ΡΡΠ°Π·ΠΈΠΌ ΡΠ΅ΡΠ΅Π· Π½Π΅Π΅. ΠΡΠ°Π²ΠΈΠ»ΠΎ ΠΏΡΠΎΡΡΠΎΠ΅ β Π²ΠΌΠ΅ΡΡΠΎ ΠΎΠ΄Π½ΠΎΠΉ ΠΈΠ· ΠΏΠ΅ΡΠ΅ΠΌΠ΅Π½Π½ΡΡ ΠΌΠΎΠΆΠ½ΠΎ Π²Π·ΡΡΡ Π»ΡΠ±ΠΎΠ΅ ΡΠΈΡΠ»ΠΎ, Π½Π΅ ΡΠ°Π²Π½ΠΎΠ΅ Π½ΡΠ»Ρ.
ΠΡΡΡΡ, Π½Π°ΠΏΡΠΈΠΌΠ΅Ρ, D = β2. Π’ΠΎΠ³Π΄Π°:
ΠΡΡΠ°Π·ΠΈΠΌ C ΠΈ B ΡΠ΅ΡΠ΅Π· A ΠΈ ΠΏΠΎΠ΄ΡΡΠ°Π²ΠΈΠΌ Π² ΡΡΠ΅ΡΡΠ΅ ΡΡΠ°Π²Π½Π΅Π½ΠΈΠ΅:
Π Π΅ΡΠΈΠ² ΡΠΈΡΡΠ΅ΠΌΡ, ΠΏΠΎΠ»ΡΡΠΈΠΌ:
Π£ΡΠ°Π²Π½Π΅Π½ΠΈΠ΅ ΠΏΠ»ΠΎΡΠΊΠΎΡΡΠΈ MNK ΠΈΠΌΠ΅Π΅Ρ Π²ΠΈΠ΄:
Π£ΠΌΠ½ΠΎΠΆΠΈΠΌ ΠΎΠ±Π΅ ΡΠ°ΡΡΠΈ ΡΡΠ°Π²Π½Π΅Π½ΠΈΡ Π½Π° β3.
Π’ΠΎΠ³Π΄Π° ΠΊΠΎΡΡΡΠΈΡΠΈΠ΅Π½ΡΡ ΡΡΠ°Π½ΡΡ ΡΠ΅Π»ΡΠΌΠΈ:
ΠΠ΅ΠΊΡΠΎΡ β ΡΡΠΎ Π½ΠΎΡΠΌΠ°Π»Ρ ΠΊ ΠΏΠ»ΠΎΡΠΊΠΎΡΡΠΈ MNK.
Π£ΡΠ°Π²Π½Π΅Π½ΠΈΠ΅ ΠΏΠ»ΠΎΡΠΊΠΎΡΡΠΈ, ΠΏΡΠΎΡ ΠΎΠ΄ΡΡΠ΅ΠΉ ΡΠ΅ΡΠ΅Π· Π·Π°Π΄Π°Π½Π½ΡΡ ΡΠΎΡΠΊΡ ΠΈΠΌΠ΅Π΅Ρ Π²ΠΈΠ΄:
Π£Π³ΠΎΠ» ΠΌΠ΅ΠΆΠ΄Ρ ΠΏΠ»ΠΎΡΠΊΠΎΡΡΡΠΌΠΈ ΡΠ°Π²Π΅Π½ ΡΠ³Π»Ρ ΠΌΠ΅ΠΆΠ΄Ρ Π½ΠΎΡΠΌΠ°Π»ΡΠΌΠΈ ΠΊ ΡΡΠΈΠΌ ΠΏΠ»ΠΎΡΠΊΠΎΡΡΡΠΌ:
ΠΠ΅ ΠΏΡΠ°Π²Π΄Π° Π»ΠΈ, Π·Π½Π°ΠΊΠΎΠΌΠ°Ρ ΡΠΎΡΠΌΡΠ»Π°? Π‘ΠΊΠ°Π»ΡΡΠ½ΠΎΠ΅ ΠΏΡΠΎΠΈΠ·Π²Π΅Π΄Π΅Π½ΠΈΠ΅ Π½ΠΎΡΠΌΠ°Π»Π΅ΠΉ ΠΏΠΎΠ΄Π΅Π»ΠΈΠ»ΠΈ Π½Π° ΠΏΡΠΎΠΈΠ·Π²Π΅Π΄Π΅Π½ΠΈΠ΅ ΠΈΡ Π΄Π»ΠΈΠ½.
ΠΠ°ΠΌΠ΅ΡΠΈΠΌ, ΡΡΠΎ ΠΏΡΠΈ ΠΏΠ΅ΡΠ΅ΡΠ΅ΡΠ΅Π½ΠΈΠΈ Π΄Π²ΡΡ ΠΏΠ»ΠΎΡΠΊΠΎΡΡΠ΅ΠΉ Π²ΠΎΠΎΠ±ΡΠ΅-ΡΠΎ ΠΎΠ±ΡΠ°Π·ΡΠ΅ΡΡΡ ΡΠ΅ΡΡΡΠ΅ ΡΠ³Π»Π°.
ΠΡ Π±Π΅ΡΠ΅ΠΌ ΠΌΠ΅Π½ΡΡΠΈΠΉ ΠΈΠ· Π½ΠΈΡ . ΠΠΎΡΡΠΎΠΌΡ Π² ΡΠΎΡΠΌΡΠ»Π΅ ΡΡΠΎΠΈΡ ΠΌΠΎΠ΄ΡΠ»Ρ ΡΠΊΠ°Π»ΡΡΠ½ΠΎΠ³ΠΎ ΠΏΡΠΎΠΈΠ·Π²Π΅Π΄Π΅Π½ΠΈΡ β ΡΡΠΎΠ±Ρ ΠΊΠΎΡΠΈΠ½ΡΡ ΡΠ³Π»Π° Π±ΡΠ» Π½Π΅ΠΎΡΡΠΈΡΠ°ΡΠ΅Π»Π΅Π½.
4. Π ΠΊΡΠ±Π΅ ABCDA1B1C1D1 ΡΠΎΡΠΊΠΈ E ΠΈ F β ΡΠ΅ΡΠ΅Π΄ΠΈΠ½Ρ ΡΠ΅Π±Π΅Ρ ΡΠΎΠΎΡΠ²Π΅ΡΡΡΠ²Π΅Π½Π½ΠΎ A1B1 ΠΈ A1D1. ΠΠ°ΠΉΠ΄ΠΈΡΠ΅ ΡΠ°Π½Π³Π΅Π½Ρ ΡΠ³Π»Π° ΠΌΠ΅ΠΆΠ΄Ρ ΠΏΠ»ΠΎΡΠΊΠΎΡΡΡΠΌΠΈ AEF ΠΈ BDD1.
Π‘ΡΡΠΎΠΈΠΌ ΡΠ΅ΡΡΠ΅ΠΆ. ΠΠΈΠ΄Π½ΠΎ, ΡΡΠΎ ΠΏΠ»ΠΎΡΠΊΠΎΡΡΠΈ AEF ΠΈ BDD1 ΠΏΠ΅ΡΠ΅ΡΠ΅ΠΊΠ°ΡΡΡΡ Π³Π΄Π΅-ΡΠΎ Π²Π½Π΅ ΠΊΡΠ±Π°. Π ΠΊΠ»Π°ΡΡΠΈΡΠ΅ΡΠΊΠΎΠΌ ΡΠ΅ΡΠ΅Π½ΠΈΠΈ ΠΏΡΠΈΡΠ»ΠΎΡΡ Π±Ρ ΡΡΡΠΎΠΈΡΡ Π»ΠΈΠ½ΠΈΡ ΠΈΡ
ΠΏΠ΅ΡΠ΅ΡΠ΅ΡΠ΅Π½ΠΈΡ. ΠΠΎ Π²Π΅ΠΊΡΠΎΡΠ½ΠΎ-ΠΊΠΎΠΎΡΠ΄ΠΈΠ½Π°ΡΠ½ΡΠΉ ΠΌΠ΅ΡΠΎΠ΄ Π·Π½Π°ΡΠΈΡΠ΅Π»ΡΠ½ΠΎ Π²ΡΡ ΡΠΏΡΠΎΡΠ°Π΅Ρ.
ΠΠ΅ Π±ΡΠ΄Π΅ΠΌ Π»ΠΎΠΌΠ°ΡΡ Π³ΠΎΠ»ΠΎΠ²Ρ Π½Π°Π΄ ΡΠ΅ΠΌ, ΠΏΠΎ ΠΊΠ°ΠΊΠΎΠΉ ΠΏΡΡΠΌΠΎΠΉ ΠΏΠ΅ΡΠ΅ΡΠ΅ΠΊΠ°ΡΡΡΡ ΠΏΠ»ΠΎΡΠΊΠΎΡΡΠΈ. ΠΡΠΎΡΡΠΎ ΠΎΡΠΌΠ΅ΡΠΈΠΌ ΠΊΠΎΠΎΡΠ΄ΠΈΠ½Π°ΡΡ Π½ΡΠΆΠ½ΡΡ
Π½Π°ΠΌ ΡΠΎΡΠ΅ΠΊ ΠΈ Π½Π°ΠΉΠ΄Π΅ΠΌ ΡΠ³ΠΎΠ» ΠΌΠ΅ΠΆΠ΄Ρ Π½ΠΎΡΠΌΠ°Π»ΡΠΌΠΈ ΠΊ ΠΏΠ»ΠΎΡΠΊΠΎΡΡΡΠΌ AEF ΠΈ BDD1.
Π‘Π½Π°ΡΠ°Π»Π° β Π½ΠΎΡΠΌΠ°Π»Ρ ΠΊ ΠΏΠ»ΠΎΡΠΊΠΎΡΡΠΈ BDD1. ΠΠΎΠ½Π΅ΡΠ½ΠΎ, ΠΌΡ ΠΌΠΎΠΆΠ΅ΠΌ ΠΏΠΎΠ΄ΡΡΠ°Π²ΠΈΡΡ ΠΊΠΎΠΎΡΠ΄ΠΈΠ½Π°ΡΡ ΡΠΎΡΠ΅ΠΊ B, D ΠΈ D1 Π² ΡΡΠ°Π²Π½Π΅Π½ΠΈΠ΅ ΠΏΠ»ΠΎΡΠΊΠΎΡΡΠΈ ΠΈ Π½Π°ΠΉΡΠΈ ΠΊΠΎΡΡΡΠΈΡΠΈΠ΅Π½ΡΡ, ΠΊΠΎΡΠΎΡΡΠ΅ ΠΈ Π±ΡΠ΄ΡΡ ΠΊΠΎΠΎΡΠ΄ΠΈΠ½Π°ΡΠ°ΠΌΠΈ Π²Π΅ΠΊΡΠΎΡΠ° Π½ΠΎΡΠΌΠ°Π»ΠΈ. Π ΠΌΠΎΠΆΠ΅ΠΌ ΡΠ΄Π΅Π»Π°ΡΡ Ρ ΠΈΡΡΠ΅Π΅ β ΡΠ²ΠΈΠ΄Π΅ΡΡ Π½ΡΠΆΠ½ΡΡ Π½ΠΎΡΠΌΠ°Π»Ρ ΠΏΡΡΠΌΠΎ Π½Π° ΡΠ΅ΡΡΠ΅ΠΆΠ΅. ΠΠ΅Π΄Ρ ΠΏΠ»ΠΎΡΠΊΠΎΡΡΡ BDD1 β ΡΡΠΎ Π΄ΠΈΠ°Π³ΠΎΠ½Π°Π»ΡΠ½ΠΎΠ΅ ΡΠ΅ΡΠ΅Π½ΠΈΠ΅ ΠΊΡΠ±Π°. ΠΠ΅ΠΊΡΠΎΡ ΠΏΠ΅ΡΠΏΠ΅Π½Π΄ΠΈΠΊΡΠ»ΡΡΠ΅Π½ ΡΡΠΎΠΉ ΠΏΠ»ΠΎΡΠΊΠΎΡΡΠΈ.
ΠΡΠ°ΠΊ, ΠΏΠ΅ΡΠ²ΡΠΉ Π²Π΅ΠΊΡΠΎΡ Π½ΠΎΡΠΌΠ°Π»ΠΈ Ρ Π½Π°Ρ ΡΠΆΠ΅ Π΅ΡΡΡ:
ΠΠ°ΠΏΠΈΡΠ΅ΠΌ ΡΡΠ°Π²Π½Π΅Π½ΠΈΠ΅ ΠΏΠ»ΠΎΡΠΊΠΎΡΡΠΈ AEF.
ΠΠ΅ΡΠ΅ΠΌ ΡΡΠ°Π²Π½Π΅Π½ΠΈΠ΅ ΠΏΠ»ΠΎΡΠΊΠΎΡΡΠΈ ΠΈ ΠΏΠΎ ΠΎΡΠ΅ΡΠ΅Π΄ΠΈ ΠΏΠΎΠ΄ΡΡΠ°Π²Π»ΡΠ΅ΠΌ Π² Π½Π΅Π³ΠΎ, Π²ΠΌΠ΅ΡΡΠΎ x, y ΠΈ z, ΡΠΎΠΎΡΠ²Π΅ΡΡΡΠ²ΡΡΡΠΈΠ΅ ΠΊΠΎΠΎΡΠ΄ΠΈΠ½Π°ΡΡ ΡΠΎΡΠ΅ΠΊ A, E ΠΈ F.
ΠΡΡΡΡ Π‘ = -1. Π’ΠΎΠ³Π΄Π° A = B = 2.
Π£ΡΠ°Π²Π½Π΅Π½ΠΈΠ΅ ΠΏΠ»ΠΎΡΠΊΠΎΡΡΠΈ AEF:
ΠΠΎΡΠΌΠ°Π»Ρ ΠΊ ΠΏΠ»ΠΎΡΠΊΠΎΡΡΠΈ AEF:
ΠΠ°ΠΉΠ΄Π΅ΠΌ ΡΠ³ΠΎΠ» ΠΌΠ΅ΠΆΠ΄Ρ ΠΏΠ»ΠΎΡΠΊΠΎΡΡΡΠΌΠΈ:
5. ΠΡΠ½ΠΎΠ²Π°Π½ΠΈΠ΅ ΠΏΡΡΠΌΠΎΠΉ ΡΠ΅ΡΡΡΠ΅Ρ
ΡΠ³ΠΎΠ»ΡΠ½ΠΎΠΉ ΠΏΡΠΈΠ·ΠΌΡ BCDA1B1C1D1 β ΠΏΡΡΠΌΠΎΡΠ³ΠΎΠ»ΡΠ½ΠΈΠΊ ABCD, Π² ΠΊΠΎΡΠΎΡΠΎΠΌ AB = 5, AD = β33.
ΠΠ°ΠΉΠ΄ΠΈΡΠ΅ ΡΠ°Π½Π³Π΅Π½Ρ ΡΠ³Π»Π° ΠΌΠ΅ΠΆΠ΄Ρ ΠΏΠ»ΠΎΡΠΊΠΎΡΡΡΡ Π³ΡΠ°Π½ΠΈ AA1D1D ΠΈ ΠΏΠ»ΠΎΡΠΊΠΎΡΡΡΡ, ΠΏΡΠΎΡ
ΠΎΠ΄ΡΡΠ΅ΠΉ ΡΠ΅ΡΠ΅Π· ΡΠ΅ΡΠ΅Π΄ΠΈΠ½Ρ ΡΠ΅Π±ΡΠ° CD ΠΏΠ΅ΡΠΏΠ΅Π½Π΄ΠΈΠΊΡΠ»ΡΡΠ½ΠΎ ΠΏΡΡΠΌΠΎΠΉ B1D, Π΅ΡΠ»ΠΈ ΡΠ°ΡΡΡΠΎΡΠ½ΠΈΠ΅ ΠΌΠ΅ΠΆΠ΄Ρ ΠΏΡΡΠΌΡΠΌΠΈ A1C1 ΠΈ BD ΡΠ°Π²Π½ΠΎ β3.
ΠΡΠ° Π·Π°Π΄Π°ΡΠ° Π½Π°Π³Π»ΡΠ΄Π½ΠΎ ΠΏΠΎΠΊΠ°Π·ΡΠ²Π°Π΅Ρ, Π½Π°ΡΠΊΠΎΠ»ΡΠΊΠΎ Π²Π΅ΠΊΡΠΎΡΠ½ΡΠΉ ΠΌΠ΅ΡΠΎΠ΄ ΠΏΡΠΎΡΠ΅ ΠΊΠ»Π°ΡΡΠΈΡΠ΅ΡΠΊΠΎΠ³ΠΎ. ΠΠΎΠΏΡΠΎΠ±ΡΠΉΡΠ΅, Π΄Π»Ρ ΡΠ°Π·Π½ΠΎΠΎΠ±ΡΠ°Π·ΠΈΡ, ΠΏΠΎΡΡΡΠΎΠΈΡΡ Π½Π΅ΠΎΠ±Ρ ΠΎΠ΄ΠΈΠΌΡΠ΅ ΡΠ΅ΡΠ΅Π½ΠΈΡ ΠΈ ΠΏΡΠΎΠ²Π΅ΡΡΠΈ Π²ΡΠ΅ Π΄ΠΎΠΊΠ°Π·Π°ΡΠ΅Π»ΡΡΡΠ²Π° β ΠΊΠ°ΠΊ ΡΡΠΎ Π΄Π΅Π»Π°Π΅ΡΡΡ Π² Β«ΠΊΠ»Π°ΡΡΠΈΠΊΠ΅Β» π
Π‘ΡΡΠΎΠΈΠΌ ΡΠ΅ΡΡΠ΅ΠΆ. ΠΡΡΠΌΡΡ ΡΠ΅ΡΡΡΠ΅Ρ ΡΠ³ΠΎΠ»ΡΠ½ΡΡ ΠΏΡΠΈΠ·ΠΌΡ ΠΌΠΎΠΆΠ½ΠΎ ΠΏΠΎ-Π΄ΡΡΠ³ΠΎΠΌΡ Π½Π°Π·Π²Π°ΡΡ «ΠΏΠ°ΡΠ°Π»Π»Π΅Π»Π΅ΠΏΠΈΠΏΠ΅Π΄».
ΠΠ°ΠΌΠ΅ΡΠ°Π΅ΠΌ, ΡΡΠΎ Π΄Π»ΠΈΠ½Π° ΠΈ ΡΠΈΡΠΈΠ½Π° ΠΏΠ°ΡΠ°Π»Π»Π΅Π»Π΅ΠΏΠΈΠΏΠ΅Π΄Π° Ρ Π½Π°Ρ Π΅ΡΡΡ, Π° Π²ΠΎΡ Π²ΡΡΠΎΡΠ° β Π²ΡΠΎΠ΄Π΅ Π½Π΅ Π΄Π°Π½Π°. ΠΠ°ΠΊ ΠΆΠ΅ Π΅Π΅ Π½Π°ΠΉΡΠΈ?
Β«Π Π°ΡΡΡΠΎΡΠ½ΠΈΠ΅ ΠΌΠ΅ΠΆΠ΄Ρ ΠΏΡΡΠΌΡΠΌΠΈ A1C1 ΠΈ BD ΡΠ°Π²Π½ΠΎ β3Β». ΠΡΡΠΌΡΠ΅ A1C1 ΠΈ BD ΡΠΊΡΠ΅ΡΠΈΠ²Π°ΡΡΡΡ. ΠΠ΄Π½Π° ΠΈΠ· Π½ΠΈΡ
β Π΄ΠΈΠ°Π³ΠΎΠ½Π°Π»Ρ Π²Π΅ΡΡ
Π½Π΅Π³ΠΎ ΠΎΡΠ½ΠΎΠ²Π°Π½ΠΈΡ, Π΄ΡΡΠ³Π°Ρ β Π΄ΠΈΠ°Π³ΠΎΠ½Π°Π»Ρ Π½ΠΈΠΆΠ½Π΅Π³ΠΎ. ΠΡΠΏΠΎΠΌΠ½ΠΈΠΌ, ΡΡΠΎ ΡΠ°ΡΡΡΠΎΡΠ½ΠΈΠ΅ ΠΌΠ΅ΠΆΠ΄Ρ ΡΠΊΡΠ΅ΡΠΈΠ²Π°ΡΡΠΈΠΌΠΈΡΡ ΠΏΡΡΠΌΡΠΌΠΈ ΡΠ°Π²Π½ΠΎ Π΄Π»ΠΈΠ½Π΅ ΠΈΡ
ΠΎΠ±ΡΠ΅Π³ΠΎ ΠΏΠ΅ΡΠΏΠ΅Π½Π΄ΠΈΠΊΡΠ»ΡΡΠ°. ΠΠ±ΡΠΈΠΉ ΠΏΠ΅ΡΠΏΠ΅Π½Π΄ΠΈΠΊΡΠ»ΡΡ ΠΊ A1C1 ΠΈ BD β ΡΡΠΎ, ΠΎΡΠ΅Π²ΠΈΠ΄Π½ΠΎ, OO1, Π³Π΄Π΅ O β ΡΠΎΡΠΊΠ° ΠΏΠ΅ΡΠ΅ΡΠ΅ΡΠ΅Π½ΠΈΡ Π΄ΠΈΠ°Π³ΠΎΠ½Π°Π»Π΅ΠΉ Π½ΠΈΠΆΠ½Π΅Π³ΠΎ ΠΎΡΠ½ΠΎΠ²Π°Π½ΠΈΡ, O1 β ΡΠΎΡΠΊΠ° ΠΏΠ΅ΡΠ΅ΡΠ΅ΡΠ΅Π½ΠΈΡ Π΄ΠΈΠ°Π³ΠΎΠ½Π°Π»Π΅ΠΉ Π²Π΅ΡΡ
Π½Π΅Π³ΠΎ.
Π ΠΎΡΡΠ΅Π·ΠΎΠΊ OO1 ΠΈ ΡΠ°Π²Π΅Π½ Π²ΡΡΠΎΡΠ΅ ΠΏΠ°ΡΠ°Π»Π»Π΅Π»Π΅ΠΏΠΈΠΏΠ΅Π΄Π°.
ΠΠ»ΠΎΡΠΊΠΎΡΡΡ AA1 D1 D β ΡΡΠΎ Π·Π°Π΄Π½ΡΡ Π³ΡΠ°Π½Ρ ΠΏΡΠΈΠ·ΠΌΡ Π½Π° Π½Π°ΡΠ΅ΠΌ ΡΠ΅ΡΡΠ΅ΠΆΠ΅. ΠΠΎΡΠΌΠ°Π»Ρ ΠΊ Π½Π΅ΠΉ β ΡΡΠΎ Π»ΡΠ±ΠΎΠΉ Π²Π΅ΠΊΡΠΎΡ, ΠΏΠ΅ΡΠΏΠ΅Π½Π΄ΠΈΠΊΡΠ»ΡΡΠ½ΡΠΉ Π·Π°Π΄Π½Π΅ΠΉ Π³ΡΠ°Π½ΠΈ, Π½Π°ΠΏΡΠΈΠΌΠ΅Ρ, Π²Π΅ΠΊΡΠΎΡ ΠΈΠ»ΠΈ, Π΅ΡΠ΅ ΠΏΡΠΎΡΠ΅, Π²Π΅ΠΊΡΠΎΡ .
ΠΡΡΠ°Π»Π°ΡΡ Π΅ΡΠ΅ Β«ΠΏΠ»ΠΎΡΠΊΠΎΡΡΡ, ΠΏΡΠΎΡ ΠΎΠ΄ΡΡΠ°Ρ ΡΠ΅ΡΠ΅Π· ΡΠ΅ΡΠ΅Π΄ΠΈΠ½Ρ ΡΠ΅Π±ΡΠ° CD ΠΏΠ΅ΡΠΏΠ΅Π½Π΄ΠΈΠΊΡΠ»ΡΡΠ½ΠΎ ΠΏΡΡΠΌΠΎΠΉ B1DΒ». ΠΠΎ ΠΏΠΎΠ·Π²ΠΎΠ»ΡΡΠ΅, Π΅ΡΠ»ΠΈ ΠΏΠ»ΠΎΡΠΊΠΎΡΡΡ ΠΏΠ΅ΡΠΏΠ΅Π½Π΄ΠΈΠΊΡΠ»ΡΡΠ½Π° ΠΏΡΡΠΌΠΎΠΉ B1D β Π·Π½Π°ΡΠΈΡ, B1D ΠΈ Π΅ΡΡΡ Π½ΠΎΡΠΌΠ°Π»Ρ ΠΊ ΡΡΠΎΠΉ ΠΏΠ»ΠΎΡΠΊΠΎΡΡΠΈ! ΠΠΎΠΎΡΠ΄ΠΈΠ½Π°ΡΡ ΡΠΎΡΠ΅ΠΊ B1 ΠΈ D ΠΈΠ·Π²Π΅ΡΡΠ½Ρ:
ΠΠΎΠΎΡΠ΄ΠΈΠ½Π°ΡΡ Π²Π΅ΠΊΡΠΎΡΠ° β ΡΠΎΠΆΠ΅:
ΠΠ°Ρ ΠΎΠ΄ΠΈΠΌ ΡΠ³ΠΎΠ» ΠΌΠ΅ΠΆΠ΄Ρ ΠΏΠ»ΠΎΡΠΊΠΎΡΡΡΠΌΠΈ, ΡΠ°Π²Π½ΡΠΉ ΡΠ³Π»Ρ ΠΌΠ΅ΠΆΠ΄Ρ Π½ΠΎΡΠΌΠ°Π»ΡΠΌΠΈ ΠΊ Π½ΠΈΠΌ:
ΠΠ½Π°Ρ ΠΊΠΎΡΠΈΠ½ΡΡ ΡΠ³Π»Π°, Π½Π°Ρ ΠΎΠ΄ΠΈΠΌ Π΅Π³ΠΎ ΡΠ°Π½Π³Π΅Π½Ρ ΠΏΠΎ ΡΠΎΡΠΌΡΠ»Π΅
ΠΠΎΠ»ΡΡΠΈΠΌ:
ΠΡΠ²Π΅Ρ:
Π£Π³ΠΎΠ» ΠΌΠ΅ΠΆΠ΄Ρ ΠΏΡΡΠΌΠΎΠΉ m ΠΈ ΠΏΠ»ΠΎΡΠΊΠΎΡΡΡΡ Ξ± ΡΠΎΠΆΠ΅ Π²ΡΡΠΈΡΠ»ΡΠ΅ΡΡΡ Ρ ΠΏΠΎΠΌΠΎΡΡΡ ΡΠΊΠ°Π»ΡΡΠ½ΠΎΠ³ΠΎ ΠΏΡΠΎΠΈΠ·Π²Π΅Π΄Π΅Π½ΠΈΡ Π²Π΅ΠΊΡΠΎΡΠΎΠ².
ΠΡΡΡΡ β Π²Π΅ΠΊΡΠΎΡ, Π»Π΅ΠΆΠ°ΡΠΈΠΉ Π½Π° ΠΏΡΡΠΌΠΎΠΉ m (ΠΈΠ»ΠΈ ΠΏΠ°ΡΠ°Π»Π»Π΅Π»ΡΠ½ΡΠΉ Π΅ΠΉ), β Π½ΠΎΡΠΌΠ°Π»Ρ ΠΊ ΠΏΠ»ΠΎΡΠΊΠΎΡΡΠΈ Ξ±.
ΠΠ°Ρ ΠΎΠ΄ΠΈΠΌ ΡΠΈΠ½ΡΡ ΡΠ³Π»Π° ΠΌΠ΅ΠΆΠ΄Ρ ΠΏΡΡΠΌΠΎΠΉ m ΠΈ ΠΏΠ»ΠΎΡΠΊΠΎΡΡΡΡ Ξ± ΠΏΠΎ ΡΠΎΡΠΌΡΠ»Π΅:
6.
Π ΠΊΡΠ±Π΅ ABCDA1B1C1D1 ΡΠΎΡΠΊΠ° E β ΡΠ΅ΡΠ΅Π΄ΠΈΠ½Π° ΡΠ΅Π±ΡΠ° A1B1. ΠΠ°ΠΉΠ΄ΠΈΡΠ΅ ΡΠΈΠ½ΡΡ ΡΠ³Π»Π° ΠΌΠ΅ΠΆΠ΄Ρ ΠΏΡΡΠΌΠΎΠΉ AE ΠΈ ΠΏΠ»ΠΎΡΠΊΠΎΡΡΡΡ BDD1.
ΠΠ°ΠΊ Π²ΡΠ΅Π³Π΄Π°, ΡΠΈΡΡΠ΅ΠΌ ΡΠ΅ΡΡΠ΅ΠΆ ΠΈ Π²ΡΠ±ΠΈΡΠ°Π΅ΠΌ ΡΠΈΡΡΠ΅ΠΌΡ ΠΊΠΎΠΎΡΠ΄ΠΈΠ½Π°Ρ
ΠΠ°Ρ ΠΎΠ΄ΠΈΠΌ ΠΊΠΎΠΎΡΠ΄ΠΈΠ½Π°ΡΡ Π²Π΅ΠΊΡΠΎΡΠ° .
ΠΡΠΆΠ½ΠΎ Π»ΠΈ Π½Π°ΠΌ ΡΡΠ°Π²Π½Π΅Π½ΠΈΠ΅ ΠΏΠ»ΠΎΡΠΊΠΎΡΡΠΈ BDD1? Π ΠΎΠ±ΡΠ΅ΠΌ-ΡΠΎ, Π±Π΅Π· Π½Π΅Π³ΠΎ ΠΌΠΎΠΆΠ½ΠΎ ΠΎΠ±ΠΎΠΉΡΠΈΡΡ. ΠΠ΅Π΄Ρ ΡΡΠ° ΠΏΠ»ΠΎΡΠΊΠΎΡΡΡ ΡΠ²Π»ΡΠ΅ΡΡΡ Π΄ΠΈΠ°Π³ΠΎΠ½Π°Π»ΡΠ½ΡΠΌ ΡΠ΅ΡΠ΅Π½ΠΈΠ΅ΠΌ ΠΊΡΠ±Π°, Π° Π·Π½Π°ΡΠΈΡ, Π½ΠΎΡΠΌΠ°Π»ΡΡ ΠΊ Π½Π΅ΠΉ Π±ΡΠ΄Π΅Ρ Π»ΡΠ±ΠΎΠΉ Π²Π΅ΠΊΡΠΎΡ, Π΅ΠΉ ΠΏΠ΅ΡΠΏΠ΅Π½Π΄ΠΈΠΊΡΠ»ΡΡΠ½ΡΠΉ. ΠΠ°ΠΏΡΠΈΠΌΠ΅Ρ, Π²Π΅ΠΊΡΠΎΡ .
ΠΠ°ΠΉΠ΄Π΅ΠΌ ΡΠ³ΠΎΠ» ΠΌΠ΅ΠΆΠ΄Ρ ΠΏΡΡΠΌΠΎΠΉ ΠΈ ΠΏΠ»ΠΎΡΠΊΠΎΡΡΡΡ:
ΠΡΠ²Π΅Ρ:
Π Π°ΡΡΡΠΎΡΠ½ΠΈΠ΅ ΠΎΡ ΡΠΎΡΠΊΠΈ M Ρ ΠΊΠΎΠΎΡΠ΄ΠΈΠ½Π°ΡΠ°ΠΌΠΈ x0, y0 ΠΈ z0 Π΄ΠΎ ΠΏΠ»ΠΎΡΠΊΠΎΡΡΠΈ Ξ±, Π·Π°Π΄Π°Π½Π½ΠΎΠΉ ΡΡΠ°Π²Π½Π΅Π½ΠΈΠ΅ΠΌ Ax + By + Cz + D = 0, ΠΌΠΎΠΆΠ½ΠΎ Π½Π°ΠΉΡΠΈ ΠΏΠΎ ΡΠΎΡΠΌΡΠ»Π΅:
7. Π ΠΎΡΠ½ΠΎΠ²Π°Π½ΠΈΠΈ ΠΏΡΡΠΌΠΎΡΠ³ΠΎΠ»ΡΠ½ΠΎΠ³ΠΎ ΠΏΠ°ΡΠ°Π»Π»Π΅Π»Π΅ΠΏΠΈΠΏΠ΅Π΄Π° BCDA1B1C1D1 Π»Π΅ΠΆΠΈΡ ΠΏΡΡΠΌΠΎΡΠ³ΠΎΠ»ΡΠ½ΠΈΠΊ ABCD ΡΠΎ ΡΡΠΎΡΠΎΠ½Π°ΠΌΠΈ AB = , AD = . ΠΡΡΠΎΡΠ° ΠΏΠ°ΡΠ°Π»Π»Π΅Π»Π΅ΠΏΠΈΠΏΠ΅Π΄Π° AA1 = .
ΠΠ°ΠΉΠ΄ΠΈΡΠ΅ ΡΠ°ΡΡΡΠΎΡΠ½ΠΈΠ΅ ΠΎΡ ΡΠΎΡΠΊΠΈ A Π΄ΠΎ ΠΏΠ»ΠΎΡΠΊΠΎΡΡΠΈ A1DB.
ΠΠΎΡΡΡΠΎΠΈΠΌ ΡΠ΅ΡΡΠ΅ΠΆ ΠΈ Π²ΡΠΏΠΈΡΠ΅ΠΌ ΠΊΠΎΠΎΡΠ΄ΠΈΠ½Π°ΡΡ ΡΠΎΡΠ΅ΠΊ:
ΠΠ°ΠΏΠΈΡΠ΅ΠΌ ΡΡΠ°Π²Π½Π΅Π½ΠΈΠ΅ ΠΏΠ»ΠΎΡΠΊΠΎΡΡΠΈ A1DB. ΠΡ ΠΏΠΎΠΌΠ½ΠΈΡΠ΅, ΠΊΠ°ΠΊ ΡΡΠΎ Π΄Π΅Π»Π°Π΅ΡΡΡ β ΠΏΠΎ ΠΎΡΠ΅ΡΠ΅Π΄ΠΈ ΠΏΠΎΠ΄ΡΡΠ°Π²Π»ΡΠ΅ΠΌ ΠΊΠΎΠΎΡΠ΄ΠΈΠ½Π°ΡΡ ΡΠΎΡΠ΅ΠΊ A1, D ΠΈ B Π² ΡΡΠ°Π²Π½Π΅Π½ΠΈΠ΅ Ax + Be + Cz + D
Π Π΅ΡΠΈΠΌ ΡΡΡ ΡΠΈΡΡΠ΅ΠΌΡ. ΠΡΠ±Π΅ΡΠ΅ΠΌ
Π’ΠΎΠ³Π΄Π°
Π£ΡΠ°Π²Π½Π΅Π½ΠΈΠ΅ ΠΏΠ»ΠΎΡΠΊΠΎΡΡΠΈ A1DB ΠΈΠΌΠ΅Π΅Ρ Π²ΠΈΠ΄:
ΠΠ°Π»ΡΡΠ΅ Π²ΡΠ΅ ΠΏΡΠΎΡΡΠΎ. ΠΠ°Ρ ΠΎΠ΄ΠΈΠΌ ΡΠ°ΡΡΡΠΎΡΠ½ΠΈΠ΅ ΠΎΡ ΡΠΎΡΠΊΠΈ A Π΄ΠΎ ΠΏΠ»ΠΎΡΠΊΠΎΡΡΠΈ A1DB:
Π Π½Π΅ΠΊΠΎΡΠΎΡΡΡ Π·Π°Π΄Π°ΡΠ°Ρ ΠΏΠΎ ΡΡΠ΅ΡΠ΅ΠΎΠΌΠ΅ΡΡΠΈΠΈ ΡΡΠ΅Π±ΡΠ΅ΡΡΡ Π½Π°ΠΉΡΠΈ ΡΠ°ΡΡΡΠΎΡΠ½ΠΈΠ΅ ΠΎΡ ΠΏΡΡΠΌΠΎΠΉ Π΄ΠΎ ΠΏΠ°ΡΠ°Π»Π»Π΅Π»ΡΠ½ΠΎΠΉ Π΅ΠΉ ΠΏΠ»ΠΎΡΠΊΠΎΡΡΠΈ. Π ΡΡΠΎΠΌ ΡΠ»ΡΡΠ°Π΅ ΠΌΠΎΠΆΠ½ΠΎ Π²ΡΠ±ΡΠ°ΡΡ Π»ΡΠ±ΡΡ ΡΠΎΡΠΊΡ, ΠΏΡΠΈΠ½Π°Π΄Π»Π΅ΠΆΠ°ΡΡΡ Π΄Π°Π½Π½ΠΎΠΉ ΠΏΡΡΠΌΠΎΠΉ.
ΠΠ²ΠΎΠ½ΠΈΡΠ΅ Π½Π°ΠΌ: 8 (800) 775-06-82 (Π±Π΅ΡΠΏΠ»Π°ΡΠ½ΡΠΉ Π·Π²ΠΎΠ½ΠΎΠΊ ΠΏΠΎ Π ΠΎΡΡΠΈΠΈ) +7 (495) 984-09-27 (Π±Π΅ΡΠΏΠ»Π°ΡΠ½ΡΠΉ Π·Π²ΠΎΠ½ΠΎΠΊ ΠΏΠΎ ΠΠΎΡΠΊΠ²Π΅)
ΠΠ»ΠΈ Π½Π°ΠΆΠΌΠΈΡΠ΅ Π½Π° ΠΊΠ½ΠΎΠΏΠΊΡ Β«Π£Π·Π½Π°ΡΡ Π±ΠΎΠ»ΡΡΠ΅Β», ΡΡΠΎΠ±Ρ Π·Π°ΠΏΠΎΠ»Π½ΠΈΡΡ ΠΊΠΎΠ½ΡΠ°ΠΊΡΠ½ΡΡ ΡΠΎΡΠΌΡ. ΠΡ ΠΎΠ±ΡΠ·Π°ΡΠ΅Π»ΡΠ½ΠΎ ΠΠ°ΠΌ ΠΏΠ΅ΡΠ΅Π·Π²ΠΎΠ½ΠΈΠΌ.
ΠΠ°Π»ΡΠΊΡΠ»ΡΡΠΎΡ ΡΠ»ΡΠΆΠΈΡ Π΄Π»Ρ Π²ΡΡΠΈΡΠ»Π΅Π½ΠΈΡ Π΄Π»ΠΈΠ½Ρ Π²Π΅ΠΊΡΠΎΡΠ° Π² ΡΡΠ΅Ρ
ΠΌΠ΅ΡΠ½ΠΎΠΌ ΠΏΡΠΎΡΡΡΠ°Π½ΡΡΠ²Π΅.
ΠΠ»ΠΈΠ½ΠΎΠΉ ΠΈΠ»ΠΈ ΠΌΠΎΠ΄ΡΠ»Π΅ΠΌ Π²Π΅ΠΊΡΠΎΡΠ° ΠΌΡ Π½Π°Π·ΡΠ²Π°Π΅ΠΌ Π΄Π»ΠΈΠ½Ρ ΠΎΡΡΠ΅Π·ΠΊΠ°, ΠΈΠ·ΠΎΠ±ΡΠ°ΠΆΠ°ΡΡΠ΅Π³ΠΎ Π΄Π°Π½Π½ΡΠΉ Π²Π΅ΠΊΡΠΎΡ.
Π€ΠΎΡΠΌΡΠ»Π° Π΄Π»Ρ ΠΎΠΏΡΠ΅Π΄Π΅Π»Π΅Π½ΠΈΡ Π΄Π»ΠΈΠ½Ρ Π²Π΅ΠΊΡΠΎΡΠ° a Π² ΡΡΠ΅Ρ ΠΌΠ΅ΡΠ½ΠΎΠΌ ΠΏΡΠΎΡΡΡΠ°Π½ΡΡΠ²Π΅ ΡΠ΅ΡΠ΅Π· Π΅Π³ΠΎ ΠΊΠΎΠΎΡΠ΄ΠΈΠ½Π°ΡΡ ΠΏΡΠ΅Π΄ΡΡΠ°Π²Π»Π΅Π½Π° Π½ΠΈΠΆΠ΅.
Π§ΡΠΎΠ±Ρ ΠΏΡΠΎΠΈΠ·Π²Π΅ΡΡΠΈ Π²ΡΡΠΈΡΠ»Π΅Π½ΠΈΠ΅ Π΄Π»ΠΈΠ½Ρ Π²Π΅ΠΊΡΠΎΡΠ° Π² ΡΡΠ΅Ρ ΠΌΠ΅ΡΠ½ΠΎΠΌ ΠΏΡΠΎΡΡΡΠ°Π½ΡΡΠ²Π΅, Π²Π²Π΅Π΄ΠΈΡΠ΅ Π² ΡΠΎΠΎΡΠ²Π΅ΡΡΡΠ²ΡΡΡΠΈΡ ΠΏΠΎΠ»ΡΡ Π·Π½Π°ΡΠ΅Π½ΠΈΠ΅ ΠΊΠΎΠΎΡΠ΄ΠΈΠ½Π°ΡΡ x ΠΈ ΠΊΠΎΠΎΡΠ΄ΠΈΠ½Π°ΡΡ y ΠΈ ΠΊΠΎΠΎΡΠ΄ΠΈΠ½Π°ΡΡ z, ΠΏΠΎΡΠ»Π΅ ΡΠ΅Π³ΠΎ Π½Π°ΠΆΠΌΠΈΡΠ΅ ΠΊΠ½ΠΎΠΏΠΊΡ » ΠΠ«Π§ΠΠ‘ΠΠΠ’Π¬ «.
Π Π΅Π·ΡΠ»ΡΡΠ°ΡΠΎΠΌ ΡΠ°Π±ΠΎΡΡ ΠΏΡΠΎΠ³ΡΠ°ΠΌΠΌΡ Π±ΡΠ΄Π΅Ρ Π·Π½Π°ΡΠ΅Π½ΠΈΠ΅ Π΄Π»ΠΈΠ½Ρ Π²Π΅ΠΊΡΠΎΡΠ° Π² ΡΡΠ΅Ρ ΠΌΠ΅ΡΠ½ΠΎΠΌ ΠΏΡΠΎΡΡΡΠ°Π½ΡΡΠ²Π΅.
ΠΠ°ΡΠ΅ΡΠ½ΡΠΉ ΡΡΠΎΠΊ ΠΏΠΎ ΡΠ΅ΠΌΠ΅: «ΠΠ΅ΡΠΎΠ΄ ΠΊΠΎΠΎΡΠ΄ΠΈΠ½Π°Ρ»
- ΠΠ»ΡΡΠΊΠΎΠ²Π° Π‘Π²Π΅ΡΠ»Π°Π½Π° ΠΠ½Π°ΡΠΎΠ»ΡΠ΅Π²Π½Π°
Π Π°Π·Π΄Π΅Π»Ρ: ΠΠ°ΡΠ΅ΠΌΠ°ΡΠΈΠΊΠ°
Π¦Π΅Π»ΠΈ ΡΡΠΎΠΊΠ°:
- ΠΏΠΎΠ΄Π²Π΅ΡΡΠΈ ΠΈΡΠΎΠ³ ΠΏΠΎ ΠΈΠ·ΡΡΠ΅Π½ΠΈΡ ΡΠ΅ΠΌΡ,
- ΠΏΡΠΎΠ²Π΅ΡΠΈΡΡ ΡΡΠ²ΠΎΠ΅Π½Π½ΡΠ΅ Π·Π½Π°Π½ΠΈΡ,
- ΠΏΠΎΠ΄Π³ΠΎΡΠΎΠ²ΠΊΠ° ΠΊ ΠΊΠΎΠ½ΡΡΠΎΠ»ΡΠ½ΠΎΠΉ ΡΠ°Π±ΠΎΡΠ΅.

ΠΠ»Π°ΡΡ ΡΠ°Π·Π±ΠΈΠ²Π°Π΅ΡΡΡ Π½Π° 5 Π³ΡΡΠΏΠΏ ΠΏΠΎ 5 ΡΠ΅Π»ΠΎΠ²Π΅ΠΊ. Π‘ΡΠ°ΡΡΠΈΠΌΠΈ Π² Π³ΡΡΠΏΠΏΠ΅ Π½Π°Π·Π½Π°ΡΠ°ΡΡΡΡ ΡΡΠΏΠ΅Π²Π°ΡΡΠΈΠ΅ ΡΡΠ΅Π½ΠΈΠΊΠΈ (ΠΏΠΎΠΌΠΎΡΠ½ΠΈΠΊΠΈ ΡΡΠΈΡΠ΅Π»Ρ).
1 ΡΡΠ°ΠΏ. ΠΡΠ³Π°Π½ΠΈΠ·Π°ΡΠΈΠΎΠ½Π½ΡΠΉ ΠΌΠΎΠΌΠ΅Π½Ρ. ΠΠΎΡΡΠ°Π½ΠΎΠ²ΠΊΠ° ΡΠ΅Π»Π΅ΠΉ
2 ΡΡΠ°ΠΏ. ΠΡΠ²Π΅ΡΡ Π½Π° Π²ΠΎΠΏΡΠΎΡΡ Ρ ΠΈΡΠΏΠΎΠ»ΡΠ·ΠΎΠ²Π°Π½ΠΈΠ΅ ΡΠΈΠ³Π½Π°Π»ΡΠ½ΡΡ ΠΊΠ°ΡΡ (15 ΠΌΠΈΠ½ΡΡ)
ΠΠ°ΠΆΠ΄ΠΎΠΉ Π³ΡΡΠΏΠΏΠ΅ Π²ΡΠ΄Π°Π΅ΡΡΡ ΠΏΡΡΠ΅Π²ΠΎΠΉ Π»ΠΈΡΡ, Π² ΠΊΠΎΡΠΎΡΠΎΠΌ ΠΏΠΎΠΌΠΎΡΠ½ΠΈΠΊΠΈ Π±ΡΠ΄ΡΡ ΠΎΡΠΌΠ΅ΡΠ°ΡΡ Π·Π½Π°Π½ΠΈΠ΅/Π½Π΅Π·Π½Π°Π½ΠΈΠ΅ ΠΎΡΠ²Π΅ΡΠ° Π½Π° Π²ΠΎΠΏΡΠΎΡ. ΠΠ°ΠΆΠ΄ΠΎΠΌΡ ΡΡΠ°ΡΠ΅ΠΌΡΡΡ Π²ΡΠ΄Π°ΡΡΡΡ Π΄Π²Π΅ ΠΊΠ°ΡΡΠΎΡΠΊΠΈ: Π·Π΅Π»Π΅Π½Π°Ρ ΠΈ ΠΊΡΠ°ΡΠ½Π°Ρ. ΠΠ°ΠΊ ΡΠΎΠ»ΡΠΊΠΎ Π²ΠΎΠΏΡΠΎΡ ΡΡΠΎΡΠΌΡΠ»ΠΈΡΠΎΠ²Π°Π½, ΡΡΠ΅Π½ΠΈΠΊΠΈ ΠΏΠΎΠ΄Π½ΠΈΠΌΠ°ΡΡ ΡΠΈΠ³Π½Π°Π»ΡΠ½ΡΠ΅ ΠΊΠ°ΡΡΡ (Π·Π½Π°Ρ ΠΎΡΠ²Π΅Ρ β Π·Π΅Π»Π΅Π½Π°Ρ ΠΊΠ°ΡΡΠΎΡΠΊΠ°, Π½Π΅ Π·Π½Π°Ρ ΠΎΡΠ²Π΅Ρ β ΠΊΡΠ°ΡΠ½Π°Ρ ΠΊΠ°ΡΡΠΎΡΠΊΠ°). ΠΠ°ΡΠ΅ΠΌ ΠΎΠ΄ΠΈΠ½ ΡΡΠ΅Π½ΠΈΠΊ ΠΏΠΎ ΠΏΡΠΎΡΡΠ±Π΅ ΡΡΠΈΡΠ΅Π»Ρ Π²ΡΠ»ΡΡ Π΄Π°Π΅Ρ ΠΎΡΠ²Π΅Ρ Π½Π° ΠΏΠΎΡΡΠ°Π²Π»Π΅Π½Π½ΡΠΉ Π²ΠΎΠΏΡΠΎΡ.
ΠΠΎΠΏΡΠΎΡ 1. Π‘ΡΠΎΡΠΌΡΠ»ΠΈΡΡΠΉΡΠ΅ Π»Π΅ΠΌΠΌΡ ΠΎ Π΄Π²ΡΡ ΠΊΠΎΠ»Π»ΠΈΠ½Π΅Π°ΡΠ½ΡΡ Π²Π΅ΠΊΡΠΎΡΠ°Ρ .
(ΠΡΠ»ΠΈ Π²Π΅ΠΊΡΠΎΡΡ ΠΈ ΠΊΠΎΠ»Π»ΠΈΠ½Π΅Π°ΡΠ½Ρ ΠΈ , ΡΠΎ ΡΡΡΠ΅ΡΡΠ²ΡΠ΅Ρ ΡΠ°ΠΊΠΎΠ΅ ΡΠΈΡΠ»ΠΎ k, ΡΡΠΎ ).
ΠΠΎΠΏΡΠΎΡ 2. Π‘ΡΠΎΡΠΌΡΠ»ΠΈΡΡΠΉΡΠ΅ ΡΠ΅ΠΎΡΠ΅ΠΌΡ ΠΎ ΡΠ°Π·Π»ΠΎΠΆΠ΅Π½ΠΈΠΈ Π²Π΅ΠΊΡΠΎΡΠ° ΠΏΠΎ Π΄Π²ΡΠΌ Π½Π΅ΠΊΠΎΠ»Π»ΠΈΠ½Π΅Π°ΡΠ½ΡΠΌ Π²Π΅ΠΊΡΠΎΡΠ°ΠΌ.
(ΠΡΠ±ΠΎΠΉ Π²Π΅ΠΊΡΠΎΡ ΠΌΠΎΠΆΠ½ΠΎ ΡΠ°Π·Π»ΠΎΠΆΠΈΡΡ ΠΏΠΎ Π΄Π²ΡΠΌ Π΄Π°Π½Π½ΡΠΌ
Π½Π΅ΠΊΠΎΠ»Π»ΠΈΠ½Π΅Π°ΡΠ½ΡΠΌ Π²Π΅ΠΊΡΠΎΡΠ°ΠΌ, ΠΏΡΠΈΡΠ΅ΠΌ ΠΊΠΎΡΡΡΠΈΡΠΈΠ΅Π½ΡΡ
ΡΠ°Π·Π»ΠΎΠΆΠ΅Π½ΠΈΡ ΠΎΠΏΡΠ΅Π΄Π΅Π»ΡΡΡΡΡ Π΅Π΄ΠΈΠ½ΡΡΠ²Π΅Π½Π½ΡΠΌ ΠΎΠ±ΡΠ°Π·ΠΎΠΌ).
ΠΠΎΠΏΡΠΎΡ 3. Π§ΡΠΎ ΡΠ°ΠΊΠΎΠ΅ ΠΊΠΎΠΎΡΠ΄ΠΈΠ½Π°ΡΡ Π²Π΅ΠΊΡΠΎΡΠ°?
(ΠΡΠΎ ΠΊΠΎΡΡΡΠΈΡΠΈΠ΅Π½ΡΡ ΡΠ°Π·Π»ΠΎΠΆΠ΅Π½ΠΈΡ Π²Π΅ΠΊΡΠΎΡΠ° ΠΏΠΎ ΠΊΠΎΠΎΡΠ΄ΠΈΠ½Π°ΡΠ½ΡΠΌ Π²Π΅ΠΊΡΠΎΡΠ°ΠΌ).
ΠΠΎΠΏΡΠΎΡ 4. Π‘ΡΠΎΡΠΌΡΠ»ΠΈΡΡΠΉΡΠ΅ ΠΏΡΠ°Π²ΠΈΠ»ΠΎ Π½Π°Ρ ΠΎΠΆΠ΄Π΅Π½ΠΈΡ ΠΊΠΎΠΎΡΠ΄ΠΈΠ½Π°Ρ ΡΡΠΌΠΌΡ Π²Π΅ΠΊΡΠΎΡΠΎΠ².
(ΠΠ°ΠΆΠ΄Π°Ρ ΠΊΠΎΠΎΡΠ΄ΠΈΠ½Π°ΡΠ° ΡΡΠΌΠΌΡ Π΄Π²ΡΡ ΠΈΠ»ΠΈ Π±ΠΎΠ»Π΅Π΅ Π²Π΅ΠΊΡΠΎΡΠΎΠ² ΡΠ°Π²Π½Π° ΡΡΠΌΠΌΠ΅ ΡΠΎΠΎΡΠ²Π΅ΡΡΡΠ²ΡΡΡΠΈΡ ΠΊΠΎΠΎΡΠ΄ΠΈΠ½Π°Ρ ΡΡΠΈΡ Π²Π΅ΠΊΡΠΎΡΠΎΠ²).
ΠΠΎΠΏΡΠΎΡ 5. Π‘ΡΠΎΡΠΌΡΠ»ΠΈΡΡΠΉΡΠ΅ ΠΏΡΠ°Π²ΠΈΠ»ΠΎ Π½Π°Ρ ΠΎΠΆΠ΄Π΅Π½ΠΈΡ ΠΊΠΎΠΎΡΠ΄ΠΈΠ½Π°Ρ ΡΠ°Π·Π½ΠΎΡΡΠΈ Π²Π΅ΠΊΡΠΎΡΠΎΠ².
(ΠΠ°ΠΆΠ΄Π°Ρ ΠΊΠΎΠΎΡΠ΄ΠΈΠ½Π°ΡΠ° ΡΠ°Π·Π½ΠΎΡΡΠΈ Π΄Π²ΡΡ ΠΈΠ»ΠΈ Π±ΠΎΠ»Π΅Π΅ Π²Π΅ΠΊΡΠΎΡΠΎΠ² ΡΠ°Π²Π½Π° ΡΠ°Π·Π½ΠΎΡΡΠΈ ΡΠΎΠΎΡΠ²Π΅ΡΡΡΠ²ΡΡΡΠΈΡ ΠΊΠΎΠΎΡΠ΄ΠΈΠ½Π°Ρ ΡΡΠΈΡ Π²Π΅ΠΊΡΠΎΡΠΎΠ²).
ΠΠΎΠΏΡΠΎΡ 6. Π‘ΡΠΎΡΠΌΡΠ»ΠΈΡΡΠΉΡΠ΅ ΠΏΡΠ°Π²ΠΈΠ»ΠΎ Π½Π°Ρ ΠΎΠΆΠ΄Π΅Π½ΠΈΡ ΠΊΠΎΠΎΡΠ΄ΠΈΠ½Π°Ρ ΠΏΡΠΎΠΈΠ·Π²Π΅Π΄Π΅Π½ΠΈΡ Π²Π΅ΠΊΡΠΎΡΠ° Π½Π° ΡΠΈΡΠ»ΠΎ.
(ΠΠ°ΠΆΠ΄Π°Ρ ΠΊΠΎΠΎΡΠ΄ΠΈΠ½Π°ΡΠ° ΠΏΡΠΎΠΈΠ·Π²Π΅Π΄Π΅Π½ΠΈΡ Π²Π΅ΠΊΡΠΎΡΠ° Π½Π° ΡΠΈΡΠ»ΠΎ ΡΠ°Π²Π½Π° ΠΏΡΠΎΠΈΠ·Π²Π΅Π΄Π΅Π½ΠΈΡ ΡΠΎΠΎΡΠ²Π΅ΡΡΡΠ²ΡΡΡΠΈΡ ΠΊΠΎΠΎΡΠ΄ΠΈΠ½Π°Ρ Π²Π΅ΠΊΡΠΎΡΠ° Π½Π° ΡΡΠΎ ΡΠΈΡΠ»ΠΎ).
ΠΠΎΠΏΡΠΎΡ 7. ΠΠ°ΠΏΠΈΡΠΈΡΠ΅ Π½Π° Π΄ΠΎΡΠΊΠ΅ ΡΠΎΡΠΌΡΠ»Ρ Π΄Π»Ρ Π²ΡΡΠΈΡΠ»Π΅Π½ΠΈΡ ΠΊΠΎΠΎΡΠ΄ΠΈΠ½Π°Ρ Π²Π΅ΠΊΡΠΎΡΠ° ΠΏΠΎ ΠΊΠΎΠΎΡΠ΄ΠΈΠ½Π°ΡΠ°ΠΌ Π΅Π³ΠΎ Π½Π°ΡΠ°Π»Π° ΠΈ ΠΊΠΎΠ½ΡΠ°.
()
ΠΠΎΠΏΡΠΎΡ 8.
ΠΠ°ΠΏΠΈΡΠΈΡΠ΅ Π½Π° Π΄ΠΎΡΠΊΠ΅ ΡΠΎΡΠΌΡΠ»Ρ Π΄Π»Ρ
Π²ΡΡΠΈΡΠ»Π΅Π½ΠΈΡ ΠΊΠΎΠΎΡΠ΄ΠΈΠ½Π°Ρ ΡΠ΅ΡΠ΅Π΄ΠΈΠ½Ρ ΠΎΡΡΠ΅Π·ΠΊΠ° ΠΏΠΎ
ΠΊΠΎΠΎΡΠ΄ΠΈΠ½Π°ΡΠ°ΠΌ Π΅Π³ΠΎ ΠΊΠΎΠ½ΡΠΎΠ².
()
ΠΠΎΠΏΡΠΎΡ 9. ΠΠ°ΠΏΠΈΡΠΈΡΠ΅ Π½Π° Π΄ΠΎΡΠΊΠ΅ ΡΠΎΡΠΌΡΠ»Ρ Π΄Π»Ρ Π²ΡΡΠΈΡΠ»Π΅Π½ΠΈΡ Π΄Π»ΠΈΠ½Ρ Π²Π΅ΠΊΡΠΎΡΠ° ΠΏΠΎ Π΅Π³ΠΎ ΠΊΠΎΠΎΡΠ΄ΠΈΠ½Π°ΡΠ°ΠΌ.
()
ΠΠΎΠΏΡΠΎΡ 10. ΠΠ°ΠΏΠΈΡΠΈΡΠ΅ Π½Π° Π΄ΠΎΡΠΊΠ΅ ΡΠΎΡΠΌΡΠ»Ρ Π΄Π»Ρ Π²ΡΡΠΈΡΠ»Π΅Π½ΠΈΡ ΡΠ°ΡΡΡΠΎΡΠ½ΠΈΡ ΠΌΠ΅ΠΆΠ΄Ρ Π΄Π²ΡΠΌΡ ΡΠΎΡΠΊΠ°ΠΌΠΈ.
()
ΠΠΎΠΏΡΠΎΡ 11. ΠΠ°ΠΏΠΈΡΠΈΡΠ΅ Π½Π° Π΄ΠΎΡΠΊΠ΅ ΡΡΠ°Π²Π½Π΅Π½ΠΈΠ΅ ΠΎΠΊΡΡΠΆΠ½ΠΎΡΡΠΈ Π΄Π°Π½Π½ΠΎΠ³ΠΎ ΡΠ°Π΄ΠΈΡΡΠ° Ρ ΡΠ΅Π½ΡΡΠΎΠΌ Π² Π΄ΠΎΠ½Π½ΠΎΠΉ ΡΠΎΡΠΊΠ΅ ΠΈ ΡΡΠ°Π²Π½Π΅Π½ΠΈΠ΅ ΠΎΠΊΡΡΠΆΠ½ΠΎΡΡΠΈ Ρ ΡΠ΅Π½ΡΡΠΎΠΌ Π² Π½Π°ΡΠ°Π»Π΅ ΠΊΠΎΠΎΡΠ΄ΠΈΠ½Π°Ρ.
ΠΠΎΠΏΡΠΎΡ 12. ΠΠ°ΠΏΠΈΡΠΈΡΠ΅ ΡΡΠ°Π²Π½Π΅Π½ΠΈΠ΅ Π΄Π°Π½Π½ΠΎΠΉ ΠΏΡΡΠΌΠΎΠΉ Π² ΠΏΡΡΠΌΠΎΡΠ³ΠΎΠ»ΡΠ½ΠΎΠΉ ΡΠΈΡΡΠ΅ΠΌΠ΅ ΠΊΠΎΠΎΡΠ΄ΠΈΠ½Π°Ρ.
(ax+by+c=0).
3 ΡΡΠ°ΠΏ. ΠΡΠΏΠΎΠ»Π½Π΅Π½ΠΈΠ΅ ΠΈΠ½Π΄ΠΈΠ²ΠΈΠ΄ΡΠ°Π»ΡΠ½ΠΎΠ³ΠΎ Π·Π°Π΄Π°Π½ΠΈΡ (10 ΠΌΠΈΠ½ΡΡ)
ΠΠ°ΠΆΠ΄ΠΎΠΌΡ ΡΡΠ΅Π½ΠΈΠΊΡ Π²ΡΠ΄Π°ΡΡΡΡ Π·Π°Π΄Π°Π½ΠΈΡ, ΠΊΠΎΡΠΎΡΡΠ΅ ΠΎΠ½ΠΈ Π΄ΠΎΠ»ΠΆΠ½Ρ Π²ΡΠΏΠΎΠ»Π½ΠΈΡΡ ΡΠ°ΠΌΠΎΡΡΠΎΡΡΠ΅Π»ΡΠ½ΠΎ.
ΠΡΠΈΠΌΠ΅ΡΠ½ΡΠ΅ Π²Π°ΡΠΈΠ°Π½ΡΡ ΠΊΠ°ΡΡΠΎΡΠ΅ΠΊ.
ΠΠ°ΡΡΠΎΡΠΊΠ° 1.
- ΠΠ΅ΠΆΠΈΡ Π»ΠΈ ΡΠΎΡΠΊΠ° A(2;-1) Π½Π° ΠΎΠΊΡΡΠΆΠ½ΠΎΡΡΠΈ, Π·Π°Π΄Π°Π½Π½ΠΎΠΉ ΡΡΠ°Π²Π½Π΅Π½ΠΈΠ΅ΠΌ ?
- ΠΠ°ΠΉΠ΄ΠΈΡΠ΅ Π΄Π»ΠΈΠ½Ρ Π²Π΅ΠΊΡΠΎΡΠ° .

- ΠΠ°ΠΉΠ΄ΠΈΡΠ΅ ΠΊΠΎΠΎΡΠ΄ΠΈΠ½Π°ΡΡ ΡΠ΅ΡΠ΅Π΄ΠΈΠ½Ρ ΠΎΡΡΠ΅Π·ΠΊΠ° PQ, Π΅ΡΠ»ΠΈ P(5;-3), Q(3;-7).
ΠΠ°ΡΡΠΎΡΠΊΠ° 2.
- ΠΠ°ΠΏΠΈΡΠΈΡΠ΅ ΡΡΠ°Π²Π½Π΅Π½ΠΈΠ΅ ΠΎΠΊΡΡΠΆΠ½ΠΎΡΡΠΈ, Π΅ΡΠ»ΠΈ Π΅Π΅ ΡΠ΅Π½ΡΡ β ΡΠΎΡΠΊΠ° (4; 5), Π° ΡΠ°Π΄ΠΈΡΡ ΡΠ°Π²Π΅Π½ 3.
- ΠΠ°ΠΉΠ΄ΠΈΡΠ΅ ΠΊΠΎΠΎΡΠ΄ΠΈΠ½Π°ΡΡ Π²Π΅ΠΊΡΠΎΡΠ° , Π΅ΡΠ»ΠΈ A(2;-5), B(-3;4).
- ΠΠ°ΠΏΠΈΡΠΈΡΠ΅ ΡΡΠ°Π²Π½Π΅Π½ΠΈΠ΅ ΠΏΡΡΠΌΠΎΠΉ, ΠΏΡΠΎΡ ΠΎΠ΄ΡΡΠ΅ΠΉ ΡΠ΅ΡΠ΅Π· ΡΠΎΡΠΊΡ M(3;-2) ΠΈ ΠΏΠ°ΡΠ°Π»Π»Π΅Π»ΡΠ½ΠΎΠΉ ΠΎΡΠΈ ΠΎΡΠ΄ΠΈΠ½Π°Ρ.
ΠΠ°ΡΡΠΎΡΠΊΠ° 3.
- ΠΠ°ΠΏΠΈΡΠΈΡΠ΅ ΡΡΠ°Π²Π½Π΅Π½ΠΈΠ΅ ΠΎΠΊΡΡΠΆΠ½ΠΎΡΡΠΈ Ρ ΡΠ΅Π½ΡΡΠΎΠΌ Π² Π½Π°ΡΠ°Π»Π΅ ΠΊΠΎΠΎΡΠ΄ΠΈΠ½Π°Ρ, Π΅ΡΠ»ΠΈ ΠΎΠ½Π° ΠΏΡΠΎΡ ΠΎΠ΄ΠΈΡ ΡΠ΅ΡΠ΅Π· ΡΠΎΡΠΊΡ C(2;-1).
- ΠΠ°ΠΉΠ΄ΠΈΡΠ΅ ΡΠ°ΡΡΡΠΎΡΠ½ΠΈΠ΅ ΠΌΠ΅ΠΆΠ΄Ρ ΡΠΎΡΠΊΠ°ΠΌΠΈ A(-1; 3) ΠΈ B(2; -1).
- ΠΠ°ΠΉΠ΄ΠΈΡΠ΅ ΠΊΠΎΠΎΡΠ΄ΠΈΠ½Π°ΡΡ Π²Π΅ΠΊΡΠΎΡΠ° , ΡΠ°Π²Π½ΠΎΠ³ΠΎ ΡΡΠΌΠΌΠ΅ Π²Π΅ΠΊΡΠΎΡΠΎΠ² ΠΈ , Π΅ΡΠ»ΠΈ .
ΠΠ°ΡΡΠΎΡΠΊΠ° 4.
- ΠΠ΅ΠΆΠΈΡ Π»ΠΈ ΡΠΎΡΠΊΠ° A(2;-1) Π½Π° ΠΏΡΡΠΌΠΎΠΉ, Π·Π°Π΄Π°Π½Π½ΠΎΠΉ ΡΡΠ°Π²Π½Π΅Π½ΠΈΠ΅ΠΌ 2x-3y-7=0.
- ΠΠ°ΠΏΠΈΡΠΈΡΠ΅ ΡΡΠ°Π²Π½Π΅Π½ΠΈΠ΅ ΠΎΠΊΡΡΠΆΠ½ΠΎΡΡΠΈ, Π΅ΡΠ»ΠΈ Π΅Π΅ ΡΠ΅Π½ΡΡ β
ΡΠΎΡΠΊΠ° (4;5), Π° ΡΠ°Π΄ΠΈΡΡ ΡΠ°Π²Π΅Π½ 2.

- ΠΠ°ΠΉΠ΄ΠΈΡΠ΅ ΠΊΠΎΠΎΡΠ΄ΠΈΠ½Π°ΡΡ Π²Π΅ΠΊΡΠΎΡΠ° , ΡΠ°Π²Π½ΠΎΠ³ΠΎ ΡΠ°Π·Π½ΠΎΡΡΠΈ Π²Π΅ΠΊΡΠΎΡΠΎΠ²Β ΠΈ , Π΅ΡΠ»ΠΈ .
ΠΠ°ΡΡΠΎΡΠΊΠ° 5.
- ΠΠ°ΠΏΠΈΡΠΈΡΠ΅ ΡΡΠ°Π²Π½Π΅Π½ΠΈΠ΅ ΠΎΠΊΡΡΠΆΠ½ΠΎΡΡΠΈ Ρ ΡΠ΅Π½ΡΡΠΎΠΌ Π² ΡΠΎΡΠΊΠ΅ P(-2;-1), Π΅ΡΠ»ΠΈ ΠΎΠ½Π° ΠΏΡΠΎΡ ΠΎΠ΄ΠΈΡ ΡΠ΅ΡΠ΅Π· ΡΠΎΡΠΊΡ Q(1;3).
- ΠΠ°ΠΉΠ΄ΠΈΡΠ΅ ΠΊΠΎΠΎΡΠ΄ΠΈΠ½Π°ΡΡ Π²Π΅ΠΊΡΠΎΡΠ° , Π΅ΡΠ»ΠΈ C(-1; 6), D(3; -2).
- ΠΠ°ΠΉΠ΄ΠΈΡΠ΅ ΠΊΠΎΠΎΡΠ΄ΠΈΠ½Π°ΡΡ Π²Π΅ΠΊΡΠΎΡΠ° , Π΅ΡΠ»ΠΈ , Π° .
4 ΡΡΠ°ΠΏ. ΠΡΡΠΏΠΏΠΎΠ²ΠΎΠ΅ ΡΠ΅ΡΠ΅Π½ΠΈΠ΅ Π·Π°Π΄Π°ΡΠΈ (15 ΠΌΠΈΠ½ΡΡ)
ΠΠ°ΠΆΠ΄ΠΎΠΉ Π³ΡΡΠΏΠΏΠ΅ Π΄Π°Π΅ΡΡΡ Π·Π°Π΄Π°ΡΠ°.
ΠΠ°Π½Ρ ΠΊΠΎΠΎΡΠ΄ΠΈΠ½Π°ΡΡ ΡΡΠ΅Ρ Π²Π΅ΡΡΠΈΠ½ ΠΏΠ°ΡΠ°Π»Π»Π΅Π»ΠΎΠ³ΡΠ°ΠΌΠΌΠ° KLMN: K(-4; 2), L(0; 5), M(12; 0). ΠΠ°ΠΉΠ΄ΠΈΡΠ΅ ΠΊΠΎΠΎΡΠ΄ΠΈΠ½Π°ΡΡ ΡΠ΅ΡΠ²Π΅ΡΡΠΎΠΉ Π²Π΅ΡΡΠΈΠ½Ρ ΠΈ ΠΏΠ΅ΡΠΈΠΌΠ΅ΡΡ Π΄Π°Π½Π½ΠΎΠ³ΠΎ ΠΏΠ°ΡΠ°Π»Π»Π΅Π»ΠΎΠ³ΡΠ°ΠΌΠΌΠ°.
ΠΡΠ΅Π΄ΡΡΠ°Π²ΠΈΡΠ΅Π»Ρ Π³ΡΡΠΏΠΏΡ, ΠΏΠ΅ΡΠ²ΠΎΠΉ ΠΏΡΠ°Π²ΠΈΠ»ΡΠ½ΠΎ ΡΠ΅ΡΠΈΠ²ΡΠ΅ΠΉ Π·Π°Π΄Π°ΡΡ, ΠΎΠ±ΡΡΡΠ½ΡΠ΅Ρ ΡΠ΅ΡΠ΅Π½ΠΈΠ΅ Ρ Π΄ΠΎΡΠΊΠΈ.
5 ΡΡΠ°ΠΏ. ΠΠΎΠΌΠ°ΡΠ½Π΅Π΅ Π·Π°Π΄Π°Π½ΠΈΠ΅
ΠΠΎ ΡΡΠ΅Π±Π½ΠΈΠΊΡ Π. Π‘. ΠΡΠ°Π½Π°ΡΡΠ½Π°, ββ 996, 1001, 1005.
2$$ ΠΈ Π·Π°Π΄Π°ΠΉΡΠ΅ ΠΊΠΎΡΡΡΠΈΡΠΈΠ΅Π½ΡΡ $a,b,c$ Π² Π²ΠΈΠ΄Π΅ ΠΊΠΎΠΎΡΠ΄ΠΈΠ½Π°ΡΠ½ΠΎΠ³ΠΎ Π²Π΅ΠΊΡΠΎΡΠ° Π²ΠΈΠ΄Π° $\langle a,b,c \rangle$ — ΠΊΠΎΠΎΡΠ΄ΠΈΠ½Π°ΡΡ ΠΎΡΠ½ΠΎΡΠΈΡΠ΅Π»ΡΠ½ΠΎ Π½ΠΎΠ²ΠΎΠ³ΠΎ Π±Π°Π·ΠΈΡΠ°.
ΠΠΎΠΌΠΈΠΌΠΎ Π»ΠΈΠ½Π΅ΠΉΠ½ΠΎ-Π°Π»Π³Π΅Π±ΡΠ°ΠΈΡΠ΅ΡΠΊΠΈΡ ΠΌΠ΅ΡΠΎΠ΄ΠΎΠ² ΡΠΌΠ΅Π½Ρ Π±Π°Π·ΠΈΡΠ°, ΠΌΠΎΠΆΠ½ΠΎ ΠΏΠ΅ΡΠ΅ΠΏΠΈΡΠ°ΡΡ ΠΌΠ½ΠΎΠ³ΠΎΡΠ»Π΅Π½ Π² ΡΡΠ΅Π±ΡΠ΅ΠΌΠΎΠΌ Π²ΠΈΠ΄Π΅ ΡΡΠ΄Π΅Π±Π½ΡΠΌ ΠΏΡΠΈΠΌΠ΅Π½Π΅Π½ΠΈΠ΅ΠΌ Π΄ΠΎΠΏΠΎΠ»Π½Π΅Π½ΠΈΡ ΠΊΠ²Π°Π΄ΡΠ°ΡΠ° ΠΈ ΡΠ°Π·Π»ΠΎΠΆΠ΅Π½ΠΈΡ Π½Π° ΠΌΠ½ΠΎΠΆΠΈΡΠ΅Π»ΠΈ, Π»ΠΈΠ±ΠΎ ΠΎΡΠ΅Π½Ρ ΠΏΡΠΎΡΡΡΠΌ ΠΏΡΠΈΠΌΠ΅Π½Π΅Π½ΠΈΠ΅ΠΌ ΡΠ΅ΠΎΡΠ΅ΠΌΡ Π’Π΅ΠΉΠ»ΠΎΡΠ° Π΄Π»Ρ Π½Π°Ρ ΠΎΠΆΠ΄Π΅Π½ΠΈΡ ΠΊΠΎΡΡΡΠΈΡΠΈΠ΅Π½ΡΠΎΠ² ΡΡΠ΄Π° Π’Π΅ΠΉΠ»ΠΎΡΠ° Ρ ΡΠ΅Π½ΡΡΠΎΠΌ Π² $t = 2$ (ΡΡΠΎ, Π²Π΅ΡΠΎΡΡΠ½ΠΎ, ΡΠ°ΠΌΡΠΉ ΡΡΡΠ΅ΠΊΡΠΈΠ²Π½ΡΠΉ ΠΌΠ΅ΡΠΎΠ΄). Π ΠΊΠ°ΡΠ΅ΡΡΠ²Π΅ Π°Π»ΡΡΠ΅ΡΠ½Π°ΡΠΈΠ²Ρ Π²Ρ ΠΌΠΎΠΆΠ΅ΡΠ΅ ΡΠ°Π·Π»ΠΎΠΆΠΈΡΡ ΠΏΠΎΠ»ΠΈΠ½ΠΎΠΌ Π²ΡΡΠ΅ ΠΈ ΡΠ΅ΡΠΈΡΡ ΡΠΈΡΡΠ΅ΠΌΡ ΡΡΠ°Π²Π½Π΅Π½ΠΈΠΉ Π΄Π»Ρ Π½Π΅ΠΎΠΏΡΠ΅Π΄Π΅Π»Π΅Π½Π½ΡΡ ΠΊΠΎΡΡΡΠΈΡΠΈΠ΅Π½ΡΠΎΠ². 92 — 4t + 4$ :
$${\bf T}= \left[\begin{array}{rrr} 1&-2&4\\ 0&1&-4\\ 0&0&1 \end{ΠΌΠ°ΡΡΠΈΠ²}\right]$$ ΠΡ Π²ΠΈΠ΄ΠΈΡΠ΅, ΠΊΠ°ΠΊΠΈΠ΅ ΡΠΈΡΠ»Π° Π΄ΠΎΠ»ΠΆΠ½Ρ Π±ΡΡΡ ΠΏΠΎΠΌΠ΅ΡΠ΅Π½Ρ Π³Π΄Π΅? Π Π·Π°ΡΠ΅ΠΌ ΡΠ΅ΡΠ°Π΅ΠΌ Π΄Π»Ρ $\bf v$ ΡΠΈΡΡΠ΅ΠΌΡ ΡΡΠ°Π²Π½Π΅Π½ΠΈΠΉ: $${\bf T}{\bf v} = \left[\begin{array}{r}2\\3\\-2\end{array }\right]$$
ΠΡΠΎ Π±ΡΠ΄Π΅Ρ Π»Π΅Π³ΠΊΠΎ, ΠΊΠΎΠ³Π΄Π° Π²Ρ Π½Π°ΡΡΠΈΡΠ΅ΡΡ ΡΠ΅ΡΠ°ΡΡ ΡΠΈΡΡΠ΅ΠΌΡ ΠΌΠ°ΡΡΠΈΡΠ½ΡΡ ΡΡΠ°Π²Π½Π΅Π½ΠΈΠΉ.
Π§ΡΠΎΠ±Ρ ΡΠ°ΡΡΠΈΡΠΈΡΡ ΡΠ΅ΡΠ΅Π½ΠΈΠ΅ JnxF (ΡΡΠΎΠ±Ρ ΠΈΠ·Π±Π΅ΠΆΠ°ΡΡ ΠΏΡΡΠ°Π½ΠΈΡΡ), ΠΌΠ°ΡΡΠΈΡΠ°, ΠΈΡΠΏΠΎΠ»ΡΠ·ΡΠ΅ΠΌΠ°Ρ Π΄Π»Ρ ΡΠΌΠ½ΠΎΠΆΠ΅Π½ΠΈΡ Π² Π΅Π³ΠΎ ΠΎΡΠ²Π΅ΡΠ΅, Π½Π° ΡΠ°ΠΌΠΎΠΌ Π΄Π΅Π»Π΅ ΡΠ²Π»ΡΠ΅ΡΡΡ 92\}$, ΡΠ°ΠΊ ΡΡΠΎ Π½Π° ΡΠ°ΠΌΠΎΠΌ Π΄Π΅Π»Π΅ ΡΡΠΎ Β«ΠΏΠ΅ΡΠ΅ΠΌΠ΅ΡΠ΅Π½ΠΈΠ΅Β» ΡΠΈΡΡΠ΅ΠΌΡ ΠΊΠΎΠΎΡΠ΄ΠΈΠ½Π°Ρ Π½Π°Π·Π°Π΄: $t\to t+2$, ΡΡΠΎ ΠΎΡΠΌΠ΅Π½ΡΠ΅Ρ ΠΏΡΠ΅Π΄ΡΠ΄ΡΡΠ΅Π΅ ΠΈΠ·ΠΌΠ΅Π½Π΅Π½ΠΈΠ΅ $t \to t-2$.
$\endgroup$
4
Π’Π²ΠΎΠΉ ΠΎΡΠ²Π΅Ρ
ΠΠ°ΡΠ΅Π³ΠΈΡΡΡΠΈΡΡΠΉΡΠ΅ΡΡ ΠΈΠ»ΠΈ Π²ΠΎΠΉΠ΄ΠΈΡΠ΅ Π² ΡΠΈΡΡΠ΅ΠΌΡ
ΠΠ°ΡΠ΅Π³ΠΈΡΡΡΠΈΡΡΠΉΡΠ΅ΡΡ Ρ ΠΏΠΎΠΌΠΎΡΡΡ Google
ΠΠ°ΡΠ΅Π³ΠΈΡΡΡΠΈΡΠΎΠ²Π°ΡΡΡΡ ΡΠ΅ΡΠ΅Π· Facebook
ΠΠ°ΡΠ΅Π³ΠΈΡΡΡΠΈΡΡΠΉΡΠ΅ΡΡ, ΠΈΡΠΏΠΎΠ»ΡΠ·ΡΡ Π°Π΄ΡΠ΅Ρ ΡΠ»Π΅ΠΊΡΡΠΎΠ½Π½ΠΎΠΉ ΠΏΠΎΡΡΡ ΠΈ ΠΏΠ°ΡΠΎΠ»Ρ
ΠΠΏΡΠ±Π»ΠΈΠΊΠΎΠ²Π°ΡΡ ΠΊΠ°ΠΊ Π³ΠΎΡΡΡ
ΠΠ»Π΅ΠΊΡΡΠΎΠ½Π½Π°Ρ ΠΏΠΎΡΡΠ°
Π’ΡΠ΅Π±ΡΠ΅ΡΡΡ, Π½ΠΎ Π½ΠΈΠΊΠΎΠ³Π΄Π° Π½Π΅ ΠΎΡΠΎΠ±ΡΠ°ΠΆΠ°Π΅ΡΡΡ
ΠΠΏΡΠ±Π»ΠΈΠΊΠΎΠ²Π°ΡΡ ΠΊΠ°ΠΊ Π³ΠΎΡΡΡ
ΠΠ»Π΅ΠΊΡΡΠΎΠ½Π½Π°Ρ ΠΏΠΎΡΡΠ°
Π’ΡΠ΅Π±ΡΠ΅ΡΡΡ, Π½ΠΎ Π½Π΅ ΠΎΡΠΎΠ±ΡΠ°ΠΆΠ°Π΅ΡΡΡ
ΠΠ°ΠΆΠΈΠΌΠ°Ρ Β«ΠΠΏΡΠ±Π»ΠΈΠΊΠΎΠ²Π°ΡΡ ΡΠ²ΠΎΠΉ ΠΎΡΠ²Π΅ΡΒ», Π²Ρ ΡΠΎΠ³Π»Π°ΡΠ°Π΅ΡΠ΅ΡΡ Ρ Π½Π°ΡΠΈΠΌΠΈ ΡΡΠ»ΠΎΠ²ΠΈΡΠΌΠΈ ΠΎΠ±ΡΠ»ΡΠΆΠΈΠ²Π°Π½ΠΈΡ, ΠΏΠΎΠ»ΠΈΡΠΈΠΊΠΎΠΉ ΠΊΠΎΠ½ΡΠΈΠ΄Π΅Π½ΡΠΈΠ°Π»ΡΠ½ΠΎΡΡΠΈ ΠΈ ΠΏΠΎΠ»ΠΈΡΠΈΠΊΠΎΠΉ ΠΈΡΠΏΠΎΠ»ΡΠ·ΠΎΠ²Π°Π½ΠΈΡ ΡΠ°ΠΉΠ»ΠΎΠ² cookie
Π»ΠΈΠ½Π΅ΠΉΠ½Π°Ρ Π°Π»Π³Π΅Π±ΡΠ° — ΠΠΏΡΠ΅Π΄Π΅Π»Π΅Π½ΠΈΠ΅ ΠΊΠΎΠΎΡΠ΄ΠΈΠ½Π°Ρ Π²Π΅ΠΊΡΠΎΡΠ° ΠΎΡΠ½ΠΎΡΠΈΡΠ΅Π»ΡΠ½ΠΎ Π±Π°Π·ΠΈΡΠ°
Π·Π°Π΄Π°Π½Π½ΡΠΉ Π²ΠΎΠΏΡΠΎΡ
ΠΠ·ΠΌΠ΅Π½Π΅Π½ΠΎ 7 Π»Π΅Ρ, 2 ΠΌΠ΅ΡΡΡΠ° Π½Π°Π·Π°Π΄
ΠΡΠΎΡΠΌΠΎΡΡΠ΅Π½ΠΎ 67 ΡΠ°Π·
$\begingroup$
ΠΠ°Π΄Π°ΡΠ°: ΠΡΡΡΡ $V = \mathbb{R}[X]_{\leq 4}$ β Π²Π΅ΠΊΡΠΎΡΠ½ΠΎΠ΅ ΠΏΡΠΎΡΡΡΠ°Π½ΡΡΠ²ΠΎ Π²ΡΠ΅Ρ
ΠΌΠ½ΠΎΠ³ΠΎΡΠ»Π΅Π½ΠΎΠ² ΡΡΠ΅ΠΏΠ΅Π½ΠΈ Π½Π΅ Π²ΡΡΠ΅ $n$, ΠΈ ΠΏΡΡΡΡ $\alpha = \left\{1 , 1+x, (1+x)^2, (1+x)^3, (1+x)^4\right\}$ β Π±Π°Π·ΠΈΡ Π΄Π»Ρ $V$.
4$ ΠΎΡΠ½ΠΎΡΠΈΡΠ΅Π»ΡΠ½ΠΎ ΡΡΠΎΠ³ΠΎ Π±Π°Π·ΠΈΡΠ°. 94) \end{align*} Π³Π΄Π΅ Ρ ΡΠ°ΡΡΠΈΡΠΈΠ» Π±Π°Π·ΠΈΡΠ½ΡΠ΅ Π²Π΅ΠΊΡΠΎΡΡ. ΠΡΠΈ ΠΏΡΠΎΠ²Π΅ΡΠΊΠ΅ ΡΠΎΠΎΡΠ²Π΅ΡΡΡΠ²ΡΡΡΠΈΡ
ΠΊΠΎΡΡΡΠΈΡΠΈΠ΅Π½ΡΠΎΠ² ΠΏΠΎΠ»ΡΡΠ°Π΅ΡΡΡ ΡΠ»Π΅Π΄ΡΡΡΠ°Ρ ΡΠΈΡΡΠ΅ΠΌΠ° ΡΡΠ°Π²Π½Π΅Π½ΠΈΠΉ: \begin{align*} \begin{cases} \lambda_1 + \lambda_2 + \lambda_3 + \lambda_4 + \lambda_5 &= 0 \\ \lambda_2 + 2 \ lambda_3 + 3\lambda_4 + 4 \lambda_5 &= 1 \\ \lambda_3 + 3 \lambda_4 + 6 \lambda_5 &= 0 \\ \lambda_4 + 4 \lambda_5 &= 1 \\ \lambda_5 &= 1 \end{cases} \end{align*} Π― ΠΏΠΎΠΌΠ΅ΡΡΠΈΠ» ΡΡΠΎ Π² ΡΠ°ΡΡΠΈΡΠ΅Π½Π½ΡΡ ΠΌΠ°ΡΡΠΈΡΡ ΠΈ ΡΠΌΠ΅Π½ΡΡΠΈΠ» ΡΡΡΠΎΠΊΡ:
\begin{Π²ΡΡΠ°Π²Π½ΠΈΠ²Π°Π½ΠΈΠ΅*} ΠΈ \left(\begin{ΠΌΠ°ΡΡΠΈΠ²}{ccccc|c}
1 ΠΈ 1 ΠΈ 1 ΠΈ 1 ΠΈ 1 ΠΈ 0 \\
0 ΠΈ 1 ΠΈ 2 ΠΈ 3 ΠΈ 4 ΠΈ 1 \\
0 ΠΈ 0 ΠΈ 1 ΠΈ 3 ΠΈ 6 ΠΈ 0 \\
0 ΠΈ 0 ΠΈ 0 ΠΈ 1 ΠΈ 4 ΠΈ 1 \\
0 ΠΈ 0 ΠΈ 0 ΠΈ 0 ΠΈ 1 ΠΈ 1
\ΠΊΠΎΠ½Π΅Ρ{ΠΌΠ°ΡΡΠΈΠ²}\ΡΠΏΡΠ°Π²Π°)
\begin{matrix} \xrightarrow{R_4 \rightarrow R_4 — 4 R_5} \\ \xrightarrow{R_3 \rightarrow R_3 — 6 R_5} \\ \xrightarrow{R_2 \rightarrow R_2 — 4R_5} \\ \xrightarrow{R_1 \rightarrow R_1 — R_5} \end{matrix} \left(\begin{array}{ccccc|c}
1 ΠΈ 1 ΠΈ 1 ΠΈ 1 ΠΈ 0 ΠΈ -1 \\
0 ΠΈ 1 ΠΈ 2 ΠΈ 3 ΠΈ 0 ΠΈ -3 \\
0 ΠΈ 0 ΠΈ 1 ΠΈ 3 ΠΈ 0 ΠΈ -6 \\
0 & 0 & 0 & 1 & 0 & -3 \\
0 ΠΈ 0 ΠΈ 0 ΠΈ 0 ΠΈ 1 ΠΈ 1
\end{ΠΌΠ°ΡΡΠΈΠ²}\right) \\ \begin{matrix} \xrightarrow{R_3 \rightarrow R_3 — 3R_4} \\ \xrightarrow{R_2 \rightarrow R_2 — 3 R_4} \\ \xrightarrow{R_1 \rightarrow R_1 — R_4} \end{ΠΌΠ°ΡΡΠΈΡΠ°} & \left(\begin{ΠΌΠ°ΡΡΠΈΠ²}{ccccc|c}
1 ΠΈ 1 ΠΈ 1 ΠΈ 0 ΠΈ 0 ΠΈ -4 \\
0 ΠΈ 1 ΠΈ 2 ΠΈ 0 ΠΈ 0 ΠΈ 6 \\
0 ΠΈ 0 ΠΈ 1 ΠΈ 0 ΠΈ 0 ΠΈ 3 \\
0 ΠΈ 0 ΠΈ 0 ΠΈ 1 ΠΈ 0 ΠΈ -3 \\
0 ΠΈ 0 ΠΈ 0 ΠΈ 0 ΠΈ 1 ΠΈ 1
\ΠΊΠΎΠ½Π΅Ρ{ΠΌΠ°ΡΡΠΈΠ²}\ΡΠΏΡΠ°Π²Π°)
\end{align*} ΠΠ°ΡΠ΅ΠΌ Ρ ΠΎΠ±ΡΠ°ΡΠ½ΠΎΠΉ Π·Π°ΠΌΠ΅Π½ΠΎΠΉ Ρ ΠΎΠ±Π½Π°ΡΡΠΆΠΈΠ», ΡΡΠΎ \begin{align*} \begin{cases} \lambda_5 &= 1 \\ \lambda_4 &= -3 \\ \lambda_3 &= 3 \\ \lambda_2 &= 0 \\ \lambda_1 &= -7 \end{cases} \end{align*} ΠΠΎ ΠΊΠΎΠ³Π΄Π° Ρ ΠΏΠΎΠ΄ΡΡΠ°Π²Π»ΡΡ ΡΡΠΎ Π² ΡΡΠ°Π²Π½Π΅Π½ΠΈΠ΅ Π²ΡΡΠ΅, ΡΡΠΎ Π½Π΅ Π΄Π°Π΅Ρ ΠΌΠ½Π΅ ΠΏΡΠ°Π²ΠΈΠ»ΡΠ½ΡΠΉ Π²Π΅ΠΊΡΠΎΡ.
ΠΠΎΠΆΠ΅Ρ ΠΊΡΠΎ-Π½ΠΈΠ±ΡΠ΄Ρ ΡΠΊΠ°Π·Π°ΡΡ ΠΌΠ½Π΅, Π³Π΄Π΅ Ρ ΠΎΡΠΈΠ±ΡΡ, ΠΏΠΎΠΆΠ°Π»ΡΠΉΡΡΠ°?
- Π»ΠΈΠ½Π΅ΠΉΠ½Π°Ρ Π°Π»Π³Π΅Π±ΡΠ°
- Π²Π΅ΠΊΡΠΎΡΠ½ΡΠ΅ ΠΏΡΠΎΡΡΡΠ°Π½ΡΡΠ²Π°
- ΡΠΈΡΡΠ΅ΠΌΡ ΡΡΠ°Π²Π½Π΅Π½ΠΈΠΉ
$\endgroup$
2
$\begingroup$
Π ΡΠΎΠΊΡΠ°ΡΠ΅Π½ΠΈΠΈ ΠΏΠΎΡΠ»Π΅Π΄Π½Π΅ΠΉ ΡΡΡΠΎΠΊΠΈ, ΠΊΠΎΠ³Π΄Π° Π²Ρ Π²ΡΠΏΠΎΠ»Π½ΡΠ΅ΡΠ΅ $R_1\leftarrow R_1-R_4$, ΠΏΠΎΡΠ»Π΅Π΄Π½Π΅Π΅ ΡΠΈΡΠ»ΠΎ Π² $R_1$ Π΄ΠΎΠ»ΠΆΠ½ΠΎ Π±ΡΡΡ $-1-(-3)=-1+3=2$, Π½ΠΎ Π²Ρ Π½Π°ΠΏΠΈΡΠ°Π»ΠΈ $-4 $. ΠΡΠΎ Π΄Π΅Π»Π°Π΅Ρ ΡΠ°ΠΌΡΡ ΠΏΠΎΡΠ»Π΅Π΄Π½ΡΡ ΠΎΠ±ΡΠ°ΡΠ½ΡΡ Π·Π°ΠΌΠ΅Π½Ρ $\lambda_1$ Π½Π΅ΠΏΡΠ°Π²ΠΈΠ»ΡΠ½ΠΎΠΉ.
Π― Π½Π΅ ΠΏΠΎΠ½ΠΈΠΌΠ°Ρ, ΡΡΠΎ Π²Ρ Π΄Π΅Π»Π°Π΅ΡΠ΅. ΠΠ°ΡΠ° ΠΈΡΡ ΠΎΠ΄Π½Π°Ρ ΠΌΠ°ΡΡΠΈΡΠ° Π±ΡΠ»Π° Π²Π΅ΡΡ Π½Π΅ΡΡΠ΅ΡΠ³ΠΎΠ»ΡΠ½ΠΎΠΉ, ΠΏΠΎΡΡΠΎΠΌΡ ΠΎΡΡΡΠ΄Π° ΠΌΠΎΠΆΠ½ΠΎ Π±ΡΠ»ΠΎ Π²ΡΠΏΠΎΠ»Π½ΠΈΡΡ ΠΎΠ±ΡΠ°ΡΠ½ΡΡ ΠΏΠΎΠ΄ΡΡΠ°Π½ΠΎΠ²ΠΊΡ, ΡΠ°ΠΊ ΡΡΠΎ Π²ΠΎΠΎΠ±ΡΠ΅ Π½Π΅ ΡΡΠ΅Π±ΠΎΠ²Π°Π»ΠΎΡΡ ΡΠΎΠΊΡΠ°ΡΠ΅Π½ΠΈΡ ΡΡΡΠΎΠΊ. Π Π²Ρ ΠΎΡΡΠ°Π½ΠΎΠ²ΠΈΠ»ΠΈΡΡ Π΄ΠΎ ΡΠΎΠ³ΠΎ, ΠΊΠ°ΠΊ Π΄ΠΎΡΠ»ΠΈ Π΄ΠΎ ΡΠΎΡΠΌΡ ΡΡΠ΅Π»ΠΎΠ½Π° Ρ ΡΠΌΠ΅Π½ΡΡΠ΅Π½Π½ΡΠΌ ΡΠΈΡΠ»ΠΎΠΌ ΡΡΡΠΎΠΊ, Π³Π΄Π΅ Π½Π΅ ΠΏΠΎΡΡΠ΅Π±ΠΎΠ²Π°Π»Π°ΡΡ Π±Ρ ΠΎΠ±ΡΠ°ΡΠ½Π°Ρ Π·Π°ΠΌΠ΅Π½Π°. ΠΠ°ΠΊΠΎΠΉ Ρ ΡΠ΅Π±Ρ Π±ΡΠ» ΡΠΌΡΡΠ» Π΄Π΅Π»Π°ΡΡ Π½Π΅ΡΠΊΠΎΠ»ΡΠΊΠΎ ΡΠΎΠΊΡΠ°ΡΠ΅Π½ΠΈΠΉ ΡΡΠ΄ΠΎΠ², Π½Π΅ ΠΏΠ΅ΡΠ΅Ρ ΠΎΠ΄Ρ ΠΏΠΎΠ»Π½ΠΎΡΡΡΡ ΠΊ ΡΠΌΠ΅Π½ΡΡΠ΅Π½Π½ΠΎΠΉ ΡΡΠ΅Π»ΠΎΠ½ΠΈΡΠΎΠ²Π°Π½Π½ΠΎΠΉ ΡΠΎΡΠΌΠ΅ ΡΡΠ΄ΠΎΠ²?
$\endgroup$
$\begingroup$
ΠΠΎΠΎΡΠ΄ΠΈΠ½Π°ΡΡ: {$-7,12,-3,-3,1$} ΡΠΌ.
Π²Π°ΡΠΈ ΠΎΠΏΠ΅ΡΠ°ΡΠΈΠΈ
$\endgroup$
2
Π’Π²ΠΎΠΉ ΠΎΡΠ²Π΅Ρ
ΠΠ°ΡΠ΅Π³ΠΈΡΡΡΠΈΡΡΠΉΡΠ΅ΡΡ ΠΈΠ»ΠΈ Π²ΠΎΠΉΠ΄ΠΈΡΠ΅
ΠΠ°ΡΠ΅Π³ΠΈΡΡΡΠΈΡΠΎΠ²Π°ΡΡΡΡ ΡΠ΅ΡΠ΅Π· Google
ΠΠ°ΡΠ΅Π³ΠΈΡΡΡΠΈΡΠΎΠ²Π°ΡΡΡΡ ΡΠ΅ΡΠ΅Π· Facebook
ΠΠ°ΡΠ΅Π³ΠΈΡΡΡΠΈΡΡΠΉΡΠ΅ΡΡ, ΠΈΡΠΏΠΎΠ»ΡΠ·ΡΡ ΡΠ»Π΅ΠΊΡΡΠΎΠ½Π½ΡΡ ΠΏΠΎΡΡΡ ΠΈ ΠΏΠ°ΡΠΎΠ»Ρ
ΠΠΏΡΠ±Π»ΠΈΠΊΠΎΠ²Π°ΡΡ ΠΊΠ°ΠΊ Π³ΠΎΡΡΡ
ΠΠ»Π΅ΠΊΡΡΠΎΠ½Π½Π°Ρ ΠΏΠΎΡΡΠ°
Π’ΡΠ΅Π±ΡΠ΅ΡΡΡ, Π½ΠΎ Π½ΠΈΠΊΠΎΠ³Π΄Π° Π½Π΅ ΠΎΡΠΎΠ±ΡΠ°ΠΆΠ°Π΅ΡΡΡ
ΠΠΏΡΠ±Π»ΠΈΠΊΠΎΠ²Π°ΡΡ ΠΊΠ°ΠΊ Π³ΠΎΡΡΡ
ΠΠ»Π΅ΠΊΡΡΠΎΠ½Π½Π°Ρ ΠΏΠΎΡΡΠ°
Π’ΡΠ΅Π±ΡΠ΅ΡΡΡ, Π½ΠΎ Π½Π΅ ΠΎΡΠΎΠ±ΡΠ°ΠΆΠ°Π΅ΡΡΡ
ΠΠ°ΠΆΠΈΠΌΠ°Ρ Β«ΠΠΏΡΠ±Π»ΠΈΠΊΠΎΠ²Π°ΡΡ ΡΠ²ΠΎΠΉ ΠΎΡΠ²Π΅ΡΒ», Π²Ρ ΡΠΎΠ³Π»Π°ΡΠ°Π΅ΡΠ΅ΡΡ Ρ Π½Π°ΡΠΈΠΌΠΈ ΡΡΠ»ΠΎΠ²ΠΈΡΠΌΠΈ ΠΎΠ±ΡΠ»ΡΠΆΠΈΠ²Π°Π½ΠΈΡ, ΠΏΠΎΠ»ΠΈΡΠΈΠΊΠΎΠΉ ΠΊΠΎΠ½ΡΠΈΠ΄Π΅Π½ΡΠΈΠ°Π»ΡΠ½ΠΎΡΡΠΈ ΠΈ ΠΏΠΎΠ»ΠΈΡΠΈΠΊΠΎΠΉ ΠΈΡΠΏΠΎΠ»ΡΠ·ΠΎΠ²Π°Π½ΠΈΡ ΡΠ°ΠΉΠ»ΠΎΠ² cookie
ΠΠΎΠΎΡΠ΄ΠΈΠ½Π°ΡΡ ΡΠΎΡΠΊΠΈ, ΠΊΠΎΠΌΠΏΠΎΠ½Π΅Π½ΡΡ Π²Π΅ΠΊΡΠΎΡΠ° ΠΈ ΡΠ΅ΡΠ΅Π΄ΠΈΠ½Ρ ΠΎΡΡΠ΅Π·ΠΊΠ°
ΠΠΎΠΎΡΠ΄ΠΈΠ½Π°ΡΡ ΡΠΎΡΠΊΠΈ Π½Π° ΠΏΠ»ΠΎΡΠΊΠΎΡΡΠΈ
ΠΠ°Π²Π°ΠΉΡΠ΅ ΠΏΠΎΡΠΌΠΎΡΡΠΈΠΌ, ΠΊΠ°ΠΊ Π²Π΅ΠΊΡΠΎΡΡ ΠΈΡΠΏΠΎΠ»ΡΠ·ΡΡΡΡΡ Π΄Π»Ρ ΠΏΡΠΈΡΠ²ΠΎΠ΅Π½ΠΈΡ ΠΊΠΎΠΎΡΠ΄ΠΈΠ½Π°Ρ ΡΠΎΡΠΊΠ°ΠΌ Π½Π° ΠΏΠ»ΠΎΡΠΊΠΎΡΡΠΈ.
Π Π°ΡΡΠΌΠΎΡΡΠΈΠΌ ΡΠΈΠΊΡΠΈΡΠΎΠ²Π°Π½Π½ΡΡ ΡΠΎΡΠΊΡ Π½Π° ΠΏΠ»ΠΎΡΠΊΠΎΡΡΠΈ $$O$$ (ΠΈΠ·Π²Π΅ΡΡΠ½ΡΡ ΠΊΠ°ΠΊ Π½Π°ΡΠ°Π»ΠΎ ΠΊΠΎΠΎΡΠ΄ΠΈΠ½Π°Ρ) ΠΈ Π±Π°Π·ΠΈΡ $$B=\{\overrightarrow{u}, \overrightarrow{v}\}$$ $$V_2$ $ (ΠΏΡΠΎΡΡΡΠ°Π½ΡΡΠ²Π΅Π½Π½ΡΠΉ Π²Π΅ΠΊΡΠΎΡ ΡΠ°Π·ΠΌΠ΅ΡΠ½ΠΎΡΡΠΈ $$2$$).
ΠΠ°ΠΏΠΎΠΌΠ½ΠΈΠΌ, ΡΡΠΎ Π² ΠΎΡΠ½ΠΎΠ²Π΅ $$V_2$$ Π»Π΅ΠΆΠ°Ρ Π΄Π²Π° Π»ΠΈΠ½Π΅ΠΉΠ½ΠΎ Π½Π΅Π·Π°Π²ΠΈΡΠΈΠΌΡΡ Π²Π΅ΠΊΡΠΎΡΠ°. ΠΠ½ΠΎΠΆΠ΅ΡΡΠ²ΠΎ, ΠΎΠ±ΡΠ°Π·ΠΎΠ²Π°Π½Π½ΠΎΠ΅ $$O$$ ΠΈ $$B=\{\overrightarrow{u}, \overrightarrow{v}\}$$, ΠΎΠ±ΡΠ°Π·ΡΠ΅Ρ ΡΠΈΡΡΠ΅ΠΌΡ ΠΎΡΡΡΠ΅ΡΠ° Π½Π° ΠΏΠ»ΠΎΡΠΊΠΎΡΡΠΈ, ΠΏΠΎΡΠΊΠΎΠ»ΡΠΊΡ ΠΏΠΎΠ·Π²ΠΎΠ»ΡΠ΅Ρ ΠΎΠΏΡΠ΅Π΄Π΅Π»ΠΈΡΡ ΠΏΠΎΠ»ΠΎΠΆΠ΅Π½ΠΈΠ΅ Π»ΡΠ±ΡΡ Π΄ΡΡΠ³ΠΈΡ ΡΠΎΡΠΊΠΈ Π½Π° ΠΏΠ»ΠΎΡΠΊΠΎΡΡΠΈ.
ΠΡΠΎ ΡΠ²ΡΠ·Π°Π½ΠΎ Ρ ΡΠ΅ΠΌ, ΡΡΠΎ Π»ΡΠ±ΡΠ΅ Π΄ΡΡΠ³ΠΈΠ΅ ΡΠΎΡΠΊΠΈ $$P$$ Π½Π° ΠΏΠ»ΠΎΡΠΊΠΎΡΡΠΈ Π²ΠΌΠ΅ΡΡΠ΅ Ρ ΡΠΎΡΠΊΠΎΠΉ $$O$$ ΠΎΠΏΡΠ΅Π΄Π΅Π»ΡΡΡ Π²Π΅ΠΊΡΠΎΡ $$\overrightarrow{OP}$$. ΠΡΡΡΡ $$(p_1,p_2)$$ β ΠΊΠΎΠΌΠΏΠΎΠ½Π΅Π½ΡΡ Π²Π΅ΠΊΡΠΎΡΠ° Π² Π±Π°Π·ΠΈΡΠ΅ $$B$$. Π’ΠΎΠ³Π΄Π° $$(p_1,p_2)$$ β ΡΡΠΎ ΠΊΠΎΠΎΡΠ΄ΠΈΠ½Π°ΡΡ ΡΠΎΡΠΊΠΈ $$P$$ Π² ΡΠΈΡΡΠ΅ΠΌΠ΅ ΠΎΡΡΡΠ΅ΡΠ° $$R=\{O;\overrightarrow{u}, \overrightarrow{v}\}$$, ΠΈ ΠΌΡ ΠΏΠΈΡΠ΅ΠΌ $$P=(p_1,p_2)$$.
ΠΡΠΎΡΠ΅Π΄ΡΡΠ° Π½Π°Ρ ΠΎΠΆΠ΄Π΅Π½ΠΈΡ ΠΊΠΎΠΎΡΠ΄ΠΈΠ½Π°Ρ ΡΠΎΡΠΊΠΈ $$P$$ Π² Π·Π°Π΄Π°Π½Π½ΠΎΠΉ ΡΠΈΡΡΠ΅ΠΌΠ΅ ΠΎΡΡΡΠ΅ΡΠ° ΡΠ»Π΅Π΄ΡΡΡΠ°Ρ:
ΠΠ· ΡΠΎΡΠ΅ΠΊ $$O$$ ΠΈ $$P$$ ΠΎΠΏΡΠ΅Π΄Π΅Π»ΡΠ΅ΠΌ Π²Π΅ΠΊΡΠΎΡ $$\overrightarrow{OP}$$
ΠΡΡΠ°Π·ΠΈΠΌ Π²Π΅ΠΊΡΠΎΡ $$\overrightarrow{OP}$$ Π² Π²ΠΈΠ΄Π΅ Π»ΠΈΠ½Π΅ΠΉΠ½ΠΎΠΉ ΠΊΠΎΠΌΠ±ΠΈΠ½Π°ΡΠΈΠΈ Π²Π΅ΠΊΡΠΎΡΠΎΠ² Π±Π°Π·ΠΈΡΠ° $$B=\{\overrightarrow{u}, \overrightarrow{v}\}$$, Ρ.
Π΅. , $$\overrightarrow{OP}=p_1 \cdot \overrightarrow{u}+p_2 \cdot \overrightarrow{v}$$- $$P=(p_1,p_2)$$
ΠΡΡΠ°Π·ΠΈΡΠ΅ ΡΠΎΡΠΊΡ $$P$$ ΡΠ΅ΡΡΠ΅ΠΆΠ° Π² ΡΠΈΡΡΠ΅ΠΌΠ΅ ΠΎΡΡΡΠ΅ΡΠ° $$R =\{O;\overrightarrow {u}, \overrightarrow{v}\}$$.
- Π ΠΈΡΡΠ΅ΠΌ Π²Π΅ΠΊΡΠΎΡ $$\overrightarrow{OP}$$:
- ΠΡΡΠ°Π·ΠΈΠΌ Π²Π΅ΠΊΡΠΎΡ $$\overrightarrow{OP}$$ Π² Π²ΠΈΠ΄Π΅ Π»ΠΈΠ½Π΅ΠΉΠ½ΠΎΠΉ ΠΊΠΎΠΌΠ±ΠΈΠ½Π°ΡΠΈΠΈ Π²Π΅ΠΊΡΠΎΡΠΎΠ² Π±Π°Π·ΠΈΡΠ° $$B=\{\overrightarrow{u}, \overrightarrow{v}\}$$:
- ΠΠΎΠ»ΡΡΠ°Π΅ΠΌ $$\overrightarrow{OP}=\overrightarrow{u}+2\overrightarrow{v}$$ ΠΈ, ΡΠ»Π΅Π΄ΠΎΠ²Π°ΡΠ΅Π»ΡΠ½ΠΎ, ΠΊΠΎΠΎΡΠ΄ΠΈΠ½Π°ΡΡ ΡΠΎΡΠΊΠΈ $$P$$ ΡΠ°Π²Π½Ρ $$P = (1 , 2 )$$
Π Π΄Π°Π»ΡΠ½Π΅ΠΉΡΠ΅ΠΌ Π² ΠΊΠ°ΡΠ΅ΡΡΠ²Π΅ ΡΠΈΡΡΠ΅ΠΌΡ ΠΎΡΡΡΠ΅ΡΠ° $$R$$ Π±ΡΠ΄Π΅ΠΌ ΡΠ°ΡΡΠΌΠ°ΡΡΠΈΠ²Π°ΡΡ ΡΠΈΡΡΠ΅ΠΌΡ, ΠΎΠ±ΡΠ°Π·ΠΎΠ²Π°Π½Π½ΡΡ Π½Π°ΡΠ°Π»ΠΎΠΌ ΠΊΠΎΠΎΡΠ΄ΠΈΠ½Π°Ρ $$O = (0, 0)$$ ΠΈ ΠΊΠ°Π½ΠΎΠ½ΠΈΡΠ΅ΡΠΊΠΈΠΌ Π±Π°Π·ΠΈΡΠΎΠΌ $$V_2$$ $$B =\{\overrightarrow{i},\overrightarrow{j}\}$$.
ΠΠΎΠΌΠΏΠΎΠ½Π΅Π½ΡΡ Π²Π΅ΠΊΡΠΎΡΠ°, ΠΎΠΏΡΠ΅Π΄Π΅Π»ΡΠ΅ΠΌΠΎΠ³ΠΎ Π΄Π²ΡΠΌΡ ΡΠΎΡΠΊΠ°ΠΌΠΈ
Π’Π΅ΠΏΠ΅ΡΡ ΠΏΠΎΡΠΌΠΎΡΡΠΈΠΌ, ΠΊΠ°ΠΊ ΠΎΠΏΡΠ΅Π΄Π΅Π»ΠΈΡΡ ΠΊΠΎΠΌΠΏΠΎΠ½Π΅Π½ΡΡ Π²Π΅ΠΊΡΠΎΡΠ°, Π΅ΡΠ»ΠΈ ΠΈΠ·Π²Π΅ΡΡΠ½Ρ ΠΊΠΎΠΎΡΠ΄ΠΈΠ½Π°ΡΡ Π΅Π³ΠΎ ΠΊΠΎΠ½ΡΠΎΠ²:
ΠΡΡΡΡ $$P =(p_1,p_2)$$ ΠΈ $$ Q = (q_1,q_2)$$ β Π΄Π²Π΅ ΡΠΎΡΠΊΠΈ ΠΏΠ»ΠΎΡΠΊΠΎΡΡΠΈ, Π° $$\overrightarrow{PQ}$$ β Π²Π΅ΠΊΡΠΎΡ, ΠΈΠ΄ΡΡΠΈΠΉ ΠΈΠ· $$P$$ Π² $$Q$$.
Π’ΠΎΠ³Π΄Π° ΠΊΠΎΠΌΠΏΠΎΠ½Π΅Π½ΡΡ Π²Π΅ΠΊΡΠΎΡΠ° $$\overrightarrow{PQ}$$ ΡΠ°Π²Π½Ρ $$\overrightarrow{PQ}=(q_1-p_1,q_2-p_2)$$.
ΠΠ°Π½ΠΎ $$P = (2, 6)$$ ΠΈ $$Q = (-3, 9)$$. ΠΠΎΠΌΠΏΠΎΠ½Π΅Π½ΡΡ Π²Π΅ΠΊΡΠΎΡΠ° $$\overrightarrow{PQ}$$ ΡΠ°ΠΊΠΎΠ²Ρ: $$\overrightarrow{PQ}= (-3 — 2, 9- 6) = (-5, 3)$$
ΠΡΠΈΠΌΠ΅Π½Π΅Π½ΠΈΠ΅ Π²Π΅ΠΊΡΠΎΡΠ° ΠΊ ΡΠΎΡΠΊΠ΅
ΠΠ»Ρ Π΄Π°Π½Π½ΠΎΠΉ ΡΠΎΡΠΊΠΈ $$P$$ ΠΈ Π²Π΅ΠΊΡΠΎΡΠ° $$\overrightarrow{v}$$ ΡΠ΅Π·ΡΠ»ΡΡΠ°Ρ ΠΏΡΠΈΠΌΠ΅Π½Π΅Π½ΠΈΡ Π²Π΅ΠΊΡΠΎΡΠ° ΠΊ ΡΠΎΡΠΊΠ° β ΡΡΠΎ Π½ΠΎΠ²Π°Ρ ΡΠΎΡΠΊΠ° $$Q$$, ΡΠ°ΡΠΏΠΎΠ»ΠΎΠΆΠ΅Π½Π½Π°Ρ Π² Π½Π°ΠΏΡΠ°Π²Π»Π΅Π½ΠΈΠΈ $$\overrightarrow{v}$$ ΠΈ Π½Π° ΡΠ°ΡΡΡΠΎΡΠ½ΠΈΠΈ $$|\overrightarrow{v}|$$. (ΠΌΠΎΠ΄ΡΠ»Ρ Π²Π΅ΠΊΡΠΎΡΠ° $$\overrightarrow{v}$$)
ΠΠΎΠΎΡΠ΄ΠΈΠ½Π°ΡΡ ΡΡΠΎΠΉ Π½ΠΎΠ²ΠΎΠΉ ΡΠΎΡΠΊΠΈ $$Q$$ Π²ΡΡΠΈΡΠ»ΡΡΡΡΡ ΠΈΠ· ΠΊΠΎΠΎΡΠ΄ΠΈΠ½Π°Ρ $$P =(p_1,p_2)$$ ΠΈ $$\overrightarrow{ v}=(v_1,v_2)$$ ΡΠ°ΠΊΠΈΠΌ ΠΎΠ±ΡΠ°Π·ΠΎΠΌ $$$Q = P +\overrightarrow{v}=(p_1+v_1,p_2+v_2)$$$
ΠΠ ΠΠΠΠ§ΠΠΠΠ. ΠΡΠ΅Π½Ρ Π²Π°ΠΆΠ½ΠΎ ΠΏΠΎΠΌΠ½ΠΈΡΡ, ΡΡΠΎ ΡΡΠ° ΠΎΠΏΠ΅ΡΠ°ΡΠΈΡ ΡΠ»ΠΎΠΆΠ΅Π½ΠΈΡ ΠΈΠΌΠ΅Π΅Ρ ΡΠΌΡΡΠ» ΡΠΎΠ»ΡΠΊΠΎ ΠΌΠ΅ΠΆΠ΄Ρ ΡΠΎΡΠΊΠΎΠΉ ΠΈ Π²Π΅ΠΊΡΠΎΡΠΎΠΌ. ΠΡ Π½ΠΈΠΊΠΎΠ³Π΄Π° Π½Π΅ Π΄ΠΎΠ»ΠΆΠ½Ρ ΡΠΊΠ»Π°Π΄ΡΠ²Π°ΡΡ Π΄Π²Π΅ ΡΠΎΡΠΊΠΈ, Π° ΡΠ΅Π·ΡΠ»ΡΡΠ°ΡΠΎΠΌ ΡΠ»ΠΎΠΆΠ΅Π½ΠΈΡ Π΄Π²ΡΡ Π²Π΅ΠΊΡΠΎΡΠΎΠ² Π±ΡΠ΄Π΅Ρ Π΄ΡΡΠ³ΠΎΠΉ Π²Π΅ΠΊΡΠΎΡ, Π° Π½Π΅ ΡΠΎΡΠΊΠ°!
Π Π°ΡΡΠΌΠΎΡΡΠ΅Π² ΡΠ»Π΅Π΄ΡΡΡΠΈΠΉ ΡΠΈΡΡΠ½ΠΎΠΊ, ΠΎΠΏΡΠ΅Π΄Π΅Π»ΠΈΡΠ΅ ΠΊΠΎΠΎΡΠ΄ΠΈΠ½Π°ΡΡ ΡΠΎΡΠΊΠΈ $$P$$ ΡΠΈΡΡΠ½ΠΊΠ°, ΠΏΠΎΠ»ΡΡΠ΅Π½Π½ΠΎΠ³ΠΎ ΠΏΡΠΈΠ»ΠΎΠΆΠ΅Π½ΠΈΠ΅ΠΌ Π²Π΅ΠΊΡΠΎΡΠ° $$\overrightarrow{v}$$ ΠΊ ΡΠΎΡΠΊΠ΅ $$A$$.
ΠΠ°ΡΠ½Π΅ΠΌ Ρ Π²ΡΡΠΈΡΠ»Π΅Π½ΠΈΡ ΠΊΠΎΠΌΠΏΠΎΠ½Π΅Π½Ρ Π²Π΅ΠΊΡΠΎΡΠ° $$\overrightarrow{v}$$:$$$\overrightarrow{v} = (2 — (-1), 4-2) = (3, 2) $$$ ΠΠΎΡΠΊΠΎΠ»ΡΠΊΡ $$P$$ ΡΠ²Π»ΡΠ΅ΡΡΡ ΡΠ΅Π·ΡΠ»ΡΡΠ°ΡΠΎΠΌ ΠΏΡΠΈΠΌΠ΅Π½Π΅Π½ΠΈΡ Π²Π΅ΠΊΡΠΎΡΠ° $$\overrightarrow{v}$$ ΠΊ ΡΠΎΡΠΊΠ΅ $$A$$, ΠΈΠΌΠ΅Π΅ΠΌ $$$P=A+\overrightarrow{v}=(0,4) +(3,2)=(3,6)$$$
Π‘Π΅ΡΠ΅Π΄ΠΈΠ½Π° ΠΎΡΡΠ΅Π·ΠΊΠ°
Π Π°ΡΡΠΌΠΎΡΡΠΈΠΌ ΡΠ΅ΠΏΠ΅ΡΡ ΠΎΡΡΠ΅Π·ΠΎΠΊ Ρ ΠΊΠΎΠ½ΡΠ°ΠΌΠΈ $$A = (a_1,a_2)$$ ΠΈ $$B = (b_1,b_2)$$. ΠΡΡΡΡ $$M =(m_1,m_2)$$ β ΡΠ΅ΡΠ΅Π΄ΠΈΠ½Π° ΡΠΊΠ°Π·Π°Π½Π½ΠΎΠ³ΠΎ Π²ΡΡΠ΅ ΠΎΡΡΠ΅Π·ΠΊΠ°. ΠΡΠ΅Π²ΠΈΠ΄Π½ΠΎ, ΡΡΠΎ ΡΠΊΠ°Π·Π°Π½Π½Π°Ρ Π²ΡΡΠ΅ ΡΠΎΡΠΊΠ° ΡΠ΄ΠΎΠ²Π»Π΅ΡΠ²ΠΎΡΡΠ΅Ρ ΡΠΎΠΌΡ, ΡΡΠΎ $$\overrightarrow{AB}=2\cdot \overrightarrow{AM}$$, ΠΈΠ»ΠΈ ΡΡΠΎ $$(b_1-a_1,b_2-a_2)=2\cdot (m_1-a_1,m_2- a_2)$$
Π Π°Π·Π΄Π΅Π»ΡΡ ΠΏΠΎΠΊΠΎΠΌΠΏΠΎΠ½Π΅Π½ΡΠ½ΠΎ ΠΏΠΎΠ»ΡΡΠ°Π΅ΠΌ: $$$\begin{ΠΌΠ°ΡΡΠΈΠ²}{rcl} b_1-a_1 & = & 2 \cdot (m_1-a_1) \\ b_2-a_2 &=& 2\cdot (m_2-a_2) \end{ΠΌΠ°ΡΡΠΈΠ²}$$$ ΠΈ ΠΈΠ·ΠΎΠ»ΠΈΡΡΡ ΠΈΠΌΠ΅Π΅ΠΌ: $$$\begin{array}{rcl} m_1 & = & \displaystyle \frac{a_1+b_1}{2}\\ m_2 &=& \displaystyle \frac{a_2+b_2}{2} \end{array} $$$ Π’Π°ΠΊ ΡΡΠΎ ΠΌΡ ΠΌΠΎΠΆΠ΅ΠΌ Π²ΡΡΠΈΡΠ»ΠΈΡΡ ΠΊΠΎΠΎΡΠ΄ΠΈΠ½Π°ΡΡ ΡΠ΅ΡΠ΅Π΄ΠΈΠ½Ρ ΠΎΡΡΠ΅Π·ΠΊΠ° ΠΏΠΎ ΠΊΠΎΠΎΡΠ΄ΠΈΠ½Π°ΡΠ°ΠΌ Π΅Π³ΠΎ ΠΊΠΎΠ½ΡΠΎΠ².
ΠΠΎ ΡΠΎΡΠΊΠ°ΠΌ $$A = (-3, 7)$$ ΠΈ $$B = (1, 2)$$ Π½Π°ΠΉΠ΄ΠΈΡΠ΅ ΡΠ΅ΡΠ΅Π΄ΠΈΠ½Ρ ΠΎΡΡΠ΅Π·ΠΊΠ°, ΠΊΠΎΡΠΎΡΡΠΉ ΠΎΠ½ΠΈ ΠΎΠΏΡΠ΅Π΄Π΅Π»ΡΡΡ.
ΠΡΠΈΠΌΠ΅Π½ΡΡ ΠΏΡΠ΅Π΄ΡΠ΄ΡΡΠΈΠ΅ ΡΠΎΡΠΌΡΠ»Ρ, ΠΈΠΌΠ΅Π΅ΠΌ: $$$\begin{array}{rcl} m_1 & = & \displaystyle \frac{a_1+b_1}{2}= \frac{-3+2}{2}=-1\\ m_2 &=& \displaystyle \frac{a_2+b_2}{2}=\frac{7+2}{2}=\frac{9}{2} \end{array}$$$ Π‘Π»Π΅Π΄ΠΎΠ²Π°ΡΠ΅Π»ΡΠ½ΠΎ, ΡΠ΅ΡΠ΅Π΄ΠΈΠ½Π° ΠΎΡΡΠ΅Π·ΠΊΠ° $$AB$$ ΡΠ°Π²Π½Π° $$M = (-1, \displaystyle \frac{9}{2})$$
2.2 Π‘ΠΈΡΡΠ΅ΠΌΡ ΠΊΠΎΠΎΡΠ΄ΠΈΠ½Π°Ρ ΠΈ ΠΊΠΎΠΌΠΏΠΎΠ½Π΅Π½ΡΡ Π²Π΅ΠΊΡΠΎΡΠ° — University Physics Volume 1
Π¦Π΅Π»ΠΈ ΠΎΠ±ΡΡΠ΅Π½ΠΈΡ
Π ΠΊΠΎΠ½ΡΡ ΡΡΠΎΠ³ΠΎ ΡΠ°Π·Π΄Π΅Π»Π° Π²Ρ ΡΠΌΠΎΠΆΠ΅ΡΠ΅:
- ΠΠΏΠΈΡΡΠ²Π°ΡΡ Π²Π΅ΠΊΡΠΎΡΠ° Π² Π΄Π²ΡΡ ΠΈ ΡΡΠ΅Ρ ΠΈΠ·ΠΌΠ΅ΡΠ΅Π½ΠΈΡΡ Ρ ΡΠΎΡΠΊΠΈ Π·ΡΠ΅Π½ΠΈΡ ΠΈΡ ΠΊΠΎΠΌΠΏΠΎΠ½Π΅Π½ΡΠΎΠ², ΠΈΡΠΏΠΎΠ»ΡΠ·ΡΡ Π΅Π΄ΠΈΠ½ΠΈΡΠ½ΡΠ΅ Π²Π΅ΠΊΡΠΎΡΡ Π²Π΄ΠΎΠ»Ρ ΠΎΡΠ΅ΠΉ.
- Π Π°Π·Π»ΠΈΡΠ°ΡΡ Π²Π΅ΠΊΡΠΎΡΠ½ΡΠ΅ ΠΊΠΎΠΌΠΏΠΎΠ½Π΅Π½ΡΡ Π²Π΅ΠΊΡΠΎΡΠ° ΠΈ ΡΠΊΠ°Π»ΡΡΠ½ΡΠ΅ ΠΊΠΎΠΌΠΏΠΎΠ½Π΅Π½ΡΡ Π²Π΅ΠΊΡΠΎΡΠ°.
- ΠΠ±ΡΡΡΠ½ΠΈΡΠ΅, ΠΊΠ°ΠΊ Π²Π΅Π»ΠΈΡΠΈΠ½Π° Π²Π΅ΠΊΡΠΎΡΠ° ΠΎΠΏΡΠ΅Π΄Π΅Π»ΡΠ΅ΡΡΡ ΡΠ΅ΡΠ΅Π· ΠΊΠΎΠΌΠΏΠΎΠ½Π΅Π½ΡΡ Π²Π΅ΠΊΡΠΎΡΠ°.
- ΠΠΏΡΠ΅Π΄Π΅Π»ΠΈΡΠ΅ ΡΠ³ΠΎΠ» Π½Π°ΠΏΡΠ°Π²Π»Π΅Π½ΠΈΡ Π²Π΅ΠΊΡΠΎΡΠ° Π½Π° ΠΏΠ»ΠΎΡΠΊΠΎΡΡΠΈ.
- ΠΠ±ΡΡΡΠ½ΠΈΡΠ΅ ΡΠ²ΡΠ·Ρ ΠΌΠ΅ΠΆΠ΄Ρ ΠΏΠΎΠ»ΡΡΠ½ΡΠΌΠΈ ΠΊΠΎΠΎΡΠ΄ΠΈΠ½Π°ΡΠ°ΠΌΠΈ ΠΈ Π΄Π΅ΠΊΠ°ΡΡΠΎΠ²ΡΠΌΠΈ ΠΊΠΎΠΎΡΠ΄ΠΈΠ½Π°ΡΠ°ΠΌΠΈ Π½Π° ΠΏΠ»ΠΎΡΠΊΠΎΡΡΠΈ.
ΠΠ΅ΠΊΡΠΎΡΡ ΠΎΠ±ΡΡΠ½ΠΎ ΠΎΠΏΠΈΡΡΠ²Π°ΡΡΡΡ ΠΈΡ
ΠΊΠΎΠΌΠΏΠΎΠ½Π΅Π½ΡΠ°ΠΌΠΈ Π² ΡΠΈΡΡΠ΅ΠΌΠ΅ ΠΊΠΎΠΎΡΠ΄ΠΈΠ½Π°Ρ.
ΠΠ°ΠΆΠ΅ Π² ΠΏΠΎΠ²ΡΠ΅Π΄Π½Π΅Π²Π½ΠΎΠΉ ΠΆΠΈΠ·Π½ΠΈ ΠΌΡ Π΅ΡΡΠ΅ΡΡΠ²Π΅Π½Π½ΡΠΌ ΠΎΠ±ΡΠ°Π·ΠΎΠΌ ΠΏΠΎΠ»ΡΠ·ΡΠ΅ΠΌΡΡ ΠΏΠΎΠ½ΡΡΠΈΠ΅ΠΌ ΠΎΡΡΠΎΠ³ΠΎΠ½Π°Π»ΡΠ½ΡΡ
ΠΏΡΠΎΠ΅ΠΊΡΠΈΠΉ Π² ΠΏΡΡΠΌΠΎΡΠ³ΠΎΠ»ΡΠ½ΠΎΠΉ ΡΠΈΡΡΠ΅ΠΌΠ΅ ΠΊΠΎΠΎΡΠ΄ΠΈΠ½Π°Ρ. ΠΠ°ΠΏΡΠΈΠΌΠ΅Ρ, Π΅ΡΠ»ΠΈ Π²Ρ ΡΠΏΡΠΎΡΠΈΡΠ΅ Ρ ΠΊΠΎΠ³ΠΎ-Π½ΠΈΠ±ΡΠ΄Ρ Π½Π°ΠΏΡΠ°Π²Π»Π΅Π½ΠΈΠ΅ ΠΊ ΠΎΠΏΡΠ΅Π΄Π΅Π»Π΅Π½Π½ΠΎΠΌΡ ΠΌΠ΅ΡΡΡ, Π²Π°ΠΌ, ΡΠΊΠΎΡΠ΅Π΅ Π²ΡΠ΅Π³ΠΎ, ΡΠΊΠ°ΠΆΡΡ ΠΏΡΠΎΠΉΡΠΈ 40 ΠΊΠΌ Π½Π° Π²ΠΎΡΡΠΎΠΊ ΠΈ 30 ΠΊΠΌ Π½Π° ΡΠ΅Π²Π΅Ρ, ΡΠ΅ΠΌ 50 ΠΊΠΌ Π² Π½Π°ΠΏΡΠ°Π²Π»Π΅Π½ΠΈΠΈ 37Β°37Β° ΠΊ ΡΠ΅Π²Π΅ΡΡ ΠΎΡ Π²ΠΎΡΡΠΎΠΊΠ°.
Π ΠΏΡΡΠΌΠΎΡΠ³ΠΎΠ»ΡΠ½ΠΎΠΉ (Π΄Π΅ΠΊΠ°ΡΡΠΎΠ²ΠΎΠΉ) xy -ΡΠΈΡΡΠ΅ΠΌΠ΅ ΠΊΠΎΠΎΡΠ΄ΠΈΠ½Π°Ρ Π½Π° ΠΏΠ»ΠΎΡΠΊΠΎΡΡΠΈ ΡΠΎΡΠΊΠ° Π½Π° ΠΏΠ»ΠΎΡΠΊΠΎΡΡΠΈ ΠΎΠΏΠΈΡΡΠ²Π°Π΅ΡΡΡ ΠΏΠ°ΡΠΎΠΉ ΠΊΠΎΠΎΡΠ΄ΠΈΠ½Π°Ρ ( x , y ). ΠΠ½Π°Π»ΠΎΠ³ΠΈΡΠ½ΡΠΌ ΠΎΠ±ΡΠ°Π·ΠΎΠΌ Π²Π΅ΠΊΡΠΎΡ AβAβ Π½Π° ΠΏΠ»ΠΎΡΠΊΠΎΡΡΠΈ ΠΎΠΏΠΈΡΡΠ²Π°Π΅ΡΡΡ ΠΏΠ°ΡΠΎΠΉ Π΅Π³ΠΎ Π²Π΅ΠΊΡΠΎΡΠ½ΡΡ
ΠΊΠΎΠΎΡΠ΄ΠΈΠ½Π°Ρ. ΠΠΎΠΎΡΠ΄ΠΈΠ½Π°ΡΠ° x Π²Π΅ΠΊΡΠΎΡΠ° AβAβ Π½Π°Π·ΡΠ²Π°Π΅ΡΡΡ Π΅Π³ΠΎ x -ΠΊΠΎΠΌΠΏΠΎΠ½Π΅Π½ΡΠΎΠΉ, Π° y -ΠΊΠΎΠΎΡΠ΄ΠΈΠ½Π°ΡΠ° Π²Π΅ΠΊΡΠΎΡΠ° AβAβ Π½Π°Π·ΡΠ²Π°Π΅ΡΡΡ Π΅Π³ΠΎ y -ΠΊΠΎΠΌΠΏΠΎΠ½Π΅Π½ΡΠΎΠΉ. ΠΠΎΠΌΠΏΠΎΠ½Π΅Π½ΡΠ° Π²Π΅ΠΊΡΠΎΡΠ° x ΠΏΡΠ΅Π΄ΡΡΠ°Π²Π»ΡΠ΅Ρ ΡΠΎΠ±ΠΎΠΉ Π²Π΅ΠΊΡΠΎΡ, ΠΎΠ±ΠΎΠ·Π½Π°ΡΠ°Π΅ΠΌΡΠΉ AβxAβx.
ΠΠ΅ΠΊΡΠΎΡ y -ΠΊΠΎΠΌΠΏΠΎΠ½Π΅Π½ΡΠ° ΠΏΡΠ΅Π΄ΡΡΠ°Π²Π»ΡΠ΅Ρ ΡΠΎΠ±ΠΎΠΉ Π²Π΅ΠΊΡΠΎΡ, ΠΎΠ±ΠΎΠ·Π½Π°ΡΠ°Π΅ΠΌΡΠΉ AβyAβy. Π Π΄Π΅ΠΊΠ°ΡΡΠΎΠ²ΠΎΠΉ ΡΠΈΡΡΠ΅ΠΌΠ΅ ΠΊΠΎΠΌΠΏΠΎΠ½Π΅Π½ΡΡ Π²Π΅ΠΊΡΠΎΡΠ° x ΠΈ y ΡΠ²Π»ΡΡΡΡΡ ΠΎΡΡΠΎΠ³ΠΎΠ½Π°Π»ΡΠ½ΡΠΌΠΈ ΠΏΡΠΎΠ΅ΠΊΡΠΈΡΠΌΠΈ ΡΡΠΎΠ³ΠΎ Π²Π΅ΠΊΡΠΎΡΠ° Π½Π° ΠΎΡΠΈ x ΠΈ y ΡΠΎΠΎΡΠ²Π΅ΡΡΡΠ²Π΅Π½Π½ΠΎ. Π’Π°ΠΊΠΈΠΌ ΠΎΠ±ΡΠ°Π·ΠΎΠΌ, ΡΠ»Π΅Π΄ΡΡ ΠΏΡΠ°Π²ΠΈΠ»Ρ ΠΏΠ°ΡΠ°Π»Π»Π΅Π»ΠΎΠ³ΡΠ°ΠΌΠΌΠ° Π΄Π»Ρ ΡΠ»ΠΎΠΆΠ΅Π½ΠΈΡ Π²Π΅ΠΊΡΠΎΡΠΎΠ², ΠΊΠ°ΠΆΠ΄ΡΠΉ Π²Π΅ΠΊΡΠΎΡ Π½Π° Π΄Π΅ΠΊΠ°ΡΡΠΎΠ²ΠΎΠΉ ΠΏΠ»ΠΎΡΠΊΠΎΡΡΠΈ ΠΌΠΎΠΆΠ΅Ρ Π±ΡΡΡ Π²ΡΡΠ°ΠΆΠ΅Π½ ΠΊΠ°ΠΊ Π²Π΅ΠΊΡΠΎΡΠ½Π°Ρ ΡΡΠΌΠΌΠ° Π΅Π³ΠΎ Π²Π΅ΠΊΡΠΎΡΠ½ΡΡ
ΠΊΠΎΠΌΠΏΠΎΠ½Π΅Π½ΡΠΎΠ²:
Aβ=Aβx+Aβy.Aβ=Aβx+Aβ Ρ.
2.10
ΠΠ°ΠΊ ΠΏΠΎΠΊΠ°Π·Π°Π½ΠΎ Π½Π° ΡΠΈΡ. 2.16, Π²Π΅ΠΊΡΠΎΡ AβAβ ΡΠ²Π»ΡΠ΅ΡΡΡ Π΄ΠΈΠ°Π³ΠΎΠ½Π°Π»ΡΡ ΠΏΡΡΠΌΠΎΡΠ³ΠΎΠ»ΡΠ½ΠΈΠΊΠ°, Π³Π΄Π΅ x -ΠΊΠΎΠΌΠΏΠΎΠ½Π΅Π½ΡΠ° AβxAβx ΡΠ²Π»ΡΠ΅ΡΡΡ ΡΡΠΎΡΠΎΠ½ΠΎΠΉ, ΠΏΠ°ΡΠ°Π»Π»Π΅Π»ΡΠ½ΠΎΠΉ ΠΎΡΠΈ x , Π° y -ΠΊΠΎΠΌΠΏΠΎΠ½Π΅Π½ΡΠ° AβyAβy ΡΠ²Π»ΡΠ΅ΡΡΡ ΡΡΠΎΡΠΎΠ½ΠΎΠΉ, ΠΏΠ°ΡΠ°Π»Π»Π΅Π»ΡΠ½ΠΎΠΉ ΠΎΡΠΈ y . ΠΠΎΠΌΠΏΠΎΠ½Π΅Π½ΡΠ° Π²Π΅ΠΊΡΠΎΡΠ° AβxAβx ΠΎΡΡΠΎΠ³ΠΎΠ½Π°Π»ΡΠ½Π° ΠΊΠΎΠΌΠΏΠΎΠ½Π΅Π½ΡΠ΅ Π²Π΅ΠΊΡΠΎΡΠ° AβyAβy.
Π ΠΈΡΡΠ½ΠΎΠΊ 2.16 ΠΠ΅ΠΊΡΠΎΡ AβAβ Π½Π° ΠΏΠ»ΠΎΡΠΊΠΎΡΡΠΈ Π² Π΄Π΅ΠΊΠ°ΡΡΠΎΠ²ΠΎΠΉ ΡΠΈΡΡΠ΅ΠΌΠ΅ ΠΊΠΎΠΎΡΠ΄ΠΈΠ½Π°Ρ ΠΏΡΠ΅Π΄ΡΡΠ°Π²Π»ΡΠ΅Ρ ΡΠΎΠ±ΠΎΠΉ Π²Π΅ΠΊΡΠΎΡΠ½ΡΡ ΡΡΠΌΠΌΡ Π΅Π³ΠΎ Π²Π΅ΠΊΡΠΎΡΠ½ΡΡ x — ΠΈ y -ΠΊΠΎΠΌΠΏΠΎΠ½Π΅Π½Ρ. ΠΠΎΠΌΠΏΠΎΠ½Π΅Π½ΡΠ° Π²Π΅ΠΊΡΠΎΡΠ° AβxAβx ΡΠ°Π·ΠΌΠ΅ΡΠΎΠΌ x ΡΠ²Π»ΡΠ΅ΡΡΡ ΠΎΡΡΠΎΠ³ΠΎΠ½Π°Π»ΡΠ½ΠΎΠΉ ΠΏΡΠΎΠ΅ΠΊΡΠΈΠ΅ΠΉ Π²Π΅ΠΊΡΠΎΡΠ° AβAβ Π½Π° 9.
2.12
ΠΡΠ»ΠΈ ΠΌΡ Π·Π½Π°Π΅ΠΌ ΠΊΠΎΠΎΡΠ΄ΠΈΠ½Π°ΡΡ b(xb,yb)b(xb,yb) Π½Π°ΡΠ°Π»ΡΠ½ΠΎΠΉ ΡΠΎΡΠΊΠΈ Π²Π΅ΠΊΡΠΎΡΠ° (Π³Π΄Π΅ b ΠΎΠ·Π½Π°ΡΠ°Π΅Ρ Β«Π½Π°ΡΠ°Π»ΠΎΒ») ΠΈ ΠΊΠΎΠΎΡΠ΄ΠΈΠ½Π°ΡΡ e(xe,ye)e(xe,ye ) ΠΊΠΎΠ½Π΅ΡΠ½ΠΎΠΉ ΡΠΎΡΠΊΠΈ Π²Π΅ΠΊΡΠΎΡΠ° (Π³Π΄Π΅ e ΠΎΠ·Π½Π°ΡΠ°Π΅Ρ Β«ΠΊΠΎΠ½Π΅ΡΒ»), ΠΌΡ ΠΌΠΎΠΆΠ΅ΠΌ ΠΏΠΎΠ»ΡΡΠΈΡΡ ΡΠΊΠ°Π»ΡΡΠ½ΡΠ΅ ΠΊΠΎΠΌΠΏΠΎΠ½Π΅Π½ΡΡ Π²Π΅ΠΊΡΠΎΡΠ°, ΠΏΡΠΎΡΡΠΎ Π²ΡΡΠΈΡΠ°Ρ ΠΊΠΎΠΎΡΠ΄ΠΈΠ½Π°ΡΡ ΠΈΡΡ ΠΎΠ΄Π½ΠΎΠΉ ΡΠΎΡΠΊΠΈ ΠΈΠ· ΠΊΠΎΠΎΡΠ΄ΠΈΠ½Π°Ρ ΠΊΠΎΠ½Π΅ΡΠ½ΠΎΠΉ ΡΠΎΡΠΊΠΈ:
{Ax=xeβxbAy =ye-yb.{Ax=xe-xbAy=ye-yb.
2.13
ΠΡΠΈΠΌΠ΅Ρ 2.3 9Π½Π°
y — ΠΎΡΡ ΡΠΊΠ°Π·ΡΠ²Π°Π΅Ρ Π²Π΅ΡΡΠΈΠΊΠ°Π»ΡΠ½ΠΎ Π²Π²Π΅ΡΡ . ΠΠ°ΡΠ°Π»ΠΎ Π²Π΅ΠΊΡΠΎΡΠ° ΡΠΌΠ΅ΡΠ΅Π½ΠΈΡ ΡΠ°ΡΠΏΠΎΠ»ΠΎΠΆΠ΅Π½ΠΎ Π² ΡΠΎΡΠΊΠ΅ b(6.0, 1.6), Π° ΠΊΠΎΠ½Π΅Ρ Π²Π΅ΠΊΡΠΎΡΠ° ΡΠΌΠ΅ΡΠ΅Π½ΠΈΡ ΡΠ°ΡΠΏΠΎΠ»ΠΎΠΆΠ΅Π½ Π² ΡΠΎΡΠΊΠ΅ e(2.
0, 4.5). ΠΠΎΠ΄ΡΡΠ°Π²ΡΡΠ΅ ΠΊΠΎΠΎΡΠ΄ΠΈΠ½Π°ΡΡ ΡΡΠΈΡ
ΡΠΎΡΠ΅ΠΊ Π² ΡΡΠ°Π²Π½Π΅Π½ΠΈΠ΅ 2.13, ΡΡΠΎΠ±Ρ Π½Π°ΠΉΡΠΈ ΡΠΊΠ°Π»ΡΡΠ½ΡΠ΅ ΠΊΠΎΠΌΠΏΠΎΠ½Π΅Π½ΡΡ DxDx ΠΈ DyDy Π²Π΅ΠΊΡΠΎΡΠ° ΡΠΌΠ΅ΡΠ΅Π½ΠΈΡ DβDβ. ΠΠ°ΠΊΠΎΠ½Π΅Ρ, ΠΏΠΎΠ΄ΡΡΠ°Π²ΡΡΠ΅ ΠΊΠΎΠΎΡΠ΄ΠΈΠ½Π°ΡΡ Π² ΡΡΠ°Π²Π½Π΅Π½ΠΈΠ΅ 2.12, ΡΡΠΎΠ±Ρ Π·Π°ΠΏΠΈΡΠ°ΡΡ Π²Π΅ΠΊΡΠΎΡ ΡΠΌΠ΅ΡΠ΅Π½ΠΈΡ Π² ΡΠΎΡΠΌΠ΅ ΠΊΠΎΠΌΠΏΠΎΠ½Π΅Π½ΡΠ° Π²Π΅ΠΊΡΠΎΡΠ°.Π Π°ΡΡΠ²ΠΎΡ
ΠΡ ΠΎΠΏΡΠ΅Π΄Π΅Π»ΡΠ΅ΠΌ xb=6,0xb=6,0, xe=2,0xe=2,0, yb=1,6yb=1,6 ΠΈ ye=4,5ye=4,5, Π³Π΄Π΅ ΡΠΈΠ·ΠΈΡΠ΅ΡΠΊΠΎΠΉ Π΅Π΄ΠΈΠ½ΠΈΡΠ΅ΠΉ ΡΠ²Π»ΡΠ΅ΡΡΡ 1 ΡΠΌ. Π‘ΠΊΠ°Π»ΡΡ 9)ΡΠΌ.
2.14
ΠΡΠΎ ΡΠ΅ΡΠ΅Π½ΠΈΠ΅ ΠΏΠΎΠΊΠ°Π·Π°Π½ΠΎ Π½Π° ΡΠΈΡ. 2.17.
Π ΠΈΡΡΠ½ΠΎΠΊ 2.17 ΠΡΠ°ΡΠΈΠΊ Π²Π΅ΠΊΡΠΎΡΠ° ΡΠΌΠ΅ΡΠ΅Π½ΠΈΡ. ΠΠ΅ΠΊΡΠΎΡ ΡΠΊΠ°Π·ΡΠ²Π°Π΅Ρ ΠΎΡ Π½Π°ΡΠ°Π»ΡΠ½ΠΎΠΉ ΡΠΎΡΠΊΠΈ b Π΄ΠΎ ΠΊΠΎΠ½Π΅ΡΠ½ΠΎΠΉ ΡΠΎΡΠΊΠΈ e .
ΠΠ½Π°ΡΠ΅Π½ΠΈΠ΅
ΠΠ±ΡΠ°ΡΠΈΡΠ΅ Π²Π½ΠΈΠΌΠ°Π½ΠΈΠ΅, ΡΡΠΎ ΡΠΈΠ·ΠΈΡΠ΅ΡΠΊΠ°Ρ Π΅Π΄ΠΈΠ½ΠΈΡΠ° β Π·Π΄Π΅ΡΡ 1 ΡΠΌ β ΠΌΠΎΠΆΠ΅Ρ Π±ΡΡΡ ΠΏΠΎΠΌΠ΅ΡΠ΅Π½Π° Π»ΠΈΠ±ΠΎ Ρ ΠΊΠ°ΠΆΠ΄ΡΠΌ ΠΊΠΎΠΌΠΏΠΎΠ½Π΅Π½ΡΠΎΠΌ Π½Π΅ΠΏΠΎΡΡΠ΅Π΄ΡΡΠ²Π΅Π½Π½ΠΎ ΠΏΠ΅ΡΠ΅Π΄ Π΅Π΄ΠΈΠ½ΠΈΡΠ½ΡΠΌ Π²Π΅ΠΊΡΠΎΡΠΎΠΌ, Π»ΠΈΠ±ΠΎ Π³Π»ΠΎΠ±Π°Π»ΡΠ½ΠΎ Π΄Π»Ρ ΠΎΠ±ΠΎΠΈΡ
ΠΊΠΎΠΌΠΏΠΎΠ½Π΅Π½ΡΠΎΠ², ΠΊΠ°ΠΊ Π² ΡΡΠ°Π²Π½Π΅Π½ΠΈΠΈ 2.14. Π§Π°ΡΡΠΎ ΠΏΠΎΡΠ»Π΅Π΄Π½ΠΈΠΉ ΡΠΏΠΎΡΠΎΠ± ΡΠ΄ΠΎΠ±Π½Π΅Π΅, ΠΏΠΎΡΠΎΠΌΡ ΡΡΠΎ ΠΎΠ½ ΠΏΡΠΎΡΠ΅.
9, ΠΊΠΎΡΠΎΡΡΠΉ ΠΏΠ°ΡΠ°Π»Π»Π΅Π»Π΅Π½ Π½Π°ΠΏΡΠ°Π²Π»Π΅Π½ΠΈΡ ΠΎΡΠΈ + y -. Π‘Π»Π΅Π΄ΠΎΠ²Π°ΡΠ΅Π»ΡΠ½ΠΎ, y -ΠΊΠΎΠΌΠΏΠΎΠ½Π΅Π½ΡΠ½ΡΠΉ Π²Π΅ΠΊΡΠΎΡ DβyDβy ΡΠΊΠ°Π·ΡΠ²Π°Π΅Ρ Π²Π²Π΅ΡΡ
, ΠΊΠ°ΠΊ ΠΏΠΎΠΊΠ°Π·Π°Π½ΠΎ Π½Π° ΡΠΈΡ. 2.17. Π‘ΠΊΠ°Π»ΡΡ y -ΠΊΠΎΠΌΠΏΠΎΠ½Π΅Π½ΡΠ° Π²Π΅ΠΊΡΠΎΡΠ° DβDβ ΡΠ°Π²Π΅Π½ Dy=+2,9Dy=+2,9. ΠΠ΅ΠΊΡΠΎΡ ΡΠΌΠ΅ΡΠ΅Π½ΠΈΡ DβDβ ΡΠ²Π»ΡΠ΅ΡΡΡ ΡΠ΅Π·ΡΠ»ΡΡΠ°ΡΠΎΠΌ Π΄Π²ΡΡ
Π΅Π³ΠΎ ΠΊΠΎΠΌΠΏΠΎΠ½Π΅Π½Ρ Π²Π΅ΠΊΡΠΎΡΠ° .
Π€ΠΎΡΠΌΠ° Π²Π΅ΠΊΡΠΎΡΠ½ΠΎΠΉ ΡΠΎΡΡΠ°Π²Π»ΡΡΡΠ΅ΠΉ Π²Π΅ΠΊΡΠΎΡΠ° ΡΠΌΠ΅ΡΠ΅Π½ΠΈΡ Π£ΡΠ°Π²Π½Π΅Π½ΠΈΠ΅ 2.14 Π³ΠΎΠ²ΠΎΡΠΈΡ Π½Π°ΠΌ, ΡΡΠΎ ΡΠΊΠ°Π·Π°ΡΠ΅Π»Ρ ΠΌΡΡΠΈ ΠΏΠ΅ΡΠ΅ΠΌΠ΅ΡΡΠΈΠ»ΡΡ Π½Π° ΠΌΠΎΠ½ΠΈΡΠΎΡΠ΅ Π½Π° 4,0 ΡΠΌ Π²Π»Π΅Π²ΠΎ ΠΈ Π½Π° 2,9 ΡΠΌ Π²Π²Π΅ΡΡ ΠΎΡ ΡΠ²ΠΎΠ΅Π³ΠΎ Π½Π°ΡΠ°Π»ΡΠ½ΠΎΠ³ΠΎ ΠΏΠΎΠ»ΠΎΠΆΠ΅Π½ΠΈΡ.
ΠΡΠΎΠ²Π΅ΡΡΡΠ΅ ΡΠ²ΠΎΠ΅ ΠΏΠΎΠ½ΠΈΠΌΠ°Π½ΠΈΠ΅ 2,4
Π‘ΠΈΠ½ΡΡ ΠΌΡΡ
Π° ΠΏΡΠΈΠ·Π΅ΠΌΠ»ΡΠ΅ΡΡΡ Π½Π° Π»ΠΈΡΡ ΠΌΠΈΠ»Π»ΠΈΠΌΠ΅ΡΡΠΎΠ²ΠΎΠΉ Π±ΡΠΌΠ°Π³ΠΈ Π² ΡΠΎΡΠΊΠ΅, ΡΠ°ΡΠΏΠΎΠ»ΠΎΠΆΠ΅Π½Π½ΠΎΠΉ Π½Π° 10,0 ΡΠΌ ΠΏΡΠ°Π²Π΅Π΅ Π΅Π³ΠΎ Π»Π΅Π²ΠΎΠ³ΠΎ ΠΊΡΠ°Ρ ΠΈ Π½Π° 8,0 ΡΠΌ Π²ΡΡΠ΅ Π΅Π³ΠΎ Π½ΠΈΠΆΠ½Π΅Π³ΠΎ ΠΊΡΠ°Ρ, ΠΈ ΠΌΠ΅Π΄Π»Π΅Π½Π½ΠΎ Π΄Π²ΠΈΠΆΠ΅ΡΡΡ ΠΊ ΡΠΎΡΠΊΠ΅, ΡΠ°ΡΠΏΠΎΠ»ΠΎΠΆΠ΅Π½Π½ΠΎΠΉ Π½Π° ΡΠ°ΡΡΡΠΎΡΠ½ΠΈΠΈ 5,0 ΡΠΌ ΠΎΡ Π»Π΅Π²ΠΎΠ³ΠΎ ΠΊΡΠ°Ρ ΠΈ 5,0 ΡΠΌ ΠΎΡ Π½ΠΈΠΆΠ½Π΅Π³ΠΎ ΠΊΡΠ°Ρ Π½ΠΈΠΆΠ½ΠΈΠΉ ΠΊΡΠ°ΠΉ. ΠΡΠ±Π΅ΡΠΈΡΠ΅ ΠΏΡΡΠΌΠΎΡΠ³ΠΎΠ»ΡΠ½ΡΡ ΡΠΈΡΡΠ΅ΠΌΡ ΠΊΠΎΠΎΡΠ΄ΠΈΠ½Π°Ρ Ρ Π½Π°ΡΠ°Π»ΠΎΠΌ Π² Π»Π΅Π²ΠΎΠΌ Π½ΠΈΠΆΠ½Π΅ΠΌ ΡΠ³Π»Ρ Π»ΠΈΡΡΠ° ΠΈ Π½Π°ΠΉΠ΄ΠΈΡΠ΅ Π²Π΅ΠΊΡΠΎΡ ΡΠΌΠ΅ΡΠ΅Π½ΠΈΡ ΠΌΡΡ
ΠΈ.
ΠΡΠΎΠΈΠ»Π»ΡΡΡΡΠΈΡΡΠΉΡΠ΅ ΡΠ²ΠΎΠ΅ ΡΠ΅ΡΠ΅Π½ΠΈΠ΅ Π³ΡΠ°ΡΠΈΠΊΠΎΠΌ.
ΠΠ½Π°Ρ ΡΠΊΠ°Π»ΡΡΠ½ΡΠ΅ ΠΊΠΎΠΌΠΏΠΎΠ½Π΅Π½ΡΡ AxAx ΠΈ AyAy Π²Π΅ΠΊΡΠΎΡΠ° AβAβ, ΠΌΡ ΠΌΠΎΠΆΠ΅ΠΌ Π½Π°ΠΉΡΠΈ Π΅Π³ΠΎ Π²Π΅Π»ΠΈΡΠΈΠ½Ρ A ΠΈ Π΅Π³ΠΎ Π΄ΠΈΡΠ΅ΠΊΡΠΈΠΎΠ½Π½ΡΠΉ ΡΠ³ΠΎΠ» ΞΈAΞΈA. Π£Π³ΠΎΠ» Π½Π°ΠΏΡΠ°Π²Π»Π΅Π½ΠΈΡ β ΠΈΠ»ΠΈ, Π΄Π»Ρ ΠΊΡΠ°ΡΠΊΠΎΡΡΠΈ, Π½Π°ΠΏΡΠ°Π²Π»Π΅Π½ΠΈΠ΅ β ΡΡΠΎ ΡΠ³ΠΎΠ», ΠΊΠΎΡΠΎΡΡΠΉ Π²Π΅ΠΊΡΠΎΡ ΠΎΠ±ΡΠ°Π·ΡΠ΅Ρ Ρ ΠΏΠΎΠ»ΠΎΠΆΠΈΡΠ΅Π»ΡΠ½ΡΠΌ Π½Π°ΠΏΡΠ°Π²Π»Π΅Π½ΠΈΠ΅ΠΌ Π½Π° ΠΎΡΠΈ x . Π£Π³ΠΎΠ» ΞΈAΞΈA ΠΈΠ·ΠΌΠ΅ΡΡΠ΅ΡΡΡ Π² Π½Π°ΠΏΡΠ°Π²Π»Π΅Π½ΠΈΠΈ ΠΏΡΠΎΡΠΈΠ² ΡΠ°ΡΠΎΠ²ΠΎΠΉ ΡΡΡΠ΅Π»ΠΊΠΈ ΠΎΡ ΠΎΡΠΈ +x ΠΊ Π²Π΅ΠΊΡΠΎΡΡ (ΡΠΈΡΡΠ½ΠΎΠΊ 2.18). ΠΠΎΡΠΊΠΎΠ»ΡΠΊΡ Π΄Π»ΠΈΠ½Ρ A , AxAx ΠΈ AyAy ΠΎΠ±ΡΠ°Π·ΡΡΡ ΠΏΡΡΠΌΠΎΡΠ³ΠΎΠ»ΡΠ½ΡΠΉ ΡΡΠ΅ΡΠ³ΠΎΠ»ΡΠ½ΠΈΠΊ, ΠΎΠ½ΠΈ ΡΠ²ΡΠ·Π°Π½Ρ ΡΠ΅ΠΎΡΠ΅ΠΌΠΎΠΉ ΠΠΈΡΠ°Π³ΠΎΡΠ°:
A2=Ax2+Ay2βA=Ax2+Ay2.A2=Ax2+Ay2βA=Ax2+Ay2.
2,15
ΠΡΠΎ ΡΡΠ°Π²Π½Π΅Π½ΠΈΠ΅ ΡΠ°Π±ΠΎΡΠ°Π΅Ρ, Π΄Π°ΠΆΠ΅ Π΅ΡΠ»ΠΈ ΡΠΊΠ°Π»ΡΡΠ½ΡΠ΅ ΠΊΠΎΠΌΠΏΠΎΠ½Π΅Π½ΡΡ Π²Π΅ΠΊΡΠΎΡΠ° ΠΎΡΡΠΈΡΠ°ΡΠ΅Π»ΡΠ½Ρ. ΠΠ°ΠΏΡΠ°Π²Π»ΡΡΡΠΈΠΉ ΡΠ³ΠΎΠ» ΞΈAΞΈA Π²Π΅ΠΊΡΠΎΡΠ° ΠΎΠΏΡΠ΅Π΄Π΅Π»ΡΠ΅ΡΡΡ ΡΠ΅ΡΠ΅Π· ΡΠ°Π½Π³Π΅Π½Ρ ΡΡΠ½ΠΊΡΠΈΠΈ ΡΠ³Π»Π° ΞΈAΞΈA Π² ΡΡΠ΅ΡΠ³ΠΎΠ»ΡΠ½ΠΈΠΊΠ΅, ΠΏΠΎΠΊΠ°Π·Π°Π½Π½ΠΎΠΌ Π½Π° ΡΠΈΡΡΠ½ΠΊΠ΅ 2.18:
2.18
ΠΠΎΠ³Π΄Π° Π²Π΅ΠΊΡΠΎΡ Π»Π΅ΠΆΠΈΡ Π»ΠΈΠ±ΠΎ Π² ΠΏΠ΅ΡΠ²ΠΎΠΌ ΠΊΠ²Π°Π΄ΡΠ°Π½ΡΠ΅, Π»ΠΈΠ±ΠΎ Π² ΡΠ΅ΡΠ²Π΅ΡΡΠΎΠΌ ΠΊΠ²Π°Π΄ΡΠ°Π½ΡΠ΅, Π³Π΄Π΅ ΡΠΎΡΡΠ°Π²Π»ΡΡΡΠ°Ρ AxAxis ΠΏΠΎΠ»ΠΎΠΆΠΈΡΠ΅Π»ΡΠ½Π° (ΡΠΈΡ.
2.19), ΡΠ³ΠΎΠ» Π½Π°ΠΏΡΠ°Π²Π»Π΅Π½ΠΈΡ ΞΈAΞΈA Π² ΡΡΠ°Π²Π½Π΅Π½ΠΈΠΈ 2.16) ΠΈΠ΄Π΅Π½ΡΠΈΡΠ΅Π½ ΡΠ³Π»Ρ ΞΈΞΈ.
ΠΠΎΠ³Π΄Π° Π²Π΅ΠΊΡΠΎΡ Π»Π΅ΠΆΠΈΡ Π»ΠΈΠ±ΠΎ Π² ΠΏΠ΅ΡΠ²ΠΎΠΌ ΠΊΠ²Π°Π΄ΡΠ°Π½ΡΠ΅, Π»ΠΈΠ±ΠΎ Π² ΡΠ΅ΡΠ²Π΅ΡΡΠΎΠΌ ΠΊΠ²Π°Π΄ΡΠ°Π½ΡΠ΅, Π³Π΄Π΅ ΡΠΎΡΡΠ°Π²Π»ΡΡΡΠ°Ρ AxAx ΠΏΠΎΠ»ΠΎΠΆΠΈΡΠ΅Π»ΡΠ½Π° (ΡΠΈΡ. 2.19), ΡΠ³ΠΎΠ» ΞΈΞΈ Π² ΡΡΠ°Π²Π½Π΅Π½ΠΈΠΈ 2.16 ΠΈΠ΄Π΅Π½ΡΠΈΡΠ΅Π½ ΡΠ³Π»Ρ Π½Π°ΠΏΡΠ°Π²Π»Π΅Π½ΠΈΡ ΞΈAΞΈA. ΠΠ»Ρ Π²Π΅ΠΊΡΠΎΡΠΎΠ² Π² ΡΠ΅ΡΠ²Π΅ΡΡΠΎΠΌ ΠΊΠ²Π°Π΄ΡΠ°Π½ΡΠ΅ ΡΠ³ΠΎΠ» ΞΈΞΈ ΠΎΡΡΠΈΡΠ°ΡΠ΅Π»Π΅Π½, ΡΡΠΎ ΠΎΠ·Π½Π°ΡΠ°Π΅Ρ, ΡΡΠΎ Π΄Π»Ρ ΡΡΠΈΡ Π²Π΅ΠΊΡΠΎΡΠΎΠ² ΡΠ³ΠΎΠ» Π½Π°ΠΏΡΠ°Π²Π»Π΅Π½ΠΈΡ ΞΈAΞΈA ΠΈΠ·ΠΌΠ΅ΡΡΠ΅ΡΡΡ Π½Π° ΠΏΠΎ ΡΠ°ΡΠΎΠ²ΠΎΠΉ ΡΡΡΠ΅Π»ΠΊΠ΅ ΠΎΡ ΠΏΠΎΠ»ΠΎΠΆΠΈΡΠ΅Π»ΡΠ½ΠΎΠΉ ΠΎΡΠΈ x . Π’ΠΎΡΠ½ΠΎ ΡΠ°ΠΊ ΠΆΠ΅ Π΄Π»Ρ Π²Π΅ΠΊΡΠΎΡΠΎΠ² Π²ΠΎ Π²ΡΠΎΡΠΎΠΌ ΠΊΠ²Π°Π΄ΡΠ°Π½ΡΠ΅ ΡΠ³ΠΎΠ» ΞΈΞΈ ΠΎΡΡΠΈΡΠ°ΡΠ΅Π»Π΅Π½. ΠΠΎΠ³Π΄Π° Π²Π΅ΠΊΡΠΎΡ Π»Π΅ΠΆΠΈΡ Π»ΠΈΠ±ΠΎ Π²ΠΎ Π²ΡΠΎΡΠΎΠΌ, Π»ΠΈΠ±ΠΎ Π² ΡΡΠ΅ΡΡΠ΅ΠΌ ΠΊΠ²Π°Π΄ΡΠ°Π½ΡΠ΅, Π³Π΄Π΅ ΠΊΠΎΠΌΠΏΠΎΠ½Π΅Π½ΡΠ° AxAx ΠΎΡΡΠΈΡΠ°ΡΠ΅Π»ΡΠ½Π°, ΡΠ³ΠΎΠ» Π½Π°ΠΏΡΠ°Π²Π»Π΅Π½ΠΈΡ ΡΠ°Π²Π΅Π½ ΞΈA=ΞΈ+180Β°, ΞΈA=ΞΈ+180Β° (ΡΠΈΡ. 2.19).).
Π ΠΈΡΡΠ½ΠΎΠΊ
2.19
Π‘ΠΊΠ°Π»ΡΡΠ½ΡΠ΅ ΠΊΠΎΠΌΠΏΠΎΠ½Π΅Π½ΡΡ Π²Π΅ΠΊΡΠΎΡΠ° ΠΌΠΎΠ³ΡΡ Π±ΡΡΡ ΠΏΠΎΠ»ΠΎΠΆΠΈΡΠ΅Π»ΡΠ½ΡΠΌΠΈ ΠΈΠ»ΠΈ ΠΎΡΡΠΈΡΠ°ΡΠ΅Π»ΡΠ½ΡΠΌΠΈ. ΠΠ΅ΠΊΡΠΎΡΡ Π² ΠΏΠ΅ΡΠ²ΠΎΠΌ ΠΊΠ²Π°Π΄ΡΠ°Π½ΡΠ΅ (I) ΠΈΠΌΠ΅ΡΡ ΠΎΠ±Π΅ ΡΠΊΠ°Π»ΡΡΠ½ΡΠ΅ ΠΊΠΎΠΌΠΏΠΎΠ½Π΅Π½ΡΡ ΠΏΠΎΠ»ΠΎΠΆΠΈΡΠ΅Π»ΡΠ½ΡΠ΅, Π° Π²Π΅ΠΊΡΠΎΡΡ Π² ΡΡΠ΅ΡΡΠ΅ΠΌ ΠΊΠ²Π°Π΄ΡΠ°Π½ΡΠ΅ ΠΈΠΌΠ΅ΡΡ ΠΎΠ±Π΅ ΡΠΊΠ°Π»ΡΡΠ½ΡΠ΅ ΠΊΠΎΠΌΠΏΠΎΠ½Π΅Π½ΡΡ ΠΎΡΡΠΈΡΠ°ΡΠ΅Π»ΡΠ½ΡΠ΅. ΠΠ»Ρ Π²Π΅ΠΊΡΠΎΡΠΎΠ² Π² ΠΊΠ²Π°Π΄ΡΠ°Π½ΡΠ°Ρ
II ΠΈ III ΡΠ³ΠΎΠ» Π½Π°ΠΏΡΠ°Π²Π»Π΅Π½ΠΈΡ Π²Π΅ΠΊΡΠΎΡΠ° ΡΠ°Π²Π΅Π½ ΞΈA=ΞΈ+180Β°, ΞΈA=ΞΈ+180Β°.
ΠΡΠΈΠΌΠ΅Ρ 2,4
ΠΠ΅Π»ΠΈΡΠΈΠ½Π° ΠΈ Π½Π°ΠΏΡΠ°Π²Π»Π΅Π½ΠΈΠ΅ Π²Π΅ΠΊΡΠΎΡΠ° ΡΠΌΠ΅ΡΠ΅Π½ΠΈΡ
ΠΡ ΠΏΠ΅ΡΠ΅ΠΌΠ΅ΡΠ°Π΅ΡΠ΅ ΡΠΊΠ°Π·Π°ΡΠ΅Π»Ρ ΠΌΡΡΠΈ Π½Π° ΠΌΠΎΠ½ΠΈΡΠΎΡΠ΅ ΠΈΠ· ΠΈΡΡ ΠΎΠ΄Π½ΠΎΠ³ΠΎ ΠΏΠΎΠ»ΠΎΠΆΠ΅Π½ΠΈΡ Π² ΡΠΎΡΠΊΠ΅ (6,0 ΡΠΌ, 1,6 ΡΠΌ) ΠΊ Π·Π½Π°ΡΠΊΡ, ΡΠ°ΡΠΏΠΎΠ»ΠΎΠΆΠ΅Π½Π½ΠΎΠΌΡ Π² ΡΠΎΡΠΊΠ΅ (2,0 ΡΠΌ, 4,5 ΡΠΌ). ΠΠ°ΠΊΠΎΠ²Ρ Π²Π΅Π»ΠΈΡΠΈΠ½Π° ΠΈ Π½Π°ΠΏΡΠ°Π²Π»Π΅Π½ΠΈΠ΅ Π²Π΅ΠΊΡΠΎΡΠ° ΡΠΌΠ΅ΡΠ΅Π½ΠΈΡ ΡΠΊΠ°Π·Π°ΡΠ΅Π»Ρ?
Π‘ΡΡΠ°ΡΠ΅Π³ΠΈΡ
Π ΠΏΡΠΈΠΌΠ΅ΡΠ΅ 2.3 ΠΌΡ Π½Π°ΡΠ»ΠΈ Π²Π΅ΠΊΡΠΎΡ ΡΠΌΠ΅ΡΠ΅Π½ΠΈΡ DβDβ ΡΠΊΠ°Π·Π°ΡΠ΅Π»Ρ ΠΌΡΡΠΈ (ΡΠΌ. ΡΡΠ°Π²Π½Π΅Π½ΠΈΠ΅ 2.14). ΠΡ ΠΈΠ΄Π΅Π½ΡΠΈΡΠΈΡΠΈΡΡΠ΅ΠΌ Π΅Π³ΠΎ ΡΠΊΠ°Π»ΡΡΠ½ΡΠ΅ ΠΊΠΎΠΌΠΏΠΎΠ½Π΅Π½ΡΡ Dx=-4,0 ΡΠΌDx=-4,0 ΡΠΌ ΠΈ Dy=+2,9 ΡΠΌDy=+2,9 ΡΠΌ ΠΈ ΠΏΠΎΠ΄ΡΡΠ°Π²Π»ΡΠ΅ΠΌ Π² ΡΡΠ°Π²Π½Π΅Π½ΠΈΡ 2.15 ΠΈ 2.16, ΡΡΠΎΠ±Ρ Π½Π°ΠΉΡΠΈ Π·Π²Π΅Π·Π΄Π½ΡΡ Π²Π΅Π»ΠΈΡΠΈΠ½Ρ D ΠΈ Π½Π°ΠΏΡΠ°Π²Π»Π΅Π½ΠΈΠ΅ ΞΈDΞΈD ΡΠΎΠΎΡΠ²Π΅ΡΡΡΠ²Π΅Π½Π½ΠΎ.
Π Π°ΡΡΠ²ΠΎΡ
ΠΠ΅Π»ΠΈΡΠΈΠ½Π° Π²Π΅ΠΊΡΠΎΡΠ° DβDβ ΡΠ°Π²Π½Π°
D=Dx2+Dy2=(β4,0 ΡΠΌ)2+(2,9 ΡΠΌ)2=(4,0)2+(2,9)2 ΡΠΌ=4,9 ΡΠΌ. D=Dx2+Dy2=(β4,0 ΡΠΌ)2+(2,9 ΡΠΌ) )2=(4,0)2+(2,9)ΡΠΌ, Π½Π°ΠΉΡΠΈ Π΅Π³ΠΎ Π²Π΅Π»ΠΈΡΠΈΠ½Ρ ΠΈ Π½Π°ΠΏΡΠ°Π²Π»Π΅Π½ΠΈΠ΅.
ΠΠΎ ΠΌΠ½ΠΎΠ³ΠΈΡ
ΠΏΡΠΈΠ»ΠΎΠΆΠ΅Π½ΠΈΡΡ
Π²Π΅Π»ΠΈΡΠΈΠ½Ρ ΠΈ Π½Π°ΠΏΡΠ°Π²Π»Π΅Π½ΠΈΡ Π²Π΅ΠΊΡΠΎΡΠ½ΡΡ
Π²Π΅Π»ΠΈΡΠΈΠ½ ΠΈΠ·Π²Π΅ΡΡΠ½Ρ, ΠΈ Π½Π°ΠΌ Π½ΡΠΆΠ½ΠΎ Π½Π°ΠΉΡΠΈ ΡΠ°Π²Π½ΠΎΠ΄Π΅ΠΉΡΡΠ²ΡΡΡΡΡ ΠΌΠ½ΠΎΠ³ΠΈΡ
Π²Π΅ΠΊΡΠΎΡΠΎΠ².
ΠΠ°ΠΏΡΠΈΠΌΠ΅Ρ, ΠΏΡΠ΅Π΄ΡΡΠ°Π²ΡΡΠ΅, ΡΡΠΎ 400 Π°Π²ΡΠΎΠΌΠΎΠ±ΠΈΠ»Π΅ΠΉ Π΄Π²ΠΈΠΆΡΡΡΡ ΠΏΠΎ ΠΌΠΎΡΡΡ ΠΠΎΠ»ΠΎΡΡΠ΅ ΠΠΎΡΠΎΡΠ° Π² Π‘Π°Π½-Π€ΡΠ°Π½ΡΠΈΡΠΊΠΎ ΠΏΡΠΈ ΡΠΈΠ»ΡΠ½ΠΎΠΌ Π²Π΅ΡΡΠ΅. ΠΠ°ΠΆΠ΄Π°Ρ ΠΌΠ°ΡΠΈΠ½Π° Π΄Π°Π΅Ρ ΠΌΠΎΡΡΡ ΡΠ°Π·Π½ΡΠ΅ ΡΠΎΠ»ΡΠΊΠΈ Π² ΡΠ°Π·Π½ΡΡ
Π½Π°ΠΏΡΠ°Π²Π»Π΅Π½ΠΈΡΡ
, ΠΈ ΠΌΡ Ρ
ΠΎΡΠ΅Π»ΠΈ Π±Ρ Π·Π½Π°ΡΡ, Π½Π°ΡΠΊΠΎΠ»ΡΠΊΠΎ Π±ΠΎΠ»ΡΡΠΈΠΌ ΠΌΠΎΠΆΠ΅Ρ Π±ΡΡΡ ΡΠ΅Π·ΡΠ»ΡΡΠΈΡΡΡΡΠΈΠΉ ΡΠΎΠ»ΡΠΎΠΊ. Π£ Π½Π°Ρ ΡΠΆΠ΅ Π΅ΡΡΡ Π½Π΅ΠΊΠΎΡΠΎΡΡΠΉ ΠΎΠΏΡΡ Π³Π΅ΠΎΠΌΠ΅ΡΡΠΈΡΠ΅ΡΠΊΠΎΠ³ΠΎ ΠΏΠΎΡΡΡΠΎΠ΅Π½ΠΈΡ Π²Π΅ΠΊΡΠΎΡΠ½ΡΡ
ΡΡΠΌΠΌ, ΠΏΠΎΡΡΠΎΠΌΡ ΠΌΡ Π·Π½Π°Π΅ΠΌ, ΡΡΠΎ Π·Π°Π΄Π°ΡΠ° Π½Π°Ρ
ΠΎΠΆΠ΄Π΅Π½ΠΈΡ ΡΠ°Π²Π½ΠΎΠ΄Π΅ΠΉΡΡΠ²ΡΡΡΠ΅ΠΉ ΠΏΡΡΠ΅ΠΌ ΡΠΈΡΠΎΠ²Π°Π½ΠΈΡ Π²Π΅ΠΊΡΠΎΡΠΎΠ² ΠΈ ΠΈΠ·ΠΌΠ΅ΡΠ΅Π½ΠΈΡ ΠΈΡ
Π΄Π»ΠΈΠ½ ΠΈ ΡΠ³Π»ΠΎΠ² ΠΌΠΎΠΆΠ΅Ρ Π΄ΠΎΠ²ΠΎΠ»ΡΠ½ΠΎ Π±ΡΡΡΡΠΎ ΡΡΠ°ΡΡ Π½Π΅ΡΠ°Π·ΡΠ΅ΡΠΈΠΌΠΎΠΉ, ΡΡΠΎ ΠΏΡΠΈΠ²Π΅Π΄Π΅Ρ ΠΊ ΠΎΠ³ΡΠΎΠΌΠ½ΡΠΌ ΠΎΡΠΈΠ±ΠΊΠ°ΠΌ. ΠΠΎΠ΄ΠΎΠ±Π½ΠΎΠ³ΠΎ Π±Π΅ΡΠΏΠΎΠΊΠΎΠΉΡΡΠ²Π° Π½Π΅ Π²ΠΎΠ·Π½ΠΈΠΊΠ°Π΅Ρ, ΠΊΠΎΠ³Π΄Π° ΠΌΡ ΠΈΡΠΏΠΎΠ»ΡΠ·ΡΠ΅ΠΌ Π°Π½Π°Π»ΠΈΡΠΈΡΠ΅ΡΠΊΠΈΠ΅ ΠΌΠ΅ΡΠΎΠ΄Ρ. Π‘Π°ΠΌΡΠΉ ΠΏΠ΅ΡΠ²ΡΠΉ ΡΠ°Π³ Π² Π°Π½Π°Π»ΠΈΡΠΈΡΠ΅ΡΠΊΠΎΠΌ ΠΏΠΎΠ΄Ρ
ΠΎΠ΄Π΅ ΡΠΎΡΡΠΎΠΈΡ Π² ΡΠΎΠΌ, ΡΡΠΎΠ±Ρ Π½Π°ΠΉΡΠΈ ΠΊΠΎΠΌΠΏΠΎΠ½Π΅Π½ΡΡ Π²Π΅ΠΊΡΠΎΡΠ°, ΠΊΠΎΠ³Π΄Π° ΠΈΠ·Π²Π΅ΡΡΠ½Ρ Π½Π°ΠΏΡΠ°Π²Π»Π΅Π½ΠΈΠ΅ ΠΈ Π²Π΅Π»ΠΈΡΠΈΠ½Π° Π²Π΅ΠΊΡΠΎΡΠ°.
ΠΠ΅ΡΠ½Π΅ΠΌΡΡ ΠΊ ΠΏΡΡΠΌΠΎΡΠ³ΠΎΠ»ΡΠ½ΠΎΠΌΡ ΡΡΠ΅ΡΠ³ΠΎΠ»ΡΠ½ΠΈΠΊΡ Π½Π° ΡΠΈΡ. 2.18. ΠΡΠ½ΠΎΡΠ΅Π½ΠΈΠ΅ ΠΏΡΠΎΡΠΈΠ²ΠΎΠ»Π΅ΠΆΠ°ΡΠ΅Π³ΠΎ ΠΊΠ°ΡΠ΅ΡΠ° AxAx ΠΊ Π³ΠΈΠΏΠΎΡΠ΅Π½ΡΠ·Π΅ A ΡΠ²Π»ΡΠ΅ΡΡΡ ΡΡΠ½ΠΊΡΠΈΠ΅ΠΉ ΠΊΠΎΡΠΈΠ½ΡΡΠ° Π½Π°ΠΏΡΠ°Π²Π»ΡΡΡΠ΅Π³ΠΎ ΡΠ³Π»Π° ΞΈAΞΈA, Ax/A=cosΞΈAAx/A=cosΞΈA, Π° ΠΎΡΠ½ΠΎΡΠ΅Π½ΠΈΠ΅ ΠΏΡΠΎΡΠΈΠ²ΠΎΠ»Π΅ΠΆΠ°ΡΠ΅Π³ΠΎ ΠΊΠ°ΡΠ΅ΡΠ° AyAy ΠΊ Π³ΠΈΠΏΠΎΡΠ΅Π½ΡΠ·Π΅ A ΡΠ²Π»ΡΠ΅ΡΡΡ ΡΡΠ½ΠΊΡΠΈΠ΅ΠΉ ΡΠΈΠ½ΡΡΠ° ΞΈAΞΈA, Ay/A=sinΞΈAAy/A=sinΞΈA.
ΠΠΎΠ³Π΄Π° ΠΈΠ·Π²Π΅ΡΡΠ½Π° Π²Π΅Π»ΠΈΡΠΈΠ½Π° A ΠΈ Π½Π°ΠΏΡΠ°Π²Π»Π΅Π½ΠΈΠ΅ ΞΈAΞΈA, ΠΌΡ ΠΌΠΎΠΆΠ΅ΠΌ ΡΠ΅ΡΠΈΡΡ ΡΡΠΈ ΡΠΎΠΎΡΠ½ΠΎΡΠ΅Π½ΠΈΡ Π΄Π»Ρ ΡΠΊΠ°Π»ΡΡΠ½ΡΡ
ΡΠΎΡΡΠ°Π²Π»ΡΡΡΠΈΡ
:
{Ax=AcosΞΈAAy=AsinΞΈA.{Ax=AcosΞΈAAy=AsinΞΈA.
2.17
ΠΡΠΈ ΡΠ°ΡΡΠ΅ΡΠ΅ ΠΊΠΎΠΌΠΏΠΎΠ½Π΅Π½ΡΠΎΠ² Π²Π΅ΠΊΡΠΎΡΠ° ΠΏΠΎ ΡΡΠ°Π²Π½Π΅Π½ΠΈΡ 2.17 Π½Π΅ΠΎΠ±Ρ ΠΎΠ΄ΠΈΠΌΠΎ ΡΠΎΠ±Π»ΡΠ΄Π°ΡΡ ΠΎΡΡΠΎΡΠΎΠΆΠ½ΠΎΡΡΡ ΠΏΡΠΈ Π²ΡΠ±ΠΎΡΠ΅ ΡΠ³Π»Π°. Π£Π³ΠΎΠ» Π½Π°ΠΏΡΠ°Π²Π»Π΅Π½ΠΈΡ ΞΈAΞΈA Π²Π΅ΠΊΡΠΎΡΠ° β ΡΡΠΎ ΡΠ³ΠΎΠ», ΠΈΠ·ΠΌΠ΅ΡΠ΅Π½Π½ΡΠΉ Π½Π° 90Β 289 ΠΏΡΠΎΡΠΈΠ² ΡΠ°ΡΠΎΠ²ΠΎΠΉ ΡΡΡΠ΅Π»ΠΊΠΈ Π½Π° 90Β 150 ΠΎΡ ΠΏΠΎΠ»ΠΎΠΆΠΈΡΠ΅Π»ΡΠ½ΠΎΠ³ΠΎ Π½Π°ΠΏΡΠ°Π²Π»Π΅Π½ΠΈΡ ΠΏΠΎ ΠΎΡΠΈ 90Β 289 x 90Β 150 Π΄ΠΎ Π²Π΅ΠΊΡΠΎΡΠ°. ΠΠ·ΠΌΠ΅ΡΠ΅Π½ΠΈΠ΅ ΠΏΠΎ ΡΠ°ΡΠΎΠ²ΠΎΠΉ ΡΡΡΠ΅Π»ΠΊΠ΅ Π΄Π°Π΅Ρ ΠΎΡΡΠΈΡΠ°ΡΠ΅Π»ΡΠ½ΡΠΉ ΡΠ³ΠΎΠ».
ΠΡΠΈΠΌΠ΅Ρ 2,5
ΠΠΎΠΌΠΏΠΎΠ½Π΅Π½ΡΡ Π²Π΅ΠΊΡΠΎΡΠΎΠ² ΡΠΌΠ΅ΡΠ΅Π½ΠΈΡ
ΠΡΡΠΏΠΏΠ° ΡΠΏΠ°ΡΠ΅Π½ΠΈΡ ΠΏΡΠΎΠΏΠ°Π²ΡΠ΅Π³ΠΎ ΡΠ΅Π±Π΅Π½ΠΊΠ° ΡΠ»Π΅Π΄ΡΠ΅Ρ Π·Π° ΠΏΠΎΠΈΡΠΊΠΎΠ²ΠΎΠΉ ΡΠΎΠ±Π°ΠΊΠΎΠΉ ΠΏΠΎ ΠΊΠ»ΠΈΡΠΊΠ΅ Π‘ΠΎΠ»Π΄Π°Ρ. ΠΠ΅ΡΠ°Π½ΡΠ½ΠΈΠΊ ΠΌΠ½ΠΎΠ³ΠΎ Π±ΡΠΎΠ΄ΠΈΡ ΠΈ Π΄Π΅Π»Π°Π΅Ρ ΠΌΠ½ΠΎΠ³ΠΎ ΠΏΡΠΎΠ±Π½ΡΡ
ΠΎΠ±Π½ΡΡ
ΠΈΠ²Π°Π½ΠΈΠΉ ΠΏΠΎ ΡΠ°Π·Π½ΡΠΌ ΡΡΠΎΠΏΠ°ΠΌ. Π‘ΠΎΠ»Π΄Π°Ρ Π² ΠΊΠΎΠ½ΡΠ΅ ΠΊΠΎΠ½ΡΠΎΠ² Π½Π°Ρ
ΠΎΠ΄ΠΈΡ ΡΠ΅Π±Π΅Π½ΠΊΠ°, ΠΈ Ρ ΠΈΡΡΠΎΡΠΈΠΈ ΡΡΠ°ΡΡΠ»ΠΈΠ²ΡΠΉ ΠΊΠΎΠ½Π΅Ρ, Π½ΠΎ Π΅Π³ΠΎ ΠΏΠ΅ΡΠ΅ΠΌΠ΅ΡΠ΅Π½ΠΈΡ Π½Π° ΡΠ°Π·Π½ΡΡ
Π½ΠΎΠ³Π°Ρ
ΠΊΠ°ΠΆΡΡΡΡ Π΄Π΅ΠΉΡΡΠ²ΠΈΡΠ΅Π»ΡΠ½ΠΎ Π·Π°ΠΏΡΡΠ°Π½Π½ΡΠΌΠΈ. ΠΠ° ΠΎΠ΄Π½ΠΎΠΉ ΠΈΠ· Π½ΠΎΠ³ ΠΎΠ½ ΠΏΡΠΎΡ
ΠΎΠ΄ΠΈΡ 200,0 ΠΌ Π½Π° ΡΠ³ΠΎ-Π²ΠΎΡΡΠΎΠΊ, Π·Π°ΡΠ΅ΠΌ Π±Π΅ΠΆΠΈΡ Π½Π° ΡΠ΅Π²Π΅Ρ ΠΎΠΊΠΎΠ»ΠΎ 300,0 ΠΌ.
ΠΠ° ΡΡΠ΅ΡΡΠ΅ΠΌ ΡΡΠ°ΠΏΠ΅ Π²Π½ΠΈΠΌΠ°ΡΠ΅Π»ΡΠ½ΠΎ ΠΈΡΡΠ»Π΅Π΄ΡΠ΅Ρ Π·Π°ΠΏΠ°Ρ
ΠΈ Π½Π° ΠΏΡΠΎΡΡΠΆΠ΅Π½ΠΈΠΈ 50,0 ΠΌ Π² Π½Π°ΠΏΡΠ°Π²Π»Π΅Π½ΠΈΠΈ 30Β°30Β° ΠΊ Π·Π°ΠΏΠ°Π΄Ρ ΠΎΡ ΡΠ΅Π²Π΅ΡΠ°. ΠΠ° ΡΠ΅ΡΠ²Π΅ΡΡΠΎΠΌ ΡΡΠ°ΠΏΠ΅ Trooper ΠΈΠ΄Π΅Ρ ΠΏΡΡΠΌΠΎ Π½Π° ΡΠ³ Π½Π° 80,0 ΠΌ, Π½Π°Ρ
ΠΎΠ΄ΠΈΡ ΡΠ²Π΅ΠΆΠΈΠΉ Π·Π°ΠΏΠ°Ρ
ΠΈ ΠΏΠΎΠ²ΠΎΡΠ°ΡΠΈΠ²Π°Π΅Ρ Π½Π° 23Β°23Β° ΠΊ Π·Π°ΠΏΠ°Π΄Ρ ΠΎΡ ΡΠ³Π° Π½Π° 150,0 ΠΌ. ΠΠ°ΠΉΠ΄ΠΈΡΠ΅ ΡΠΊΠ°Π»ΡΡΠ½ΡΠ΅ ΠΊΠΎΠΌΠΏΠΎΠ½Π΅Π½ΡΡ Π²Π΅ΠΊΡΠΎΡΠΎΠ² ΡΠΌΠ΅ΡΠ΅Π½ΠΈΡ ΠΠ΅ΡΠ°Π½ΡΠ½ΠΈΠΊΠ° ΠΈ Π΅Π³ΠΎ Π²Π΅ΠΊΡΠΎΡΠΎΠ² ΡΠΌΠ΅ΡΠ΅Π½ΠΈΡ Π² ΡΠΎΡΠΌΠ΅ Π²Π΅ΠΊΡΠΎΡΠ½ΡΡ
ΠΊΠΎΠΌΠΏΠΎΠ½Π΅Π½Ρ Π΄Π»Ρ ΠΊΠ°ΠΆΠ΄ΠΎΠΉ Π½ΠΎΠ³ΠΈ.
9ΠΎΡΠΈ ΠΈ ΡΠΊΠ°Π·ΡΠ²Π°Π΅Ρ Π½Π° ΡΠ΅Π²Π΅Ρ. ΠΠ΅ΡΠ°Π½ΡΠ½ΠΈΠΊ Π΄Π΅Π»Π°Π΅Ρ ΠΏΡΡΡ Π½ΠΎΠ³, ΠΏΠΎΡΡΠΎΠΌΡ Π΅ΡΡΡ ΠΏΡΡΡ Π²Π΅ΠΊΡΠΎΡΠΎΠ² ΡΠΌΠ΅ΡΠ΅Π½ΠΈΡ. ΠΠ°ΡΠ½Π΅ΠΌ Ρ ΠΎΠΏΡΠ΅Π΄Π΅Π»Π΅Π½ΠΈΡ ΠΈΡ
Π²Π΅Π»ΠΈΡΠΈΠ½ ΠΈ ΡΠ³Π»ΠΎΠ² Π½Π°ΠΏΡΠ°Π²Π»Π΅Π½ΠΈΡ, Π·Π°ΡΠ΅ΠΌ Π²ΠΎΡΠΏΠΎΠ»ΡΠ·ΡΠ΅ΠΌΡΡ ΡΡΠ°Π²Π½Π΅Π½ΠΈΠ΅ΠΌ 2.17, ΡΡΠΎΠ±Ρ Π½Π°ΠΉΡΠΈ ΡΠΊΠ°Π»ΡΡΠ½ΡΠ΅ ΠΊΠΎΠΌΠΏΠΎΠ½Π΅Π½ΡΡ ΡΠΌΠ΅ΡΠ΅Π½ΠΈΠΉ, ΠΈ ΡΡΠ°Π²Π½Π΅Π½ΠΈΠ΅ΠΌ 2.12, ΡΡΠΎΠ±Ρ Π½Π°ΠΉΡΠΈ Π²Π΅ΠΊΡΠΎΡΠ° ΡΠΌΠ΅ΡΠ΅Π½ΠΈΡ.
Π Π°ΡΡΠ²ΠΎΡ
ΠΠ° ΠΏΠ΅ΡΠ²ΠΎΠΌ ΡΡΠ°ΡΡΠΊΠ΅ Π²Π΅Π»ΠΈΡΠΈΠ½Π° ΡΠΌΠ΅ΡΠ΅Π½ΠΈΡ L1=200,0ΠΌL1=200,0ΠΌ ΠΈ Π½Π°ΠΏΡΠ°Π²Π»Π΅Π½ΠΈΠ΅ ΡΠ³ΠΎ-Π²ΠΎΡΡΠΎΡΠ½ΠΎΠ΅. Π ΠΊΠ°ΡΠ΅ΡΡΠ²Π΅ Π΄ΠΈΡΠ΅ΠΊΡΠΈΠΎΠ½Π½ΠΎΠ³ΠΎ ΡΠ³Π»Π° ΞΈ1ΞΈ1 ΠΌΡ ΠΌΠΎΠΆΠ΅ΠΌ ΠΏΡΠΈΠ½ΡΡΡ Π»ΠΈΠ±ΠΎ 45Β°45Β°, ΠΈΠ·ΠΌΠ΅ΡΠ΅Π½Π½ΡΠ΅ ΠΏΠΎ ΡΠ°ΡΠΎΠ²ΠΎΠΉ ΡΡΡΠ΅Π»ΠΊΠ΅ Ρ Π²ΠΎΡΡΠΎΡΠ½ΠΎΠ³ΠΎ Π½Π°ΠΏΡΠ°Π²Π»Π΅Π½ΠΈΡ, Π»ΠΈΠ±ΠΎ 45Β°+270Β°45Β°+270Β°, ΠΈΠ·ΠΌΠ΅ΡΠ΅Π½Π½ΡΠ΅ ΠΏΡΠΎΡΠΈΠ² ΡΠ°ΡΠΎΠ²ΠΎΠΉ ΡΡΡΠ΅Π»ΠΊΠΈ Ρ Π²ΠΎΡΡΠΎΠΊΠ°.
ΠΡΠΈ ΠΏΠ΅ΡΠ²ΠΎΠΌ Π²ΡΠ±ΠΎΡΠ΅ ΞΈ1=β45°θ1=β45Β°. ΠΡΠΈ Π²ΡΠΎΡΠΎΠΌ Π²ΡΠ±ΠΎΡΠ΅ ΞΈ1=+315°θ1=+315Β°. ΠΡ ΠΌΠΎΠΆΠ΅ΠΌ ΠΈΡΠΏΠΎΠ»ΡΠ·ΠΎΠ²Π°ΡΡ Π»ΡΠ±ΠΎΠΉ ΠΈΠ· ΡΡΠΈΡ
Π΄Π²ΡΡ
ΡΠ³Π»ΠΎΠ². ΠΠΎΠΌΠΏΠΎΠ½Π΅Π½ΡΡ
9Π²Π΄ΠΎΠ»Ρ ΠΎΡΠΈ x ΠΈ ΠΎΡΠΈ y ΡΠΎΠΎΡΠ²Π΅ΡΡΡΠ²Π΅Π½Π½ΠΎ. ΠΠ΅ΠΊΠ°ΡΡΠΎΠ²Ρ ΡΠΈΡΡΠ΅ΠΌΡ ΠΊΠΎΠΎΡΠ΄ΠΈΠ½Π°Ρ ΠΎΡΠ΅Π½Ρ ΡΠ΄ΠΎΠ±Π½ΠΎ ΠΈΡΠΏΠΎΠ»ΡΠ·ΠΎΠ²Π°ΡΡ ΠΏΡΠΈ ΠΎΠΏΠΈΡΠ°Π½ΠΈΠΈ ΠΏΠ΅ΡΠ΅ΠΌΠ΅ΡΠ΅Π½ΠΈΠΉ ΠΈ ΡΠΊΠΎΡΠΎΡΡΠ΅ΠΉ ΡΠ΅Π» ΠΈ Π΄Π΅ΠΉΡΡΠ²ΡΡΡΠΈΡ
Π½Π° Π½ΠΈΡ
ΡΠΈΠ». ΠΠ΄Π½Π°ΠΊΠΎ ΡΡΠΎ ΡΡΠ°Π½ΠΎΠ²ΠΈΡΡΡ Π³ΡΠΎΠΌΠΎΠ·Π΄ΠΊΠΈΠΌ, ΠΊΠΎΠ³Π΄Π° Π½Π°ΠΌ Π½ΡΠΆΠ½ΠΎ ΠΎΠΏΠΈΡΠ°ΡΡ Π²ΡΠ°ΡΠ΅Π½ΠΈΠ΅ ΠΎΠ±ΡΠ΅ΠΊΡΠΎΠ². ΠΡΠΈ ΠΎΠΏΠΈΡΠ°Π½ΠΈΠΈ Π²ΡΠ°ΡΠ΅Π½ΠΈΡ ΠΌΡ ΠΎΠ±ΡΡΠ½ΠΎ ΡΠ°Π±ΠΎΡΠ°Π΅ΠΌ Π² ΠΏΠΎΠ»ΡΡΠ½ΠΎΠΉ ΡΠΈΡΡΠ΅ΠΌΠ΅ ΠΊΠΎΠΎΡΠ΄ΠΈΠ½Π°Ρ.
Π ΠΏΠΎΠ»ΡΡΠ½ΠΎΠΉ ΡΠΈΡΡΠ΅ΠΌΠ΅ ΠΊΠΎΠΎΡΠ΄ΠΈΠ½Π°Ρ ΠΏΠΎΠ»ΠΎΠΆΠ΅Π½ΠΈΠ΅ ΡΠΎΡΠΊΠΈ P Π½Π° ΠΏΠ»ΠΎΡΠΊΠΎΡΡΠΈ Π·Π°Π΄Π°Π΅ΡΡΡ Π΄Π²ΡΠΌΡ ΠΏΠΎΠ»ΡΡΠ½ΡΠΌΠΈ ΠΊΠΎΠΎΡΠ΄ΠΈΠ½Π°ΡΠ°ΠΌΠΈ (ΡΠΈΡ. 2.20). ΠΠ΅ΡΠ²Π°Ρ ΠΏΠΎΠ»ΡΡΠ½Π°Ρ ΠΊΠΎΠΎΡΠ΄ΠΈΠ½Π°ΡΠ° — ΡΡΠΎ ΡΠ°Π΄ΠΈΠ°Π»ΡΠ½Π°Ρ ΠΊΠΎΠΎΡΠ΄ΠΈΠ½Π°ΡΠ° 9Π½Π°ΠΏΡΠ°Π²Π»Π΅Π½ΠΈΠ΅ ΠΏΠΎΠΊΠ°Π·ΡΠ²Π°Π΅Ρ, ΠΊΠ°ΠΊ ΡΠ³ΠΎΠ» ΟΟ ΠΈΠ·ΠΌΠ΅Π½ΡΠ΅ΡΡΡ Π² Π½Π°ΠΏΡΠ°Π²Π»Π΅Π½ΠΈΠΈ ΠΏΡΠΎΡΠΈΠ² ΡΠ°ΡΠΎΠ²ΠΎΠΉ ΡΡΡΠ΅Π»ΠΊΠΈ. Π’Π°ΠΊΠΈΠΌ ΠΎΠ±ΡΠ°Π·ΠΎΠΌ, ΡΠΎΡΠΊΠ° P , ΠΈΠΌΠ΅ΡΡΠ°Ρ ΠΊΠΎΠΎΡΠ΄ΠΈΠ½Π°ΡΡ ( x , y ) Π² ΠΏΡΡΠΌΠΎΡΠ³ΠΎΠ»ΡΠ½ΠΎΠΉ ΡΠΈΡΡΠ΅ΠΌΠ΅, ΠΌΠΎΠΆΠ΅Ρ Π±ΡΡΡ ΡΠΊΠ²ΠΈΠ²Π°Π»Π΅Π½ΡΠ½ΠΎ ΠΎΠΏΠΈΡΠ°Π½Π° Π² ΠΏΠΎΠ»ΡΡΠ½ΠΎΠΉ ΡΠΈΡΡΠ΅ΠΌΠ΅ ΠΊΠΎΠΎΡΠ΄ΠΈΠ½Π°Ρ Π΄Π²ΡΠΌΡ ΠΏΠΎΠ»ΡΡΠ½ΡΠΌΠΈ ΠΊΠΎΠΎΡΠ΄ΠΈΠ½Π°ΡΠ°ΠΌΠΈ (r,Ο)(r,Ο).
Π£ΡΠ°Π²Π½Π΅Π½ΠΈΠ΅ 2.17 ΡΠΏΡΠ°Π²Π΅Π΄Π»ΠΈΠ²ΠΎ Π΄Π»Ρ Π»ΡΠ±ΠΎΠ³ΠΎ Π²Π΅ΠΊΡΠΎΡΠ°, ΠΏΠΎΡΡΠΎΠΌΡ ΠΌΡ ΠΌΠΎΠΆΠ΅ΠΌ ΠΈΡΠΏΠΎΠ»ΡΠ·ΠΎΠ²Π°ΡΡ Π΅Π³ΠΎ Π΄Π»Ρ Π²ΡΡΠ°ΠΆΠ΅Π½ΠΈΡ ΠΊΠΎΠΎΡΠ΄ΠΈΠ½Π°Ρ x ΠΈ y Π²Π΅ΠΊΡΠΎΡΠ° rβrβ. Π’Π°ΠΊΠΈΠΌ ΠΎΠ±ΡΠ°Π·ΠΎΠΌ, ΠΌΡ ΠΏΠΎΠ»ΡΡΠ°Π΅ΠΌ ΡΠ²ΡΠ·Ρ ΠΌΠ΅ΠΆΠ΄Ρ ΠΏΠΎΠ»ΡΡΠ½ΡΠΌΠΈ ΠΊΠΎΠΎΡΠ΄ΠΈΠ½Π°ΡΠ°ΠΌΠΈ ΠΈ ΠΏΡΡΠΌΠΎΡΠ³ΠΎΠ»ΡΠ½ΡΠΌΠΈ ΠΊΠΎΠΎΡΠ΄ΠΈΠ½Π°ΡΠ°ΠΌΠΈ ΡΠΎΡΠΊΠΈ 9ΠΎΠΏΡΠ΅Π΄Π΅Π»ΡΠ΅Ρ ΠΏΠΎΠ»ΠΎΠΆΠΈΡΠ΅Π»ΡΠ½ΠΎΠ΅ Π½Π°ΠΏΡΠ°Π²Π»Π΅Π½ΠΈΠ΅ Π²ΡΠ°ΡΠ΅Π½ΠΈΡ ΡΠ³Π»ΠΎΠΌ ΟΟ.
ΠΡΠΈΠΌΠ΅Ρ 2,6
ΠΠΎΠ»ΡΡΠ½ΡΠ΅ ΠΊΠΎΠΎΡΠ΄ΠΈΠ½Π°ΡΡ
ΠΠ»Π°Π΄ΠΎΠΈΡΠΊΠ°ΡΠ΅Π»Ρ Π½Π°Ρ ΠΎΠ΄ΠΈΡ ΠΎΠ΄Π½Ρ ΡΠ΅ΡΠ΅Π±ΡΡΠ½ΡΡ ΠΌΠΎΠ½Π΅ΡΡ Π½Π° ΡΠ°ΡΡΡΠΎΡΠ½ΠΈΠΈ 20,0 ΠΌ ΠΎΡ ΡΡΡ ΠΎΠ³ΠΎ ΠΊΠΎΠ»ΠΎΠ΄ΡΠ° Π² Π½Π°ΠΏΡΠ°Π²Π»Π΅Π½ΠΈΠΈ 20Β°20Β° ΡΠ΅Π²Π΅ΡΠ½ΠΎΠΉ Π²ΠΎΡΡΠΎΠΊΠ° ΠΈ ΠΎΠ΄Π½Ρ Π·ΠΎΠ»ΠΎΡΡΡ ΠΌΠΎΠ½Π΅ΡΡ Π½Π° ΡΠ°ΡΡΡΠΎΡΠ½ΠΈΠΈ 10,0 ΠΌ ΠΎΡ ΠΊΠΎΠ»ΠΎΠ΄ΡΠ° Π² Π½Π°ΠΏΡΠ°Π²Π»Π΅Π½ΠΈΠΈ 20Β°20Β° ΡΠ΅Π²Π΅ΡΠ½ΠΎΠΉ ΡΠΈΡΠΎΡΡ. Π·Π°ΠΏΠ°Π΄. ΠΠ°ΠΊΠΎΠ²Ρ ΠΏΠΎΠ»ΡΡΠ½ΡΠ΅ ΠΈ ΠΏΡΡΠΌΠΎΡΠ³ΠΎΠ»ΡΠ½ΡΠ΅ ΠΊΠΎΠΎΡΠ΄ΠΈΠ½Π°ΡΡ ΡΡΠΈΡ Π½Π°Ρ ΠΎΠ΄ΠΎΠΊ ΠΎΡΠ½ΠΎΡΠΈΡΠ΅Π»ΡΠ½ΠΎ ΡΠΊΠ²Π°ΠΆΠΈΠ½Ρ?
Π‘ΡΡΠ°ΡΠ΅Π³ΠΈΡ
ΠΠΎΠ»ΠΎΠ΄Π΅Ρ ΠΎΡΠΌΠ΅ΡΠ°Π΅Ρ Π½Π°ΡΠ°Π»ΠΎ ΡΠΈΡΡΠ΅ΠΌΡ ΠΊΠΎΠΎΡΠ΄ΠΈΠ½Π°Ρ, Π° Π²ΠΎΡΡΠΎΠΊ — + 9.0289 x — Π½Π°ΠΏΡΠ°Π²Π»Π΅Π½ΠΈΠ΅. ΠΡ ΠΎΠΏΡΠ΅Π΄Π΅Π»ΡΠ΅ΠΌ ΡΠ°Π΄ΠΈΠ°Π»ΡΠ½ΡΠ΅ ΡΠ°ΡΡΡΠΎΡΠ½ΠΈΡ ΠΎΡ ΠΌΠ΅ΡΡΠΎΠΏΠΎΠ»ΠΎΠΆΠ΅Π½ΠΈΠΉ Π΄ΠΎ Π½Π°ΡΠ°Π»Π° ΠΊΠΎΠΎΡΠ΄ΠΈΠ½Π°Ρ, ΠΊΠΎΡΠΎΡΡΠ΅ ΡΠΎΡΡΠ°Π²Π»ΡΡΡ rS=20,0mrS=20,0ΠΌ (Π΄Π»Ρ ΡΠ΅ΡΠ΅Π±ΡΡΠ½ΠΎΠΉ ΠΌΠΎΠ½Π΅ΡΡ) ΠΈ rG=10,0mrG=10,0ΠΌ (Π΄Π»Ρ Π·ΠΎΠ»ΠΎΡΠΎΠΉ ΠΌΠΎΠ½Π΅ΡΡ).
Π§ΡΠΎΠ±Ρ Π½Π°ΠΉΡΠΈ ΡΠ³Π»ΠΎΠ²ΡΠ΅ ΠΊΠΎΠΎΡΠ΄ΠΈΠ½Π°ΡΡ, ΠΌΡ ΠΊΠΎΠ½Π²Π΅ΡΡΠΈΡΡΠ΅ΠΌ 20Β°20Β° Π² ΡΠ°Π΄ΠΈΠ°Π½Ρ: 20Β°=Ο20/180=Ο/920Β°=Ο20/180=Ο/9. ΠΡ ΠΈΡΠΏΠΎΠ»ΡΠ·ΡΠ΅ΠΌ ΡΡΠ°Π²Π½Π΅Π½ΠΈΠ΅ 2.18, ΡΡΠΎΠ±Ρ Π½Π°ΠΉΡΠΈ ΠΊΠΎΠΎΡΠ΄ΠΈΠ½Π°ΡΡ x ΠΈ y ΠΌΠΎΠ½Π΅Ρ.
Π Π°ΡΡΠ²ΠΎΡ
Π£Π³Π»ΠΎΠ²Π°Ρ ΠΊΠΎΠΎΡΠ΄ΠΈΠ½Π°ΡΠ° ΡΠ΅ΡΠ΅Π±ΡΡΠ½ΠΎΠΉ ΠΌΠΎΠ½Π΅ΡΡ ΟS=Ο/9ΟS=Ο/9, ΡΠΎΠ³Π΄Π° ΠΊΠ°ΠΊ ΡΠ³Π»ΠΎΠ²Π°Ρ ΠΊΠΎΠΎΡΠ΄ΠΈΠ½Π°ΡΠ° Π·ΠΎΠ»ΠΎΡΠΎΠΉ ΠΌΠΎΠ½Π΅ΡΡ ΟG=ΟβΟ/9.=8Ο/9ΟG=ΟβΟ/9=8Ο/9. Π‘Π»Π΅Π΄ΠΎΠ²Π°ΡΠ΅Π»ΡΠ½ΠΎ, ΠΏΠΎΠ»ΡΡΠ½ΡΠ΅ ΠΊΠΎΠΎΡΠ΄ΠΈΠ½Π°ΡΡ ΡΠ΅ΡΠ΅Π±ΡΡΠ½ΠΎΠΉ ΠΌΠΎΠ½Π΅ΡΡ ΡΠ°Π²Π½Ρ (rS,ΟS)=(20,0m,Ο/9)(rS,ΟS)=(20,0m,Ο/9), Π° ΠΏΠΎΠ»ΡΡΠ½ΡΠ΅ ΠΊΠΎΠΎΡΠ΄ΠΈΠ½Π°ΡΡ Π·ΠΎΠ»ΠΎΡΠΎΠΉ ΠΌΠΎΠ½Π΅ΡΡ ΡΠ°Π²Π½Ρ (rG,ΟG) =(10,0ΠΌ,8Ο/9)(rG,ΟG)=(10,0ΠΌ,8Ο/9). ΠΡ ΠΏΠΎΠ΄ΡΡΠ°Π²Π»ΡΠ΅ΠΌ ΡΡΠΈ ΠΊΠΎΠΎΡΠ΄ΠΈΠ½Π°ΡΡ Π² ΡΡΠ°Π²Π½Π΅Π½ΠΈΠ΅ 2.18, ΡΡΠΎΠ±Ρ ΠΏΠΎΠ»ΡΡΠΈΡΡ ΠΏΡΡΠΌΠΎΡΠ³ΠΎΠ»ΡΠ½ΡΠ΅ ΠΊΠΎΠΎΡΠ΄ΠΈΠ½Π°ΡΡ. ΠΠ»Ρ Π·ΠΎΠ»ΠΎΡΠΎΠΉ ΠΌΠΎΠ½Π΅ΡΡ ΠΊΠΎΠΎΡΠ΄ΠΈΠ½Π°ΡΡ
{xG=rGcosΟG=(10,0m)cos8Ο/9=β9,4myG=rGsinΟG=(10,0m)sin8Ο/9=3,4mβ(xG,yG)=(β9,4m,3,4m).{xG= rGcosΟG=(10,0m)cos8Ο/9=-9,4myG=rGsinΟG=(10,0m)sin8Ο/9=3,4mβ(xG,yG)=(-9,4m,3,4m).
ΠΠ»Ρ ΡΠ΅ΡΠ΅Π±ΡΡΠ½ΠΎΠΉ ΠΌΠΎΠ½Π΅ΡΡ ΠΊΠΎΠΎΡΠ΄ΠΈΠ½Π°ΡΡ
{xS=rScosΟS=(20,0m)cosΟ/9=18,9myS=rSsinΟS=(20,0m)sinΟ/9=6,8mβ(xS,yS)=(18,9m,6,8m).
{xS=rScosΟS= (20,0m)cosΟ/9=18,9myS=rSsinΟS=(20,0m)sinΟ/9=6,8mβ(xS,yS)=(18,9m,6,8m).
ΠΠ΅ΠΊΡΠΎΡΡ Π² ΡΡΠ΅Ρ ΠΈΠ·ΠΌΠ΅ΡΠ΅Π½ΠΈΡΡ
Π§ΡΠΎΠ±Ρ Π·Π°Π΄Π°ΡΡ ΠΏΠΎΠ»ΠΎΠΆΠ΅Π½ΠΈΠ΅ ΡΠΎΡΠΊΠΈ Π² ΠΏΡΠΎΡΡΡΠ°Π½ΡΡΠ²Π΅, Π½Π°ΠΌ Π½ΡΠΆΠ½Ρ ΡΡΠΈ ΠΊΠΎΠΎΡΠ΄ΠΈΠ½Π°ΡΡ ( x , y , z ), Π³Π΄Π΅ ΠΊΠΎΠΎΡΠ΄ΠΈΠ½Π°ΡΡ x ΠΈ y Π·Π°Π΄Π°ΡΡ ΡΠ°ΡΠΏΠΎΠ»ΠΎΠΆΠ΅Π½ΠΈΠ΅ Π½Π° ΠΏΠ»ΠΎΡΠΊΠΎΡΡΠΈ, Π° ΠΊΠΎΠΎΡΠ΄ΠΈΠ½Π°ΡΠ° z Π΄Π°Π΅Ρ Π²Π΅ΡΡΠΈΠΊΠ°Π»ΡΠ½ΠΎΠ΅ ΠΏΠΎΠ»ΠΎΠΆΠ΅Π½ΠΈΠ΅ Π½Π°Π΄ ΠΈΠ»ΠΈ ΠΏΠΎΠ΄ ΠΏΠ»ΠΎΡΠΊΠΎΡΡΡΡ. Π’ΡΠ΅Ρ ΠΌΠ΅ΡΠ½ΠΎΠ΅ ΠΏΡΠΎΡΡΡΠ°Π½ΡΡΠ²ΠΎ ΠΈΠΌΠ΅Π΅Ρ ΡΡΠΈ ΠΎΡΡΠΎΠ³ΠΎΠ½Π°Π»ΡΠ½ΡΡ Π½Π°ΠΏΡΠ°Π²Π»Π΅Π½ΠΈΡ, ΠΏΠΎΡΡΠΎΠΌΡ Π½Π°ΠΌ Π½ΡΠΆΠ½ΠΎ Π½Π΅ Π΄Π²Π°, Π° 9.
2,19
ΠΡΠ»ΠΈ ΠΈΠ·Π²Π΅ΡΡΠ½Ρ ΠΊΠΎΠΎΡΠ΄ΠΈΠ½Π°ΡΡ Π΅Π³ΠΎ Π½Π°ΡΠ°Π»Π° b(xb,yb,zb)b(xb,yb,zb) ΠΈ ΠΊΠΎΠ½ΡΠ° e(xe,ye,ze)e(xe,ye,ze), Π΅Π³ΠΎ ΡΠΊΠ°Π»ΡΡΠ½ΡΠ΅ ΠΊΠΎΠΌΠΏΠΎΠ½Π΅Π½ΡΡ ΡΠ°Π²Π½Ρ ΠΏΠΎΠ»ΡΡΠ°Π΅ΡΡΡ ΠΏΡΡΠ΅ΠΌ Π²Π·ΡΡΠΈΡ ΠΈΡ ΡΠ°Π·Π½ΠΎΡΡΠ΅ΠΉ: AxAx ΠΈ AyAy Π·Π°Π΄Π°ΡΡΡΡ ΡΡΠ°Π²Π½Π΅Π½ΠΈΠ΅ΠΌ 2.13, Π° z -ΠΊΠΎΠΌΠΏΠΎΠ½Π΅Π½ΡΠ° Π·Π°Π΄Π°Π΅ΡΡΡ
Az=ze-zb.Az=ze-zb.
2,20
ΠΠ΅Π»ΠΈΡΠΈΠ½Π° A ΠΏΠΎΠ»ΡΡΠ°Π΅ΡΡΡ ΠΏΡΡΠ΅ΠΌ ΠΎΠ±ΠΎΠ±ΡΠ΅Π½ΠΈΡ ΡΡΠ°Π²Π½Π΅Π½ΠΈΡ 2.
15 Π½Π° ΡΡΠΈ ΠΈΠ·ΠΌΠ΅ΡΠ΅Π½ΠΈΡ:
A=Ax2+Ay2+Az2.A=Ax2+Ay2+Az2.
2,21
ΠΡΠΎ Π²ΡΡΠ°ΠΆΠ΅Π½ΠΈΠ΅ Π΄Π»Ρ Π²Π΅Π»ΠΈΡΠΈΠ½Ρ Π²Π΅ΠΊΡΠΎΡΠ° ΠΏΠΎΠ»ΡΡΠ°Π΅ΡΡΡ ΠΈΠ· Π΄Π²ΠΎΠΉΠ½ΠΎΠ³ΠΎ ΠΏΡΠΈΠΌΠ΅Π½Π΅Π½ΠΈΡ ΡΠ΅ΠΎΡΠ΅ΠΌΡ ΠΠΈΡΠ°Π³ΠΎΡΠ°. ΠΠ°ΠΊ Π²ΠΈΠ΄Π½ΠΎ Π½Π° ΡΠΈΡ. 2.22, Π΄ΠΈΠ°Π³ΠΎΠ½Π°Π»Ρ Π² ΠΏΠ»ΠΎΡΠΊΠΎΡΡΠΈ xy ΠΈΠΌΠ΅Π΅Ρ Π΄Π»ΠΈΠ½Ρ Ax2+Ay2Ax2+Ay2, Π° Π΅Π΅ ΠΊΠ²Π°Π΄ΡΠ°Ρ Π² ΡΡΠΌΠΌΠ΅ Ρ ΠΊΠ²Π°Π΄ΡΠ°ΡΠΎΠΌ Az2Az2 Π΄Π°Π΅Ρ A2A2. ΠΠ±ΡΠ°ΡΠΈΡΠ΅ Π²Π½ΠΈΠΌΠ°Π½ΠΈΠ΅, ΡΡΠΎ ΠΊΠΎΠ³Π΄Π° z -ΠΊΠΎΠΌΠΏΠΎΠ½Π΅Π½ΡΠ° ΡΠ°Π²Π½Π° Π½ΡΠ»Ρ, Π²Π΅ΠΊΡΠΎΡ ΠΏΠΎΠ»Π½ΠΎΡΡΡΡ Π»Π΅ΠΆΠΈΡ Π² xy -ΠΏΠ»ΠΎΡΠΊΠΎΡΡΠΈ ΠΈ Π΅Π³ΠΎ ΠΎΠΏΠΈΡΠ°Π½ΠΈΠ΅ ΡΠ²ΠΎΠ΄ΠΈΡΡΡ ΠΊ Π΄Π²ΡΠΌ ΠΈΠ·ΠΌΠ΅ΡΠ΅Π½ΠΈΡΠΌ.
Π ΠΈΡΡΠ½ΠΎΠΊ 2,22 ΠΠ΅ΠΊΡΠΎΡ Π² ΡΡΠ΅Ρ ΠΌΠ΅ΡΠ½ΠΎΠΌ ΠΏΡΠΎΡΡΡΠ°Π½ΡΡΠ²Π΅ ΠΏΡΠ΅Π΄ΡΡΠ°Π²Π»ΡΠ΅Ρ ΡΠΎΠ±ΠΎΠΉ Π²Π΅ΠΊΡΠΎΡΠ½ΡΡ ΡΡΠΌΠΌΡ ΡΡΠ΅Ρ Π΅Π³ΠΎ Π²Π΅ΠΊΡΠΎΡΠ½ΡΡ ΠΊΠΎΠΌΠΏΠΎΠ½Π΅Π½Ρ.
ΠΡΠΈΠΌΠ΅Ρ 2,7
ΠΠ·Π»Π΅Ρ Π΄ΡΠΎΠ½Π°
ΠΡΠΈ Π²Π·Π»Π΅ΡΠ΅ IAI Heron (ΡΠΈΡ. 2.23) Π΅Π³ΠΎ ΠΏΠΎΠ»ΠΎΠΆΠ΅Π½ΠΈΠ΅ ΠΎΡΠ½ΠΎΡΠΈΡΠ΅Π»ΡΠ½ΠΎ Π΄ΠΈΡΠΏΠ΅ΡΡΠ΅ΡΡΠΊΠΎΠΉ Π²ΡΡΠΊΠΈ ΡΠΎΡΡΠ°Π²Π»ΡΠ΅Ρ 100 ΠΌ Π½Π°Π΄ Π·Π΅ΠΌΠ»Π΅ΠΉ, 300 ΠΌ Π²ΠΎΡΡΠΎΡΠ½Π΅Π΅ ΠΈ 200 ΠΌ ΡΠ΅Π²Π΅ΡΠ½Π΅Π΅. Π§Π΅ΡΠ΅Π· ΠΎΠ΄Π½Ρ ΠΌΠΈΠ½ΡΡΡ Π΅Π³ΠΎ ΠΏΠΎΠ»ΠΎΠΆΠ΅Π½ΠΈΠ΅ 250 ΠΌ Π½Π°Π΄ Π·Π΅ΠΌΠ»Π΅ΠΉ, 1200 ΠΌ Π½Π° Π²ΠΎΡΡΠΎΠΊ ΠΈ 2100 ΠΌ Π½Π° ΡΠ΅Π²Π΅Ρ.
ΠΠ°ΠΊΠΎΠ² Π²Π΅ΠΊΡΠΎΡ ΡΠΌΠ΅ΡΠ΅Π½ΠΈΡ Π΄ΡΠΎΠ½Π° ΠΎΡΠ½ΠΎΡΠΈΡΠ΅Π»ΡΠ½ΠΎ Π΄ΠΈΡΠΏΠ΅ΡΡΠ΅ΡΡΠΊΠΎΠΉ? ΠΠ°ΠΊΠΎΠ²Π° Π²Π΅Π»ΠΈΡΠΈΠ½Π° Π΅Π³ΠΎ Π²Π΅ΠΊΡΠΎΡΠ° ΡΠΌΠ΅ΡΠ΅Π½ΠΈΡ?
Π ΠΈΡΡΠ½ΠΎΠΊ 2,23 ΠΡΠΎΠ½ IAI Heron Π² ΠΏΠΎΠ»Π΅ΡΠ΅. (ΠΊΡΠ΅Π΄ΠΈΡ: ΡΠ΅ΡΠΆΠ°Π½Ρ Π Π΅ΠΉΠ½Π°Π»ΡΠ΄ΠΎ Π Π°ΠΌΠΎΠ½, ΠΠΠ‘ Π‘Π¨Π) 9, ΠΊΠΎΡΠΎΡΡΠΉ ΡΠΊΠ°Π·ΡΠ²Π°Π΅Ρ Π²Π²Π΅ΡΡ ΠΎΡ Π·Π΅ΠΌΠ»ΠΈ. ΠΠ΅ΡΠ²Π°Ρ ΠΏΠΎΠ·ΠΈΡΠΈΡ Π΄ΡΠΎΠ½Π° β ΡΡΠΎ Π½Π°ΡΠ°Π»ΠΎ (ΠΈΠ»ΠΈ, ΡΡΠΎ ΡΠΎ ΠΆΠ΅ ΡΠ°ΠΌΠΎΠ΅, Π½Π°ΡΠ°Π»ΠΎ) Π²Π΅ΠΊΡΠΎΡΠ° ΡΠΌΠ΅ΡΠ΅Π½ΠΈΡ, Π° Π΅Π³ΠΎ Π²ΡΠΎΡΠ°Ρ ΠΏΠΎΠ·ΠΈΡΠΈΡ β ΠΊΠΎΠ½Π΅Ρ Π²Π΅ΠΊΡΠΎΡΠ° ΡΠΌΠ΅ΡΠ΅Π½ΠΈΡ.
Π Π΅ΡΠ΅Π½ΠΈΠ΅
ΠΡ ΠΈΠ΄Π΅Π½ΡΠΈΡΠΈΡΠΈΡΡΠ΅ΠΌ b (300,0 ΠΌ, 200,0 ΠΌ, 100,0 ΠΌ) ΠΈ e (1200 ΠΌ, 2100 ΠΌ, 250 ΠΌ) ΠΈ ΠΈΡΠΏΠΎΠ»ΡΠ·ΡΠ΅ΠΌ ΡΡΠ°Π²Π½Π΅Π½ΠΈΡ 2.13 ΠΈ 2.20, ΡΡΠΎΠ±Ρ Π½Π°ΠΉΡΠΈ ΡΠΊΠ°Π»ΡΡΠ½ΡΠ΅ ΠΊΠΎΠΌΠΏΠΎΠ½Π΅Π½ΡΡ Π΄ΡΠΎΠ½Π°. Π²Π΅ΠΊΡΠΎΡ ΡΠΌΠ΅ΡΠ΅Π½ΠΈΡ:
{Dx=xe-xb=1200,0 ΠΌ-300,0 ΠΌ=900,0 ΠΌ,Dy=ye-yb=2100,0 ΠΌ-200,0 ΠΌ=19) ΠΌ/Ρ, ΠΊΠ°ΠΊΠΎΠ²Π° Π²Π΅Π»ΠΈΡΠΈΠ½Π° Π²Π΅ΠΊΡΠΎΡΠ° ΡΠΊΠΎΡΠΎΡΡΠΈ Π΄ΡΠΎΠ½Π°?
ΠΠ΅ΠΊΡΠΎΡ ΠΏΠΎΠ»ΠΎΠΆΠ΅Π½ΠΈΡ — ΠΎΠΏΡΠ΅Π΄Π΅Π»Π΅Π½ΠΈΠ΅, ΡΠΎΡΠΌΡΠ»Π°, ΠΏΡΠΈΠΌΠ΅ΡΡ, ΡΠ°ΡΡΠΎ Π·Π°Π΄Π°Π²Π°Π΅ΠΌΡΠ΅ Π²ΠΎΠΏΡΠΎΡΡ
ΠΠ΅ΠΊΡΠΎΡ ΠΏΠΎΠ»ΠΎΠΆΠ΅Π½ΠΈΡ ΠΈΡΠΏΠΎΠ»ΡΠ·ΡΠ΅ΡΡΡ, ΡΡΠΎΠ±Ρ ΠΏΠΎΠΌΠΎΡΡ Π½Π°ΠΌ Π½Π°ΠΉΡΠΈ ΠΏΠΎΠ»ΠΎΠΆΠ΅Π½ΠΈΠ΅ ΠΎΠ΄Π½ΠΎΠ³ΠΎ ΠΎΠ±ΡΠ΅ΠΊΡΠ° ΠΎΡΠ½ΠΎΡΠΈΡΠ΅Π»ΡΠ½ΠΎ Π΄ΡΡΠ³ΠΎΠ³ΠΎ ΠΎΠ±ΡΠ΅ΠΊΡΠ°.
ΠΠ΅ΠΊΡΠΎΡΡ ΠΏΠΎΠ»ΠΎΠΆΠ΅Π½ΠΈΡ ΠΎΠ±ΡΡΠ½ΠΎ Π½Π°ΡΠΈΠ½Π°ΡΡΡΡ Π² Π½Π°ΡΠ°Π»Π΅ ΠΊΠΎΠΎΡΠ΄ΠΈΠ½Π°Ρ, Π° Π·Π°ΡΠ΅ΠΌ Π·Π°ΠΊΠ°Π½ΡΠΈΠ²Π°ΡΡΡΡ Π² Π»ΡΠ±ΠΎΠΉ Π΄ΡΡΠ³ΠΎΠΉ ΠΏΡΠΎΠΈΠ·Π²ΠΎΠ»ΡΠ½ΠΎΠΉ ΡΠΎΡΠΊΠ΅. Π’Π°ΠΊΠΈΠΌ ΠΎΠ±ΡΠ°Π·ΠΎΠΌ, ΡΡΠΈ Π²Π΅ΠΊΡΠΎΡΡ ΠΈΡΠΏΠΎΠ»ΡΠ·ΡΡΡΡΡ Π΄Π»Ρ ΠΎΠΏΡΠ΅Π΄Π΅Π»Π΅Π½ΠΈΡ ΠΏΠΎΠ»ΠΎΠΆΠ΅Π½ΠΈΡ ΠΊΠΎΠ½ΠΊΡΠ΅ΡΠ½ΠΎΠΉ ΡΠΎΡΠΊΠΈ ΠΎΡΠ½ΠΎΡΠΈΡΠ΅Π»ΡΠ½ΠΎ Π΅Π΅ Π½Π°ΡΠ°Π»Π°.
Π ΡΡΠΎΠΉ ΡΡΠ°ΡΡΠ΅ Π΄Π°Π²Π°ΠΉΡΠ΅ ΡΠ·Π½Π°Π΅ΠΌ ΠΎ Π²Π΅ΠΊΡΠΎΡΠ°Ρ ΠΏΠΎΠ»ΠΎΠΆΠ΅Π½ΠΈΡ, ΠΈΡ ΠΎΠΏΡΠ΅Π΄Π΅Π»Π΅Π½ΠΈΠΈ, ΡΠΎΡΠΌΡΠ»Π°Ρ Ρ ΡΠ΅ΡΠ΅Π½Π½ΡΠΌΠΈ ΠΏΡΠΈΠΌΠ΅ΡΠ°ΠΌΠΈ.
| 1. | Π§ΡΠΎ ΡΠ°ΠΊΠΎΠ΅ Π²Π΅ΠΊΡΠΎΡ ΠΏΠΎΠ»ΠΎΠΆΠ΅Π½ΠΈΡ? |
| 2. | ΠΠ°ΠΊ Π½Π°ΠΉΡΠΈ Π²Π΅ΠΊΡΠΎΡ ΠΏΠΎΠ»ΠΎΠΆΠ΅Π½ΠΈΡ? |
| 3. | Π€ΠΎΡΠΌΡΠ»Π° Π²Π΅ΠΊΡΠΎΡΠ° ΠΏΠΎΠ»ΠΎΠΆΠ΅Π½ΠΈΡ |
| 4. | Π§Π°ΡΡΠΎ Π·Π°Π΄Π°Π²Π°Π΅ΠΌΡΠ΅ Π²ΠΎΠΏΡΠΎΡΡ ΠΎ Π²Π΅ΠΊΡΠΎΡΠ΅ ΠΏΠΎΠ»ΠΎΠΆΠ΅Π½ΠΈΡ |
Π§ΡΠΎ ΡΠ°ΠΊΠΎΠ΅ Π²Π΅ΠΊΡΠΎΡ ΠΏΠΎΠ»ΠΎΠΆΠ΅Π½ΠΈΡ?
ΠΠ΅ΠΊΡΠΎΡ ΠΏΠΎΠ»ΠΎΠΆΠ΅Π½ΠΈΡ ΠΏΡΠ΅Π΄ΡΡΠ°Π²Π»ΡΠ΅Ρ ΡΠΎΠ±ΠΎΠΉ ΠΏΡΡΠΌΡΡ Π»ΠΈΠ½ΠΈΡ, ΠΎΠ΄ΠΈΠ½ ΠΊΠΎΠ½Π΅Ρ ΠΊΠΎΡΠΎΡΠΎΠΉ ΠΏΡΠΈΠΊΡΠ΅ΠΏΠ»Π΅Π½ ΠΊ ΡΠ΅Π»Ρ, Π° Π΄ΡΡΠ³ΠΎΠΉ ΠΊΠΎΠ½Π΅Ρ ΠΏΡΠΈΠΊΡΠ΅ΠΏΠ»Π΅Π½ ΠΊ Π΄Π²ΠΈΠΆΡΡΠ΅ΠΉΡΡ ΡΠΎΡΠΊΠ΅, ΠΈ ΠΈΡΠΏΠΎΠ»ΡΠ·ΡΠ΅ΡΡΡ Π΄Π»Ρ ΠΎΠΏΠΈΡΠ°Π½ΠΈΡ ΠΏΠΎΠ»ΠΎΠΆΠ΅Π½ΠΈΡ ΡΠΎΡΠΊΠΈ ΠΎΡΠ½ΠΎΡΠΈΡΠ΅Π»ΡΠ½ΠΎ ΡΠ΅Π»Π°. ΠΠΎ ΠΌΠ΅ΡΠ΅ ΠΏΠ΅ΡΠ΅ΠΌΠ΅ΡΠ΅Π½ΠΈΡ ΡΠΎΡΠΊΠΈ Π²Π΅ΠΊΡΠΎΡ ΠΏΠΎΠ»ΠΎΠΆΠ΅Π½ΠΈΡ Π±ΡΠ΄Π΅Ρ ΠΈΠ·ΠΌΠ΅Π½ΡΡΡΡΡ ΠΏΠΎ Π΄Π»ΠΈΠ½Π΅ ΠΈΠ»ΠΈ ΠΏΠΎ Π½Π°ΠΏΡΠ°Π²Π»Π΅Π½ΠΈΡ, ΠΈΠ»ΠΈ ΠΏΠΎ Π΄Π»ΠΈΠ½Π΅ ΠΈ ΠΏΠΎ Π½Π°ΠΏΡΠ°Π²Π»Π΅Π½ΠΈΡ ΠΎΠ΄Π½ΠΎΠ²ΡΠ΅ΠΌΠ΅Π½Π½ΠΎ.
ΠΠΏΡΠ΅Π΄Π΅Π»Π΅Π½ΠΈΠ΅ Π²Π΅ΠΊΡΠΎΡΠ° ΠΏΠΎΠ»ΠΎΠΆΠ΅Π½ΠΈΡ
ΠΠ΅ΠΊΡΠΎΡ ΠΏΠΎΠ»ΠΎΠΆΠ΅Π½ΠΈΡ ΠΎΠΏΡΠ΅Π΄Π΅Π»ΡΠ΅ΡΡΡ ΠΊΠ°ΠΊ Π²Π΅ΠΊΡΠΎΡ, ΠΊΠΎΡΠΎΡΡΠΉ ΡΠΊΠ°Π·ΡΠ²Π°Π΅Ρ Π»ΠΈΠ±ΠΎ ΠΏΠΎΠ»ΠΎΠΆΠ΅Π½ΠΈΠ΅, Π»ΠΈΠ±ΠΎ ΠΏΠΎΠ»ΠΎΠΆΠ΅Π½ΠΈΠ΅ Π»ΡΠ±ΠΎΠΉ Π·Π°Π΄Π°Π½Π½ΠΎΠΉ ΡΠΎΡΠΊΠΈ ΠΎΡΠ½ΠΎΡΠΈΡΠ΅Π»ΡΠ½ΠΎ Π»ΡΠ±ΠΎΠΉ ΠΏΡΠΎΠΈΠ·Π²ΠΎΠ»ΡΠ½ΠΎΠΉ ΠΊΠΎΠ½ΡΡΠΎΠ»ΡΠ½ΠΎΠΉ ΡΠΎΡΠΊΠΈ, ΡΠ°ΠΊΠΎΠΉ ΠΊΠ°ΠΊ Π½Π°ΡΠ°Π»ΠΎ ΠΊΠΎΠΎΡΠ΄ΠΈΠ½Π°Ρ. ΠΠ°ΠΏΡΠ°Π²Π»Π΅Π½ΠΈΠ΅ Π²Π΅ΠΊΡΠΎΡΠ° ΠΏΠΎΠ»ΠΎΠΆΠ΅Π½ΠΈΡ Π²ΡΠ΅Π³Π΄Π° ΡΠΊΠ°Π·ΡΠ²Π°Π΅Ρ ΠΎΡ Π½Π°ΡΠ°Π»Π° ΡΡΠΎΠ³ΠΎ Π²Π΅ΠΊΡΠΎΡΠ° ΠΊ Π·Π°Π΄Π°Π½Π½ΠΎΠΉ ΡΠΎΡΠΊΠ΅.
- Π Π΄Π΅ΠΊΠ°ΡΡΠΎΠ²ΠΎΠΉ ΡΠΈΡΡΠ΅ΠΌΠ΅ ΠΊΠΎΠΎΡΠ΄ΠΈΠ½Π°Ρ, Π΅ΡΠ»ΠΈ O ΡΠ²Π»ΡΠ΅ΡΡΡ Π½Π°ΡΠ°Π»ΠΎΠΌ ΠΊΠΎΠΎΡΠ΄ΠΈΠ½Π°Ρ, Π° P(x 1 , y 1 ) ΡΠ²Π»ΡΠ΅ΡΡΡ Π΄ΡΡΠ³ΠΎΠΉ ΡΠΎΡΠΊΠΎΠΉ, ΡΠΎ Π²Π΅ΠΊΡΠΎΡ ΠΏΠΎΠ»ΠΎΠΆΠ΅Π½ΠΈΡ, Π½Π°ΠΏΡΠ°Π²Π»Π΅Π½Π½ΡΠΉ ΠΈΠ· ΡΠΎΡΠΊΠΈ O Π² ΡΠΎΡΠΊΡ P, ΠΌΠΎΠΆΠ΅Ρ Π±ΡΡΡ ΠΏΡΠ΅Π΄ΡΡΠ°Π²Π»Π΅Π½ ΠΊΠ°ΠΊ ΠΠ.
- Π ΡΡΠ΅Ρ ΠΌΠ΅ΡΠ½ΠΎΠΌ ΠΏΡΠΎΡΡΡΠ°Π½ΡΡΠ²Π΅, Π΅ΡΠ»ΠΈ Π½Π°ΡΠ°Π»ΠΎ ΠΊΠΎΠΎΡΠ΄ΠΈΠ½Π°Ρ O = (0,0,0) ΠΈ P = (x 1 , y 1 , z 1 ), ΡΠΎ Π²Π΅ΠΊΡΠΎΡ ΠΏΠΎΠ»ΠΎΠΆΠ΅Π½ΠΈΡ v ΡΠΎΡΠΊΠΈ P ΠΌΠΎΠΆΠ΅Ρ Π±ΡΡΡ ΠΏΡΠ΅Π΄ΡΡΠ°Π²Π»Π΅Π½ΠΎ ΠΊΠ°ΠΊ: v = x 1 i + y 1 j + z 1 k
Π Π°ΡΡΠΌΠΎΡΡΠΈΠΌ Π΄Π²Π° Π²Π΅ΠΊΡΠΎΡΠ° P ΠΈ Q Ρ Π²Π΅ΠΊΡΠΎΡΠ°ΠΌΠΈ ΠΏΠΎΠ»ΠΎΠΆΠ΅Π½ΠΈΡ p = (2,4) ΠΈ q = (3, 5) ΡΠΎΠΎΡΠ²Π΅ΡΡΡΠ²Π΅Π½Π½ΠΎ. ΠΠΎΠΎΡΠ΄ΠΈΠ½Π°ΡΡ Π²Π΅ΠΊΡΠΎΡΠΎΠ² P ΠΈ Q ΠΌΠΎΠΆΠ½ΠΎ Π·Π°ΠΏΠΈΡΠ°ΡΡ Π² Π²ΠΈΠ΄Π΅:Β P = (2,4), Q = (3, 5).
ΠΠ°Π²Π°ΠΉΡΠ΅ ΡΠ°ΡΡΠΌΠΎΡΡΠΈΠΌ Π½Π°ΡΠ°Π»ΠΎ ΠΊΠΎΠΎΡΠ΄ΠΈΠ½Π°Ρ O, ΠΊΠ°ΠΊ ΠΏΠΎΠΊΠ°Π·Π°Π½ΠΎ Π½Π° ΠΈΠ·ΠΎΠ±ΡΠ°ΠΆΠ΅Π½ΠΈΠΈ Π½ΠΈΠΆΠ΅. ΠΡ Π±ΡΠ΄Π΅ΠΌ ΡΠ°ΡΡΠΌΠ°ΡΡΠΈΠ²Π°ΡΡ ΡΠ°ΡΡΠΈΡΡ, ΠΊΠΎΡΠΎΡΠ°Ρ Π΄Π²ΠΈΠΆΠ΅ΡΡΡ ΠΈΠ· ΡΠΎΡΠΊΠΈ P Π² ΡΠΎΡΠΊΡ Q. ΠΠ΅ΠΊΡΠΎΡ ΠΏΠΎΠ»ΠΎΠΆΠ΅Π½ΠΈΡ ΡΠ°ΡΡΠΈΡΡ ΠΌΠΎΠΆΠ½ΠΎ ΠΎΠΏΡΠ΅Π΄Π΅Π»ΠΈΡΡ ΠΊΠ°ΠΊ Π²Π΅ΠΊΡΠΎΡ, ΠΊΠΎΡΠΎΡΡΠΉ Π½Π°ΡΠΈΠ½Π°Π΅ΡΡΡ ΠΎΡ Π½Π°ΡΠ°Π»Π° ΠΊΠΎΠΎΡΠ΄ΠΈΠ½Π°Ρ Π΄ΠΎ ΡΠΎΡΠΊΠΈ, Π² ΠΊΠΎΡΠΎΡΠΎΠΉ Π½Π°Ρ
ΠΎΠ΄ΠΈΡΡΡ ΡΠ°ΡΡΠΈΡΠ°.
ΠΠ° ΠΏΡΠΈΠ²Π΅Π΄Π΅Π½Π½ΠΎΠΉ Π²ΡΡΠ΅ Π΄ΠΈΠ°Π³ΡΠ°ΠΌΠΌΠ΅ Π²Π΅ΠΊΡΠΎΡ ΠΏΠΎΠ»ΠΎΠΆΠ΅Π½ΠΈΡ ΡΠ°ΡΡΠΈΡΡ, ΠΊΠΎΠ³Π΄Π° ΠΎΠ½Π° Π½Π°Ρ ΠΎΠ΄ΠΈΡΡΡ Π² ΡΠΎΡΠΊΠ΅ P, ΡΠ²Π»ΡΠ΅ΡΡΡ Π²Π΅ΠΊΡΠΎΡΠΎΠΌ OP, Π° ΠΊΠΎΠ³Π΄Π° ΠΎΠ½Π° Π½Π°Ρ ΠΎΠ΄ΠΈΡΡΡ Π² ΡΠΎΡΠΊΠ΅ Q, ΡΡΠΎ Π²Π΅ΠΊΡΠΎΡ OQ.
ΠΠ°ΠΊ Π½Π°ΠΉΡΠΈ Π²Π΅ΠΊΡΠΎΡ ΠΏΠΎΠ»ΠΎΠΆΠ΅Π½ΠΈΡ?
ΠΡΠ΅Π½Ρ Π²Π°ΠΆΠ½ΠΎ ΡΠ½Π°ΡΠ°Π»Π° ΠΎΠΏΡΠ΅Π΄Π΅Π»ΠΈΡΡ ΠΊΠΎΠΎΡΠ΄ΠΈΠ½Π°ΡΡ ΡΠΎΡΠΊΠΈ, ΠΏΡΠ΅ΠΆΠ΄Π΅ ΡΠ΅ΠΌ Π½Π°ΠΉΡΠΈ Π²Π΅ΠΊΡΠΎΡ ΠΏΠΎΠ»ΠΎΠΆΠ΅Π½ΠΈΡ ΡΡΠΎΠΉ ΡΠΎΡΠΊΠΈ. Π Π°ΡΡΠΌΠΎΡΡΠΈΠΌ Π΄Π²Π΅ ΡΠΎΡΠΊΠΈ, A ΠΈ B, Π³Π΄Π΅ A = (x 1 , y 1 ) ΠΈ B = (x 2 , y 2 ).
- ΠΠ°Π»Π΅Π΅ Π½Π°ΠΉΠ΄Π΅ΠΌ Π²Π΅ΠΊΡΠΎΡ ΠΏΠΎΠ»ΠΎΠΆΠ΅Π½ΠΈΡ ΠΈΠ· ΡΠΎΡΠΊΠΈ Π Π² ΡΠΎΡΠΊΡ Π, Π²Π΅ΠΊΡΠΎΡ ΠΠ.
- Π§ΡΠΎΠ±Ρ ΠΎΠΏΡΠ΅Π΄Π΅Π»ΠΈΡΡ ΡΡΠΎΡ Π²Π΅ΠΊΡΠΎΡ ΠΏΠΎΠ»ΠΎΠΆΠ΅Π½ΠΈΡ, Π½Π°ΠΌ Π½ΡΠΆΠ½ΠΎ Π²ΡΡΠ΅ΡΡΡ ΡΠΎΠΎΡΠ²Π΅ΡΡΡΠ²ΡΡΡΠΈΠ΅ ΠΊΠΎΠΌΠΏΠΎΠ½Π΅Π½ΡΡ A ΠΈΠ· B: AB = (x 2 Β — x 1 , y 2 Β — y 1 ) = (x 2 Β — x 1 ) Ρ + (Ρ 2 Β — Ρ 1 ) Ρ
Π€ΠΎΡΠΌΡΠ»Π° Π²Π΅ΠΊΡΠΎΡΠ° ΠΏΠΎΠ»ΠΎΠΆΠ΅Π½ΠΈΡ
ΠΡΠ»ΠΈ ΠΌΡ Π·Π½Π°Π΅ΠΌ ΠΏΠΎΠ»ΠΎΠΆΠ΅Π½ΠΈΠ΅ Π»ΡΠ±ΠΎΠΉ ΡΠΎΡΠΊΠΈ Π² ΠΏΠ»ΠΎΡΠΊΠΎΡΡΠΈ xy, ΡΠΎ ΠΌΡ ΠΌΠΎΠΆΠ΅ΠΌ ΠΈΡΠΏΠΎΠ»ΡΠ·ΠΎΠ²Π°ΡΡ ΡΠΎΡΠΌΡΠ»Ρ Π΄Π»Ρ ΠΎΠΏΡΠ΅Π΄Π΅Π»Π΅Π½ΠΈΡ Π²Π΅ΠΊΡΠΎΡΠ° ΠΏΠΎΠ»ΠΎΠΆΠ΅Π½ΠΈΡ ΠΌΠ΅ΠΆΠ΄Ρ ΡΡΠΈΠΌΠΈ Π΄Π²ΡΠΌΡ ΡΠΎΡΠΊΠ°ΠΌΠΈ.
ΠΠ°ΠΏΡΠΈΠΌΠ΅Ρ, ΡΠ°ΡΡΠΌΠΎΡΡΠΈΠΌ ΡΠΎΡΠΊΡ A, ΠΊΠΎΡΠΎΡΠ°Ρ ΠΈΠΌΠ΅Π΅Ρ ΠΊΠΎΠΎΡΠ΄ΠΈΠ½Π°ΡΡ (x k , y k ) Π² ΠΏΠ»ΠΎΡΠΊΠΎΡΡΠΈ xy ΠΈ Π΅ΡΠ΅ ΠΎΠ΄Π½Π° ΡΠΎΡΠΊΠ° B, ΠΈΠΌΠ΅ΡΡΠ°Ρ ΠΊΠΎΠΎΡΠ΄ΠΈΠ½Π°ΡΡ (x k+1 , y k+1 ).
- Π€ΠΎΡΠΌΡΠ»Π° Π΄Π»Ρ ΠΎΠΏΡΠ΅Π΄Π΅Π»Π΅Π½ΠΈΡ Π²Π΅ΠΊΡΠΎΡΠ° ΠΏΠΎΠ»ΠΎΠΆΠ΅Π½ΠΈΡ ΠΎΡ A Π΄ΠΎ B: AB = (x k+1 Β — x k , y k+1 Β — y k ).
- ΠΠ΅ΠΊΡΠΎΡ ΠΏΠΎΠ»ΠΎΠΆΠ΅Π½ΠΈΡ AB ΠΎΡΠ½ΠΎΡΠΈΡΡΡ ΠΊ Π²Π΅ΠΊΡΠΎΡΡ, ΠΊΠΎΡΠΎΡΡΠΉ Π½Π°ΡΠΈΠ½Π°Π΅ΡΡΡ Π² ΡΠΎΡΠΊΠ΅ A ΠΈ Π·Π°ΠΊΠ°Π½ΡΠΈΠ²Π°Π΅ΡΡΡ Π² ΡΠΎΡΠΊΠ΅ B.
- ΠΠ½Π°Π»ΠΎΠ³ΠΈΡΠ½ΠΎ, Π΅ΡΠ»ΠΈ ΠΌΡ Ρ ΠΎΡΠΈΠΌ Π½Π°ΠΉΡΠΈ Π²Π΅ΠΊΡΠΎΡ ΠΏΠΎΠ»ΠΎΠΆΠ΅Π½ΠΈΡ ΠΈΠ· ΡΠΎΡΠΊΠΈ B Π² ΡΠΎΡΠΊΡ A, ΡΠΎ ΠΌΡ ΠΌΠΎΠΆΠ΅ΠΌ ΠΈΡΠΏΠΎΠ»ΡΠ·ΠΎΠ²Π°ΡΡ: BA = (x ΠΊ Β — Ρ ΠΊ+1 , Ρ ΠΊ Β — Ρ ΠΊ+1 )
ΠΠΎΡ ΠΎΠΆΠΈΠ΅ ΡΡΠ°ΡΡΠΈ ΠΎ Π²Π΅ΠΊΡΠΎΡΠ΅ ΠΏΠΎΠ»ΠΎΠΆΠ΅Π½ΠΈΡ
ΠΠ·Π½Π°ΠΊΠΎΠΌΡΡΠ΅ΡΡ ΡΠΎ ΡΠ»Π΅Π΄ΡΡΡΠΈΠΌΠΈ ΡΡΡΠ°Π½ΠΈΡΠ°ΠΌΠΈ, ΡΠ²ΡΠ·Π°Π½Π½ΡΠΌΠΈ Ρ Π²Π΅ΠΊΡΠΎΡΠΎΠΌ ΠΏΠΎΠ»ΠΎΠΆΠ΅Π½ΠΈΡ
- ΠΠΎΠ±Π°Π²Π»Π΅Π½ΠΈΠ΅ ΠΊΠ°Π»ΡΠΊΡΠ»ΡΡΠΎΡΠ° Π²Π΅ΠΊΡΠΎΡΠΎΠ²
- ΠΠ°Π»ΡΠΊΡΠ»ΡΡΠΎΡ ΡΠ³Π»Π° ΠΌΠ΅ΠΆΠ΄Ρ Π΄Π²ΡΠΌΡ Π²Π΅ΠΊΡΠΎΡΠ°ΠΌΠΈ
- ΠΠ±ΡΠ°Π±ΠΎΡΠΊΠ° Π²Π΅ΠΊΡΠΎΡΠΎΠ², ΡΠΊΠ°Π·Π°Π½Π½ΡΡ Π² ΡΠΎΡΠΌΠ΅ i-j
- ΠΠ΅ΡΠ°Π²Π΅Π½ΡΡΠ²ΠΎ ΡΡΠ΅ΡΠ³ΠΎΠ»ΡΠ½ΠΈΠΊΠΎΠ² Π² Π²Π΅ΠΊΡΠΎΡΠ°Ρ
- ΠΡΡΠΈΡΠ°Π½ΠΈΠ΅ Π΄Π²ΡΡ Π²Π΅ΠΊΡΠΎΡΠΎΠ²
ΠΠ°ΠΆΠ½ΡΠ΅ ΠΏΡΠΈΠΌΠ΅ΡΠ°Π½ΠΈΡ ΠΏΠΎ Π²Π΅ΠΊΡΠΎΡΡ ΠΏΠΎΠ»ΠΎΠΆΠ΅Π½ΠΈΡ
ΠΠΈΠΆΠ΅ ΠΏΡΠΈΠ²Π΅Π΄Π΅Π½ ΡΠΏΠΈΡΠΎΠΊ Π½Π΅ΡΠΊΠΎΠ»ΡΠΊΠΈΡ
ΡΠΎΡΠ΅ΠΊ, ΠΊΠΎΡΠΎΡΡΠ΅ ΡΠ»Π΅Π΄ΡΠ΅Ρ ΠΏΠΎΠΌΠ½ΠΈΡΡ ΠΏΡΠΈ ΠΈΠ·ΡΡΠ΅Π½ΠΈΠΈ Π²Π΅ΠΊΡΠΎΡΠ° ΠΏΠΎΠ»ΠΎΠΆΠ΅Π½ΠΈΡ ΠΎΡΠ½ΠΎΡΠΈΡΠ΅Π»ΡΠ½ΠΎ Π»ΡΠ±ΠΎΠΉ ΠΏΡΠΎΠΈΠ·Π²ΠΎΠ»ΡΠ½ΠΎΠΉ ΡΠΎΡΠΊΠΈ ΠΎΡΡΡΠ΅ΡΠ°, ΡΠ°ΠΊΠΎΠΉ ΠΊΠ°ΠΊ Π½Π°ΡΠ°Π»ΠΎ ΠΊΠΎΠΎΡΠ΄ΠΈΠ½Π°Ρ.
Β
ΠΡΠΈΠΌΠ΅ΡΡ Π²Π΅ΠΊΡΠΎΡΠ° ΠΏΠΎΠ»ΠΎΠΆΠ΅Π½ΠΈΡ
ΠΡΠΈΠΌΠ΅Ρ 1: ΠΠ»Ρ Π΄Π²ΡΡ ΡΠΎΡΠ΅ΠΊ P = (-4, 6) ΠΈ Q = (5, 11) ΠΎΠΏΡΠ΅Π΄Π΅Π»ΠΈΡΠ΅ Π²Π΅ΠΊΡΠΎΡ ΠΏΠΎΠ»ΠΎΠΆΠ΅Π½ΠΈΡ PQ.
Π Π΅ΡΠ΅Π½ΠΈΠ΅: ΠΡΠ»ΠΈ Π² ΡΠΈΡΡΠ΅ΠΌΠ΅ ΠΊΠΎΠΎΡΠ΄ΠΈΠ½Π°Ρ xy Π·Π°Π΄Π°Π½Ρ Π΄Π²Π΅ ΡΠΎΡΠΊΠΈ, ΡΠΎ ΠΌΡ ΠΌΠΎΠΆΠ΅ΠΌ ΠΈΡΠΏΠΎΠ»ΡΠ·ΠΎΠ²Π°ΡΡ ΡΠ»Π΅Π΄ΡΡΡΡΡ ΡΠΎΡΠΌΡΠ»Ρ Π΄Π»Ρ Π½Π°Ρ ΠΎΠΆΠ΄Π΅Π½ΠΈΡ Π²Π΅ΠΊΡΠΎΡΠ° ΠΏΠΎΠ»ΠΎΠΆΠ΅Π½ΠΈΡ PQ: — y 1 )ΠΠ΄Π΅ (x 1 , y 1 ) ΠΏΡΠ΅Π΄ΡΡΠ°Π²Π»ΡΠ΅Ρ ΠΊΠΎΠΎΡΠ΄ΠΈΠ½Π°ΡΡ ΡΠΎΡΠΊΠΈ P ΠΈ (x 2 , y 2 ) ΠΏΡΠ΅Π΄ΡΡΠ°Π²Π»ΡΠ΅Ρ ΠΊΠΎΠΎΡΠ΄ΠΈΠ½Π°ΡΡ ΡΠΎΡΠΊΠΈ Q. Π’Π°ΠΊΠΈΠΌ ΠΎΠ±ΡΠ°Π·ΠΎΠΌ, ΠΏΡΠΎΡΡΠΎ ΠΏΠΎΠ΄ΡΡΠ°Π²ΠΈΠ² Π·Π½Π°ΡΠ΅Π½ΠΈΡ ΡΠΎΡΠ΅ΠΊ P ΠΈ Q Π² ΠΏΡΠΈΠ²Π΅Π΄Π΅Π½Π½ΠΎΠ΅ Π²ΡΡΠ΅ ΡΡΠ°Π²Π½Π΅Π½ΠΈΠ΅, ΠΌΡ ΠΌΠΎΠΆΠ΅ΠΌ Π½Π°ΠΉΡΠΈ Π²Π΅ΠΊΡΠΎΡ ΠΏΠΎΠ»ΠΎΠΆΠ΅Π½ΠΈΡ PQ:
PQ = (5-(-4), 11-6)
PQ = ((5+ 4 ), 11-6)
PQ = (9, 5) = 9 i + 5 j
Π’Π°ΠΊΠΈΠΌ ΠΎΠ±ΡΠ°Π·ΠΎΠΌ, Π²Π΅ΠΊΡΠΎΡ ΠΏΠΎΠ»ΠΎΠΆΠ΅Π½ΠΈΡ PQ ΡΠΊΠ²ΠΈΠ²Π°Π»Π΅Π½ΡΠ΅Π½ Π²Π΅ΠΊΡΠΎΡΡ, ΠΊΠΎΡΠΎΡΡΠΉ Π½Π°ΡΠΈΠ½Π°Π΅ΡΡΡ Π² Π½Π°ΡΠ°Π»Π΅ ΠΊΠΎΠΎΡΠ΄ΠΈΠ½Π°Ρ. ΠΡΠΎΡ Π²Π΅ΠΊΡΠΎΡ Π½Π°ΠΏΡΠ°Π²Π»Π΅Π½ Π² ΡΠΎΡΠΊΡ Π½Π° 9 Π΅Π΄ΠΈΠ½ΠΈΡ Π²ΠΏΡΠ°Π²ΠΎ ΠΏΠΎ ΠΎΡΠΈ x ΠΈ Π½Π° 5 Π΅Π΄ΠΈΠ½ΠΈΡ Π²Π²Π΅ΡΡ ΠΏΠΎ ΠΎΡΠΈ y.

ΠΡΠΈΠΌΠ΅Ρ 2: ΠΠ»Ρ Π΄Π²ΡΡ ΡΠΎΡΠ΅ΠΊ P = (-4, 6) ΠΈ Q = (5, 11) ΠΎΠΏΡΠ΅Π΄Π΅Π»ΠΈΡΠ΅ Π²Π΅ΠΊΡΠΎΡ ΠΏΠΎΠ»ΠΎΠΆΠ΅Π½ΠΈΡ QP.
Π Π΅ΡΠ΅Π½ΠΈΠ΅: ΠΡΠ»ΠΈ Π² ΡΠΈΡΡΠ΅ΠΌΠ΅ ΠΊΠΎΠΎΡΠ΄ΠΈΠ½Π°Ρ xy Π·Π°Π΄Π°Π½Ρ Π΄Π²Π΅ ΡΠΎΡΠΊΠΈ, ΡΠΎ ΠΌΡ ΠΌΠΎΠΆΠ΅ΠΌ ΠΈΡΠΏΠΎΠ»ΡΠ·ΠΎΠ²Π°ΡΡ ΡΠ»Π΅Π΄ΡΡΡΡΡ ΡΠΎΡΠΌΡΠ»Ρ Π΄Π»Ρ Π½Π°Ρ ΠΎΠΆΠ΄Π΅Π½ΠΈΡ Π²Π΅ΠΊΡΠΎΡΠ° ΠΏΠΎΠ»ΠΎΠΆΠ΅Π½ΠΈΡ QP: Β — y 2 )
ΠΠ΄Π΅ (x 1 , y 1 ) ΠΏΡΠ΅Π΄ΡΡΠ°Π²Π»ΡΠ΅Ρ ΠΊΠΎΠΎΡΠ΄ΠΈΠ½Π°ΡΡ ΡΠΎΡΠΊΠΈ P ΠΈ (x 2 , y 2 ) ΠΏΡΠ΅Π΄ΡΡΠ°Π²Π»ΡΠ΅Ρ ΠΊΠΎΠΎΡΠ΄ΠΈΠ½Π°ΡΡ ΡΠΎΡΠΊΠΈ Q. ΠΠ±ΡΠ°ΡΠΈΡΠ΅ Π²Π½ΠΈΠΌΠ°Π½ΠΈΠ΅, ΡΡΠΎ Π²Π΅ΠΊΡΠΎΡ ΠΏΠΎΠ»ΠΎΠΆΠ΅Π½ΠΈΡ QP ΠΏΡΠ΅Π΄ΡΡΠ°Π²Π»ΡΠ΅Ρ ΡΠΎΠ±ΠΎΠΉ Π²Π΅ΠΊΡΠΎΡ, Π½Π°ΠΏΡΠ°Π²Π»Π΅Π½Π½ΡΠΉ ΠΎΡ ΡΠΎΡΠΊΠΈ Q ΠΊ ΡΠΎΡΠΊΠ΅ P. ΠΠ½ ΠΎΡΠ»ΠΈΡΠ°Π΅ΡΡΡ ΠΎΡ Π²Π΅ΠΊΡΠΎΡΠ° ΠΏΠΎΠ»ΠΎΠΆΠ΅Π½ΠΈΡ PQ, ΠΊΠΎΡΠΎΡΡΠΉ Π½Π°ΠΏΡΠ°Π²Π»Π΅Π½ ΠΎΡ P ΠΊ Q. Π’Π°ΠΊΠΈΠΌ ΠΎΠ±ΡΠ°Π·ΠΎΠΌ, ΠΏΡΠΎΡΡΠΎ ΠΏΠΎΠ΄ΡΡΠ°Π²ΠΈΠ² Π·Π½Π°ΡΠ΅Π½ΠΈΡ ΡΠΎΡΠ΅ΠΊ P ΠΈ Q Π² ΠΏΡΠΈΠ²Π΅Π΄Π΅Π½Π½ΠΎΠ΅ Π²ΡΡΠ΅ ΡΡΠ°Π²Π½Π΅Π½ΠΈΠ΅ , ΠΌΡ ΠΌΠΎΠΆΠ΅ΠΌ Π½Π°ΠΉΡΠΈ Π²Π΅ΠΊΡΠΎΡ ΠΏΠΎΠ»ΠΎΠΆΠ΅Π½ΠΈΡ QP:
QP = (-4-5, 6-11)
QP = (-9, -5) = -9 i — 5 j
Π’Π°ΠΊΠΈΠΌ ΠΎΠ±ΡΠ°Π·ΠΎΠΌ, Π²Π΅ΠΊΡΠΎΡ ΠΏΠΎΠ»ΠΎΠΆΠ΅Π½ΠΈΡ QP ΡΠ°Π²Π΅Π½ ΡΠΊΠ²ΠΈΠ²Π°Π»Π΅Π½ΡΠ΅Π½ Π²Π΅ΠΊΡΠΎΡΡ, ΠΊΠΎΡΠΎΡΡΠΉ Π½Π°ΡΠΈΠ½Π°Π΅ΡΡΡ Π² Π½Π°ΡΠ°Π»Π΅ ΠΊΠΎΠΎΡΠ΄ΠΈΠ½Π°Ρ. ΠΡΠΎΡ Π²Π΅ΠΊΡΠΎΡ Π½Π°ΠΏΡΠ°Π²Π»Π΅Π½ Π² ΡΠΎΡΠΊΡ 9Π΅Π΄ΠΈΠ½ΠΈΡ Π²Π»Π΅Π²ΠΎ ΠΏΠΎ ΠΎΡΠΈ x ΠΈ Π½Π° 5 Π΅Π΄ΠΈΠ½ΠΈΡ Π²Π½ΠΈΠ· ΠΏΠΎ ΠΎΡΠΈ y.

ΠΏΠ΅ΡΠ΅ΠΉΡΠΈ ΠΊ ΡΠ»Π°ΠΉΠ΄ΡΠΏΠ΅ΡΠ΅ΠΉΡΠΈ ΠΊ ΡΠ»Π°ΠΉΠ΄Ρ
ΠΡΠ»ΠΈΡΠ½ΠΎΠ΅ ΠΎΠ±ΡΡΠ΅Π½ΠΈΠ΅ Π² ΡΡΠ°ΡΡΠ΅ΠΉ ΡΠΊΠΎΠ»Π΅ Ρ ΠΈΡΠΏΠΎΠ»ΡΠ·ΠΎΠ²Π°Π½ΠΈΠ΅ΠΌ ΠΏΡΠΎΡΡΡΡ ΠΏΠΎΠ΄ΡΠΊΠ°Π·ΠΎΠΊ
Π£Π²Π»Π΅ΠΊΠ°ΡΡΡ Π·ΡΠ±ΡΠ΅ΠΆΠΊΠΎΠΉ, Π²Ρ, ΡΠΊΠΎΡΠ΅Π΅ Π²ΡΠ΅Π³ΠΎ, Π·Π°Π±ΡΠ΄Π΅ΡΠ΅ ΠΏΠΎΠ½ΡΡΠΈΡ. Π‘ Cuemath Π²Ρ Π±ΡΠ΄Π΅ΡΠ΅ ΡΡΠΈΡΡΡΡ Π²ΠΈΠ·ΡΠ°Π»ΡΠ½ΠΎ ΠΈ Π±ΡΠ΄Π΅ΡΠ΅ ΡΠ΄ΠΈΠ²Π»Π΅Π½Ρ ΡΠ΅Π·ΡΠ»ΡΡΠ°ΡΠ°ΠΌΠΈ.
ΠΠ°ΠΏΠΈΡΠ°ΡΡΡΡ Π½Π° Π±Π΅ΡΠΏΠ»Π°ΡΠ½ΡΠΉ ΠΏΡΠΎΠ±Π½ΡΠΉ ΡΡΠΎΠΊ
ΠΡΠ°ΠΊΡΠΈΡΠ΅ΡΠΊΠΈΠ΅ Π²ΠΎΠΏΡΠΎΡΡ ΠΏΠΎ ΠΏΠΎΠ·ΠΈΡΠΈΠΎΠ½Π½ΠΎΠΌΡ Π²Π΅ΠΊΡΠΎΡΡ
Β
ΠΏΠ΅ΡΠ΅ΠΉΡΠΈ ΠΊ ΡΠ»Π°ΠΉΠ΄ΡΠΏΠ΅ΡΠ΅ΠΉΡΠΈ ΠΊ ΡΠ»Π°ΠΉΠ΄Ρ
Π§Π°ΡΡΠΎ Π·Π°Π΄Π°Π²Π°Π΅ΠΌΡΠ΅ Π²ΠΎΠΏΡΠΎΡΡ ΠΎ Π²Π΅ΠΊΡΠΎΡΠ΅ ΠΏΠΎΠ»ΠΎΠΆΠ΅Π½ΠΈΡ
Π§ΡΠΎ ΡΠ°ΠΊΠΎΠ΅
Π²Π΅ΠΊΡΠΎΡ ΠΏΠΎΠ»ΠΎΠΆΠ΅Π½ΠΈΡ Π² ΠΌΠ°ΡΠ΅ΠΌΠ°ΡΠΈΠΊΠ΅?ΠΠ΅ΠΊΡΠΎΡ ΠΏΠΎΠ»ΠΎΠΆΠ΅Π½ΠΈΡ ΠΏΡΠ΅Π΄ΡΡΠ°Π²Π»ΡΠ΅Ρ ΡΠΎΠ±ΠΎΠΉ ΠΏΡΡΠΌΡΡ Π»ΠΈΠ½ΠΈΡ, ΠΎΠ΄ΠΈΠ½ ΠΊΠΎΠ½Π΅Ρ ΠΊΠΎΡΠΎΡΠΎΠΉ ΠΏΡΠΈΠΊΡΠ΅ΠΏΠ»Π΅Π½ ΠΊ ΡΠ΅Π»Ρ, Π° Π΄ΡΡΠ³ΠΎΠΉ ΠΊΠΎΠ½Π΅Ρ ΠΏΡΠΈΠΊΡΠ΅ΠΏΠ»Π΅Π½ ΠΊ Π΄Π²ΠΈΠΆΡΡΠ΅ΠΉΡΡ ΡΠΎΡΠΊΠ΅, ΠΈ ΠΈΡΠΏΠΎΠ»ΡΠ·ΡΠ΅ΡΡΡ Π΄Π»Ρ ΠΎΠΏΠΈΡΠ°Π½ΠΈΡ ΠΏΠΎΠ»ΠΎΠΆΠ΅Π½ΠΈΡ ΡΠΎΡΠΊΠΈ ΠΎΡΠ½ΠΎΡΠΈΡΠ΅Π»ΡΠ½ΠΎ ΡΠ΅Π»Π°. ΠΠ°ΠΏΡΠ°Π²Π»Π΅Π½ΠΈΠ΅ Π²Π΅ΠΊΡΠΎΡΠ° ΠΏΠΎΠ»ΠΎΠΆΠ΅Π½ΠΈΡ Π²ΡΠ΅Π³Π΄Π° ΡΠΊΠ°Π·ΡΠ²Π°Π΅Ρ ΠΎΡ Π½Π°ΡΠ°Π»Π° ΡΡΠΎΠ³ΠΎ Π²Π΅ΠΊΡΠΎΡΠ° ΠΊ Π·Π°Π΄Π°Π½Π½ΠΎΠΉ ΡΠΎΡΠΊΠ΅.
ΠΠ°ΠΊ Π½Π°ΠΉΡΠΈ Π²Π΅ΠΊΡΠΎΡ ΠΏΠΎΠ»ΠΎΠΆΠ΅Π½ΠΈΡ?
ΠΡΠ΅Π½Ρ Π²Π°ΠΆΠ½ΠΎ ΡΠ½Π°ΡΠ°Π»Π° ΠΎΠΏΡΠ΅Π΄Π΅Π»ΠΈΡΡ ΠΊΠΎΠΎΡΠ΄ΠΈΠ½Π°ΡΡ ΡΠΎΡΠΊΠΈ, ΠΏΡΠ΅ΠΆΠ΄Π΅ ΡΠ΅ΠΌ Π½Π°ΠΉΡΠΈ Π²Π΅ΠΊΡΠΎΡ ΠΏΠΎΠ»ΠΎΠΆΠ΅Π½ΠΈΡ ΡΡΠΎΠΉ ΡΠΎΡΠΊΠΈ.
Π Π°ΡΡΠΌΠΎΡΡΠΈΠΌ Π΄Π²Π΅ ΡΠΎΡΠΊΠΈ, A ΠΈ B, Π³Π΄Π΅ A = (x 1 , y 1 ) ΠΈ B = (x 2 , y 2 ).
- ΠΠ°Π»Π΅Π΅ Π½Π°ΠΉΠ΄Π΅ΠΌ Π²Π΅ΠΊΡΠΎΡ ΠΏΠΎΠ»ΠΎΠΆΠ΅Π½ΠΈΡ ΠΈΠ· ΡΠΎΡΠΊΠΈ Π Π² ΡΠΎΡΠΊΡ Π, Π²Π΅ΠΊΡΠΎΡ ΠΠ.
- Π§ΡΠΎΠ±Ρ ΠΎΠΏΡΠ΅Π΄Π΅Π»ΠΈΡΡ ΡΡΠΎΡ Π²Π΅ΠΊΡΠΎΡ ΠΏΠΎΠ»ΠΎΠΆΠ΅Π½ΠΈΡ, Π½Π°ΠΌ Π½ΡΠΆΠ½ΠΎ Π²ΡΡΠ΅ΡΡΡ ΡΠΎΠΎΡΠ²Π΅ΡΡΡΠ²ΡΡΡΠΈΠ΅ ΠΊΠΎΠΌΠΏΠΎΠ½Π΅Π½ΡΡ A ΠΈΠ· B: AB = (x 2 Β — x 1 , y 2 Β — Ρ 1 )
Π ΡΠ΅ΠΌ ΡΠ°Π·Π½ΠΈΡΠ° ΠΌΠ΅ΠΆΠ΄Ρ Π²Π΅ΠΊΡΠΎΡΠΎΠΌ ΠΏΠΎΠ»ΠΎΠΆΠ΅Π½ΠΈΡ ΠΈ Π²Π΅ΠΊΡΠΎΡΠΎΠΌ ΡΠΌΠ΅ΡΠ΅Π½ΠΈΡ?
ΠΠ΅ΠΊΡΠΎΡ ΠΏΠΎΠ»ΠΎΠΆΠ΅Π½ΠΈΡ ΠΎΠΏΡΠ΅Π΄Π΅Π»ΡΠ΅ΡΡΡ ΠΊΠ°ΠΊ Π²Π΅ΠΊΡΠΎΡ, ΡΠΊΠ°Π·ΡΠ²Π°ΡΡΠΈΠΉ Π»ΠΈΠ±ΠΎ ΠΏΠΎΠ»ΠΎΠΆΠ΅Π½ΠΈΠ΅, Π»ΠΈΠ±ΠΎ ΠΌΠ΅ΡΡΠΎΠΏΠΎΠ»ΠΎΠΆΠ΅Π½ΠΈΠ΅ Π»ΡΠ±ΠΎΠΉ Π·Π°Π΄Π°Π½Π½ΠΎΠΉ ΡΠΎΡΠΊΠΈ ΠΎΡΠ½ΠΎΡΠΈΡΠ΅Π»ΡΠ½ΠΎ Π»ΡΠ±ΠΎΠΉ ΠΏΡΠΎΠΈΠ·Π²ΠΎΠ»ΡΠ½ΠΎΠΉ ΠΊΠΎΠ½ΡΡΠΎΠ»ΡΠ½ΠΎΠΉ ΡΠΎΡΠΊΠΈ, ΡΠ°ΠΊΠΎΠΉ ΠΊΠ°ΠΊ Π½Π°ΡΠ°Π»ΠΎ ΠΊΠΎΠΎΡΠ΄ΠΈΠ½Π°Ρ. ΠΡΠΈΠ½ΠΈΠΌΠ°Ρ Π²ΠΎ Π²Π½ΠΈΠΌΠ°Π½ΠΈΠ΅, ΡΡΠΎ Π²Π΅ΠΊΡΠΎΡ ΡΠΌΠ΅ΡΠ΅Π½ΠΈΡ ΠΏΠΎΠΌΠΎΠ³Π°Π΅Ρ Π½Π°ΠΌ Π½Π°ΠΉΡΠΈ ΠΈΠ·ΠΌΠ΅Π½Π΅Π½ΠΈΠ΅ Π²Π΅ΠΊΡΠΎΡΠ° ΠΏΠΎΠ»ΠΎΠΆΠ΅Π½ΠΈΡ Π΄Π°Π½Π½ΠΎΠ³ΠΎ ΠΎΠ±ΡΠ΅ΠΊΡΠ°.
ΠΠ΄Π΅ Π²ΡΠ΅Π³Π΄Π° Π½Π°ΡΠΈΠ½Π°Π΅ΡΡΡ Π²Π΅ΠΊΡΠΎΡ ΠΏΠΎΠ»ΠΎΠΆΠ΅Π½ΠΈΡ?
ΠΠ΅ΠΊΡΠΎΡ ΠΏΠΎΠ»ΠΎΠΆΠ΅Π½ΠΈΡ Π½Π°ΡΠΈΠ½Π°Π΅ΡΡΡ Π² Π½Π°ΡΠ°Π»Π΅ ΠΊΠΎΠΎΡΠ΄ΠΈΠ½Π°Ρ ΠΈ Π·Π°ΠΊΠ°Π½ΡΠΈΠ²Π°Π΅ΡΡΡ Π² Π»ΡΠ±ΠΎΠΉ Π΄ΡΡΠ³ΠΎΠΉ ΠΏΡΠΎΠΈΠ·Π²ΠΎΠ»ΡΠ½ΠΎΠΉ ΡΠΎΡΠΊΠ΅. ΠΠ½ΠΈ ΠΈΡΠΏΠΎΠ»ΡΠ·ΡΡΡΡΡ Π΄Π»Ρ ΠΎΠΏΡΠ΅Π΄Π΅Π»Π΅Π½ΠΈΡ ΠΏΠΎΠ»ΠΎΠΆΠ΅Π½ΠΈΡ ΡΠΎΡΠΊΠΈ ΠΎΡΠ½ΠΎΡΠΈΡΠ΅Π»ΡΠ½ΠΎ Π½Π°ΡΠ°Π»Π° ΠΊΠΎΠΎΡΠ΄ΠΈΠ½Π°Ρ.
Π ΡΠ΅ΠΌ ΡΠ°Π·Π½ΠΈΡΠ° ΠΌΠ΅ΠΆΠ΄Ρ Π²Π΅ΠΊΡΠΎΡΠΎΠΌ ΠΏΠΎΠ»ΠΎΠΆΠ΅Π½ΠΈΡ ΠΈ Π΅Π΄ΠΈΠ½ΠΈΡΠ½ΡΠΌ Π²Π΅ΠΊΡΠΎΡΠΎΠΌ?
ΠΠ΅ΠΊΡΠΎΡ ΡΡΠΈΡΠ°Π΅ΡΡΡ Π΅Π΄ΠΈΠ½ΠΈΡΠ½ΡΠΌ Π²Π΅ΠΊΡΠΎΡΠΎΠΌ, Π΅ΡΠ»ΠΈ ΠΎΠ½ ΠΈΡΠΏΠΎΠ»ΡΠ·ΡΠ΅ΡΡΡ Π΄Π»Ρ ΡΠΊΠ°Π·Π°Π½ΠΈΡ ΡΠΎΠ»ΡΠΊΠΎ Π½Π°ΠΏΡΠ°Π²Π»Π΅Π½ΠΈΡ ΠΈ ΠΈΠΌΠ΅Π΅Ρ Π²Π΅Π»ΠΈΡΠΈΠ½Ρ, ΡΠ°Π²Π½ΡΡ 1. ΠΠ»Ρ Π½Π°ΠΏΡΠ°Π²Π»Π΅Π½ΠΈΡ Π½Π΅ ΡΡΠ΅Π±ΡΠ΅ΡΡΡ Π²Π΅Π»ΠΈΡΠΈΠ½Π°, ΠΏΠΎΡΡΠΎΠΌΡ Π²Π΅Π»ΠΈΡΠΈΠ½Π° Π΅Π΄ΠΈΠ½ΠΈΡΠ½ΠΎΠ³ΠΎ Π²Π΅ΠΊΡΠΎΡΠ° Π²ΡΠ΅Π³Π΄Π° ΡΠ°Π²Π½Π° 1. A Π²Π΅ΠΊΡΠΎΡ ΠΏΠΎΠ»ΠΎΠΆΠ΅Π½ΠΈΡ ΠΎΠΏΡΠ΅Π΄Π΅Π»ΡΠ΅ΡΡΡ ΠΊΠ°ΠΊ Π²Π΅ΠΊΡΠΎΡ, ΠΊΠΎΡΠΎΡΡΠΉ ΡΠΊΠ°Π·ΡΠ²Π°Π΅Ρ Π»ΠΈΠ±ΠΎ ΠΏΠΎΠ»ΠΎΠΆΠ΅Π½ΠΈΠ΅, Π»ΠΈΠ±ΠΎ ΠΌΠ΅ΡΡΠΎΠΏΠΎΠ»ΠΎΠΆΠ΅Π½ΠΈΠ΅ Π»ΡΠ±ΠΎΠΉ Π·Π°Π΄Π°Π½Π½ΠΎΠΉ ΡΠΎΡΠΊΠΈ ΠΎΡΠ½ΠΎΡΠΈΡΠ΅Π»ΡΠ½ΠΎ Π»ΡΠ±ΠΎΠΉ ΠΏΡΠΎΠΈΠ·Π²ΠΎΠ»ΡΠ½ΠΎΠΉ ΠΎΠΏΠΎΡΠ½ΠΎΠΉ ΡΠΎΡΠΊΠΈ, ΡΠ°ΠΊΠΎΠΉ ΠΊΠ°ΠΊ Π½Π°ΡΠ°Π»ΠΎ ΠΊΠΎΠΎΡΠ΄ΠΈΠ½Π°Ρ.
ΠΠ°ΠΊΠ°Ρ ΡΠΎΡΠΌΡΠ»Π° Π΄Π»Ρ Π²Π΅ΠΊΡΠΎΡΠ° ΠΏΠΎΠ»ΠΎΠΆΠ΅Π½ΠΈΡ?
ΠΡΠ»ΠΈ ΠΌΡ Π·Π½Π°Π΅ΠΌ ΠΏΠΎΠ»ΠΎΠΆΠ΅Π½ΠΈΠ΅ Π»ΡΠ±ΠΎΠΉ ΡΠΎΡΠΊΠΈ Π² ΠΏΠ»ΠΎΡΠΊΠΎΡΡΠΈ xy, ΡΠΎ ΠΌΡ ΠΌΠΎΠΆΠ΅ΠΌ ΠΈΡΠΏΠΎΠ»ΡΠ·ΠΎΠ²Π°ΡΡ ΡΠΎΡΠΌΡΠ»Ρ Π΄Π»Ρ ΠΎΠΏΡΠ΅Π΄Π΅Π»Π΅Π½ΠΈΡ Π²Π΅ΠΊΡΠΎΡΠ° ΠΏΠΎΠ»ΠΎΠΆΠ΅Π½ΠΈΡ ΠΌΠ΅ΠΆΠ΄Ρ ΡΡΠΈΠΌΠΈ Π΄Π²ΡΠΌΡ ΡΠΎΡΠΊΠ°ΠΌΠΈ. ΠΠ°ΠΏΡΠΈΠΌΠ΅Ρ, ΡΠ°ΡΡΠΌΠΎΡΡΠΈΠΌ ΡΠΎΡΠΊΡ A Ρ ΠΊΠΎΠΎΡΠ΄ΠΈΠ½Π°ΡΠ°ΠΌΠΈ (x k , y k ) Π² ΠΏΠ»ΠΎΡΠΊΠΎΡΡΠΈ xy ΠΈ Π΄ΡΡΠ³ΡΡ ΡΠΎΡΠΊΡ B Ρ ΠΊΠΎΠΎΡΠ΄ΠΈΠ½Π°ΡΠ°ΠΌΠΈ (x k+1 , y k+ 1 ). Π€ΠΎΡΠΌΡΠ»Π° Π΄Π»Ρ ΠΎΠΏΡΠ΅Π΄Π΅Π»Π΅Π½ΠΈΡ Π²Π΅ΠΊΡΠΎΡΠ° ΠΏΠΎΠ»ΠΎΠΆΠ΅Π½ΠΈΡ ΠΎΡ A Π΄ΠΎ B: AB = (x k+1 Β — x k , y ΠΊ+1 Β — Ρ ΠΊ ).
Π―Π²Π»ΡΠ΅ΡΡΡ Π»ΠΈ ΡΠΌΠ΅ΡΠ΅Π½ΠΈΠ΅ Π²Π΅ΠΊΡΠΎΡΠΎΠΌ ΠΏΠΎΠ»ΠΎΠΆΠ΅Π½ΠΈΡ?
ΠΠ΅Ρ, ΡΠΌΠ΅ΡΠ΅Π½ΠΈΠ΅ Π½Π΅ ΡΠ²Π»ΡΠ΅ΡΡΡ Π²Π΅ΠΊΡΠΎΡΠΎΠΌ ΠΏΠΎΠ»ΠΎΠΆΠ΅Π½ΠΈΡ. ΠΠ΅ΠΊΡΠΎΡ ΠΏΠΎΠ»ΠΎΠΆΠ΅Π½ΠΈΡ ΠΈΡΠΏΠΎΠ»ΡΠ·ΡΠ΅ΡΡΡ, ΡΡΠΎΠ±Ρ ΠΏΠΎΠΌΠΎΡΡ Π½Π°ΠΌ Π½Π°ΠΉΡΠΈ ΠΌΠ΅ΡΡΠΎΠΏΠΎΠ»ΠΎΠΆΠ΅Π½ΠΈΠ΅ ΠΎΠ΄Π½ΠΎΠ³ΠΎ ΠΎΠ±ΡΠ΅ΠΊΡΠ° ΠΎΡΠ½ΠΎΡΠΈΡΠ΅Π»ΡΠ½ΠΎ Π΄ΡΡΠ³ΠΎΠ³ΠΎ ΠΎΠ±ΡΠ΅ΠΊΡΠ°. ΠΠ°ΠΆΠ½ΠΎ Π·Π½Π°ΡΡ ΠΏΠΎΠ»ΠΎΠΆΠ΅Π½ΠΈΠ΅ ΡΠ΅Π»Π°, ΠΊΠΎΠ³Π΄Π° ΠΌΡ ΠΎΠΏΠΈΡΡΠ²Π°Π΅ΠΌ Π΅Π³ΠΎ Π΄Π²ΠΈΠΆΠ΅Π½ΠΈΠ΅. ΠΠΎ ΡΠΎΠ³Π΄Π° Π²Π΅ΠΊΡΠΎΡ ΡΠΌΠ΅ΡΠ΅Π½ΠΈΡ ΠΌΠΎΠΆΠ½ΠΎ ΠΎΠΏΡΠ΅Π΄Π΅Π»ΠΈΡΡ ΠΊΠ°ΠΊ ΠΈΠ·ΠΌΠ΅Π½Π΅Π½ΠΈΠ΅ ΠΈΠ»ΠΈ Π²Π°ΡΠΈΠ°ΡΠΈΡ Π·Π°Π΄Π°Π½Π½ΠΎΠ³ΠΎ Π²Π΅ΠΊΡΠΎΡΠ° ΠΏΠΎΠ»ΠΎΠΆΠ΅Π½ΠΈΡ.
ΠΠ·ΠΌΠ΅Π½Π΅Π½ΠΈΠ΅ Π±Π°Π·ΠΈΡΠ° β Π£ΡΠ΅Π±Π½ΡΠ΅ ΠΏΠΎΡΠΎΠ±ΠΈΡ ΠΏΠΎ ΡΠ°ΡΡΠ΅ΡΡ
ΠΠ·ΠΌΠ΅Π½Π΅Π½ΠΈΠ΅ Π±Π°Π·ΠΈΡΠ° β Π£ΡΠ΅Π±Π½ΠΎΠ΅ ΠΏΠΎΡΠΎΠ±ΠΈΠ΅ ΠΏΠΎ ΡΠ°ΡΡΠ΅ΡΡ HMCΠΡΡΡΡ $V$ β Π²Π΅ΠΊΡΠΎΡΠ½ΠΎΠ΅ ΠΏΡΠΎΡΡΡΠ°Π½ΡΡΠ²ΠΎ, ΠΈ ΠΏΡΡΡΡ $S = \{{\bf v_1,v_2, \ldots, v_n}\}$ β ΠΌΠ½ΠΎΠΆΠ΅ΡΡΠ²ΠΎ Π²Π΅ΠΊΡΠΎΡΠΎΠ² Π² $V$. ΠΠ°ΠΏΠΎΠΌΠ½ΠΈΠΌ, ΡΡΠΎ $S$ ΠΎΠ±ΡΠ°Π·ΡΠ΅Ρ Π±Π°Π·ΠΈΡ Π΄Π»Ρ $V$, Π΅ΡΠ»ΠΈ Π²ΡΠΏΠΎΠ»Π½ΡΡΡΡΡ ΡΠ»Π΅Π΄ΡΡΡΠΈΠ΅ Π΄Π²Π° ΡΡΠ»ΠΎΠ²ΠΈΡ: ΠΏΡΡΡΡ $V$ β Π²Π΅ΠΊΡΠΎΡΠ½ΠΎΠ΅ ΠΏΡΠΎΡΡΡΠ°Π½ΡΡΠ²ΠΎ ΠΈ ΠΏΡΡΡΡ $S = \{{\bf v_1,v_2, \ldots, v_n}\} $ β Π½Π°Π±ΠΎΡ Π²Π΅ΠΊΡΠΎΡΠΎΠ² Π² $V$. ΠΠ°ΠΏΠΎΠΌΠ½ΠΈΠΌ, ΡΡΠΎ $S$ ΠΎΠ±ΡΠ°Π·ΡΠ΅Ρ Π±Π°Π·ΠΈΡ Π΄Π»Ρ $V$, Π΅ΡΠ»ΠΈ Π²ΡΠΏΠΎΠ»Π½ΡΡΡΡΡ ΡΠ»Π΅Π΄ΡΡΡΠΈΠ΅ Π΄Π²Π° ΡΡΠ»ΠΎΠ²ΠΈΡ:
ΠΡΠ»ΠΈ $S = \{{\bf v_1,v_2, \ldots, v_n}\}$ ΡΠ²Π»ΡΠ΅ΡΡΡ Π±Π°Π·ΠΈΡΠΎΠΌ Π΄Π»Ρ $V $, ΡΠΎ ΠΊΠ°ΠΆΠ΄ΡΠΉ Π²Π΅ΠΊΡΠΎΡ ${\bf v} \in V$ ΠΌΠΎΠΆΠ΅Ρ Π±ΡΡΡ Π²ΡΡΠ°ΠΆΠ΅Π½ ΠΎΠ΄Π½ΠΎΠ·Π½Π°ΡΠ½ΠΎ ΠΊΠ°ΠΊ Π»ΠΈΠ½Π΅ΠΉΠ½Π°Ρ ΠΊΠΎΠΌΠ±ΠΈΠ½Π°ΡΠΈΡ ${\bf v_1,v_2, \ldots, v_n}$: $$ {\bf v} = c_1{\bf v_1} + c_2{\bf v_2} + \cdots + c_n{ \bf v_n}.
$$ ΠΡΠΌΠ°ΠΉΡΠ΅ ΠΎ $\left[\begin{array}{c} c_1\\ c_2\\ \vdots \\ c_n \end{array}\right]$ ΠΊΠ°ΠΊ ΠΎ ΠΊΠΎΠΎΡΠ΄ΠΈΠ½Π°ΡΠ°Ρ
ΠΎΡΠ½ΠΎΡΠΈΡΠ΅Π»ΡΠ½ΠΎΠ³ΠΎ ${\bf v}$ ΠΊ Π±Π°Π·Π΅ $S$. ΠΡΠ»ΠΈ $V$ ΠΈΠΌΠ΅Π΅Ρ ΡΠ°Π·ΠΌΠ΅ΡΠ½ΠΎΡΡΡ , ΡΡΠΎ ΠΊΠΎΠ»ΠΈΡΠ΅ΡΡΠ²ΠΎ Π²Π΅ΠΊΡΠΎΡΠΎΠ², Π½Π΅ΠΎΠ±Ρ
ΠΎΠ΄ΠΈΠΌΡΡ
Π΄Π»Ρ ΡΠΎΡΠΌΠΈΡΠΎΠ²Π°Π½ΠΈΡ ΠΎΡΠ½ΠΎΠ²Ρ. $n$, ΡΠΎ ΠΊΠ°ΠΆΠ΄ΡΠΉ Π½Π°Π±ΠΎΡ ΠΈΠ· $n$ Π»ΠΈΠ½Π΅ΠΉΠ½ΠΎ Π½Π΅Π·Π°Π²ΠΈΡΠΈΠΌΡΡ
Π²Π΅ΠΊΡΠΎΡΠΎΠ² Π² $V$ ΠΎΠ±ΡΠ°Π·ΡΠ΅Ρ Π±Π°Π·ΠΈΡ Π΄Π»Ρ $V$. Π ΠΊΠ°ΠΆΠ΄ΠΎΠΌ ΠΏΡΠΈΠ»ΠΎΠΆΠ΅Π½ΠΈΠΈ Ρ Π½Π°Ρ Π΅ΡΡΡ Π²ΡΠ±ΠΎΡ, ΠΊΠ°ΠΊΡΡ ΠΎΡΠ½ΠΎΠ²Ρ ΠΌΡ ΠΈΡΠΏΠΎΠ»ΡΠ·ΡΠ΅ΠΌ. Π ΡΡΠΎΠΌ ΡΡΠΎΠΊΠ΅ ΠΌΡ ΠΎΠΏΠΈΡΠ΅ΠΌ ΠΏΡΠ΅ΠΎΠ±ΡΠ°Π·ΠΎΠ²Π°Π½ΠΈΠ΅ ΠΊΠΎΠΎΡΠ΄ΠΈΠ½Π°Ρ Π²Π΅ΠΊΡΠΎΡΠΎΠ² ΠΏΡΠΈ ΡΠΌΠ΅Π½Π΅ Π±Π°Π·ΠΈΡΠ°. 92$. ΠΠ»Ρ Π²Π΅ΠΊΡΠΎΡΠ° ${\bf v} \in V$ ΠΏΠΎ Π·Π°Π΄Π°Π½Π½ΡΠΌ Π΅Π³ΠΎ ΠΊΠΎΠΎΡΠ΄ΠΈΠ½Π°ΡΠ°ΠΌ $[{\bf
v}]_B$ Π² Π±Π°Π·ΠΈΡΠ΅ $B$ ΠΌΡ Ρ
ΠΎΡΠ΅Π»ΠΈ Π±Ρ ΠΈΠΌΠ΅ΡΡ Π²ΠΎΠ·ΠΌΠΎΠΆΠ½ΠΎΡΡΡ Π²ΡΡΠ°Π·ΠΈΡΡ ${\bf v}$ Π²
Π΅Π³ΠΎ ΠΊΠΎΠΎΡΠ΄ΠΈΠ½Π°Ρ $[{\bf v}]_{Bβ}$ Π² Π±Π°Π·ΠΈΡΠ΅ $Bβ$, ΠΈ Π½Π°ΠΎΠ±ΠΎΡΠΎΡ.
ΠΡΠ΅Π΄ΠΏΠΎΠ»ΠΎΠΆΠΈΠΌ, ΡΡΠΎ Π±Π°Π·ΠΈΡΠ½ΡΠ΅ Π²Π΅ΠΊΡΠΎΡΡ ${\bf uβ}$ ΠΈ ${\bf wβ}$ Π΄Π»Ρ $Bβ$ ΠΈΠΌΠ΅ΡΡ ΡΠ»Π΅Π΄ΡΡΡΠΈΠ΅ ΠΊΠΎΠΎΡΠ΄ΠΈΠ½Π°ΡΡ ΠΎΡΠ½ΠΎΡΠΈΡΠ΅Π»ΡΠ½ΠΎ Π±Π°Π·ΠΈΡΠ° $B$:
\begin{Π²ΡΡΠ°Π²Π½ΠΈΠ²Π°Π½ΠΈΠ΅*} ~[{\bf uβ}]_B & = & \left[\begin{array}{c} a \\ b \end{ΠΌΠ°ΡΡΠΈΠ²}\right] \qquad \\ ~[{\bf wβ}]_B & = & \left[\begin{array}{c} c \\ d \end{ΠΌΠ°ΡΡΠΈΠ²}\right]. \qquad \end{Π²ΡΡΠ°Π²Π½ΠΈΠ²Π°Π½ΠΈΠ΅*}
ΠΡΠΎ ΠΎΠ·Π½Π°ΡΠ°Π΅Ρ, ΡΡΠΎ \begin{Π²ΡΡΠ°Π²Π½ΠΈΠ²Π°Π½ΠΈΠ΅*} {\bf u’} & = & a{\bf u} + b{\bf w} \\ {\bf Ρ’} & = & c{\bf u} + d{\bf Ρ} \end{Π²ΡΡΠ°Π²Π½ΠΈΠ²Π°Π½ΠΈΠ΅*}
ΠΈΠ·ΠΌΠ΅Π½Π΅Π½ΠΈΠ΅ ΠΌΠ°ΡΡΠΈΡΡ ΠΊΠΎΠΎΡΠ΄ΠΈΠ½Π°Ρ Ρ $B’$ Π½Π° $B$ $$ P = \left[\begin{array}{cc} a & c \\ b & d \\ \end{array} \right ] $$ ΠΎΠΏΡΠ΅Π΄Π΅Π»ΡΠ΅Ρ ΠΈΠ·ΠΌΠ΅Π½Π΅Π½ΠΈΠ΅ ΠΊΠΎΠΎΡΠ΄ΠΈΠ½Π°Ρ ${\bf v} \in V$ ΠΏΡΠΈ Π·Π°ΠΌΠ΅Π½Π΅ Π±Π°Π·ΠΈΡΠ° Ρ $B’$ Π½Π° $B$.
{-1} = \left[\begin{ΠΌΠ°ΡΡΠΈΠ²}{cc}
\frac{1}{5} & \frac{2}{5} \\
-\frac{1}{5} ΠΈ \frac{3}{5}
\ΠΊΠΎΠ½Π΅Ρ{ΠΌΠ°ΡΡΠΈΠ²}\ΡΠΏΡΠ°Π²Π°],
$$
ΠΌΡ ΠΌΠΎΠΆΠ΅ΠΌ ΡΠ±Π΅Π΄ΠΈΡΡΡΡ, ΡΡΠΎ
$$
[{\bf v}]_{Bβ} = \left[\begin{ΠΌΠ°ΡΡΠΈΠ²}{cc}
\frac{1}{5} & \frac{2}{5} \\
-\frac{1}{5} ΠΈ \frac{3}{5}
\end{ΠΌΠ°ΡΡΠΈΠ²}\right]\left[\begin{ΠΌΠ°ΡΡΠΈΠ²}{c}
4\3
\end{ΠΌΠ°ΡΡΠΈΠ²}\right] = \left[\begin{ΠΌΠ°ΡΡΠΈΠ²}{c}
2\1
\ΠΊΠΎΠ½Π΅Ρ{ΠΌΠ°ΡΡΠΈΠ²}\ΡΠΏΡΠ°Π²Π°]
$$
Ρ ΡΠ΅Π³ΠΎ ΠΌΡ Π½Π°ΡΠ°Π»ΠΈ.
Π ΡΠ»Π΅Π΄ΡΡΡΠ΅ΠΌ ΠΏΡΠΈΠΌΠ΅ΡΠ΅ ΠΌΡ Π²Π²ΠΎΠ΄ΠΈΠΌ ΡΡΠ΅ΡΠΈΠΉ Π±Π°Π·ΠΈΡ, ΡΡΠΎΠ±Ρ ΠΏΠΎΡΠΌΠΎΡΡΠ΅ΡΡ Π½Π° ΠΎΡΠ½ΠΎΡΠ΅Π½ΠΈΡ ΠΌΠ΅ΠΆΠ΄Ρ Π΄Π²ΡΠΌΡ Π½Π΅ΡΡΠ°Π½Π΄Π°ΡΡΠ½ΡΠΌΠΈ Π±Π°Π·Π°ΠΌΠΈ .
ΠΡΠΈΠΌΠ΅Ρ
ΠΡΡΡΡ $Bβ = \left\{ \left[ {2 \top 1} \right],\left[ {1 \top 4} \right]\right\}$. Π§ΡΠΎΠ±Ρ Π½Π°ΠΉΡΠΈ ΠΈΠ·ΠΌΠ΅Π½Π΅Π½ΠΈΠ΅ ΠΌΠ°ΡΡΠΈΡΡ ΠΊΠΎΠΎΡΠ΄ΠΈΠ½Π°Ρ ΠΈΠ· Π±Π°Π·ΠΈΡΠ° $B’$ ΠΈΠ· ΠΏΡΠ΅Π΄ΡΠ΄ΡΡΠ΅Π³ΠΎ ΠΏΡΠΈΠΌΠ΅ΡΠ° Π² $Bβ$, ΡΠ½Π°ΡΠ°Π»Π° Π²ΡΡΠ°Π·ΠΈΠΌ Π±Π°Π·ΠΈΡΠ½ΡΠ΅ Π²Π΅ΠΊΡΠΎΡΡ $\left[ {3 \atop 1} \right]$ ΠΈ $\left[ { -2 \ΠΏΠΎΠ²Π΅ΡΡ
1} \ΡΠΏΡΠ°Π²Π°]$ $B’$ ΠΊΠ°ΠΊ Π»ΠΈΠ½Π΅ΠΉΠ½ΡΠ΅ ΠΊΠΎΠΌΠ±ΠΈΠ½Π°ΡΠΈΠΈ Π±Π°Π·ΠΈΡΠ½ΡΡ
Π²Π΅ΠΊΡΠΎΡΠΎΠ² $\left[ {2 \ΠΏΠΎΠ²Π΅ΡΡ
1} \ΡΠΏΡΠ°Π²Π°]$ ΠΈ $\left[ {1 \ΠΏΠΎΠ²Π΅ΡΡ
4} \ΡΠΏΡΠ°Π²Π°] $ of $Bβ$: \begin{eqnarray*} \mbox{Set }\left[ \begin{array}{c} 3 \\ 1 \end{array}\right] & = & a\left[\begin {array}{c} 2 \\ 1 \end{ΠΌΠ°ΡΡΠΈΠ²}\right] + b\left[\begin{array}{c} 1 \\ 4 \end{ΠΌΠ°ΡΡΠΈΠ²}\right] \\ \left[\begin {array}{c} -2 \\ 1 \end{ΠΌΠ°ΡΡΠΈΠ²}\right] & = & c \left[\begin{array}{c} 2 \\ 1 \end{ΠΌΠ°ΡΡΠΈΠ²}\right] + d\left [\begin{array}{c} 1 \\ 4 \end{array}\right] \end{eqnarray*} ΠΈ ΡΠ΅ΡΠΈΡΡ ΠΏΠΎΠ»ΡΡΠ΅Π½Π½ΡΠ΅ ΡΠΈΡΡΠ΅ΠΌΡ r $a,b,c,$ ΠΈ $d$: \begin{ eqnarray*} \left[ \begin{array}{c} 3 \\ 1 \end{ΠΌΠ°ΡΡΠΈΠ²}\right] & = & \frac{11}{7} \left[\begin{array}{c} 2 \ \ 1 \end{ΠΌΠ°ΡΡΠΈΠ²}\right] β \frac{1}{7}\left[\begin{array}{c} 1 \\ 4 \end{ΠΌΠ°ΡΡΠΈΠ²}\right] \\ \left[\begin{ array}{c} -2 \\ 1 \end{array}\right] & = & \frac{- 9}{7}\left[\begin{array}{c} 2 \\ 1 \end{ΠΌΠ°ΡΡΠΈΠ²}\right] + \frac{4}{7}\left[\begin{array}{c} 1 \\ 4 \end{array}\right] \end{eqnarray*} Π’Π°ΠΊΠΈΠΌ ΠΎΠ±ΡΠ°Π·ΠΎΠΌ, ΠΌΠ°ΡΡΠΈΡΠ½Π°Ρ ΡΠΎΡΠΌΠ° ΠΏΠ΅ΡΠ΅Ρ
ΠΎΠ΄Π° $B’$ Π² $Bβ$ ΡΠ°Π²Π½Π° $$ \left[\begin{array}{cc} \frac{11}{ 7} & \frac{-9}{7} \\ \frac{-1}{7} & \frac{4}{7} \end{ΠΌΠ°ΡΡΠΈΠ²}\right].
$$ ΠΠ΅ΠΊΡΠΎΡ ${\bf v}$ Ρ ΠΊΠΎΠΎΡΠ΄ΠΈΠ½Π°ΡΠ°ΠΌΠΈ $\left[ {2 \atop 1} \right]$ ΠΎΡΠ½ΠΎΡΠΈΡΠ΅Π»ΡΠ½ΠΎ Π±Π°Π·ΠΈΡΠ° $B’$ ΠΈΠΌΠ΅Π΅Ρ ΠΊΠΎΠΎΡΠ΄ΠΈΠ½Π°ΡΡ $$ \left[\begin{array}{cc} \frac{11}{7} & \frac{-9}{7} \\ \frac{-1}{9} & \frac{4}{7} \end{ΠΌΠ°ΡΡΠΈΠ²}\right]\left[\ begin{array}{c} 2 \\ 1 \end{array}\right] = \left[\begin{array}{c} \frac{13}{7} \\ \frac{2}{7} \ end{array}\right] $$ ΠΎΡΠ½ΠΎΡΠΈΡΠ΅Π»ΡΠ½ΠΎ Π±Π°Π·ΠΈΡΠ° $Bβ$. ΠΠ΅ΡΠ½Π΅ΠΌΡΡ ΠΊ ΡΡΠ°Π½Π΄Π°ΡΡΠ½ΠΎΠΌΡ Π±Π°Π·ΠΈΡΡ: $$ [ {\bf v} ]_B = \frac{13}{7}\left[\begin{array}{c} 2 \\ 1 \end{array}\right ] + \frac{2}{7}\left[\begin{array}{c} 1 \\ 4 \end{array}\right] = \left[\begin{array}{c} 4 \\ 3 \ end{array}\right], $$, ΡΡΠΎ ΡΠΎΠ³Π»Π°ΡΡΠ΅ΡΡΡ Ρ ΡΠ΅Π·ΡΠ»ΡΡΠ°ΡΠ°ΠΌΠΈ ΠΏΡΠ΅Π΄ΡΠ΄ΡΡΠ΅Π³ΠΎ ΠΏΡΠΈΠΌΠ΅ΡΠ°.
ΠΡΠ°ΡΠ΅Π½ΠΈΠ΅ ΠΎΡΠ΅ΠΉ ΠΊΠΎΠΎΡΠ΄ΠΈΠ½Π°Ρ
ΠΡΠ΅Π΄ΠΏΠΎΠ»ΠΎΠΆΠΈΠΌ, ΠΌΡ ΠΏΠΎΠ»ΡΡΠ°Π΅ΠΌ Π½ΠΎΠ²ΡΡ ΡΠΈΡΡΠ΅ΠΌΡ ΠΊΠΎΠΎΡΠ΄ΠΈΠ½Π°Ρ ΠΈΠ· ΡΡΠ°Π½Π΄Π°ΡΡΠ½ΠΎΠΉ
ΠΏΡΡΠΌΠΎΡΠ³ΠΎΠ»ΡΠ½ΠΎΠΉ ΡΠΈΡΡΠ΅ΠΌΠ΅ ΠΊΠΎΠΎΡΠ΄ΠΈΠ½Π°Ρ, Π²ΡΠ°ΡΠ°Ρ ΠΎΡΠΈ ΠΏΡΠΎΡΠΈΠ² ΡΠ°ΡΠΎΠ²ΠΎΠΉ ΡΡΡΠ΅Π»ΠΊΠΈ Π½Π°
ΡΠ³ΠΎΠ» $\theta$. ΠΠΎΠ²ΡΠΉ Π±Π°Π·ΠΈΡ $Bβ = \left\{{\bf uβ, vβ}\right\}$
Π΅Π΄ΠΈΠ½ΠΈΡΠ½ΡΡ
Π²Π΅ΠΊΡΠΎΡΠΎΠ² Π²Π΄ΠΎΠ»Ρ ΠΎΡΠ΅ΠΉ $xβ$ ΠΈ $yβ$ ΡΠΎΠΎΡΠ²Π΅ΡΡΡΠ²Π΅Π½Π½ΠΎ, ΠΈΠΌΠ΅Π΅Ρ
ΠΊΠΎΠΎΡΠ΄ΠΈΠ½Π°ΡΡ
\begin{Π²ΡΡΠ°Π²Π½ΠΈΠ²Π°Π½ΠΈΠ΅*}
~[{\bf u’}]_B & = & \left[\begin{ΠΌΠ°ΡΡΠΈΠ²}{c}
\cos\ΡΠ΅ΡΠ° \\ \sin\ΡΠ΅ΡΠ°
\ΠΊΠΎΠ½Π΅Ρ{ΠΌΠ°ΡΡΠΈΠ²}\ΡΠΏΡΠ°Π²Π°] \\
~[{\bf v’}]_B & = & \left[\begin{ΠΌΠ°ΡΡΠΈΠ²}{c}
-\sin\theta \\ \cos\theta
\end{ΠΌΠ°ΡΡΠΈΠ²} \right]
\end{Π²ΡΡΠ°Π²Π½ΠΈΠ²Π°Π½ΠΈΠ΅*}
Π² ΠΈΡΡ
ΠΎΠ΄Π½ΠΎΠΉ ΡΠΈΡΡΠ΅ΠΌΠ΅ ΠΊΠΎΠΎΡΠ΄ΠΈΠ½Π°Ρ.







Π΅. , $$\overrightarrow{OP}=p_1 \cdot \overrightarrow{u}+p_2 \cdot \overrightarrow{v}$$
